The Audio PANCE and PANRE Physician Assistant Board Review Podcast http://podcast.thepalife.com/ Multiple Choice Physician Assistant Board Review and Rotation Exam Questions on the Go – Listen and Learn Tue, 15 Aug 2023 10:00:36 +0000 en-US hourly 1 http://podcast.thepalife.com/wp-content/uploads/2017/04/cropped-The-Audio-PANCE-and-PANRE-512x512-32x32.jpg The Audio PANCE and PANRE Physician Assistant Board Review Podcast http://podcast.thepalife.com/ 32 32 The Audio PANCE and PANRE is an audio Board and Rotation Review Series that includes ten Multiple Choice PANCE and PANRE Board Review Questions in Each Episode. Now you can study for your PANCE, PANRE, and End or Rotation Exams in the gym, in the car, on a run, or while relaxing on the beach. This free series is limited to every other episode. To download the complete series join Smarty PANCE at https://smartypance.com/sign-up/ The Physician Assistant Life | Smarty PANCE clean The Physician Assistant Life | Smarty PANCE The PA Life, INC The PA Life, INC podcast Multiple Choice Physician Assistant Board and Rotation Review Questions on the Go - Listen and Learn The Audio PANCE and PANRE Physician Assistant Board Review Podcast http://podcast.thepalife.com/wp-content/uploads/powerpress/THE_AUDIO_PANCE_AND_PANRE_ACADEMY_PODCAST.png http://podcast.thepalife.com TV-G a2b772cd-52dc-5ac3-acca-7283a4cdfece 78572745 Podcast Episode 107: This vs. That – PANCE Blueprint Comparisons You Need to Know (Part 1) http://podcast.thepalife.com/episode-107/ Tue, 15 Aug 2023 10:00:36 +0000 https://podcast.thepalife.com/?p=591 http://podcast.thepalife.com/episode-107/#respond http://podcast.thepalife.com/episode-107/feed/ 0 <p>Listen to Podcast Episode 107: This vs. That – PANCE Blueprint Comparisons You Need to Know (Episode 1) In today’s session, we will be discussing five questions related to PANCE/PANRE Blueprint topics. These questions will cover similar presentations and crucial comparisons that are important for you to know. These topics are often used by PANCE/PANRE […]</p> <p>The post <a rel="nofollow" href="http://podcast.thepalife.com/episode-107/">Podcast Episode 107: This vs. That – PANCE Blueprint Comparisons You Need to Know (Part 1)</a> appeared first on <a rel="nofollow" href="http://podcast.thepalife.com">The Audio PANCE and PANRE</a>.</p> Listen to Podcast Episode 107: This vs. That – PANCE Blueprint Comparisons You Need to Know (Episode 1)

In today’s session, we will be discussing five questions related to PANCE/PANRE Blueprint topics. These questions will cover similar presentations and crucial comparisons that are important for you to know. These topics are often used by PANCE/PANRE test question writers, so it’s essential to learn how to differentiate between them. This is the first part (episode 1) of a series.

If you can’t see the audio player, click here to listen to the full episode.

Podcast Episode 107 This vs. That - PANCE Blueprint Comparisons You Need to Know (Part 1)

Links from today’s episode:

I hope you enjoy this free audio component of the examination portion of this site. Smarty PANCE includes over 2,000 interactive board review questions, along with flashcards, ReelDx cases, integrated Picmonics, and lessons covering every blueprint topic available to all Smarty PANCE members.

Interactive exam to complement today’s podcast

1. A 32-year-old woman presents with a 6-month history of loose bowel movements, approximately eight per day. Blood has been present in many of them. She has lost 30 pounds. For the past 6 weeks, she has had intermittent fever. She has had no previous gastrointestinal (GI) problems, and there is no family history of GI problems. On examination, the patient looks ill. Her blood pressure is 130/ 70 mm Hg. Her pulse is 108 beats/ minute and regular. There is generalized abdominal tenderness with no rebound. A sigmoidoscopy reveals a friable rectal mucosa with multiple bleeding points. Which of the following is the most likely diagnosis?

A) Crohn’s Disease
B) Ulcerative Colitis
C) Infectious Colitis
D) Irritable Bowel Syndrome (IBS)
E) Ischemic Colitis

Answer and topic summary

The answer is B) Ulcerative Colitis

The patient’s symptoms of chronic bloody diarrhea, weight loss, fever, and the sigmoidoscopy findings of a friable rectal mucosa with multiple bleeding points are consistent with a diagnosis of ulcerative colitis (UC), which is a form of inflammatory bowel disease (IBD). UC typically involves the rectum and may extend proximally to involve other parts of the colon.

Incorrect answers:

A) Crohn’s Disease: This is another type of IBD. However, Crohn’s usually presents with non-bloody diarrhea, abdominal pain, and may involve any part of the GI tract from mouth to anus, often with skip lesions. In this case, the bloody diarrhea and the findings on sigmoidoscopy are more indicative of ulcerative colitis.

C) Infectious Colitis: Although infectious causes can lead to similar symptoms, the duration of this patient’s symptoms (6 months) is much longer than typically seen with infectious colitis. Additionally, fever is less common in infectious colitis.

D) Irritable Bowel Syndrome (IBS): IBS is a functional GI disorder characterized by abdominal pain with a change in bowel habit. It does not cause weight loss, fever, or bloody stools.

E) Ischemic Colitis: This typically presents acutely in older patients or those with vascular risk factors. The clinical presentation often includes abrupt onset of abdominal pain and bloody diarrhea. The duration and pattern of symptoms in this patient are more consistent with IBD.

Smarty PANCE Content Blueprint Review:

Covered under ⇒ PANCE Blueprint GI and NutritionColorectal disordersInflammatory bowel disease


2. A 27-year-old female presents to the emergency department with a 3-day history of a widespread painful rash. She reports having started a new medication for her seizures 1 week ago. On examination, you note erythematous macules that are coalescing into large areas of epidermal detachment. The mucous membranes of her mouth, eyes, and genital region are also affected, and the skin involvement covers more than 30% of her body surface area. A skin biopsy reveals full-thickness epidermal necrosis. Based on her presentation and the medication history, which of the following diagnoses is she most likely suffering from, and which medication most likely contributed to this condition?

A) Erythema multiforme major secondary to Levetiracetam (Keppra)
B) Toxic Epidermal Necrolysis (TEN) secondary to Carbamazepine
C) Stevens-Johnson syndrome (SJS) secondary to Metformin
D) Acute generalized exanthematous pustulosis secondary to Lisinopril
E) Stevens-Johnson syndrome (SJS) secondary to Atorvastatin

Answer and topic summary

The answer is B) Toxic Epidermal Necrolysis (TEN) secondary to Carbamazepine

The patient’s presentation with widespread epidermal necrosis, mucous membrane involvement, and skin detachment affecting more than 30% of her body surface area is suggestive of Toxic Epidermal Necrolysis (TEN). SJS and TEN are considered a spectrum of the same disease, with SJS affecting less than 10% of body surface area, SJS-TEN overlap affecting 10-30%, and TEN affecting more than 30%. The diagnosis can be confirmed by biopsy (showing necrotic epithelium) if clinical characteristics (eg, target lesions progressing to bullae, ocular and mucous membrane involvement, Nikolsky sign, desquamation in sheets) are inconclusive. Immediate discontinuation of the offending drug is paramount. Patients with TEN often require transfer to a burn unit or an intensive care unit for supportive care. Fluid and electrolyte balance, pain control, and prevention of secondary infections are critical. Immunomodulatory agents like IVIG (intravenous immunoglobulin) or cyclosporine may be considered, but their efficacy is still under debate.

Option A: Erythema multiforme major: Presents with targetoid lesions and is less severe than SJS and TEN. Levetiracetam is not strongly associated with SJS or TEN.

Option D: Acute generalized exanthematous pustulosis is an extensive formation of nonfollicular sterile pustules on erythematous background combined with fever and peripheral blood leukocytosis. This uncommon eruption is most often an allergic reaction because of drugs such as aminopenicillins and sulfonamides inter alia.

A good way to remember the body surface area affected in TEN is “T hree x T en = T hirty percent in Toxic Epidermal Necrolysis

The distinction between SJS, SJS/TEN overlap, and TEN is based on the type of lesions and the amount of the body surface area with blisters and erosions

  • Blisters and erosions cover between 3% and 10% of the body in SJS
  • 11–30% in SJS/TEN overlap
  • over 30% in TEN

Smarty PANCE Content Blueprint Review:

Covered under ⇒  PANCE Blueprint DermatologyDesquamationToxic epidermal necrolysis and Stevens-Johnson syndrome


3. A 32-year-old male presents to his primary care physician complaining of persistent fatigue, unintentional weight loss over the past 3 months, and a painless swelling in the left side of his neck. On examination, there is a non-tender, rubbery 3cm lymphadenopathy in the left cervical region. A subsequent excisional biopsy of the node is performed, and microscopy reveals large atypical cells with abundant cytoplasm and bilobed nuclei, reminiscent of an “owl’s eye” appearance. What is the most likely diagnosis?

A) Acute lymphoblastic leukemia (ALL)
B) Non-Hodgkin’s Lymphoma
C) Burkitt’s lymphoma
D) Hodgkin’s Lymphoma
E) Chronic lymphocytic leukemia (CLL)

Answer and topic summary

The answer is D) Hodgkin’s Lymphoma

The patient’s clinical presentation with painless cervical lymphadenopathy and constitutional symptoms, coupled with the histological finding of large cells bearing the “owl’s eye” appearance (Reed-Sternberg cells), is characteristic of Hodgkin’s Lymphoma. Hodkin’s lymphoma is the most common type of lymphoma and usually presents as a solitary cervical lymph node that has been there for > 30 days. It is commonly manifested with painless cervical adenopathy; there may be splenomegaly or enlargement of other immune tissue, fever, weight loss, fatigue, or night sweats. Upper body lymph nodes are the most common. Diagnosis is based on clinical presentation and confirmed by an excisional biopsy of an involved lymph node. Histology often reveals the pathognomonic Reed-Sternberg cells, which have bilobed nuclei. A chest radiograph should be obtained to search for mediastinal adenopathy. Treatment for Hodgkin’s Lymphoma depends on the stage of the disease and typically involves a combination of chemotherapy, most commonly the ABVD regimen (Adriamycin, Bleomycin, Vinblastine, Dacarbazine). Radiation therapy might be added, especially for localized disease.

Answer Choices Explanations:

A) Acute lymphoblastic leukemia (ALL): Affects mainly children and presents with bone marrow failure symptoms.

B) Non-Hodgkin’s Lymphoma: A group of lymphoid malignancies. Reed-Sternberg cells distinguish Hodgkin’s Lymphoma from Non-Hodgkin’s types.

C) Burkitt’s lymphoma: Fast-growing non-Hodgkin’s lymphoma linked to the Epstein-Barr virus. Exhibits a “starry sky” appearance on microscopy.

E) Chronic lymphocytic leukemia (CLL): A malignancy of mature B cells in older adults. Increased lymphocyte count and smudge cells on a peripheral blood smear.

Smarty PANCE Content Blueprint Review:

Covered under ⇒ PANCE Blueprint HematologyNeoplasms, premalignancies, and malignanciesLymphoma (ReelDx + Lecture)


4. A 65-year-old male presents to your office complaining of fatigue and shortness of breath with exertion. The patient reports minimal cough. On physical exam, you note a thin, barrel-chested man with decreased heart and breath sounds, pursed-lip breathing, end-expiratory wheezing, and scattered rhonchi. Chest X-ray reveals a flattened diaphragm, hyperinflation, and a small, thin-appearing heart. PFTs show a decreased FEV1 / FVC ratio.

A) Asthma
B) Chronic bronchitis
C) Emphysema
D) Pulmonary fibrosis
E) Bronchiectasis

Answer and topic summary

The answer is C. Emphysema

The patient’s presentation of shortness of breath with exertion, barrel-chested appearance, pursed-lip breathing, findings on chest X-ray (e.g., flattened diaphragm, hyperinflation, and a small heart), and pulmonary function test results all align with the diagnosis of emphysema.

Emphysema is a lung disease that causes shortness of breath. It is one of the two main conditions that make up chronic obstructive pulmonary disease (COPD). The other condition is chronic bronchitis.

In emphysema, the air sacs (alveoli) in the lungs are damaged. Over time, the inner walls of the air sacs weaken and rupture — creating larger air spaces instead of many small ones. This reduces the surface area of the lungs and, in turn, the amount of oxygen that reaches your bloodstream.

The main cause of emphysema is cigarette smoking. Other causes include air pollution and chemical fumes. A small percentage of cases are caused by a familial or genetic disorder, alpha-1-antitrypsin deficiency.

  • The body’s natural response to ↓ lung function is chronic hyperventilation = Pink Puffers! CO2 Retainers – the body must increase ventilation to blow off CO2
  • Minimal cough (compared to chronic bronchitis), quiet lungs
  • Minimal sputum (compared to chronic bronchitis)
  • Thin, underweight, and barrel chest

Answer Choices Explanations:

A) Asthma: An obstructive lung disease that is reversible and is marked by bronchoconstriction. It typically presents with episodes of wheezing triggered by factors such as allergens, exercise, or infections.

B) Chronic bronchitis: Defined by a productive cough for 3 consecutive months in 2 successive years. It’s a subtype of chronic obstructive pulmonary disease (COPD), with its main causative factor being smoking. The primary concern is mucus production, as opposed to the alveolar wall destruction seen in emphysema.

D) Pulmonary fibrosis: Represents a set of disorders causing scarring of the lung tissue, leading to a restrictive lung disease pattern. Symptoms might include a dry cough, finger clubbing, and inspiratory crackles upon examination.

E) Bronchiectasis: It is marked by the chronic dilation of bronchi or bronchioles due to repeated infections and inflammation. Patients often have a chronic cough and produce significant amounts of sputum.

Smarty PANCE Content Blueprint Review:

Covered under ⇒ PANCE Blueprint Pulmonary ⇒ Chronic obstructive pulmonary diseases ⇒ Emphysema

Also covered as part of the Internal Medicine EORFamily Medicine EOR, and General Surgery EOR topic lists


5. A 68-year-old woman presents to the emergency department with chest pain that started 5 hours ago. She describes the pain as a heavy pressure radiating to her left arm. It began at rest and has persisted. She has a history of hypertension and diabetes mellitus. An ECG shows ST-segment depressions in leads II, III, and aVF. Troponin I levels are elevated at 0.55 ng/mL (normal: <0.04 ng/mL). Which of the following is the most likely diagnosis?

A) Stable angina
B) Unstable angina
C) Non-ST segment elevation myocardial infarction (NSTEMI)
D) ST-segment elevation myocardial infarction (STEMI)
E) Prinzmetal angina

Answer and topic summary

The answer is C) Non-ST segment elevation myocardial infarction (NSTEMI)

This patient’s presentation with chest pain at rest, ST-segment depressions on ECG, and elevated troponin I levels are consistent with NSTEMI. NSTEMI is characterized by myocardial ischemia severe enough to result in myocyte injury and elevated cardiac biomarkers but not sufficient to cause ST-segment elevation on ECG.

NSTEMI is diagnosed clinically with supporting findings from ECG changes, especially ST-segment depressions or T-wave inversions, and elevated cardiac biomarkers (troponins or CK-MB). Elevated troponins, in particular, differentiate NSTEMI from unstable angina.

Immediate management includes antiplatelet agents (e.g., aspirin, clopidogrel), anticoagulation (e.g., heparin or enoxaparin), and nitrates for symptom relief. A coronary angiography, followed by possible revascularization (percutaneous coronary intervention or coronary artery bypass grafting), might be required based on risk assessment.

Answer Choices Explanations:

A) Stable angina: Characterized by predictable chest pain or discomfort with exertion or stress, which is relieved by rest or nitroglycerin.

B) Unstable angina: Presents as chest pain at rest, new-onset angina, or angina that is more frequent, longer in duration, or not relieved by rest/nitroglycerin. Crucially, cardiac biomarkers (e.g., troponins) remain normal, differentiating it from NSTEMI.

D) ST-segment elevation myocardial infarction (STEMI): Acute myocardial infarction characterized by ST-segment elevation on ECG. Requires immediate reperfusion therapy.

E) Prinzmetal angina: Caused by coronary artery spasm leading to transient ST-segment elevation. Pain typically occurs at rest, often in the early morning hours.

Smarty PANCE Content Blueprint Review:

PANCE Blueprint Cardiology => Coronary Heart Disease (PEARLS) => Non-ST-Segment Elevation MI (NSTEMI) ReelDx


This podcast is available on every device!

You can download and listen to past FREE episodes here, on iTunes, Spotify, Google Podcasts, Stitcher, Amazon Music, and all podcasting apps.

Download Interactive Content Blueprint Checklists for the PANCE, PANRE, EOR, and PANRE-LA

Interactive Content Blueprints for the PANCE PANRE and PANRE-LA

Follow this link to download your FREE copy of the PANCE/PANRE/EOR Content Blueprint Checklists.

Print it up and start crossing out the topics you understand, marking the ones you don’t, and making notes of key terms you should remember. The PDF version is interactive and linked directly to the individual lessons on Smarty PANCE.

Smarty PANCE is not sponsored or endorsed by, or affiliated with, the National Commission on Certification of Physician Assistants.

The post Podcast Episode 107: This vs. That – PANCE Blueprint Comparisons You Need to Know (Part 1) appeared first on The Audio PANCE and PANRE.

]]>
Listen to Podcast Episode 107: This vs. That – PANCE Blueprint Comparisons You Need to Know (Episode 1) In today’s session, we will be discussing five questions related to PANCE/PANRE Blueprint topics. These questions will cover similar presentations a... In today’s session, we will be discussing five questions related to PANCE/PANRE Blueprint topics. These questions will cover similar presentations and crucial comparisons that are important for you to know. These topics are often used by PANCE/PANRE test question writers, so it’s essential to learn how to differentiate between them. This is the first part (episode 1) of a series.
If you can’t see the audio player, click here to listen to the full episode.

Links from today’s episode:

* Sign up for our new PANCE and PANRE Test-Taking Masterclass.
* Check out my first blog in our “This vs. That” Blueprint series: The PANCE Blueprint Showdown: Crohn’s Disease vs. Ulcerative Colitis.
* Want a question of the day that covers the Blueprint – ALL of it? Sign up for the Entire Blueprint Email Series.
* Follow Smarty PANCE and The Daily PANCE Blueprint on Instagram and Facebook for more daily questions.
* Join the Smarty PANCE Member’s Community, then sign up for a study group to get updates about upcoming webinars.

I hope you enjoy this free audio component of the examination portion of this site. Smarty PANCE includes over 2,000 interactive board review questions, along with flashcards, ReelDx cases, integrated Picmonics, and lessons covering every blueprint topic available to all Smarty PANCE members.

* You can download and listen to past FREE episodes here, on iTunes, Spotify, Google Podcasts, Stitcher, Amazon Music, and all podcasting apps.
* You can listen to all the latest episodes, take interactive quizzes, and download more resources on each episode page.

Interactive exam to complement today’s podcast
1. A 32-year-old woman presents with a 6-month history of loose bowel movements, approximately eight per day. Blood has been present in many of them. She has lost 30 pounds. For the past 6 weeks, she has had intermittent fever. She has had no previous gastrointestinal (GI) problems, and there is no family history of GI problems. On examination,]]>
The Physician Assistant Life | Smarty PANCE full 30:56 591
Podcast Episode 106: Ten PANCE, PANRE, and Rotation Questions + Review of Adrenal Insufficiency http://podcast.thepalife.com/episode-106/ Wed, 14 Jun 2023 09:00:45 +0000 https://podcast.thepalife.com/?p=581 http://podcast.thepalife.com/episode-106/#respond http://podcast.thepalife.com/episode-106/feed/ 0 <p>Listen to Podcast Episode 106: Ten PANCE, PANRE, and Rotation Review Questions + Review of Adrenal Insufficiency If you can’t see the audio player, click here to listen to the full episode. Welcome to episode 106 of the Audio PANCE and PANRE Physician Assistant/Associate Board Review Podcast. Join me today as we cover ten board […]</p> <p>The post <a rel="nofollow" href="http://podcast.thepalife.com/episode-106/">Podcast Episode 106: Ten PANCE, PANRE, and Rotation Questions + Review of Adrenal Insufficiency</a> appeared first on <a rel="nofollow" href="http://podcast.thepalife.com">The Audio PANCE and PANRE</a>.</p> Listen to Podcast Episode 106: Ten PANCE, PANRE, and Rotation Review Questions + Review of Adrenal Insufficiency

If you can’t see the audio player, click here to listen to the full episode.

Podcast Episode 106 - The Audio PANCE and PANRE Board Review PodcastWelcome to episode 106 of the Audio PANCE and PANRE Physician Assistant/Associate Board Review Podcast.

Join me today as we cover ten board review questions for your PANCE, PANRE, EOR, and EOC exams.

Links from today’s episode:

I hope you enjoy this free audio component of the examination portion of this site. Smarty PANCE includes over 2,000 interactive board review questions, along with flashcards, ReelDx cases, integrated Picmonics, and lessons covering every blueprint topic available to all Smarty PANCE members.

Interactive exam to complement today’s podcast

1. A 22-year-old female with an unknown past medical history presents to the ER with a prolonged seizure lasting more than 5 minutes per EMS. Her airway was supported, and IV access was obtained en route to the ER. Her blood glucose is 120. Her vitals are stable, and laboratory studies are relatively unremarkable. Which of the following would be an appropriate medication to give to this patient?

A. Calcium gluconate
B. Propofol infusion
C. Lorazepam
D. Clonidine
E. Narcan

Answer and topic summary

The answer is C. Lorazepam

The patient has status epilepticus, which is defined as >5 minutes of continuous seizures or > 2 discrete seizures between which there is incomplete recovery of consciousness. Most episodes of status epilepticus in adults are due to a brain lesion or a toxic/metabolic disturbance (e.g., alcohol withdrawal, hypoglycemia, etc). Immediate management of status epilepticus includes stabilizing the airway, placing pulse oximetry and cardiorespiratory monitors, establishing IV access, and getting a glucose level. The first-line pharmacological agent given is a benzodiazepine (e.g., lorazepam or diazepam). Keppra, valproate, or other anti-seizure medication can be given as well.

Smarty PANCE Content Blueprint Review:

Covered under ⇒ PANCE Blueprint NeurologySeizure disordersStatus epilepticus

Also covered as part of the Internal Medicine EOR and Emergency Medicine EOR topic list


2. A 41-year-old male presents to the ER after a construction accident that left him with severe right eye pain and decreased visual acuity. On a physical exam, you notice a teardrop-shaped pupil. Which of the following is the most likely diagnosis?

A. Globe rupture
B. Acute angle-closure glaucoma
C. Retinal detachment
D. Corneal laceration
E. Metallic foreign body

Answer and topic summary

The answer is A. Globe rupture

Mechanical globe injuries occur when there is a laceration or full-thickness rupture through the cornea and/or sclera. Globe rupture (also called an open globe) follows blunt eye injury (e.g., motor vehicle crash, assault, thrown ball, etc). Globe lacerations occur after trauma from a sharp-penetrating object (e.g., knife or high-velocity projectile). PE signs include subconjunctival hemorrhage, irregularly-shaped pupil, hyphema (anterior chamber bleeding), decreased visual acuity, and limited EOM. This is an emergency and a patient should see an ophthalmologist immediately.

Smarty PANCE Content Blueprint Review:

Covered under ⇒ PANCE Blueprint EENTTraumatic disordersGlobe rupture


3. A 45-year-old male veteran presents to your clinic complaining of the following for 4 months: problems with concentration, difficulty staying asleep, persistent negative state, inability to remember events that happened while he was in Iraq, feelings of detachment from others, avoidance of other veterans, and distress when seeing violence on the TV. He denies hallucinations. He has been a veteran for 5 years. Which of the following is the most likely diagnosis?

A. Acute stress disorder
B. Cyclothymic disorder
C. Brief psychotic episode
D. Derealization/depersonalization disorder
E. Post-traumatic stress disorder

Answer and topic summary

The answer is E. Post-traumatic stress disorder

Post-traumatic stress disorder is a complex disorder defined by the behavioral, somatic, cognitive, and emotional effects of trauma. The most common causes are sexual relationship violence (33%), death or traumatic event of a loved one (30%), interpersonal violence like physical assault (12%), and participation in organized violence like combat (11%). The DSM-V criteria for PTSD is:

Exposure to a traumatic event(s). And then for ≥1 month of each of these that started/worsened after a traumatic event and are associated with the traumatic event:

1) Intrusion symptoms

2) Avoidance of stimuli

3) Alterations in cognitions and mood

4) Alterations in arousal/reactivity

Treatment is SSRIs + counseling/psychotherapy. About 50% of patients will recover in 3 months with treatment.

Smarty PANCE Content Blueprint Review:

Covered under ⇒ PANCE Blueprint PsychiatryTrauma and stressor-related disordersPost-traumatic stress disorder

Also covered in Emergency Medicine EOR, and Family Medicine EOR Blueprint


4. Which of the following clinical findings would you likely expect in a patient with primary adrenal insufficiency?

A. Hypernatremia, hypokalemia, tired
B. Moon facies, depression, bruising
C. Polyuria, polydipsia, intense thirst
D. Fatigue, hyponatremia, hyperpigmentation
E. Hypothermia, bradycardia, goiter

Answer and topic summary

The answer is d. Fatigue, hyponatremia, hyperpigmentation

Adrenal insufficiency is defined as insufficient production of hormones by the adrenal glands. Remember, the MOST common cause of adrenal insufficiency overall is the sudden withdrawal of exogenous steroids. The most common primary cause of adrenal insufficiency is autoimmune adrenalitis (aka Addison disease, aka 21-hydroxylase antibodies against the adrenal gland). Patients will have symptoms and signs of glucocorticoid & mineralocorticoid deficiency, such as fatigue (MC), weight loss, nausea, vomiting, hyponatremia, hyperkalemia, skin hyperpigmentation (due to increased production of proopiomelanocortin prohormone), hypotension, and salt craving. Remember, secondary adrenal insufficiency does NOT lead to hyperpigmentation, only primary adrenal insufficiency (this is commonly asked in tests). Diagnosis of adrenal insufficiency is made by serum cortisol level and then a cosyntropin stimulation test. Treatment is glucocorticoids +/- mineralocorticoids.

Smarty PANCE Content Blueprint Review:

Covered under ⇒ PANCE Blueprint EndocrinologyAdrenal DisordersPrimary adrenal insufficiency

Also covered as part of the Family Medicine EOR, Internal Medicine EOR, and Emergency Medicine EOR topic list


5. A 42-year-old male presents with right upper extremity numbness, dysesthesia, weakness, and arm pain with exertion. The symptoms are aggravated by activity and reaching overhead. Which of the following is the most likely diagnosis?

A. Thoracic outlet syndrome
B. Median nerve compression
C. Biceps tendon rupture
D. Carpal tunnel syndrome
E. Subclavian steal syndrome

Answer and topic summary

The answer is A. Thoracic outlet syndrome

Thoracic outlet syndrome (TOS) refers to a constellation of signs & symptoms that occur due to compression of the neurovascular bundle in the space of the thoracic outlet. Causes include trauma, repetitive injuries, anatomical defects, and pregnancy. The clinical presentation depends on what is being compressed (i.e., nerve, vein, and/or artery), but may include the following: pain, numbness, weakness, muscle weakness, fatigue, swelling, or coldness. Often these symptoms are aggravated by activity or raising the arm overhead. Diagnostic tests include ultrasound, x-ray, CT scan, EMG, MRI, etc. Treatment options depend on what type of TOS the patient has (e.g., physical therapy for neurogenic TOS).

Smarty PANCE Content Blueprint Review:

Covered under ⇒ PANCE Blueprint MusculoskeletalSpinal DisordersThoracic outlet syndrome


6. A 34-year-old female is diagnosed with immune thrombocytopenic purpura. What is NOT a known cause or risk factor of ITP?

A. Hepatitis C
B. Systemic lupus erythematosus
C. HIV
D. COVID-19
E. Warfarin

Answer and topic summary

The answer is E. Warfarin

Immune thrombocytopenic purpura (ITP) is acquired thrombocytopenia caused by autoantibodies against platelet antigens. Primary ITP is specifically due to autoimmune mechanisms, whereas secondary ITP is associated with other conditions or drugs. Some examples of causes of secondary ITP include systemic lupus erythematosus (SLE), HIV infection, hepatitis C infection, COVID-19, and thyroid dysfunction. There are also many drugs that can lead to ITP, but warfarin is not known to be one of them. Clinical symptoms include petechiae, purpura, easy bleeding, bruising. Obviously, platelets will be low. Work-up for ITP should include HIV and HCV testing, TSH, ANA, coagulation studies, and peripheral blood smear. A bone marrow biopsy can be done to rule out something more pathological. Treatment includes steroids (high-dose) and IVIG. Definitive treatment is splenectomy.

Smarty PANCE Content Blueprint Review:

Covered under ⇒ PANCE Blueprint HematologyCoagulation DisordersThrombocytopeniaIdiopathic thrombocytopenic purpura

Also covered as part of the Internal Medicine EOR topic list


7. A 72-year-old African American male presents to the clinic complaining of weight loss, fatigue, and back pain. During the physical exam, you palpate an enlarged, irregular, nodular prostate. Which of the following is the most important risk factor for the development of this most likely diagnosis?

A. Genetic factors
B. Ethnicity
C. Increasing age
D. Obesity
E. High meat diet

Answer and topic summary

The answer is C. Increasing age

Prostate cancer is the most common cancer among men (excluding skin cancer) — as about 1 in 8 men will get it. The most important risk factor for the development of prostate cancer is increasing age. Other risk factors include ethnicity, genetic factors, obesity, tobacco, and a diet high in animal fat. Clinical manifestations may include non-specific urinary symptoms (frequency, urgency, etc), hematuria, bone pain, elevated PSA, and an indurated/asymmetrical prostate with nodules on digital rectal exam. Diagnosis is made by a transrectal biopsy of the prostate.

Smarty PANCE Content Blueprint Review:

Covered under ⇒ PANCE Blueprint GenitourinaryNeoplasms of the Genitourinary SystemProstate cancer

Also found in Internal Medicine EOR PAEA blueprint


8. A 52-year-old female with a history of previous abdominal surgeries presents to the ER with nausea, emesis, abdominal pain, and inability to pass gas. On physical exam, you notice abdominal distention and auscultate high-pitched tinkering sounds in the upper abdominal quadrants. Which of the following is the most likely diagnosis?

A. Mesenteric ischemia
B. Diverticulitis
C. Gastric ulcer perforation
D. Small bowel obstruction
E. Ischemic colitis

Answer and topic summary

The answer is D. Small bowel obstruction

A bowel obstruction happens when the normal flow of the GI tract is interrupted. The most common cause of small bowel obstruction is intraperitoneal adhesions (s/t previous surgeries). Other causes include hernias or neoplasms. Clinical features include nausea, vomiting, crampy abdominal pain, and obstipation (inability to pass stool). On physical exam you may note abdominal distention, tinkling high-pitch sounds on auscultation, and hyperresonance to percussion. Initial diagnostic modalities include an abdominal x-ray and CT with contrast. These patients need to be admitted for further evaluation and management. Surgery should be consulted.

Smarty PANCE Content Blueprint Review:

Covered under ⇒ PANCE Blueprint GI and NutritionDiseases of the Small IntestineSmall bowel obstruction

Also covered as part of the Family Medicine EOR, Emergency Medicine EOR, and General Surgery EOR topic list


9. Which of the following is the most common cause of acute cor pulmonale?

A. Cardiac tamponade
B. Pulmonary embolism
C. Mitral regurgitation
D. Congestive heart failure
E. Portopulmonary syndrome

Answer and topic summary

The answer is B. Pulmonary embolism

The most common cause of acute cor pulmonale is a massive pulmonary embolism (PE). Cor pulmonale is defined by RV failure/enlargement due to a pulmonary issue. The reason a PE leads to acute right heart failure is because a PE is a mechanical obstruction that increases vascular resistance in lungs and increases RV afterload. The RV can’t unload sufficiently and this results in dilation of the RV. The dilated RV impedes on the LV, leading to decreased LV output and decreased supply to the coronary arteries. It also causes RV wall tension/pressure to build and thus coronary perfusion is impeded to the right heart, further leading to ischemia. Also, the dilation of the RV can lead to tricuspid regurgitation, which further reduces blood flow. As the RV becomes ischemic, it can’t contract as well and this further decreases RV output and LV output — leading to a dangerous cycle toward shock. You can risk stratify the degree of RV compromise by getting an echocardiogram, BNP, & troponin.

Smarty PANCE Content Blueprint Review:

Covered under ⇒ PANCE Blueprint PulmonaryPulmonary Circulation Cor pulmonale


10. A 25-year-old female with no past medical history presents with nausea, emesis, and pelvic pain. She is normotensive, afebrile, and slightly tachycardic (HR 102 bpm). On physical exam, you palpate an ovarian/pelvic mass. Beta-hCG is negative. The pelvic US is pending. Which of the following is the most likely diagnosis?

A. Ectopic pregnancy
B. Appendicitis
C. Pelvic inflammatory disorder
D. Ovarian torsion
E. Placenta abruption

Answer and topic summary

D. Ovarian torsion

Ovarian torsion refers to the rotation of the ovary, often leading to complete or partial obstruction of blood supply. The most common predisposing factor is an ovarian mass. Presenting features include pelvic pain (90%), nausea, emesis, and fever. On physical exam they may have pelvic and/or abdominal tenderness. A pelvic ultrasound is the mainstay of evaluation when ovarian torsion is suspected. You can get a beta-hCG to rule out an ectopic pregnancy. The definitive diagnosis is made by directly visualizing a rotated ovary at the time of surgical evaluation. Surgery and OB-GYN should be consulted for further management.

Smarty PANCE Content Blueprint Review:

Covered under ⇒ PANCE Reproductive System BlueprintOvarian Torsion

Also covered as part of the Women’s Health PAEA EOR topic lists


This podcast is available on every device!

You can download and listen to past FREE episodes here, on iTunes, Spotify, Google Podcasts, Stitcher, Amazon Music, and all podcasting apps.

Download Interactive Content Blueprint Checklists for the PANCE, PANRE, EOR, and PANRE-LA

Interactive Content Blueprints for the PANCE PANRE and PANRE-LA

Follow this link to download your FREE copy of the PANCE/PANRE/EOR Content Blueprint Checklists.

Print it up and start crossing out the topics you understand, marking the ones you don’t, and making notes of key terms you should remember. The PDF version is interactive and linked directly to the individual lessons on Smarty PANCE.

Smarty PANCE is not sponsored or endorsed by, or affiliated with, the National Commission on Certification of Physician Assistants.

The post Podcast Episode 106: Ten PANCE, PANRE, and Rotation Questions + Review of Adrenal Insufficiency appeared first on The Audio PANCE and PANRE.

]]>
Listen to Podcast Episode 106: Ten PANCE, PANRE, and Rotation Review Questions + Review of Adrenal Insufficiency If you can’t see the audio player, click here to listen to the full episode. Welcome to episode 106 of the Audio PANCE and PANRE Physician ... If you can’t see the audio player, click here to listen to the full episode.
Welcome to episode 106 of the Audio PANCE and PANRE Physician Assistant/Associate Board Review Podcast.
Join me today as we cover ten board review questions for your PANCE, PANRE, EOR, and EOC exams.
Links from today’s episode:

* Sign up for our new PANCE and PANRE Test Taking Masterclass
* Sign up for the Entire Blueprint Email Series
* Follow Smarty PANCE and The Daily PANCE Blueprint on Instagram and Facebook
* Review adrenal insufficiency and adrenal disorders
* Join the Smarty PANCE Member’s Community, then sign up for a study group to get updates about upcoming webinars.

I hope you enjoy this free audio component of the examination portion of this site. Smarty PANCE includes over 2,000 interactive board review questions, along with flashcards, ReelDx cases, integrated Picmonics, and lessons covering every blueprint topic available to all Smarty PANCE members.

* You can download and listen to past FREE episodes here, on iTunes, Spotify, Google Podcasts, Stitcher, Amazon Music, and all podcasting apps
* You can listen to all the latest episodes, take interactive quizzes, and download more resources on each episode page.

Interactive exam to complement today’s podcast
1. A 22-year-old female with an unknown past medical history presents to the ER with a prolonged seizure lasting more than 5 minutes per EMS. Her airway was supported, and IV access was obtained en route to the ER. Her blood glucose is 120. Her vitals are stable, and laboratory studies are relatively unremarkable. Which of the following would be an appropriate medication to give to this patient?
A. Calcium gluconate
B. Propofol infusion
C. Lorazepam
D. Clonidine
E. Narcan
Smarty PANCE Content Blueprint Review:
Covered under ⇒ PANCE Blueprint Neurology full 28:55 581
Podcast Episode 105: Ten PANCE, PANRE, and Rotation Review Questions http://podcast.thepalife.com/podcast-episode-105-ten-pance-panre-and-rotation-review-questions/ Mon, 08 May 2023 09:00:03 +0000 https://podcast.thepalife.com/?p=573 http://podcast.thepalife.com/podcast-episode-105-ten-pance-panre-and-rotation-review-questions/#respond http://podcast.thepalife.com/podcast-episode-105-ten-pance-panre-and-rotation-review-questions/feed/ 0 <p>Listen to Podcast Episode 105: Ten PANCE, PANRE, and Rotation Review Questions If you can’t see the audio player, click here to listen to the full episode. Welcome to episode 105 of the Audio PANCE and PANRE Physician Assistant/Associate Board Review Podcast. Join me today as we cover ten board review questions for your PANCE, […]</p> <p>The post <a rel="nofollow" href="http://podcast.thepalife.com/podcast-episode-105-ten-pance-panre-and-rotation-review-questions/">Podcast Episode 105: Ten PANCE, PANRE, and Rotation Review Questions</a> appeared first on <a rel="nofollow" href="http://podcast.thepalife.com">The Audio PANCE and PANRE</a>.</p> Listen to Podcast Episode 105: Ten PANCE, PANRE, and Rotation Review Questions

If you can’t see the audio player, click here to listen to the full episode.

Podcast Episode 105 - The Audio PANCE and PANRE Board Review PodcastWelcome to episode 105 of the Audio PANCE and PANRE Physician Assistant/Associate Board Review Podcast.

Join me today as we cover ten board review questions for your PANCE, PANRE, EOR, and EOC exams.

Links from today’s episode:

I hope you enjoy this free audio component of the examination portion of this site. Smarty PANCE includes over 2,000 interactive board review questions, along with flashcards, ReelDx cases, integrated Picmonics, and lessons covering every blueprint topic available to all Smarty PANCE members.

Interactive exam to complement today’s podcast

1. A 68-year-old male farmer presents with a flesh-colored papule with a rolled border located on the right side of his forehead. As you examine the lesion closely, you notice something else about the lesion. Which of the following physical exam findings would make you more suspicious of malignancy?

A. Telangiectasia
B. Nikolsky sign
C. Hypopigmentation
D. Tenderness to palpation
E. Central umbilication

Answer and topic summary

The answer is A. Telangiectasia

The patient has basal cell carcinoma, which is a skin cancer with low metastatic potential. It commonly occurs on the face (70% of the time). There are different types of BCC (nodular vs. superficial vs. infiltrative). However, nodular is the most common (80%) and typical characteristics you may see include a papule with a rolled border, pearly-like look, flesh-colored, and telangiectasia. Risk factors for BCC include UV radiation, certain genes, inherited disorders, etc.

Smarty PANCE Content Blueprint Review:

Covered under ⇒ PANCE Blueprint DermatologyDermatologic NeoplasmsBasal cell carcinoma

Also covered as part of the General Surgery EOR and Family Medicine EOR topic list

2. Which of the following is the most common type of elder abuse?

A. Neglect
B. Emotional abuse
C. Physical abuse
D. Sexual abuse
E. Financial exploitation

Answer and topic summary

The answer is A. Neglect

The most common type of elder abuse is neglect, which refers to the failure of a trusted person to protect an older person from harm or provide for their needs. Self-neglect is also common, which is when an older person can’t proide their own care. Some warning signs include bruising, lacerations, skin tears, spiral fractures, malnutrition, pressure ulcers, and dehydration. If you even suspect an older adult is being neglected, you should report this immediately to adult protective services (or similar agencies) and treat the medical complications immediately.

Smarty PANCE Content Blueprint Review:

Covered under ⇒ PANCE Blueprint PsychiatryAbuse and NeglectChild/elder abuse

3. Which of the following is the most common bacterial cause of a hordeolum?

A. Streptococcus pyogenes
B. Staphylococcus aureus
C. Clostridium perfringens
D. Propionibacterium species
E. Enterobacter aerogenes

Answer and topic summary

The answer is B. Staphylococcus aureus

A hordeolum (stye) is an abscess of the eyelid that is acute. It will usually present with pain and swelling of the eyelid. The most common pathogen indicated is Staphylococcus aureus. Risk factors include dirty eye makeup, rosacea, and seborrheic dermatitis. A way to differentiate hordeolums from chalazions is that Hordeolums are Hot (aka warm/tender).

Treatment may include warm compresses (10 min up to 5 times daily) plus massaging the eyelid to aid in drainage.

Smarty PANCE Content Blueprint Review:

Covered under ⇒ PANCE Blueprint EENTDisorders of the Eye Lid disordersHordeolum

Also covered as part of the Family Medicine EOR topic list

4. A 19-year-old female presents to the clinic with fatigue, weight loss, arthralgia, and an erythematous rash in a malar distribution over the cheeks and nose (that spares the nasolabial folds). Which of the following is the most common cardiac manifestation of the patient’s likely diagnosis?

A. Third-degree AV block
B. Mitral regurgitation
C. Myocarditis
D. Pericarditis
E. Bradydysrhythmias

Answer and topic summary

The answer is D. Pericarditis

Systemic lupus erythematosus (SLE) is a chronic autoimmune disease that can impact almost any organ. It’s important to be aware of how SLE clinically presents in order to get to a diagnosis and treatment.

The most common clinical features overall are fatigue (80-100%), arthralgia/arthritis (>90%), fever (>50%), and Raynaud phenomenon (~50%). You may also see the classic malar rash that spares nasolabial folds.

Cardiac complications are common (50%) in SLE, and they lead to morbidity and mortality. The most common cardiac manifestation is pericarditis. Other cardiac issues these patients may have include pericardial effusion, myocarditis (rare), and valvular dysfunction.

Smarty PANCE Content Blueprint Review:

Covered under ⇒ PANCE Blueprint MusculoskeletalRheumatologic DisordersSystemic lupus erythematosus

Also covered as part of the Internal Medicine EOR and Family Medicine EOR topic list

5. Which of the following is the most common and important risk factor for aortic dissection?

A. Bicuspid aortic valve
B. Connective tissue disorder
C. Turner’s syndrome
D. Systemic hypertension
E. Aortic surgery

Answer and topic summary

The answer is D. Systemic hypertension

Aortic dissection is a life-threatening emergency and is defined by a tear of the inner layer of the aorta. On an exam, you may be asked about the classifications:

Standford Classification: Stanford A is ascending aorta and Standford B is descending aorta

DeBakey Classifications: type I is Both (ascending + descending aorta), type II is Ascending aorta, and III is Descending aorta. (Hint: It spells out BAD.)

The most important & common risk factor is systemic hypertension. Known causes of acute, severe increases in blood pressure include weight lifting, energy drinks, and cocaine. The classic presentation is “tearing and ripping chest pain with pulse deficits.” Treatment is lowering the BP via IV beta-blockers for both Type A and B and then surgery (for Type A).

Smarty PANCE Content Blueprint Review:

Covered under ⇒ PANCE Blueprint CardiologyVascular DiseaseAortic aneurysm/dissection

6. Which of the following is characterized by a rash that has a “slapped cheek” appearance at first and then progresses into a maculopapular rash on the arms and trunk in a reticular pattern?

A. Roseola infantum
B. Rubella
C. Measles
D. Erythema infectiosum
E. Kawasaki disease

Answer and topic summary

The answer is D. Erythema infectiosum

Erythema infectiosum is a condition characterized by a fever with a rash. It is caused by parvovirus B19 infection. The classic presentation is a “slapped cheek” rash on the face and an erythematous, lacy reticular rash on the trunk and extremities. Some call it the “fifth disease.” It is a common childhood viral exanthem. Diagnosis is usually clinical (serology can be done though). The disease is self-limiting and no specific therapy is warranted (most of the time).

Smarty PANCE Content Blueprint Review:

Covered under ⇒ PANCE Blueprint Infectious DiseaseViral Infectious DiseaseErythema infectiosum

Also covered as part of the Pediatric Rotation EOR topic list

7. Which of the following is the most common cause of generalized musculoskeletal pain in young adult females?

A. Multiple sclerosis
B. Rheumatoid arthritis
C. Polymyalgia rheumatica
D. Menstruation
E. Fibromyalgia

Answer and topic summary

The answer is E. Fibromyalgia

Fibromyalgia is the most common cause of generalized muscle/bone pain in young adult females. Patients with fibromyalgia also usually have fatigue, cognitive fog, sleep disturbances, morning stiffness, headaches, paresthesias, and depression. On a physical exam, you will see tenderness to palpation at multiple sites. Initial treatment includes patient education, aerobic exercise, management of concurrent issues (e.g., sleep and psychiatric), & tricyclic antidepressants (e.g., amitriptyline 10 mg).

Smarty PANCE Content Blueprint Review:

Covered under ⇒ PANCE Blueprint MusculoskeletalRheumatologic DisordersFibromyalgia

Also covered as part of the Internal Medicine EOR and Family Medicine EOR topic list

8. Which of the following is the best first-line treatment for a post-menopausal woman with osteoporosis and no pathological fractures?

A. Bisphosphonates
B. Parathyroid hormone analogs
C. RANKL inhibitors
D. Selective estrogen receptor modulators
E.Calcitonin

Answer and topic summary

The answer is A. Bisphosphonates

Osteoporosis is characterized by low bone mass and skeletal fragility leading to decreased bone strength. There are no clinical symptoms of osteoporosis until a patient has an actual fracture. Vertebral fractures in particular, are the MOST common clinical manifestation of osteoporosis.

Diagnosis is made by a DEXA scan (T-score < -2.5 standard deviations) or a fragility fracture.

Lifestyle changes that can be made include exercise and cessation of smoking. The first-line pharmacological agent is oral bisphosphonates (e.g., alendronate or risedronate).

Remember bisphosphonates should be administered first thing in the AM and >30 min before the first food. Patients should stay upright for >30 minutes.

Smarty PANCE Content Blueprint Review:

Covered under ⇒ PANCE Blueprint MusculoskeletalRheumatologic DisordersOsteoporosis

Also covered as part of the Family Medicine EOR topic list

9. A 72-year-old female with sick sinus syndrome (s/p pacemaker) presents to your clinic with fever and chills. On a physical exam, you appreciate a new murmur located at the left 5th intercostal space at the midclavicular line. You also notice petechiae on the patient’s conjunctivae. Which of the following is the most likely diagnosis?

A. Bacterial pneumonia
B. Pericarditis
C. Rheumatic fever
D. Infective endocarditis
E. Pleural effusion

Answer and topic summary

The answer is D. Infective endocarditis

The patient has infective endocarditis (IE), which is an infection of the endocardial surface of the heart. Risk factors include pre-existing valvular or congenital heart disease, IVDU, indwelling cardiac devices, IV catheters, or recent dental/surgical procedures. The most common symptoms include fever (90%), new murmur (85%), and petechiae (20-50%). Other textbook findings include Janeway lesions (non-tender macules on palms/soles), Osler nodes (tender nodules on pads of fingers/toes), and Roth spots (hemorrhagic lesions of the retina). The best diagnostic test is a transesophageal echocardiogram. The modified Duke criteria can be used as a diagnostic guide but must be interpreted in light of pretest probability. At least 3 sets of blood cultures should be obtained. Treatment includes IV antibiotics and removal of any infected device.

Smarty PANCE Content Blueprint Review:

Covered under ⇒ PANCE Blueprint CardiologyTraumatic, infectious, and inflammatory heart conditionsAcute and subacute bacterial endocarditis

Also covered as part of the Emergency Medicine EOR, Internal Medicine EOR, and Family Medicine EOR topic list

10. Which of the following is not considered diagnostic for diabetes mellitus?

A. Random plasma glucose > 200 mg/dL with symptoms of hyperglycemia
B. A1c > 6.5
C. Fasting plasma glucose > 126 mg/dL on at least two separate occasions
D. Plasma glucose > 200 mg/dL measured after a glucose tolerance test
E. All of the above are diagnostic

Answer and topic summary

The answer is E. All of the above are diagnostic 

Type 2 diabetes mellitus is a very prevalent, serious disease that impacts about 9% of adults in the United States. Risk factors include family history, obesity, sedentary lifestyle, smoking, and poor diet habits. Initial symptoms include polyuria, polydipsia, polyphagia, and blurred vision. Diagnosis is made by one of the following answer choices seen on the first slide. First-line therapies include metformin and lifestyle changes (i.e., weight loss + exercise). Early combination therapy can be considered at treatment initiation to extend the time to treatment failure. It is VERY important to reach tight glycemic control as soon as possible in diabetic patients. Complications of diabetes include microvascular (retinopathy, nephropathy, neuropathy) and macrovascular (stroke, myocardial infarction).

Smarty PANCE Content Blueprint Review:

Covered under ⇒ PANCE Blueprint EndocrinologyDiabetes MellitusDiabetes Mellitus Type 2

Also covered as part of the Family Medicine EOR and Emergency Medicine EOR topic list

This podcast is available on every device!

You can download and listen to past FREE episodes here, on iTunes, Spotify, Google Podcasts, Stitcher, Amazon Music, and all podcasting apps.

Download Interactive Content Blueprint Checklists for the PANCE, PANRE, EOR, and PANRE-LA

Interactive Content Blueprints for the PANCE PANRE and PANRE-LA

Follow this link to download your FREE copy of the PANCE/PANRE/EOR Content Blueprint Checklists.

Print it up and start crossing out the topics you understand, marking the ones you don’t, and making notes of key terms you should remember. The PDF version is interactive and linked directly to the individual lessons on Smarty PANCE.

Smarty PANCE is not sponsored or endorsed by, or affiliated with, the National Commission on Certification of Physician Assistants.

The post Podcast Episode 105: Ten PANCE, PANRE, and Rotation Review Questions appeared first on The Audio PANCE and PANRE.

]]> Listen to Podcast Episode 105: Ten PANCE, PANRE, and Rotation Review Questions If you can’t see the audio player, click here to listen to the full episode. Welcome to episode 105 of the Audio PANCE and PANRE Physician Assistant/Associate Board Review P... If you can’t see the audio player, click here to listen to the full episode.
Welcome to episode 105 of the Audio PANCE and PANRE Physician Assistant/Associate Board Review Podcast.
Join me today as we cover ten board review questions for your PANCE, PANRE, EOR, and EOC exams.
Links from today’s episode:

* Sign up for the Entire Blueprint Email Series
* Follow Smarty PANCE and The Daily PANCE Blueprint on Instagram
* Follow Smarty PANCE and The Daily PANCE Blueprint on Facebook
* Review systemic lupus erythematosus (SLE)
* Review the diabetes diagnostic guidelines
* Review gestational diabetes screening guidelines
* Review basal cell carcinoma and our comparison tables of the Blueprint dermatologic neoplasms
* Join the Smarty PANCE Member’s Community, then sign up for a study group to get updates about upcoming webinars.

I hope you enjoy this free audio component of the examination portion of this site. Smarty PANCE includes over 2,000 interactive board review questions, along with flashcards, ReelDx cases, integrated Picmonics, and lessons covering every blueprint topic available to all Smarty PANCE members.

* You can download and listen to past FREE episodes here, on iTunes, Spotify, Google Podcasts, Stitcher, Amazon Music, and all podcasting apps
* You can listen to all the latest episodes, take interactive quizzes, and download more resources on each episode page.

Interactive exam to complement today’s podcast
1. A 68-year-old male farmer presents with a flesh-colored papule with a rolled border located on the right side of his forehead. As you examine the lesion closely, you notice something else about the lesion. Which of the following physical exam findings would make you more suspicious of malignancy?
A. Telangiectasia
B. Nikolsky sign
C. Hypopigmentation
D. Tenderness to palpation
E.]]>
The Physician Assistant Life | Smarty PANCE full 25:57 573 Podcast Episode 104: Ten PANCE, PANRE, and Rotation Review Questions http://podcast.thepalife.com/episode-104/ Mon, 20 Mar 2023 17:04:50 +0000 https://podcast.thepalife.com/?p=568 http://podcast.thepalife.com/episode-104/#respond http://podcast.thepalife.com/episode-104/feed/ 0 <p>Listen to Podcast Episode 104: Ten PANCE, PANRE, and Rotation Review Questions If you can’t see the audio player, click here to listen to the full episode. Welcome to episode 104 of the Audio PANCE and PANRE Physician Assistant/Associate Board Review Podcast. Join me today as we cover ten board review questions for your PANCE, […]</p> <p>The post <a rel="nofollow" href="http://podcast.thepalife.com/episode-104/">Podcast Episode 104: Ten PANCE, PANRE, and Rotation Review Questions</a> appeared first on <a rel="nofollow" href="http://podcast.thepalife.com">The Audio PANCE and PANRE</a>.</p> Listen to Podcast Episode 104: Ten PANCE, PANRE, and Rotation Review Questions

If you can’t see the audio player, click here to listen to the full episode.

Welcome to episode 104 of the Audio PANCE and PANRE Physician Assistant/Associate Board Review Podcast.

Join me today as we cover ten board review questions for your PANCE, PANRE, EOR, and EOC exams.

Links from today’s episode:

I hope you enjoy this free audio component of the examination portion of this site. Smarty PANCE includes over 2,000 interactive board review questions, along with flashcards, ReelDx cases, integrated Picmonics, and lessons covering every blueprint topic available to all Smarty PANCE members.

Smarty PANCE is not sponsored or endorsed by, or affiliated with, the National Commission on Certification of Physician Assistants.

Interactive exam to complement today’s podcast

When is screening for gestational diabetes done?

A. 16 weeks
B. 22 weeks
C. 24 weeks
D. 32 weeks
E. 34 weeks

Answer and topic summary

The answer is C. 24 weeks

Prenatal care is extremely important. Screening for gestational diabetes is routinely done in pregnant patients at 24 weeks of gestation (typically until 28 weeks). Pregnancy is associated with insulin resistance, mostly because of the placenta’s secretion of human placental lactogen. There are bad consequences of gestational diabetes, so it is critical it is diagnosed and treated adequately.

The initial test is a one-hour 50-gram oral glucose tolerance test (GTT). A positive test >135 mg/dL. If a patient tests positive, they need to undergo the second test, which is a three-hour 100 mg oral GTT. The cut-offs are debated, but generally, the following are positive results: fasting > 95 mg/dL, 1 hour>180 mg/dL, 2 hours>155 mg/dL, 3 hours>140 mg/dL.

Smarty PANCE Content Blueprint Review:

Covered under ⇒ PANCE Blueprint Reproductive System ⇒ Complicated Pregnancy ⇒ Gestational diabetes

Also covered as part of the Women’s Health EOR topic list

2. A 60-year-old male with a history of alcohol abuse and esophageal varices is brought to the ER with lethargy, delirium, weakness, and nausea. He is normotensive and afebrile. On physical exam, he is ill-appearing with jaundice, spider angiomas, a distended abdomen, and 3+ pretibial pitting edema. Based on his history and clinical presentation, which of the following electrolyte abnormalities would you expect to see in this patient?

A. Hyponatremia
B. Hypocalcemia
C. Hypercalcemia
D. Hyperphosphatemia
E. Hypermagnesemia

Answer and topic summary

The answer is A. Hyponatremia

The patient has hypervolemic hyponatremia secondary to cirrhosis. The causes of hypervolemic hyponatremia are cirrhosis, nephrotic syndrome, and CHF. Symptoms include nausea, headache, lethargy, and seizures. It’s important to have an approach to hyponatremia since it is the most common electrolyte abnormality in the hospital.

  • First, it’s important to rule out pseudohyponatremia due to proteins, glucose, or mannitol. Also, make sure it’s not a diuretic causing hyponatremia.
  • Next, consider the volume status – are they hypervolemic, hypovolemic, or euvolemic?
  • Hypovolemic causes are more obvious (emesis, hemorrhage, etc.); however, urinary sodium can help differentiate between hypovolemia and euvolemia. If uNA < 20, then this means the renin-angiotensin-aldosterone system is on and trying to maintain pressure/volume; therefore, it is likely the patient is hypovolemic.

Treatment of hyponatremia depends on the cause. It usually involves fluid restriction and possibly (and carefully) a hypertonic solution. Remember – rapid correction of hyponatremia can lead to central pontine myelinolysis.

Smarty PANCE Content Blueprint Review:

Covered under ⇒ PANCE Blueprint Renal System ⇒ Fluid and Electrolyte Disorders ⇒ Hyponatremia

Also covered as part of the Internal Medicine EOR topic list

3. A 12-year-old male patient presents with a circular, expanding rash located where a tick had bitten him. He had recently been camping in the upper Midwest. He also complains of a headache and mild fever. Which of the following is a known cardiac complication of the patient’s most likely diagnosis?

A. Supraventricular tachycardia
B. Atrioventricular block
C. Wandering atrial pacemaker
D. Sinus bradycardia
E. None of the above

Answer and topic summary

The answer is B. Atrioventricular block

The patient most likely has Lyme disease, which is a tick-borne illness usually caused by Borrelia burgdorferi. It is more prevalent in the Northeast and upper Midwest. Erythema migrans, which is a circular, expanding rash that may look like a “bull’s eye” is seen about 20% of the time with Lyme disease. Clinical features in the early localized disease include fever, headaches, malaise, arthralgia, and lymphadenopathy. Treatment is doxycycline for non-pregnant adults and children. Amoxicillin is a second-line treatment. A well-known cardiac complication is an atrioventricular block.

Smarty PANCE Content Blueprint Review:

Covered under ⇒ PANCE Blueprint Infectious Disease ⇒ Spirochetal Disease ⇒ Lyme disease

Also covered as part of the Internal Medicine EOR and Family Medicine EOR topic list

4. Which of the following is not a manifestation of congenital rubella syndrome?

A. Deafness
B. Cataracts
C. Heart defects
D. Microcephaly
E. Fetal growth acceleration

Answer and topic summary

The answer is E. Fetal growth acceleration

Clinical features of a congenital rubella infection (CRI) include deafness, cataracts, and cardiac disease. Patients may also have CNS abnormalities, such as microcephaly. Typically, a congenital rubella infection will lead to fetal growth restriction, not fetal growth acceleration. A classic finding that you may see on exams is petechiae and purpura — some call these “blueberry muffin lesions.” In general, CRI should be considered in patients who have a suspected rubella infection during their pregnancy. Laboratory studies on the child (before the age of one) are done to help confirm the diagnosis. There is no treatment, and supportive care/surveillance is the main way to manage a congenital rubella infection.

Smarty PANCE Content Blueprint Review:

Covered under ⇒ PANCE Blueprint Infectious Disease ⇒ Viral Infectious Disease ⇒ Rubella (German Measles)

Also covered as part of the Pediatric EOR topic list

5. Which of the following is the most common cause of cardiogenic shock?

A. Right ventricle failure
B. Tachydysrhythmias
C. Myocardial infarction
D. Coronary vasospasms
E. Papillary muscle rupture

Answer and topic summary

The answer is C. Myocardial Infarction

Cardiogenic shock is a type of shock that occurs due to cardiac pump failure; essentially, it is the most severe expression of heart failure. It is defined by systemic tissue hypoperfusion secondary to poor cardiac output despite adequate intravascular volume. The most common cause of cardiogenic shock is a myocardial infarction. Other causes include atrial/ventricular tachyarrhythmias, aortic or mitral valve insufficiency, septal defects, free wall rupture, etc. Therapy options vary but may include inotropic support (e.g., dobutamine), intra-aortic balloon pump, and percutaneous ventricular assist devices.

Smarty PANCE Content Blueprint Review:

Covered under ⇒ PANCE Blueprint Cardiology ⇒ Hypotension ⇒ Cardiogenic shock

Also covered as part of the Emergency Medicine EOR topic list

6. A 20-year-old female with asthma presents to the clinic with “white stuff” in her mouth for the past few days. On physical exam, you notice a few white plaques on the buccal mucosa and tongue. The lesions are easily scraped off with a tongue depressor. Which of the following is the best initial treatment of choice for the most likely diagnosis?

A. Fluconazole
B. Amphotericin B
C. Lidocaine solution
D. Nystatin
E. Bactrim

Answer and topic summary

The answer is D. Nystatin

The patient has oropharyngeal candidiasis, which is a local infection in patients with certain risk factors (e.g., AIDs, dentures, radiation, inhaled glucocorticoids, etc.). This patient has asthma so the likely causative culprit is her inhaled glucocorticoids. The most common cause of oropharyngeal candidiasis is C. albicans. On a physical exam, you will see white plaques in the mouth (which can be scrapped). In smokers, the lesions may look more yellow/brown.

Diagnosis is usually clinical but can be confirmed by scraping the lesion. A KOH test can be performed on the scrapings — budding yeast with pseudohyphae are seen. The treatment of choice is nystatin swish and swallow or topical clotrimazole. If a patient has a severe disease or fails topical therapy, oral fluconazole can be given.

Smarty PANCE Content Blueprint Review:

Covered under ⇒ PANCE Blueprint Dermatology ⇒ Fungal Infections ⇒ Candidiasis

Also covered as part of the Internal Medicine EOR and Pediatric Rotation EOR topic list

7. A 51-year-old male presents to the clinic. His wife explains his symptoms: brief, involuntary movements of his limbs, depression, impaired thinking, and weight loss. Which of the following is the most likely diagnosis?

A. Parkinson disease
B. Multiple sclerosis
C. Alzheimer disease
D. Vascular dementia
E. Huntington’s disease

Answer and topic summary

The answer is E. Huntington’s disease

Huntington’s disease is an autosomal-dominant disorder characterized by progressive neurodegeneration. It is due to the expansion of the cytosine-adenine-guanine (CAG) trinucleotide in the HTT gene. The main clinical features include choreiform movementsdementiacognitive impairment, and psychiatric problems. Patients typically present during mid-life. Diagnosis is confirmed by genetic testing. A brain MRI may reveal caudate atrophy. Treatment is generally supportive and symptomatic. There is no cure for Huntington’s disease.

Smarty PANCE Content Blueprint Review:

Covered under ⇒ PANCE Blueprint Neurology ⇒ Movement Disorders ⇒ Huntington Disease

Also covered as part of the Internal Medicine EOR topic list

8. A 42-year-old male with a history of diabetes presents the ER with a “bad rash” on his left thigh. He said he had previously gotten a bug bite there. He is febrile (102.2F), tachycardic (110 bpm), and hypotensive (80/50 mmHg). On physical exam, you notice a 2-inch erythematous circular lesion that is warm to the touch. Which of the following is the best next step in the management of this patient?

A. Initiate IV antibiotics and fluids
B. Surgical consult for amputation
C. Outpatient dermatology referral
D. Order venous ultrasound of the leg
E. Send home on oral antibiotics

Answer and topic summary

The answer is A. Initiate IV antibiotics and fluids

This patient has a severe case of cellulitis and sepsis. The best next step is to initiate intravenous antibiotics and fluids. A broad-spectrum antibiotic (like cefepime) plus coverage for MRSA (vancomycin) would be appropriate.

The most common cause of cellulitis is Group A beta-hemolytic Streptococcus pyogenes. What differentiates cellulitis from erysipelas is that erysipelas is more well-defined. Antibiotic options for more mild cellulitis cases include Bactrimclindamycin, and Keflex.

It’s important to keep in mind that events don’t happen in a vacuum — the bug bite led to cellulitis, and the patient’s body responded to the cutaneous infection in a complex and dysregulated way (i.e. sepsis), eventually leading to hypotension and instability.

Smarty PANCE Content Blueprint Review:

Covered under ⇒ PANCE Blueprint Dermatology ⇒ Dermatologic Infectious diseases ⇒ Bacterial Infections ⇒ Cellulitis

Also covered as part of the Family Medicine EOREmergency Medicine EOR, and General Surgery EOR topic list

9. An 8-year-old male is brought into the clinic by his mother, who is concerned about his behavior in school and at home for the past 7 months. In both settings, he lacks the ability to pay attention, does not follow directions or listen, is easily distracted, is reluctant to do tasks, and constantly loses his homework and pencils. Which of the following is a common side effect of the first-line treatment for this condition?

A. Cognitive fog
B. Appetite suppression
C. Constipation
D. Somnolence
E. Xerosis

Answer and topic summary

The answer is B. Appetite suppression

The patient has attention-deficit hyperactivity disorder (ADHD). There are two main patterns: inattentive or hyperactive-impulse (some may have a combined presentation). The DSM V has criteria for diagnosing ADHD and the various subtypes, but in general, symptoms should be seen in multiple settings, last for 6 months or more, and negatively impact social/academic activities. The first-line pharmacological treatment for ADHD, in most cases, is a stimulant (e.g., methylphenidate, dextroamphetamine, etc.). The most adverse effects of stimulants include moodiness and irritability, and appetite suppression. It’s important to monitor children’s growth and weight when they are taking a stimulant for this very reason.

Smarty PANCE Content Blueprint Review:

Covered under ⇒ PANCE Blueprint Psychiatry ⇒ Neurodevelopmental disorders ⇒ Attention-Deficit/Hyperactivity Disorder

Also covered as part of the Pediatric Rotation and Psychiatry EOR topic list

10. Which of the following patients should be screened for lung cancer, according to the USPSTF?

A. 82-year-old with 34 pack-years who quit 2 years ago
B. 51-year-old with 31 pack-years who quit 10 years ago
C. 70-year-old with 14 pack-years who quit 5 years ago
D. 40-year-old with 20 pack-years who quit 2 weeks ago

Answer and topic summary

The answer is B. 51-year-old with 31 pack-years who quit 10 years ago

The U.S. Preventive Services Task Force (USPSTF) recently changed the lung cancer screening guidelines in March 2021.

An annual low-dose chest CT scan (NOT chest x-ray) should be done for those who are 50 to 80 years old with at least a 20-pack-year history and who currently smoke or have quit within the past 15 years.

It is also important to note that these guidelines do not apply to those who have another illness that already severely limits their life expectancy.

Smarty PANCE Content Blueprint Review:

Covered under ⇒ PANCE Blueprint Pulmonary ⇒ Pulmonary Neoplasms ⇒ Lung cancer

Also covered as part of the Family Medicine EOR and Emergency Medicine EOR topic list

This podcast is available on every device!

You can download and listen to past FREE episodes here, on iTunes, Spotify, Google Podcasts, Stitcher, Amazon Music, and all podcasting apps.

Download Interactive Content Blueprint Checklists for the PANCE, PANRE, EOR, and PANRE-LA

Interactive Content Blueprints for the PANCE PANRE and PANRE-LA

Follow this link to download your FREE copy of the PANCE/PANRE/EOR Content Blueprint Checklists.

Print it up and start crossing out the topics you understand, marking the ones you don’t, and making notes of key terms you should remember. The PDF version is interactive and linked directly to the individual lessons on Smarty PANCE.

The post Podcast Episode 104: Ten PANCE, PANRE, and Rotation Review Questions appeared first on The Audio PANCE and PANRE.

]]>
Listen to Podcast Episode 104: Ten PANCE, PANRE, and Rotation Review Questions If you can’t see the audio player, click here to listen to the full episode. Welcome to episode 104 of the Audio PANCE and PANRE Physician Assistant/Associate Board Review P... If you can’t see the audio player, click here to listen to the full episode.
Welcome to episode 104 of the Audio PANCE and PANRE Physician Assistant/Associate Board Review Podcast.
Join me today as we cover ten board review questions for your PANCE, PANRE, EOR, and EOC exams.
Links from today’s episode:

* Sign up for the Entire Blueprint Email Series
* Follow Smarty PANCE and The Daily PANCE Blueprint on Instagram
* Follow Smarty PANCE and The Daily PANCE Blueprint on Facebook
* Join the Smarty PANCE Member’s Community, then sign up for a study group to get updates about upcoming webinars.

I hope you enjoy this free audio component of the examination portion of this site. Smarty PANCE includes over 2,000 interactive board review questions, along with flashcards, ReelDx cases, integrated Picmonics, and lessons covering every blueprint topic available to all Smarty PANCE members.

* You can download and listen to past FREE episodes here, on iTunes, Spotify, Google Podcasts, Stitcher, Amazon Music, and all podcasting apps
* You can listen to all the latest episodes, take interactive quizzes, and download more resources on each episode page.

Smarty PANCE is not sponsored or endorsed by, or affiliated with, the National Commission on Certification of Physician Assistants.
Interactive exam to complement today’s podcast
When is screening for gestational diabetes done?
A. 16 weeks
B. 22 weeks
C. 24 weeks
D. 32 weeks
E. 34 weeks
2. A 60-year-old male with a history of alcohol abuse and esophageal varices is brought to the ER with lethargy, delirium, weakness, and nausea. He is normotensive and afebrile. On physical exam, he is ill-appearing with jaundice, spider angiomas, a distended abdomen, and 3+ pretibial pitting edema. Based on his history and clinical presentation, which of the following electrolyte abnormalities would you expect to see in this patient?
A. Hyponatremia
B. Hypocalcemia
C. Hypercalcemia
D. Hyperphosphatemia
E. Hypermagnesemia
3. A 12-year-old male patient presents with a circular, expanding rash located where a tick had bitten him. He had recently been camping in the upper Midwest. He also complains of a headache and mild fever. Which of the following is a known cardiac complication of the patient’s most likely diagnosis?
A.]]>
The Physician Assistant Life | Smarty PANCE full 24:25 568
Podcast Episode 103: Ten PANRE & PANRE-LA Intervention Complex Practice Questions http://podcast.thepalife.com/podcast-episode-103/ Wed, 08 Mar 2023 12:00:42 +0000 https://podcast.thepalife.com/?p=562 http://podcast.thepalife.com/podcast-episode-103/#respond http://podcast.thepalife.com/podcast-episode-103/feed/ 0 <p>Listen to Podcast Episode 103: Ten PANRE & PANRE-LA Intervention Complex Practice Question If you can’t see the audio player, click here to listen to the full episode. Welcome to episode 103 of the Audio PANCE and PANRE Physician Assistant/Associate Board Review Podcast. Join me today as we cover ten NCCPA-style board review questions for […]</p> <p>The post <a rel="nofollow" href="http://podcast.thepalife.com/podcast-episode-103/">Podcast Episode 103: Ten PANRE & PANRE-LA Intervention Complex Practice Questions</a> appeared first on <a rel="nofollow" href="http://podcast.thepalife.com">The Audio PANCE and PANRE</a>.</p> Listen to Podcast Episode 103: Ten PANRE & PANRE-LA Intervention Complex Practice Question

If you can’t see the audio player, click here to listen to the full episode.

Podcast Episode 103 Ten PANRE & PANRE-LA Intervention Complex Practice QuestionsWelcome to episode 103 of the Audio PANCE and PANRE Physician Assistant/Associate Board Review Podcast.

Join me today as we cover ten NCCPA-style board review questions for your PANRE and PANRE-LA exams.

Special from today’s episode:

I hope you enjoy this free audio component of the examination portion of this site. Smarty PANCE includes over 2,000 interactive board review questions, along with flashcards, ReelDx cases, integrated Picmonics, and lessons covering every blueprint topic available to all Smarty PANCE members.

Here is an interactive exam to complement today’s podcast

1. A 65-year-old man presents to your office with complaints of constipation for the past six months. He says that he has difficulty passing stools, which are hard and dry. He also reports occasional abdominal pain and bloating. He denies any weight loss, blood in stools, fever, or night sweats. His medical history is significant for hypertension and type 2 diabetes mellitus. His medications include metformin, lisinopril, and aspirin. He does not smoke or drink alcohol. On physical examination, his vital signs are normal. His abdomen is soft and nontender, with normal bowel sounds. There are no masses or organomegaly palpable. Which of the following is the most appropriate next step in evaluating this patient?

A) Colonoscopy
B) Barium enema
C) Thyroid function tests
D) Stool osmolarity
E) Dietary modification

Answer and topic summary

The correct answer is A) Colonoscopy

Colonoscopy is a procedure that involves inserting a flexible tube with a camera into the colon to visualize the mucosa and detect any abnormalities such as polyps, tumors, inflammation, or bleeding. It is indicated for patients with chronic constipation who are older than 50 years or have any red flag features for colorectal malignancy, such as weight loss, blood in stools, anemia, or a family history of colon cancer. This patient meets the age criterion and should undergo colonoscopy to rule out any serious causes of his constipation.

Answer explanations:

  • Barium enema is an imaging test that involves injecting a contrast agent (barium sulfate) into the rectum and taking X-rays of the colon. It can show structural abnormalities such as diverticula, strictures, masses, or volvulus. However, it is less sensitive and specific than colonoscopy for detecting colorectal malignancy. Therefore, it is not the preferred test for this patient.
  • Thyroid function tests are blood tests that measure the levels of thyroid hormones (T3, T4) and thyroid-stimulating hormone (TSH). They can help diagnose thyroid disorders such as hypothyroidism or hyperthyroidism. Hypothyroidism can cause constipation due to decreased gastrointestinal motility. However, this patient has no other signs or symptoms of hypothyroidism such as fatigue, cold intolerance, dry skin, hair loss, or bradycardia. Therefore, thyroid function tests are not necessary for this patient.
  • Stool osmolarity is a test that measures the concentration of solutes in stool water. It can help differentiate between osmotic diarrhea (high stool osmolarity) and secretory diarrhea (low stool osmolarity). However, this test is not useful for evaluating constipation which is defined by infrequent or difficult defecation.
  • Dietary modification is a nonpharmacological measure that involves increasing fiber and fluid intake to improve stool consistency and frequency. It may be effective for patients with primary constipation which has no identifiable organic cause. However, this patient has chronic constipation which requires further evaluation before initiating treatment.

(Review PANRE Blueprint Topic: Constipation)

2. A 60-year-old male presents to the emergency department with chest pain and shortness of breath. ECG reveals ST segment elevation in the anterior leads. Which of the following laboratory tests is the most specific for the diagnosis of acute myocardial infarction (AMI)?

A) Troponin
B) Creatinine kinase-MB (CK-MB)
C) Myoglobin
D) C-reactive protein (CRP)
E) Brain natriuretic peptide (BNP)

Answer and topic summary

The correct answer is A) Troponin

Acute myocardial infarction (AMI) is a serious medical emergency that requires prompt diagnosis and treatment. Cardiac biomarkers such as troponin, creatinine kinase-MB (CK-MB), and myoglobin are commonly used to diagnose AMI. Among these biomarkers, troponin is the most specific for the diagnosis of AMI. Troponin is a protein found in cardiac muscle cells, and its release into the bloodstream is a specific marker of myocardial injury. Elevated troponin levels are typically seen within 3-4 hours after the onset of symptoms and can remain elevated for up to 10-14 days after an AMI.

Incorrect answer explanations:

  • Creatinine kinase (CK) is an enzyme found in various tissues, including skeletal and cardiac muscle. CK-MB is a specific isoform of CK found predominantly in cardiac muscle cells. Elevated levels of CK-MB can be seen in the early stages of AMI, but CK-MB is less specific for the diagnosis of AMI compared to troponin.
  • Myoglobin is a protein found in skeletal and cardiac muscle cells. Elevated myoglobin levels can be seen within 1-3 hours after the onset of symptoms, but myoglobin is less specific for the diagnosis of AMI compared to troponin. Elevated myoglobin levels can also be seen in other conditions that cause muscle injury, such as rhabdomyolysis.
  • C-reactive protein (CRP) is an acute-phase protein that is elevated in response to tissue injury, inflammation, and infection. While elevated CRP levels can be seen in patients with AMI, CRP is not specific for the diagnosis of AMI and cannot be used as a diagnostic tool on its own.
  • Brain natriuretic peptide (BNP) is a hormone released by the heart in response to increased pressure and volume in the cardiac chambers. Elevated BNP levels can be seen in patients with heart failure and other cardiac conditions, but BNP is not specific for the diagnosis of AMI.

(Review PANRE Blueprint Topic: Acute myocardial infarction: ST-segment elevation myocardial infarction (STEMI))

3. A 65-year-old woman with a history of atrial fibrillation, hypertension, and obesity presents to the emergency department with acute onset of severe left leg pain and swelling. She has been taking warfarin for anticoagulation but admits to missing some doses in the past week. Her blood pressure is 180/100 mmHg, pulse is 110 beats/min and irregular, and respiratory rate is 20 breaths/min. Her physical examination reveals a warm, tender, erythematous left lower extremity with prominent superficial veins and a positive Homan’s sign. Her international normalized ratio (INR) is 1.5. What is the most appropriate next step in the management of this patient?

A) Start heparin infusion and adjust warfarin dose
B) Order duplex ultrasonography of the lower extremities
C) Administer tissue plasminogen activator (tPA)
D) Perform venography of the left leg
E) Apply compression stockings and elevate the leg

Answer and topic summary

The correct answer is B) Order duplex ultrasonography of the lower extremities

Duplex ultrasonography is a non-invasive test that uses sound waves to visualize the blood flow in the veins of the legs. It can detect thrombi by measuring the diameter, compressibility, and flow characteristics of the veins. It has high sensitivity and specificity for diagnosing DVT. It is considered the first-line diagnostic test for patients with suspected DVT. If negative, it can be repeated in one week or combined with D-dimer testing to rule out DVT.

Incorrect answer explanations:

  • Heparin infusion and warfarin dose adjustment are indicated for patients with confirmed deep vein thrombosis (DVT) who have subtherapeutic INR levels. However, this patient has not yet been diagnosed with DVT and needs further diagnostic testing before initiating anticoagulation therapy.
  • tPA is a thrombolytic agent that dissolves blood clots by activating plasminogen into plasmin. Plasmin degrades fibrin, which forms the meshwork of clots. tPA may be used for patients with massive or life-threatening DVT who have contraindications to anticoagulation or who have failed anticoagulation therapy. However, it carries a high risk of bleeding complications and should not be used without confirming DVT diagnosis first.
  • Venography is an invasive procedure that involves injecting contrast dye into a vein and taking X-ray images to visualize any obstruction or narrowing in the venous system. It was once considered the gold standard for diagnosing DVT but has been largely replaced by duplex ultrasonography due to its higher cost, invasiveness, discomfort, and risk of allergic reactions or nephrotoxicity from contrast dye.
  • Compression stockings are elastic garments that apply graduated pressure on the legs to improve venous return and prevent edema formation. They may be used as an adjunctive treatment for patients with confirmed DVT, along with anticoagulation therapy. Elevation of the leg can also reduce swelling and pain by decreasing hydrostatic pressure in the veins. However, these interventions do not address the underlying cause of DVT or prevent clot propagation or embolization. They should not be used as the sole therapy for suspected or confirmed DVT without diagnostic testing or anticoagulation therapy.

(Review PANRE Blueprint Topic: Deep venous thrombosis)

4. A 7-year-old child presents with honey-colored crusting lesions on the face and extremities. The lesions started as small red papules that quickly progressed to vesicles and then ruptured, leaving behind a honey-colored crust. The child has no fever and is otherwise healthy. What is the most appropriate initial treatment for this condition?

A) Topical corticosteroids
B) Oral antihistamines
C) Oral doxycycline
D) Topical mupirocin
E) Oral acyclovir

Answer and topic summary

The correct answer is D) Topical mupirocin

The presentation of honey-colored crusting lesions on the face and extremities in a child is consistent with impetigo, a bacterial skin infection caused by Staphylococcus aureus or Streptococcus pyogenes. The most appropriate initial treatment for impetigo is topical antibiotics such as mupirocin. This is because impetigo is a localized skin infection, and topical antibiotics can effectively treat it without the need for systemic antibiotics. The topical antibiotic is applied to the affected area 2-3 times per day for 5-7 days.

Incorrect answer explanations:

  • Topical corticosteroids are not appropriate for impetigo as they can exacerbate the infection by suppressing the immune response and promoting bacterial growth.
  • Oral antihistamines are used to treat allergic reactions and are not effective in treating bacterial skin infections such as impetigo.
  • Oral doxycycline can be used to treat impetigo, but it is not the initial treatment of choice. Oral antibiotics are reserved for cases of extensive or severe impetigo or for patients who cannot tolerate topical antibiotics.
  • Oral acyclovir is an antiviral medication used to treat viral skin infections such as herpes simplex virus (HSV). It is not effective in treating bacterial skin infections such as impetigo.

(Review PANRE Blueprint Topic: Impetigo)

5. A 72-year-old woman with a history of hypertension, diabetes mellitus, and atrial fibrillation presents to the emergency department with sudden onset of left-sided weakness and slurred speech. She was last seen normal 2 hours ago by her daughter. Her vital signs are: blood pressure 180/100 mmHg, pulse 110 beats/min irregularly irregular, respiratory rate 18 breaths/min, temperature 36.5°C (97.7°F), and oxygen saturation 98% on room air. On physical examination, she has left facial droop, left hemiparesis (4/5 strength), and dysarthria. Her NIH Stroke Scale score is 8. A non-contrast head CT scan shows no evidence of hemorrhage. What is the most appropriate next step in management?

A) Administer intravenous alteplase
B) Administer intravenous heparin
C) Perform carotid endarterectomy
D) Perform mechanical thrombectomy
E) Start oral aspirin

Answer and topic summary

The correct answer is A. Administer intravenous alteplase

The patient has a clinical diagnosis of acute ischemic stroke, which is caused by occlusion of a cerebral artery by a thrombus or embolus. The most important factor in determining the treatment of acute ischemic stroke is the time from symptom onset to presentation. If the patient presents within 4.5 hours of symptom onset and has no contraindications, intravenous alteplase (a tissue plasminogen activator or tPA) should be administered as soon as possible to dissolve the clot and restore blood flow to the ischemic brain tissue. Intravenous alteplase has been shown to improve functional outcomes and reduce disability after acute ischemic stroke. The patient meets the criteria for intravenous alteplase because she presented within 4.5 hours of symptom onset, has no evidence of hemorrhage on head CT scan, and has no other contraindications such as recent surgery, active bleeding, severe hypertension (>185/110 mmHg), or use of anticoagulants. The other answer choices are incorrect because they are not indicated or effective in this scenario.

Incorrect answer explanations:

  • Intravenous heparin administration is not recommended for acute ischemic stroke because it does not improve outcomes and may increase the risk of bleeding complications.
  • Carotid endarterectomy is a surgical procedure that removes plaque from the carotid artery to prevent recurrent strokes in patients with significant carotid stenosis (>70%) who are asymptomatic or have had a transient ischemic attack (TIA) or minor stroke within the past 6 months. It is not indicated for patients with acute ischemic stroke who have not been stabilized medically.
  • Mechanical thrombectomy is an endovascular procedure that uses a catheter-based device to remove large vessel occlusions in patients with acute ischemic stroke who have a large penumbra (area of potentially salvageable brain tissue). It can be performed within 24 hours of symptom onset if intravenous alteplase is contraindicated or ineffective. However, it should not delay or replace intravenous alteplase if eligible.
  • Oral aspirin is recommended for secondary prevention of ischemic stroke after initial treatment with intravenous alteplase or mechanical thrombectomy. It can also be used as initial treatment for patients who present more than 24 hours after symptom onset or who have contraindications to intravenous alteplase. However, it should not be given within 24 hours of receiving intravenous alteplase because it may increase the risk of bleeding complications.

(Review PANRE Blueprint Topic: Stroke)

[dt_sc_button type=”type2″ link=”https://smartypance.com/lessons/vascular-disorders/stroke-reeldx266/” size=”small” target=”_blank” timeline_button=”no”]View blueprint lesson[/dt_sc_button]

6. A 65-year-old man with a history of hypertension and peptic ulcer disease presents to your clinic for a routine follow-up. He reports feeling well and has no complaints. His medications include lisinopril and omeprazole. His vital signs are normal. A complete blood count (CBC) shows:

  • Hemoglobin: 10 g/dL (normal: 13-17 g/dL)
  • Hematocrit: 30% (normal: 40-50%)
  • Mean corpuscular volume (MCV): 70 fL (normal: 80-100 fL)
  • Red cell distribution width (RDW): 18% (normal: 11-15%)
  • White blood cell count: 6 x 10^9/L (normal: 4-11 x 10^9/L)
  • Platelet count: 250 x 10^9/L (normal: 150-450 x 10^9/L)

What is the most likely cause of this patient’s anemia?

A) Chronic kidney disease
B) Folate deficiency
C) Gastrointestinal bleeding
D) Thalassemia trait
E) Vitamin B12 deficiency

Answer and topic summary

The answer is C. Gastrointestinal bleeding

The patient has a microcytic anemia, which is characterized by a low MCV (<80 fL). The most common cause of microcytic anemia is iron deficiency. Iron deficiency can result from inadequate dietary intake, malabsorption, increased demand, or chronic blood loss. In this patient, the most likely source of chronic blood loss is gastrointestinal bleeding due to his history of peptic ulcer disease and the use of omeprazole. Omeprazole can mask the symptoms of gastrointestinal bleeding by reducing acid secretion and healing ulcers but does not prevent recurrence or complications. The patient may also have occult blood loss that is not visible in the stool. The RDW is elevated (>15%), indicating increased variation in red blood cell size due to iron deficiency. To confirm iron deficiency, iron studies such as serum ferritin, serum iron, total iron-binding capacity (TIBC), and transferrin saturation should be ordered. The patient should also undergo endoscopy to evaluate for the source and severity of gastrointestinal bleeding and rule out malignancy. The other answer choices are incorrect because they are not causes of microcytic anemia but rather causes of normocytic or macrocytic anemia.

Incorrect answer explanations:

  • Chronic kidney disease can cause normocytic anemia due to reduced production of erythropoietin by the kidneys. The MCV would be normal (80-100 fL).
  • Folate deficiency can cause macrocytic anemia due to impaired DNA synthesis in red blood cell precursors. The MCV would be high (>100 fL).
  • Thalassemia trait can cause microcytic anemia due to reduced synthesis of alpha or beta globin chains that form hemoglobin. However, thalassemia trait usually has a normal or low RDW (<15%) because red blood cells are uniformly small. Thalassemia trait also has a genetic basis and is more common in people of Mediterranean, African, or Southeast Asian descent.
  • Vitamin B12 deficiency can also cause macrocytic anemia due to impaired DNA synthesis in red blood cell precursors as well as neurological symptoms such as peripheral neuropathy, ataxia, dementia, or psychosis. The MCV would be high (>100 fL).

(Review PANRE Blueprint Topic: Iron Deficiency Anemia)

7. A 42-year-old woman with a history of GERD presents to your clinic for follow-up. She has been taking omeprazole 20 mg daily for the past 6 months and reports significant improvement in her heartburn and regurgitation symptoms. She has also made lifestyle modifications such as avoiding spicy and fatty foods, quitting smoking, and elevating the head of her bed. She asks you if she can stop taking omeprazole or reduce the dose. What is the most appropriate next step in management?

A) Continue omeprazole 20 mg daily indefinitely
B) Discontinue omeprazole and monitor symptoms
C) Switch to famotidine 20 mg twice daily
D) Taper omeprazole to every other day for 4 weeks
E) Perform an upper endoscopy

Answer and topic summary

The correct answer is D) Taper omeprazole to every other day for 4 weeks

Explanation:

The patient has a history of GERD that has responded well to PPI therapy and lifestyle modifications. The goal of treatment is to achieve symptom relief and prevent complications such as esophagitis, stricture, Barrett’s esophagus, or adenocarcinoma. PPIs are more effective than H2 blockers or antacids for healing erosive esophagitis and maintaining remission. However, long-term use of PPIs may be associated with adverse effects such as increased risk of fractures, infections, hypomagnesemia, vitamin B12 deficiency, and chronic kidney disease. Therefore, it is reasonable to attempt a step-down approach after achieving symptom control with PPIs for at least 8 weeks. This involves tapering the dose of PPI gradually over 2-4 weeks to avoid rebound acid hypersecretion. If symptoms recur after discontinuation of PPIs, then switching to an H2 blocker or restarting PPI maintenance therapy with the lowest effective dose may be indicated.

Incorrect answer choices:

  • Continuing omeprazole 20 mg daily may expose the patient to unnecessary risks of long-term PPI use without attempting a trial of dose reduction or discontinuation.
  • Discontinuing omeprazole and monitoring symptoms may cause rebound acid hypersecretion and recurrence of GERD symptoms due to abrupt withdrawal of PPI therapy.
  • Immediately switching to an H2 blocker (such as famotidine) may not provide adequate symptom control for GERD, as H2 blockers are less potent than PPIs in suppressing gastric acid secretion. Slowly taper off the PPI first over 2-4 weeks (the higher the dose, the longer the taper). If symptoms return, it would be appropriate to start again with an H2 blocker.  If long-term treatment is needed, H2 blockers allow better absorption of nutrients than PPIs and so potentially have fewer long-term adverse effects. If symptoms are still difficult to control, consider adding the PPI back at the lowest effective dose.
  • Perform an upper endoscopy: This option is not indicated for patients with uncomplicated GERD who have responded well to medical therapy. Endoscopy is reserved for patients who have alarm features such as dysphagia, odynophagia, weight loss, anemia,

(Review PANRE Blueprint Topic: GERD)

8. Which of the following is an appropriate step-down therapy for a patient with well-controlled asthma on medium-dose inhaled corticosteroids (ICS) and long-acting beta agonists (LABA)?

A) Discontinue LABA and continue medium-dose ICS
B) Discontinue ICS and continue LABA
C) Reduce ICS dose by 50% and continue LABA
D) Reduce both ICS and LABA doses by 50%
E) Switch to low-dose ICS/formoterol as needed

Answer and topic summary

The answer is E. Switch to low-dose ICS/formoterol as needed

This patient has well-controlled asthma on medium-dose ICS and LABA, which corresponds to step 4 of the asthma treatment algorithm. Step-down therapy can be considered for patients with at least 3 months of continuous good control of asthma. The goal of step-down therapy is to reduce medication use to the lowest effective dose while maintaining asthma control. According to the Global Initiative for Asthma (GINA), one option for stepping down from step 4 is to switch to low-dose ICS/formoterol as needed. This regimen involves using a combination inhaler containing low-dose ICS (budesonide) and formoterol (a fast-acting LABA) both as maintenance therapy and as reliever therapy instead of a short-acting beta agonist (SABA). This strategy has been shown to reduce exacerbations, improve symptom control, and decrease steroid exposure compared with conventional maintenance therapy with higher doses of ICS/LABA plus SABA as needed.

(Review PANRE Blueprint Topic: Asthma)

9. A 35-year-old woman presents to her primary care provider with complaints of chronic worry and nervousness for the past 8 months. She says she worries about everything, such as her health, her family, her work, and her finances. She has difficulty sleeping, concentrating, and relaxing. She also experiences muscle tension, headaches, and palpitations. She denies any history of trauma, substance abuse, or other psychiatric disorders. She has no medical problems and takes no medications. Her vital signs are normal and her physical examination is unremarkable. Which of the following is the most likely diagnosis?

A) Panic disorder
B) Obsessive-compulsive disorder
C) Post-traumatic stress disorder
D) Generalized anxiety disorder
E) Adjustment disorder

Answer and topic summary

The correct answer is D) Generalized anxiety disorder

Generalized anxiety disorder (GAD) is characterized by persistent, excessive, and unrealistic worry about everyday things that lasts for at least 6 months. The worry causes significant distress or impairment in social, occupational, or other areas of functioning. The anxiety is not attributable to any specific triggers or stressors. The diagnosis of GAD requires at least 3 of the following symptoms: restlessness, fatigue, difficulty concentrating, irritability, muscle tension, and sleep disturbance.

  • Panic disorder is characterized by recurrent unexpected panic attacks that cause fear of having another attack or avoidance of situations that might trigger an attack. Panic attacks are sudden episodes of intense fear or discomfort that peak within minutes and are accompanied by at least 4 physical or cognitive symptoms such as palpitations, sweating, trembling, shortness of breath, chest pain, nausea, dizziness, derealization/depersonalization, fear of losing control or dying. The patient does not report having panic attacks.
  • Obsessive-compulsive disorder (OCD) is characterized by recurrent obsessions (intrusive thoughts or images that cause anxiety) and/or compulsions (repetitive behaviors or mental acts that aim to reduce anxiety). The patient does not report having obsessions or compulsions.
  • Post-traumatic stress disorder (PTSD) is characterized by exposure to a traumatic event that involved actual or threatened death, serious injury, or sexual violence, and subsequent re-experiencing, avoidance, negative alterations in cognition and mood, and increased arousal related to the event. The patient denies any history of trauma.
  • Adjustment disorder is characterized by emotional or behavioral symptoms that develop within 3 months of an identifiable psychosocial stressor and cause significant impairment in social, occupational, or other areas of functioning. The symptoms usually resolve within 6 months after the termination of the stressor unless it has chronic consequences. The patient’s anxiety is not related to any specific stressors and has lasted longer than 6 months.

(Review PANRE Blueprint Topic: Generalized Anxiety Disorder)

10. A 65-year-old man with a history of type 2 diabetes mellitus presents to the emergency department with altered mental status, polyuria, and polydipsia. He has been feeling unwell for the past week with a urinary tract infection that he has been self-treating with cranberry juice. His vital signs are: blood pressure 180/100 mmHg, heart rate 110 beats per minute, respiratory rate 24 breaths per minute, temperature 37.8°C (100°F), and oxygen saturation 95% on room air. His physical examination reveals dry mucous membranes, poor skin turgor, and decreased level of consciousness. His laboratory tests show:

  • Serum glucose: 900 mg/dL
  • Serum sodium: 150 mEq/L
  • Serum potassium: 4.0 mEq/L
  • Serum bicarbonate: 18 mEq/L
  • Serum osmolality: 350 mOsm/kg
  • Urine ketones: negative

Which of the following is the most appropriate initial treatment?

A) Intravenous insulin infusion
B) Intravenous normal saline infusion
C) Intravenous sodium bicarbonate infusion
D) Intravenous potassium chloride infusion
E) Subcutaneous insulin glargine injection

Answer and topic summary

The correct answer is B) Intravenous normal saline infusion

This patient has hyperosmolar hyperglycemic syndrome (HHS), which is a complication of diabetes mellitus characterized by severe hyperglycemia (>600 mg/dL), hyperosmolality (>320 mOsm/kg), and dehydration in the absence of ketoacidosis. HHS is often triggered by an acute stressor such as infection, medication noncompliance, or excessive carbohydrate intake. The mainstay of treatment for HHS is fluid replacement with isotonic saline to correct dehydration, lower serum glucose and osmolality, and improve renal function. Insulin therapy can be initiated after adequate fluid resuscitation, usually at a low dose (0.05-0.1 units/kg/hour). Electrolyte abnormalities such as hypokalemia or acidosis should be corrected as needed.

Incorrect answer explanations:

  • Insulin infusion is not the first-line treatment for HHS, as it can worsen dehydration, hypokalemia, and cerebral edema if given before adequate fluid replacement. Insulin should be started after fluid resuscitation at a low dose to avoid rapid drops in serum glucose and osmolality.
  • Sodium bicarbonate infusion is not indicated for HHS unless there is severe acidosis (pH <7.0). The patient’s serum bicarbonate level is mildly low (18 mEq/L), but not enough to warrant bicarbonate therapy. Moreover, sodium bicarbonate can increase serum osmolality and worsen cerebral edema.
  • Intravenous potassium chloride infusion is unnecessary for HHS unless there is hypokalemia (<3.5 mEq/L). The patient’s serum potassium level is normal (4.0 mEq/L), so potassium supplementation is not required. However, potassium levels should be monitored closely during fluid and insulin therapy, as they may drop rapidly due to intracellular shifts.
  • Insulin glargine injection is a long-acting insulin that provides basal coverage for up to 24 hours. It is not suitable for treating acute hyperglycemia in HHS, as it has a slow onset of action and cannot be titrated easily according to blood glucose levels. Moreover, subcutaneous insulin administration may be unreliable in patients with poor perfusion due to dehydration.

This podcast is available on every device!

You can download and listen to past FREE episodes here, on iTunes, Spotify, Google Podcasts, Stitcher, Amazon Music, and all podcasting apps.

Download Interactive Content Blueprint Checklists for the PANCE, PANRE, EOR, and PANRE-LA

PANRE and PANRE LA Interactive Content Blueprints

Follow this link to download your FREE copy of the PANCE/PANRE/EOR Content Blueprint Checklists.

Print it up and start crossing out the topics you understand, marking the ones you don’t, and making notes of key terms you should remember. The PDF version is interactive and linked directly to the individual lessons on Smarty PANCE.

The post Podcast Episode 103: Ten PANRE & PANRE-LA Intervention Complex Practice Questions appeared first on The Audio PANCE and PANRE.

]]>
Listen to Podcast Episode 103: Ten PANRE & PANRE-LA Intervention Complex Practice Question If you can’t see the audio player, click here to listen to the full episode. Welcome to episode 103 of the Audio PANCE and PANRE Physician Assistant/Associate Bo... If you can’t see the audio player, click here to listen to the full episode.
Welcome to episode 103 of the Audio PANCE and PANRE Physician Assistant/Associate Board Review Podcast.
Join me today as we cover ten NCCPA-style board review questions for your PANRE and PANRE-LA exams.
Special from today’s episode:

* Take the new PANRE & PANRE-LA (Intervention Complex) Practice Exam: Covers all the topics tested within the new PANRE (Intervention Complex) performance expectation with links to Smarty PANCE lessons.
* PANRE & PANRE-LA Blueprint 8-Week Schedule and Study Planner
* Read The New 2023 PANRE and PANRE-LA: Everything you Need to Know
* Members can try out the newly updated PANRE-LA Smart Search Tool (you must log in to access the search bar)
* Sign up for the Entire Blueprint Email Series to get daily questions for the next 478 days!
* Follow Smarty PANCE and The Daily PANCE Blueprint on Instagram
* Follow Smarty PANCE and The Daily PANCE Blueprint on Facebook
* Join the Smarty PANCE Member’s Community, then sign up for a study group to get updates about upcoming webinars

I hope you enjoy this free audio component of the examination portion of this site. Smarty PANCE includes over 2,000 interactive board review questions, along with flashcards, ReelDx cases, integrated Picmonics, and lessons covering every blueprint topic available to all Smarty PANCE members.

* You can download and listen to past FREE episodes here, on iTunes, Spotify, Google Podcasts, Stitcher, and most podcasting apps.
* You can listen to all the latest episodes, take interactive quizzes, and download more resources on each episode page.

Here is an interactive exam to complement today’s podcast
1. A 65-year-old man presents to your office with complaints of constipation for the past six months. He says that he has difficulty passing stools, which are hard and dry. He also reports occasional abdominal pain and bloating. He denies any weight loss, blood in stools, fever, or night sweats. His medical history is significant for hypertension and type 2 diabetes mellitus.]]>
The Physician Assistant Life | Smarty PANCE full 34:55 562
Podcast Episode 102: Ten PANCE, PANRE, and Rotation Review Questions http://podcast.thepalife.com/podcast-episode-102-ten-pance-panre-and-rotation-review-questions/ Thu, 09 Feb 2023 23:40:51 +0000 https://podcast.thepalife.com/?p=556 http://podcast.thepalife.com/podcast-episode-102-ten-pance-panre-and-rotation-review-questions/#respond http://podcast.thepalife.com/podcast-episode-102-ten-pance-panre-and-rotation-review-questions/feed/ 0 <p>Listen to Podcast Episode 102: Ten PANCE, PANRE, and Rotation Review Questions If you can’t see the audio player, click here to listen to the full episode. Welcome to episode 102 of the Audio PANCE and PANRE Physician Assistant/Associate Board Review Podcast. Join me today as we cover ten board review questions for your PANCE, […]</p> <p>The post <a rel="nofollow" href="http://podcast.thepalife.com/podcast-episode-102-ten-pance-panre-and-rotation-review-questions/">Podcast Episode 102: Ten PANCE, PANRE, and Rotation Review Questions</a> appeared first on <a rel="nofollow" href="http://podcast.thepalife.com">The Audio PANCE and PANRE</a>.</p> Listen to Podcast Episode 102: Ten PANCE, PANRE, and Rotation Review Questions

If you can’t see the audio player, click here to listen to the full episode.

Podcast Episode 102: The Audio PANCE and PANREWelcome to episode 102 of the Audio PANCE and PANRE Physician Assistant/Associate Board Review Podcast.

Join me today as we cover ten board review questions for your PANCE, PANRE, EOR™, and EOC™ exams.

Special from today’s episode:

I hope you enjoy this free audio component of the examination portion of this site. Smarty PANCE includes over 2,000 interactive board review questions, along with flashcards, ReelDx cases, integrated Picmonics, and lessons covering every blueprint topic available to all Smarty PANCE members.

Interactive exam to complement today’s podcast

1. Which of the following is NOT true about a non-ST elevation myocardial infarction?

A. Non-enteric-coated, chewable aspirin 325 mg should be given
B. Troponins are elevated
C. It happens due to a partially occluded epicardial coronary artery
D. Patients need a 12-lead EKG
E. You will always see ST depressions

Answer and topic summary

The answer is E. You will always see ST depressions

A non-ST elevation myocardial infarction (NSTEMI) is defined by the absence of persistent ST-elevation with elevated cardiac biomarkers (e.g., troponin I or T, CKMB, etc). It happens due to a partially occluded epicardial coronary artery (leading to subendocardial ischemia). NSTEMI typically presents as pressure-type chest pain.

Patients with a suspected NSTEMI should receive a 12-lead EKG within 10 minutes of arrival. ST depression, transient ST-elevation, and/or T-wave inversions may be seen on EKG, but they are NOT required for the diagnosis of NSTEMI. The most important medication to give is non-enteric-coated chewable aspirin 325 mg. Other meds include sublingual nitroglycerin, oxygen as needed, beta-blockers (assuming no C/I), high-intensity statin, ACE inhibitors (if CKD, DM, or EF < 40%), P2Y12 inhibitor (e.g., clopidogrel), anticoagulation, and possibly PCI with stenting or CABG.

Smarty PANCE Content Blueprint Review:

Covered under ⇒ PANCE Blueprint Cardiology ⇒ Coronary Heart Disease ⇒ Acute myocardial infarction ⇒ Non-ST-Segment Elevation MI (NSTEMI)

Also covered as part of the Family Medicine EORInternal Medicine EOREmergency Medicine EOR topic list

2. A 22-year-old G1P0 female at 28 weeks gestation with a history of diabetes presents to the clinic with fever, chills, and dysuria. Vitals show tachycardia (115 bpm), tachypnea (22 bpm), and hypotension (90/58 mmHg). Physical exam reveals suprapubic tenderness. Labs reveal leukocytosis, hyponatremia, and hyperglycemia. Urine dipstick is positive for nitrites, blood, glucose, and ketones. Which of the following is the next best step?

A. Send home with oral antibiotics and analgesics
B. Reassurance and oral rehydration solution
C. Admit to hospital for antibiotics, fluids, & insulin
D. Order outpatient CT scan of the abdomen
E. Refer to an outpatient nephrologist for workup

Answer and topic summary

The answer is C. Admit to hospital for antibiotics, fluids, & insulin

The patient most likely has pyelonephritis, which is an infection of the upper urinary tract and kidneys. It is one of the most common causes of sepsis in pregnancy. The pyelonephritis is also precipitating diabetic ketoacidosis in this diabetic patient.

Clinical features of pyelonephritis include fever, flank pain, N/V, and possible CVA tenderness. Pregnant patients are at high risk for obstetric and medical complications from the infection. It is recommended that pregnant women with acute pyelonephritis should be admitted for IV antibiotics (typically at least until the woman is febrile for 1-2 days and symptomatically improved). Antibiotic options include cefepime, piperacillin-tazobactam, and meropenem. This patient will also need insulin and fluids.

Smarty PANCE Content Blueprint Review:

Covered under ⇒ PANCE Blueprint Genitourinary ⇒ Infectious Disorders ⇒ Pyelonephritis

Also covered as part of the Family Medicine EORInternal Medicine EOR, and Emergency Medicine EOR topic list

3. Which of the following is the most common benign neoplasm of the liver?

A. Hepatocellular carcinoma
B. Hepatic hemangioma
C. Liver angiosarcoma
D. Hepatocellular adenoma
E. Hepatoblastoma

Answer and topic summary

The answer is B. Hepatic hemangioma

Hepatic hemangiomas are the most common benign liver lesions. The typical patient is a 30 to a 50-year-old woman. Exposure to estrogen may increase the size of hepatic hemangiomas. Patients are typically asymptomatic; however, if they do have symptoms they may have RUQ abdominal pain. An ultrasound will show a homogeneous, hyperechoic mass. If lesions are < 5 cm, you usually don’t need to do anything. If > 5 cm, monitoring is needed Q6-12 months via MRI.

Smarty PANCE Content Blueprint Review:

Covered under ⇒ PANCE Blueprint GI and Nutrition ⇒ Gastrointestinal System Neoplasms ⇒ Liver neoplasms

4. A 12-year-old male presents with a sore throat, fever, dysphagia, and a muffled voice. On physical exam, the uvula is deviated. Which of the following is the most likely diagnosis?

A. Pharyngitis only
B. Retropharyngeal abscess
C. Peritonsillar abscess
D. Oral candidiasis
E. Foreign body

Answer and topic summary

The answer is C. Peritonsillar abscess

A peritonsillar abscess is a collection of pus near the tonsils. The most common causes are Streptococcus pyogenes (group A), Streptococcus anginosus, & Staphylococcus aureus. Clinical features include muffled voice, sore throat, fever, drooling, neck pain, fatigue, and decreased PO intake. On physical exam, the uvula may be deviated. Diagnostic tests that can be done include ultrasound or CT with contrast (depends). Treatment is antibiotics, drainage, and supportive care.

Smarty PANCE Content Blueprint Review:

Covered under ⇒ PANCE Blueprint EENT ⇒ Oropharyngeal disorders ⇒ Infectious and inflammatory disorders ⇒ Peritonsillar abscess

Also covered as part of the Pediatric EORFamily Medicine EOR, and Emergency Medicine EOR topic list

5. Which of the following is the most common cause of acute epiglottitis?

A. Staphylococcus aureus
B. Haemophilus influenzae
C. Streptococcus pneumoniae
D. Streptococcus pyogenes
E. Neisseria meningitidis

Answer and topic summary

The answer is B. Haemophilus influenzae

Acute epiglottitis is an inflammatory condition of the epiglottis (usually due to a bacterial infection). In severe situations, it is life-threatening. The most common cause overall is said to be Haemophilus influenzae; however, in adults, Streptococcal species now tend to be more common. The management of this disease should focus on giving antibiotics and steroids. Airway management and securing an airway is the most important part of treatment.

Smarty PANCE Content Blueprint Review:

Covered under ⇒ PANCE Blueprint Pulmonary ⇒ Infectious Pulmonary Disorders ⇒ Acute epiglottitis

Also covered as part of the Pediatric EOR and Emergency Medicine EOR topic list

6. A 32-year-old female presents with pain around the nail fold on the index finger of her right hand. She is afebrile. On physical exam, you appreciate erythema and swelling of the proximal nail fold. The area is not fluctuant. Which of the following is the best treatment option?

A. Topical antibiotics and warm water soaks
B. Intravenous antibiotics
C. Oral antibiotics and incision and drainage
D. Surgical consult for removal
E. None of the above

Answer and topic summary

The answer is A. Topical antibiotics and warm water soaks

The patient has acute paronychia, which is a very common condition and essentially is just inflammation of the nail folds. The most common bacterial causes are Staph aureus and Strep pyogenes. Risk factors include manicures, nail biting, and picking at nails. Clinical features include sudden onset of painful erythema and swelling. Sometimes an abscess will be present. In this patient’s case, there was no fluctuance and so topical antibiotics and warm water soaks would be appropriate.

If the patient had an abscess, oral antibiotics and incision/drainage (with number 11 surgical blade) would be reasonable.

Smarty PANCE Content Blueprint Review:

Covered under ⇒ PANCE Blueprint Infectious DiseaseBacterial DiseaseMethicillin-resistant Staphylococcus aureus infection

7. A 60-year-old male presents with fatigue and weight loss. His physical exam is remarkable for gingival hyperplasia and splenomegaly. Labs reveal pancytopenia. A bone marrow biopsy reveals Auer rods. What is the likely diagnosis?

A. Acute myeloid leukemia
B. Acute lymphocytic leukemia
C. Chronic myeloid leukemia
D. Chronic lymphocytic leukemia
E. Non-Hodgkin’s lymphoma

Answer and topic summary

The answer is A. Acute myeloid leukemia

The patient has acute myeloid leukemia (AML), which is a group of cancers that involve the myeloid precursor cells. It is characterized by clonal proliferation (excessive growth) of abnormal myeloid precursor cells. Risk factors include smoking and chemotherapy/radiation.

Clinical features include fatigue, pallor, weakness, bone pain, gingival bleeding, and organomegaly. Lab findings are variable but may include pancytopenia (decrease in RBCs, WBCs, and platelets), electrolyte derangements, and/or hypoxemia. Workup should include a peripheral blood smear and bone marrow biopsy. The bone marrow biopsy will classically show Auer rods and >20% blasts.

Smarty PANCE Content Blueprint Review:

Covered under ⇒ PANCE Blueprint HematologyNeoplasms, premalignancies, and malignanciesAcute and chronic myelogenous leukemia

Also covered as part of the Family Medicine EOR topic list

8. A 42-year-old female presents for an annual physical exam with some complaints of fatigue and dyspnea. On physical exam, you appreciate a fixed S2 split. Which of the following is the most likely diagnosis?

A. Ventricular septal defect
B. Mitral regurgitation
C. Aortic stenosis
D. Atrial septal defect
E. Patent ductus arteriosus

Answer and topic summary

The answer is D. Atrial septal defect

Atrial septal defect is the most common congenital heart lesion in adults. People often don’t have symptoms until adulthood. The most common type is ostium secundum. Symptoms include fatigue and dyspnea. A classic physical exam finding on a test would be a systolic ejection crescendo-decrescendo flow murmur @ LUSB with FIXED S2, loud S1. Echocardiography is the initial imaging modality of choice. Keep in mind that a complication of ASD is that the left to right shunt can cause volume and pressure overload of the right heart and pulmonary circulation, leading to pulmonary hypertension.

Smarty PANCE Content Blueprint Review:

Covered under ⇒ PANCE Blueprint Cardiology ⇒ Congenital Heart Disease ⇒ Atrial septal defect

Also covered as part of the Pediatric EOR topic list

9. Which of the following is not a test for acute appendicitis?

A. Murphy sign
B. Rovsing sign
C. Obturator sign
D. Psoas sign
E. McBurney sign

Answer and topic summary

The answer is A. Murphy sign

Appendicitis is one of the most common indications for emergent abdominal surgery. The most common physical exam finding is RLQ tenderness. There are some tests that may help with the diagnosis — keep in mind though that these tests/signs are NOT sensitive.

  • Rovsing sign: pain in the RLQ with palpation of the LLQ (indicative of peritoneal irritation)
  • McBurney sign: tenderness about 2 inches from the ASIS (on a straight line from ASIS to the umbilicus)
  • Psoas sign: RLQ pain with passive right hip extension
  • Obturator sign: flexion of the right hip and knee, followed by internal rotation of the right hip elicits RLQ pain

Murphy’s sign is positive in acute cholecystitis, not appendicitis. A positive test is RUQ pain on inspiration.

Smarty PANCE Content Blueprint Review:

Covered under ⇒ PANCE Blueprint GI and Nutrition ⇒ Diseases of the Small Intestine ⇒ Appendicitis

Also covered as part of the Emergency Medicine EORFamily Medicine EORPediatric EOR, and General Surgery EOR topic list

10. A 31-year-old female presents with pain and numbness in her anterior tibial region. On physical exam, the area is pale, cold, and pulseless. Which of the following is the best definitive treatment for this likely diagnosis?

A. Antiplatelets
B. Amputation
C. Physical therapy
D. Fasciotomy
E. IV heparin drip

Answer and topic summary

The answer is D. Fasciotomy

The patient has acute compartment syndrome, which is when the tissue pressure within a closed muscle compartment exceeds the perfusion pressure. It can result in ischemia. The most common location is the calf. Many cases of acute compartment syndrome in the lower extremity are associated with fractures, burn injuries, crush injuries, or soft tissue infections. The “5 P’s” can help you remember the symptoms – painpallorparesthesiapulselessnessparalysis. Extremity fasciotomy is the treatment for acute compartment syndrome.

Smarty PANCE Content Blueprint Review:

Covered under ⇒ PANCE Blueprint Musculoskeletal  ⇒ Compartment Syndrome

This podcast is available on every device!

You can download and listen to past FREE episodes here, on iTunes, Spotify, Google Podcasts, Stitcher, Amazon Music, and all podcasting apps.

Download Interactive Content Blueprint Checklists for the PANCE, PANRE, EOR, and PANRE-LA

Interactive Content Blueprints for the PANCE PANRE and PANRE-LA

Follow this link to download your FREE copy of the PANCE/PANRE/EOR Content Blueprint Checklists.

Print it up and start crossing out the topics you understand, marking the ones you don’t, and making notes of key terms you should remember. The PDF version is interactive and linked directly to the individual lessons on Smarty PANCE.

The post Podcast Episode 102: Ten PANCE, PANRE, and Rotation Review Questions appeared first on The Audio PANCE and PANRE.

]]>
Listen to Podcast Episode 102: Ten PANCE, PANRE, and Rotation Review Questions If you can’t see the audio player, click here to listen to the full episode. Welcome to episode 102 of the Audio PANCE and PANRE Physician Assistant/Associate Board Review P... If you can’t see the audio player, click here to listen to the full episode.
Welcome to episode 102 of the Audio PANCE and PANRE Physician Assistant/Associate Board Review Podcast.
Join me today as we cover ten board review questions for your PANCE, PANRE, EOR™, and EOC™ exams.
Special from today’s episode:

* Read The New 2023 PANRE and PANRE-LA: Everything you Need to Know
* Members can try out the newly updated PANRE-LA Smart Search (you must log in to access the search bar)
* Sign up for the Entire Blueprint Email Series
* Follow Smarty PANCE and The Daily PANCE Blueprint on Instagram
* Follow Smarty PANCE and The Daily PANCE Blueprint on Facebook
* Join the Smarty PANCE Member’s Community then sign up for a study group to get updates about upcoming webinars.

I hope you enjoy this free audio component of the examination portion of this site. Smarty PANCE includes over 2,000 interactive board review questions, along with flashcards, ReelDx cases, integrated Picmonics, and lessons covering every blueprint topic available to all Smarty PANCE members.

* You can download and listen to past FREE episodes here, on iTunes, Spotify, Google Podcasts, Stitcher, and most podcasting apps.
* You can listen to all the latest episodes, take interactive quizzes, and download more resources on each episode page.

Interactive exam to complement today’s podcast
1. Which of the following is NOT true about a non-ST elevation myocardial infarction?
A. Non-enteric-coated, chewable aspirin 325 mg should be given
B. Troponins are elevated
C. It happens due to a partially occluded epicardial coronary artery
D. Patients need a 12-lead EKG
E. You will always see ST depressions
2. A 22-year-old G1P0 female at 28 weeks gestation with a history of diabetes presents to the clinic with fever, chills, and dysuria. Vitals show tachycardia (115 bpm), tachypnea (22 bpm), and hypotension (90/58 mmHg). Physical exam reveals suprapubic tenderness. Labs reveal leukocytosis, hyponatremia, and hyperglycemia. Urine dipstick is positive for nitrites, blood, glucose, and ketones. Which of the following is the next best step?
A. Send home with oral antibiotics and analgesics
B. Reassurance and oral rehydration solution
C. Admit to hospital for antibiotics, fluids, & insulin
D. Order outpatient CT scan of the abdomen
E.]]>
The Physician Assistant Life | Smarty PANCE full 21:57 556
Podcast Episode 101: Ten PANCE, PANRE, and Rotation Review Questions http://podcast.thepalife.com/podcast-episode-101-ten-pance-panre-and-rotation-review-questions/ Wed, 02 Nov 2022 04:26:35 +0000 https://podcast.thepalife.com/?p=535 http://podcast.thepalife.com/podcast-episode-101-ten-pance-panre-and-rotation-review-questions/#respond http://podcast.thepalife.com/podcast-episode-101-ten-pance-panre-and-rotation-review-questions/feed/ 0 <p>Welcome to episode 101 of the Audio PANCE and PANRE Physician Assistant/Associate Board Review Podcast. Join me today as we cover ten board review questions for your PANCE, PANRE, and rotation exams. Special from today’s episode: Join the Smarty PANCE Member’s Community then sign up for a study group to get updates about upcoming webinars. […]</p> <p>The post <a rel="nofollow" href="http://podcast.thepalife.com/podcast-episode-101-ten-pance-panre-and-rotation-review-questions/">Podcast Episode 101: Ten PANCE, PANRE, and Rotation Review Questions</a> appeared first on <a rel="nofollow" href="http://podcast.thepalife.com">The Audio PANCE and PANRE</a>.</p> Podcast Episode 101 - Ten Questions For Your PANCE PANRE and EOC EOR ExamsWelcome to episode 101 of the Audio PANCE and PANRE Physician Assistant/Associate Board Review Podcast.

Join me today as we cover ten board review questions for your PANCE, PANRE, and rotation exams.

Special from today’s episode:

I hope you enjoy this free audio component of the examination portion of the Smarty PANCE website. The full board review website includes over 2,000 interactive board review questions, flashcards, and blueprint lessons available to all members of Smarty PANCE.

Listen to Podcast Episode 101: Ten PANCE, PANRE, and Rotation Review Questions

If you can’t see the audio player, click here to listen to the full episode.

Interactive exam to complement today’s podcast

1. A 75-year-female smoker with a history of atrial fibrillation and hypertension presents to the ER complaining of a 2-hour history of right-sided weakness and aphasia that has now resolved. Her physical exam and vital signs are completely unremarkable. CT head is unremarkable. Which of the following is the most likely diagnosis?

A. Subarachnoid hemorrhage
B. Transient ischemic attack
C. Cerebral venous sinus thrombosis
D. Multiple sclerosis
E. Complicated migraine

Answer and topic summary

The answer is B. Transient ischemic attack

The patient had a transient ischemic attack (TIA), which is characterized by transient neurological symptoms without objective evidence of acute infarction. Symptoms vary, but patients may have hemiparesis, hemiplegia, aphasia, or vision loss. Risk factors include alcohol, hypertension, smoking, diabetes, illicit drug use, and atrial fibrillation.

Urgent evaluation is needed in patients with symptoms of TIA (e.g., coagulation studies, TTE, EKG, CT or MRI head, imaging of the cervicocephalic vasculature via carotid US, CTA, or MRA). Treatment for high-risk patients includes dual antiplatelet therapy (ASA + clopidogrel) for 21 days and risk factor management (statin, exercise, etc). Remember the risk of an actual stroke is high after a TIA.

View blueprint lesson

Smarty PANCE Content Blueprint Review:

Covered under ⇒ PANCE Blueprint NeurologyVascular DisorderTransient ischemic attack

Also covered as part of the Internal Medicine EOR, Family Medicine EOR, and Emergency Medicine EOR topic list

2. A 26-year-old female presents with a history of miscarriages and recurrent pulmonary embolism. She is diagnosed with antiphospholipid syndrome. Which of the following is the mainstay of treatment for this condition?

A. Dual antiplatelet therapy (DAPT)
B. Aspirin and heparin
C. Dabigatran
D. Warfarin
E. Heparin

Answer and topic summary

The answer is D. Warfarin

Antiphospholipid syndrome (APS) is an autoimmune disease defined by venous thromboembolism, arterial thrombosis, and obstetric morbidity in the presence of circulating antiphospholipid antibodies (aPLs). It is the most common form of acquired thrombophilia.

aPLs include lupus anticoagulant, anticardiolipin, anti-b2-glycoprotein I antibodies. Clinical features include DVTs (32%), thrombocytopenia (22%), livedo reticularis (20%), stroke (13%), PEs (9%), and fetal loss (8%). The mainstay of treatment is warfarin. Other anticoagulants have been found to be less effective than warfarin so far.

View blueprint lesson

Smarty PANCE Content Blueprint Review:

Covered under ⇒ PANCE Blueprint HematologyCoagulation DisordersHypercoagulable states

Also covered as part of the Internal Medicine EOR and Emergency Medicine EOR topic list

3. Which of the following is the most common cause of acute bronchitis in the United States?

A. Virus
B. Bacteria
C. Fungus
D. Allergies
E. Tuberculosis

Answer and topic summary

The answer is A. Virus

Acute bronchitis is characterized by a cough due to inflammation of the large airways and trachea with no evidence of pneumonia. Acute bronchitis often follows an upper respiratory infection. The most common cause of acute bronchitis is viral. Symptoms include cough (10-20 days), headache, shortness of breath, and wheezing. The presence of fever should make you consider pneumonia or influenza.

Diagnosis is clinical. A chest radiograph should be given if it is hard to clinically distinguish between pneumonia and bronchitis. Treatment is mainly supportive care (rest, hydration) and symptomatic management (antitussives for cough, bronchodilators for wheezing, etc). Antibiotics should NOT be given for acute bronchitis.

View blueprint lesson

Smarty PANCE Content Blueprint Review:

Covered under ⇒ PANCE Blueprint PulmonaryInfectious Pulmonary DisordersAcute bronchitis

Also covered as part of the Emergency Medicine EOR topic list

4. A 27-year-old male presents to the clinic with shortness of breath, chest pain, and fatigue. His physical exam is remarkable for a crescendo-decrescendo systolic murmur heard at the apex. An echocardiogram reveals asymmetric septal hypertrophy and left ventricular hypertrophy. Which of the following is the best initial treatment option?

A. Procainamide
B. Hydrochlorothiazide
C. Beta-blockers
D. Aspirin
E. ACE inhibitor

Answer and topic summary

The answer is C. Beta-blockers

The patient has hypertrophic cardiomyopathy (HCM), which is a genetic disease of the heart muscle due to mutations in the sarcomere genes. It often presents with fatigue, dyspnea, chest pain, or syncope. The murmur on the physical exam is due to LV obstruction; it will be a harsh crescendo-decrescendo systolic murmur heard at the apex and LLSB. It’s important to keep in mind that the murmur intensity will decrease with more venous return (squatting) and increase with less venous return (Valsalva).

Diagnostic tests include an EKG, echocardiogram, and exercise stress testing. Treatment is indicated for symptomatic patients and includes a negative inotropic agent (i.e., nondihydropyridine CCBs or beta-blockers) and diuretics as needed for volume overload. If patients are refractory to medications, septal myectomy and percutaneous septal ablation can be performed.

View blueprint lesson

Smarty PANCE Content Blueprint Review:

Covered under ⇒ PANCE Blueprint CardiologyCardiomyopathyHypertrophic Cardiomyopathy

Also covered as part of the Pediatric Rotation EOR topic list

5. A 22-year-old female presents to the ER with crampy lower abdominal pain and vaginal bleeding for the past 2 hours. She missed her last menstrual cycle. The physical exam is remarkable for an open cervical os. No products of conception are observed. Which of the following is the most likely diagnosis?

A. Threatened abortion
B. Incomplete abortion
C. Missed abortion
D. Septic abortion
E. Inevitable abortion

Answer and topic summary

The answer is E. Inevitable abortion

The patient most likely has a spontaneous abortion, which is a nonviable intrauterine pregnancy up to 20 weeks gestation. The most common cause of miscarriages in the first trimester is chromosomal abnormalities. There are many different types of spontaneous abortions.

The patient specifically has an inevitable abortion, which typically presents with vaginal bleeding, crampy abdominal pain, and an open external cervical os without expulsion of products of conception. The fetus is not viable.

View blueprint lesson

Smarty PANCE Content Blueprint Review:

Covered under ⇒ PANCE Blueprint Reproductive SystemComplicated PregnancyAbortion

Also covered as part of the Women’s Health EOR topic list

6. Which of the following is not a risk factor for Methicillin-resistant Staphylococcus aureus (MRSA)?

A. Previous antibiotic use
B. Injection drug use
C. Indwelling hemodialysis catheter
D. Presence in long-term facility
E. All of the above are risk factors

Answer and topic summary

The answer is E. All of the above are risk factors 

Methicillin-resistant Staphylococcus aureus (MRSA) infection is caused by a certain type of S. aureus that is resistant to several antibiotics. Risk factors include previous antibiotic use, injection drug use, indwelling hemodialysis catheter, living in a long-term care facility, HIV infection, and recent hospitalization.

The most common site for MRSA infections to occur is the skin/soft tissue. Oral antibiotics that cover MRSA include Bactrim, doxycycline, and clindamycin. For severe MRSA infections, IV vancomycin is preferred.

View blueprint lesson

Smarty PANCE Content Blueprint Review:

Covered under ⇒ PANCE Blueprint Infectious DiseaseBacterial DiseaseMethicillin-resistant Staphylococcus aureus infection

7. A 60-year-old male presents with fatigue and weight loss. His physical exam is remarkable for gingival hyperplasia and splenomegaly. Labs reveal pancytopenia. A bone marrow biopsy reveals Auer rods. What is the likely diagnosis?

A. Acute myeloid leukemia
B. Acute lymphocytic leukemia
C. Chronic myeloid leukemia
D. Chronic lymphocytic leukemia
E. Non-Hodgkin’s lymphoma

Answer and topic summary

The answer is A. Acute myeloid leukemia

The patient has acute myeloid leukemia (AML), which is a group of cancers that involve the myeloid precursor cells. It is characterized by clonal proliferation (excessive growth) of abnormal myeloid precursor cells. Risk factors include smoking and chemotherapy/radiation.

Clinical features include fatigue, pallor, weakness, bone pain, gingival bleeding, and organomegaly. Lab findings are variable but may include pancytopenia (decrease in RBCs, WBCs, and platelets), electrolyte derangements, and/or hypoxemia. The workup should include a peripheral blood smear and bone marrow biopsy. The bone marrow biopsy will classically show Auer rods and >20% blasts.

View blueprint lesson

Smarty PANCE Content Blueprint Review:

Covered under ⇒ PANCE Blueprint HematologyNeoplasms, premalignancies, and malignanciesAcute and chronic myelogenous leukemia

Also covered as part of the Internal Medicine EOR, Emergency Medicine EOR, Family Medicine EOR, and Pediatric EOR topic list

8. Giant Cell Arteritis: The Daily PANCE Blueprint

A 55-year-old female presents with right-sided headaches, jaw pain upon chewing, and mild dizziness for the past few months. On physical exam, the right temporal artery is tender to palpation. Labs reveal an elevated ESR and CRP. What is the definitive diagnosis for this condition?

A. CT head
B. Temporal artery biopsy
C. PET scan
D. Ultrasound
E. IgA antibody level

Answer and topic summary

The answer is B. Temporal artery biopsy

The patient has temporal arteritis, which is a vasculitis of large and medium vessels. Clinical features often include fever, fatigue, weight loss, headache, jaw claudication, and transient vision loss. The typical clinical vignette is an older female presenting with a headache and jaw claudication. On physical exam, you may appreciate a temporal artery that is thickened, tender, or erythematous.

ESR and CRP are sensitive, but not specific for the diagnosis. The definitive diagnosis of temporal arteritis is a temporal artery biopsy. Treatment is high-dose steroids. It’s important that temporal arteritis is treated because it can lead to blindness, thrombotic events, and other complications.

View blueprint lesson

Smarty PANCE Content Blueprint Review:

Covered under ⇒ PANCE Blueprint CardiologyVascular DiseaseGiant cell arteritis

Also covered as part of the Internal Medicine EOR topic list

9. Which of the following is one of the most common initial clinical symptoms in multiple sclerosis?

A. Vertigo
B. Extreme weakness
C. Depression
D. Arthralgias
E. Optic neuritis

Answer and topic summary

The answer is E. Optic neuritis

Multiple sclerosis (MS) is a demyelinating disease of the central nervous system. There are many types of MS patterns. Clinical features suggestive of MS include sensory loss in limbs or one side of the face, gait/balance issues, heat sensitivity (Uhthoff), fatigue, and Lhermitte sign. The most common presenting symptom is optic neuritis.

MRI of the brain (+/- spinal cord) is the diagnostic test of choice. The McDonald diagnostic criteria can be used to help make the diagnosis of MS. Treatment of MS includes disease-modifying pharmacotherapy, such as glatiramer acetate (Copaxone), dimethyl fumarate, natalizumab (Tysabri), interferons, and many more. Acute exacerbations of MS are managed with short-term, high-dose glucocorticoids.

View blueprint lesson

Smarty PANCE Content Blueprint Review:

Covered under ⇒ PANCE Blueprint NeurologyNeuromuscular disordersMultiple sclerosis

Also covered as part of the Internal Medicine EOR topic list

10. A 52-year-old male presents to the clinic complaining of “sharp burning” in his lower chest after eating a large meal. The pain goes away when he raises the head of the bed. Which of the following is the best pharmacological agent for this likely diagnosis?

A. Esomeprazole
B. Nitroglycerin
C. Aspirin
D. Codeine
E. Sucralfate

Answer and topic summary

The answer is A. Esomeprazole

The patient has gastrointestinal reflux disease (GERD), which is a common condition characterized by stomach acid flowing up into the esophagus. Classic symptoms include heartburn (burning sensation around the sternum) after eating, water brash, regurgitation, and dysphagia.

The diagnosis is usually clinical; however, if the patient has alarming symptoms (e.g., weight loss, anemia, etc), it is appropriate to get an upper endoscopy. The gold standard is an ambulatory 24-hour pH monitoring system. Initial management includes weight loss, elevating the head of the bed, and eliminating triggers (spicy foods, chocolate, etc). Most recommend starting with histamine 2 receptor antagonist therapy first (e.g., ranitidine) and then transitioning to proton pump inhibitors (e.g., esomeprazole).

View blueprint lesson

Smarty PANCE Content Blueprint Review:

Covered under ⇒ PANCE Blueprint GI and NutritionEsophageal DisordersGastroesophageal reflux disease

Also covered as part of the Family Medicine EOR, Internal Medicine EOR, and Pediatric EOR topic list

This podcast is available on every device!

You can download and listen to past FREE episodes here, on iTunes, Spotify, Google Podcasts, Stitcher, Amazon Music, and all podcasting apps.

Download Interactive Content Blueprint Checklists for the PANCE, PANRE, EOR, and PANRE-LA

Interactive Content Blueprints for the PANCE PANRE and PANRE-LA

Follow this link to download your FREE copy of the PANCE/PANRE/EOR Content Blueprint Checklists.

Print it up and start crossing out the topics you understand, marking the ones you don’t, and making notes of key terms you should remember. The PDF version is interactive and linked directly to the individual lessons on Smarty PANCE.

The post Podcast Episode 101: Ten PANCE, PANRE, and Rotation Review Questions appeared first on The Audio PANCE and PANRE.

]]>
Welcome to episode 101 of the Audio PANCE and PANRE Physician Assistant/Associate Board Review Podcast. Join me today as we cover ten board review questions for your PANCE, PANRE, and rotation exams. Special from today’s episode: Join the Smarty PANCE ... Welcome to episode 101 of the Audio PANCE and PANRE Physician Assistant/Associate Board Review Podcast.
Join me today as we cover ten board review questions for your PANCE, PANRE, and rotation exams.
Special from today’s episode:

* Join the Smarty PANCE Member’s Community then sign up for a study group to get updates about upcoming webinars.
* Check out our updated End of Curriculum™ (EOC) Exam Course
* Sign up for the Entire Blueprint Email Series
* Follow Smarty PANCE and The Daily PANCE Blueprint on Instagram
* Follow Smarty PANCE and The Daily PANCE Blueprint on Facebook

I hope you enjoy this free audio component of the examination portion of the Smarty PANCE website. The full board review website includes over 2,000 interactive board review questions, flashcards, and blueprint lessons available to all members of Smarty PANCE.

* You can download and listen to past FREE episodes here, on iTunes, Spotify, Google Podcasts, Stitcher, and most podcasting apps.
* You can listen to all the latest episodes, take interactive quizzes, and download more resources on each episode page.

Listen to Podcast Episode 101: Ten PANCE, PANRE, and Rotation Review Questions
If you can’t see the audio player, click here to listen to the full episode.
Interactive exam to complement today’s podcast
1. A 75-year-female smoker with a history of atrial fibrillation and hypertension presents to the ER complaining of a 2-hour history of right-sided weakness and aphasia that has now resolved. Her physical exam and vital signs are completely unremarkable. CT head is unremarkable. Which of the following is the most likely diagnosis?
A. Subarachnoid hemorrhage
B. Transient ischemic attack
C. Cerebral venous sinus thrombosis
D. Multiple sclerosis
E. Complicated migraine
2. A 26-year-old female presents with a history of miscarriages and recurrent pulmonary embolism. She is diagnosed with antiphospholipid syndrome. Which of the following is the mainstay of treatment for this condition?
A. Dual antiplatelet therapy (DAPT)
B. Aspirin and heparin
C. Dabigatran
D. Warfarin
E. Heparin
3. Which of the following is the most common cause of acute bronchitis in the United States?
A. Virus
B. Bacteria
C. Fungus
D. Allergies
E. Tuberculosis
4. A 27-year-old male presents to the clinic with shortness of breath, chest pain, and fatigue. His physical exam is remarkable for a crescendo-decrescendo systolic murmur heard at the apex.]]>
The Physician Assistant Life | Smarty PANCE full 20:21 535
Podcast Episode 100: Atrial Fibrillation for the PANCE and PANRE http://podcast.thepalife.com/episode-100/ Tue, 20 Sep 2022 18:15:06 +0000 http://podcast.thepalife.com/?p=523 http://podcast.thepalife.com/episode-100/#respond http://podcast.thepalife.com/episode-100/feed/ 0 <p>Welcome to episode one hundred of the Audio PANCE and PANRE Physician Assistant/Associate Board Review Podcast. Join me today as we cover atrial fibrillation for the PANCE, PANRE, and EOR™ exams. Special from today’s episode: Join the Smarty PANCE Member’s Community then sign up for the Sunday Funday Study Group. Once you have signed up, […]</p> <p>The post <a rel="nofollow" href="http://podcast.thepalife.com/episode-100/">Podcast Episode 100: Atrial Fibrillation for the PANCE and PANRE</a> appeared first on <a rel="nofollow" href="http://podcast.thepalife.com">The Audio PANCE and PANRE</a>.</p> Welcome to episode one hundred of the Audio PANCE and PANRE Physician Assistant/Associate Board Review Podcast.

Join me today as we cover atrial fibrillation for the PANCE, PANRE, and EOR™ exams.

Special from today’s episode:

Below you will find the audio of today’s podcast.

The Audio PANCE/PANRE and EOR PA Board Review Podcast

I hope you enjoy this free audio component to the examination portion of this site. The full board review course includes over 2,000 interactive board review questions and is available to all members of Smarty PANCE.

Listen to Podcast Episode 100: Atrial Fibrillation for the PANCE and PANRE

If you can’t see the audio player, click here to listen to the full episode.

The post Podcast Episode 100: Atrial Fibrillation for the PANCE and PANRE appeared first on The Audio PANCE and PANRE.

]]>
Welcome to episode one hundred of the Audio PANCE and PANRE Physician Assistant/Associate Board Review Podcast. Join me today as we cover atrial fibrillation for the PANCE, PANRE, and EOR™ exams. Special from today’s episode: Join the Smarty PANCE Memb... Welcome to episode one hundred of the Audio PANCE and PANRE Physician Assistant/Associate Board Review Podcast.
Join me today as we cover atrial fibrillation for the PANCE, PANRE, and EOR™ exams.
Special from today’s episode:

* Join the Smarty PANCE Member’s Community then sign up for the Sunday Funday Study Group. Once you have signed up, you can access the Webinar replay of this episode here
* Check out our updated End of Curriculum™ (EOC) Exam Course (now with orthopedics!)
* Sign up for the Entire Blueprint email series
* Follow Smarty PANCE and The Daily PANCE Blueprint on Instagram
* Follow Smarty PANCE and The Daily PANCE Blueprint on Facebook

Below you will find the audio of today’s podcast.
The Audio PANCE/PANRE and EOR PA Board Review Podcast
I hope you enjoy this free audio component to the examination portion of this site. The full board review course includes over 2,000 interactive board review questions and is available to all members of Smarty PANCE.

* You can download and listen to past FREE episodes here, on iTunes, Spotify, Google Podcasts, Stitcher, and most podcasting apps.
* You can listen to all the latest episodes, take interactive quizzes, and download more resources on each episode page.

Listen to Podcast Episode 100: Atrial Fibrillation for the PANCE and PANRE
If you can’t see the audio player, click here to listen to the full episode.
]]>
The Physician Assistant Life | Smarty PANCE full 59:06 523
Podcast Episode 99: Ten PANCE, PANRE, and Rotation Review Questions http://podcast.thepalife.com/podcast-episode-99-ten-pance-panre-and-rotation-review-questions/ Thu, 21 Jul 2022 14:08:30 +0000 http://podcast.thepalife.com/?p=505 http://podcast.thepalife.com/podcast-episode-99-ten-pance-panre-and-rotation-review-questions/#respond http://podcast.thepalife.com/podcast-episode-99-ten-pance-panre-and-rotation-review-questions/feed/ 0 <p>Welcome to episode 99 of the Audio PANCE and PANRE Physician Assistant/Associate Board Review Podcast. Join me as I cover ten PANCE, PANRE, and EOR™ review questions from the Smarty PANCE Instagram/Facebook page and the smartypance.com board review website. Special from today’s episode: Join the Smarty PANCE Member’s Community Check out our all-new End of […]</p> <p>The post <a rel="nofollow" href="http://podcast.thepalife.com/podcast-episode-99-ten-pance-panre-and-rotation-review-questions/">Podcast Episode 99: Ten PANCE, PANRE, and Rotation Review Questions</a> appeared first on <a rel="nofollow" href="http://podcast.thepalife.com">The Audio PANCE and PANRE</a>.</p> The Audio PANCE and PANRE Episode 99Welcome to episode 99 of the Audio PANCE and PANRE Physician Assistant/Associate Board Review Podcast.

Join me as I cover ten PANCE, PANRE, and EOR™ review questions from the Smarty PANCE Instagram/Facebook page and the smartypance.com board review website.

Special from today’s episode:

Below you will find an interactive exam to complement today’s podcast.

The Audio PANCE/PANRE and EOR PA Board Review Podcast

I hope you enjoy this free audio component to the examination portion of this site. The full board review course includes over 2,000 interactive board review questions and is available to all members of Smarty PANCE.

Listen Carefully Then Take the Practice Exam

If you can’t see the audio player, click here to listen to the full episode.

Podcast Episode 99: Ten PANCE/PANRE and EOR Topic Blueprint Questions

1. A 42-year-old male on lithium presents with polyuria, nocturia, and polydipsia. Laboratory findings are remarkable for slightly elevated sodium. Which of the following is the most likely diagnosis?

A. Neurogenic diabetes insipidus
B. Nephrogenic diabetes insipidus
C. Type 2 diabetes mellitus
D. SIADH
E. Adrenal insufficiency

Click here to see the answer

The answer is B. Nephrogenic diabetes insipidus

Nephrogenic diabetes insipidus (DI) is a condition where the kidneys are resistant to the effects of ADH. Nephrogenic DI can occur secondary to lithium toxicity or chronic lithium usepregnancyinherited disorders, and electrolyte issues. Clinical features include polyuria, nocturia, and polydipsia. Serum sodium is either normal or high.

In this patient, lithium has entered the collecting duct, accumulated, and interfered with ADH’s capacity to increase water permeability. Treatment options for this patient include amiloride or thiazide diuretics. In general, whether or not lithium is discontinued depends on many factors.

VIEW BLUEPRINT LESSON

Smarty PANCE Content Blueprint Review:

Covered under ⇒ PANCE Blueprint Endocrinology ⇒ Pituitary Disorders ⇒ Diabetes insipidus

Also covered as part of the Internal Medicine EOR and Emergency Medicine PAEA EOR topic list

2. A 50-year-old female presents with poor appetite, low energy, poor concentration, and feelings of hopelessness on most days for the past 3 years. She denies suicidal ideation. She has never had a past manic or hypomanic episode. Which of the following is the best treatment option?

A. Haloperidol
B. Fluoxetine
C. Lorazepam
D. Amitriptyline
E. Risperidone

Click here to see the answer

The answer is B. Fluoxetine

This patient has dysthymia, or persistent depressive disorder. The DSM V criteria follow:

  • Depressed mood ≥ 2 years on most days
  • At least 2 of the following: appetite changes, sleep changes, low energy, low self-esteem, poor concentration, hopelessness
  • Not without symptoms > 2 months at a time
  • No mania or hypomania episodes, ever

Like other psychiatric disorders, symptoms can’t be attributable to drugs, and the symptoms must cause distress/impairment. 1st line treatment is selective serotonin reuptake inhibitors and psychotherapy.

VIEW BLUEPRINT LESSON

Smarty PANCE Content Blueprint Review:

Covered under ⇒ PANCE Blueprint Psychiatry ⇒ Depressive disorders ⇒ Persistent depressive disorder (dysthymia)

Also covered as part of the Psychiatry EOREmergency Medicine EOR, and Pediatric PAEA EOR topic list

3. An 85-year-old male with a history of chronic kidney disease presents to the ER with muscle cramps. Laboratory studies reveal potassium of 7.8 mEq/L. EKG reveals peaked T waves. Which of the following is the best initial med to give?

A. Insulin
B. Albuterol
C. Furosemide
D. Sodium bicarbonate
E. Calcium gluconate

Click here to see the answer

The answer is E. Calcium gluconate

Hyperkalemia is caused by many things: iatrogenic (ACEI/ARBs), cellular destruction (hemolysis, tumor lysis syndrome, burns), renal failure, adrenal insufficiency, etc. Symptoms are nonspecific and include muscle weakness, N/V, decreased DTRs, etc. Classically on EKG, you may see peaked T waves (there are many other findings though…such as QRS widening, PR interval prolongation, sine-wave, etc). Management for severe hyperkalemia includes calcium gluconate (stabilizes the resting membrane potential of the myocardial membrane), shifting potassium intracellularly (via beta-agonists, sodium bicarbonate, insulin), and removing potassium (via diuretics, kayexalate, or hemodialysis).

VIEW BLUEPRINT LESSON

Smarty PANCE Content Blueprint Review:

Covered under ⇒ PANCE Blueprint Renal SystemFluid and Electrolyte DisordersHyperkalemia/hypokalemia

Also covered as part of the Emergency Medicine EOR and General Surgery PAEA EOR topic list

4. A 28-year-old male with sickle cell disease presents to the ER with chest pain, dyspnea, and a cough for the past day. Vitals are remarkable for SpO2 91% and T 102.2F. A CXR reveals bilateral pulmonary infiltrates. On physical exam the patient is alert, speaking in full sentences, and breathing without accessory muscle use. Which of the following is the most appropriate management option for this patient?

A. Surgical consultation, antibiotics, plasmapheresis, IVIG
B. Pain control, hydration, blood transfusions, oxygen, antibiotics, VTE prophylaxis
C. Immediate endotracheal intubation, aggressive intravenous fluids, antibiotics, blood transfusions
D. Blood transfusions and oxygen only
E. Empiric antibiotics and oxygen only

Click here to see the answer

The answer is B. Pain control, hydration, blood transfusions, oxygen, antibiotics, VTE prophylaxis

The patient has acute chest syndrome (ACS), which is defined as a new radiodensity on CXR with fever and/or pulmonary symptoms. It occurs due to vaso-occlusion in the pulmonary microvasculature, subsequently leading to deoxygenation of hemoglobin and sickling of RBCs. ACS is the main cause of death in patients with sickle cell disease (SCD). About 1 in 2 patients with SCD get ACS.

Patients may present with chest pain, extremity pain, and shortness of breath. Causes of ACS include fat emboli, infection, asthma, oversedation, and post-op issues. Treatment typically includes fluidsoxygenpain controlblood transfusionsantibiotics, and VTE prophylaxis.

VIEW BLUEPRINT LESSON

Smarty PANCE Content Blueprint Review:

Covered under ⇒ PANCE Blueprint Hematology ⇒ Hemoglobinopathies ⇒ Sickle cell anemia

Also covered as part of the Internal Medicine EOR and Emergency Medicine PAEA EOR topic list

5. Which of the following is the most common cause of bacterial sialadenitis?

A. Bacteroides
B. Staphylococcus aureus
C. Streptococcus pneumoniae
D. Escherichia coli
E. Streptococcus viridans

Click here to see the answer

The answer is B. Staphylococcus aureus

Bacterial sialadenitis is defined by inflammation of a salivary gland due to a bacterial infection. It is most commonly caused by Staphylococcus aureusUsually, bacterial sialadenitis happens in the setting of a salivary gland stone (reduced salivary flow leads to bacteria building up in the mouth). Other risk factors include bad oral hygiene and old age. The most common gland affected is the parotid gland.

Clinical features include acute onset of fever, chills, and swelling/tenderness of the affected gland (possible to see purulent drainage). The treatment is broad-spectrum antibiotics: IV ampicillin-sulbactam. Surgical drainage is needed if an abscess develo

VIEW BLUEPRINT LESSON

Smarty PANCE Content Blueprint Review:

Covered under ⇒ PANCE Blueprint EENT ⇒ Salivary disorders ⇒ Sialadenitis

Also covered as part of the Family Medicine PAEA EOR topic list

6. A 52-year-old female presents to the clinic complaining of chronic bone pain, constipation, and fatigue. Her last lab results reveal a decline in the glomerular filtration rate. Which of the following is the most likely diagnosis?

A. Acute myelogenous leukemia
B. Bronchogenic carcinoma
C. Multiple myeloma
D. Polymyalgia rheumatica
E. Colorectal cancer

Click here to see the answer

The answer is C. Multiple myeloma

The patient has multiple myeloma, which is a clonal proliferation of abnormal plasma cells. It can present with “BREAK” symptoms: Bone pain, Recurrent infections, Elevated calcium, Anemia, and Kidney failure. A serum and urine protein electrophoresis will reveal a monoclonal spike. Other diagnostic studies include a CT or MRI (reveals lytic bone lesions), peripheral blood smear (normocytic anemia in rouleaux formation), and urinalysis (Bence Jones proteins aka free light chains).

The definitive diagnosis is a bone marrow biopsy. Treatment is high-dose chemotherapy with autologous hematopoietic cell transplantation.

VIEW BLUEPRINT LESSON

Smarty PANCE Content Blueprint Review:

Covered under ⇒ PANCE Blueprint HematologyNeoplasms, premalignancies, and malignanciesMultiple myeloma

Also covered as part of the Internal Medicine PAEA EOR topic list

7. A 56-year-old post-menopausal G0P0 female presents to the clinic complaining of abnormal uterine bleeding and weight loss for the past 5 months. Physical exam is unremarkable. Which of the following is not a risk factor for this patient’s likely diagnosis?

A. Nulliparity
B. Tamoxifen
C. Chronic anovulation
D. Obesity
E. Late menarche

Click here to see the answer

The answer is E. Late menarche

The patient has endometrial cancer, the most commonly encountered gynecologic cancer. The cardinal symptom is abnormal uterine bleeding. Other clinical features include pelvic pain, pelvic masses, and weight loss.

Risk factors include conditions or medications that lead to chronically increased estrogen levels, such as nulliparity, tamoxifen, anovulation, and obesity. Early menarche and late menopause would be risk factors, not late menarche.

VIEW BLUEPRINT LESSON

Smarty PANCE Content Blueprint Review:

Covered under ⇒ PANCE Blueprint Reproductive SystemNeoplasms of the breast and reproductive tractEndometrial cancer

Also covered as part of the Women’s Health PAEA EOR topic list

8. Which of the following are the two most common etiologies of peptic ulcer disease?

A. H. pylori and NSAIDs
B. Spicy foods and acute stress
C. Chemical ingestion and GERD
D. Smoking and alcohol use
E. Chronic stress and radiation

Click here to see the answer

The answer is A. H. pylori and NSAIDs

Peptic ulcer disease (PUD) is a common GI disease defined by a defect in the mucosal lining of the stomach or duodenum. The two most common causes are H. pylori and NSAIDs. Other more rare culprits include Zollinger-Ellison Syndrome, cancer, stress, and radiation. Surprisingly, patients are asymptomatic about 70% of the time. Common symptoms are epigastric abdominal paindyspepsia, and bloating. Overall, an upper endoscopy is the best diagnostic test. A biopsy should be done for ulcers with malignant features on all gastric ulcers.

The preferred treatment is PPIs. If H. pylori is the cause, an antibiotic regimen is warranted (e.g., amoxicillin, clarithromycin, and a PPI).

VIEW BLUEPRINT LESSON

Smarty PANCE Content Blueprint Review:

Covered under ⇒ PANCE Blueprint GI and Nutrition ⇒ Gastric Disorders ⇒ Peptic ulcer disease

Also covered as part of the Internal Medicine EOREmergency Medicine EORFamily Medicine EOR, and General Surgery EOR topic list

9. A 72-year-old male smoker with a history of cancer presents to the clinic for an annual physical. Vitals are unremarkable. On physical exam, you notice erythema along the course of a superficial vein on his left leg. The area is mildly tender to palpation. The left leg is also larger than the other leg. Which of the following is the next best step?

A. Treat with NSAIDs and warm compresses
B. Order a duplex ultrasound
C. Initial anticoagulation immediately
D. Admit to the hospital
E. Reassurance and send home

Click here to see the answer

The answer is B. Order a duplex ultrasound

The patient has superficial thrombophlebitis, which is inflammation involving a superficial vein along with the presence of a superficial clot. Risk factors include varicose veins, estrogen, prior DVT, cancer, and hypercoagulable states. Clinical features include tendernessindurationpain, and erythematous skin over a superficial vein.

In many cases, duplex ultrasound is needed to rule out a concurrent DVT (studies show that up to 25-50% of patients with superficial thrombophlebitis have a concurrent DVT). Treatment includes NSAIDs, warm/cold compressesextremity elevation, and compression therapy. The decision to initiate anticoagulation for superficial thrombophlebitis depends on many factors.

VIEW BLUEPRINT LESSON

Smarty PANCE Content Blueprint Review:

Covered under ⇒ PANCE Blueprint Cardiology ⇒ Vascular Disease ⇒ Phlebitis/thrombophlebitis

10. A 63-year-old male with a history of hepatitis C presents with a pruritic rash located on the flexor surfaces of his wrist. On physical exam, you observe purplish papules with flat-tops. Which of the following is the most likely diagnosis?

A. Atopic dermatitis
B. Psoriasis
C. Lichen planus
D. Pemphigus vulgaris
E. Scabies

Click here to see the answer

The answer is C. Lichen planus

Lichen planus is a skin disorder characterized by a rash associated with the 5 Pspolygonal, pruritic, papules, plaques, and purple. It commonly affects the wrists and ankles. Lichen planus can also cause the classic Wickham’s striae, which are white-lacelike lines (usually in the oral mucosa). A risk factor is hepatitis C.

skin biopsy can help confirm the diagnosis. Treatment includes high potency topical corticosteroids. If a patient has an extensive form of lichen planus, then phototherapy, acitretin, or systemic steroids may help.

VIEW BLUEPRINT LESSON

Smarty PANCE Content Blueprint Review:

Covered under ⇒ PANCE Blueprint Dermatology ⇒ Papulosquamous Disorders ⇒ Lichen planus

Also covered as part of the Family Medicine EOR and Pediatric EOR topic list

Looking for all the podcast episodes?

This FREE podcast series is limited to every other episode, you can download and enjoy the complete audio series by becoming a Smarty PANCE member.

I will be releasing new episodes every few weeks. Smarty PANCE is now discounted, so sign up now before it’s too late!

Additional resources and links from the show

This Podcast is available on iOS and Android

Download the Interactive Content Blueprint Checklist

Interactive Content Blueprint for the 2022-2023 PANCE

Follow this link to download your FREE copy of the PANCE/PANRE/EOR™ Content Blueprint Checklists

Print it up and start crossing out the topics you understand, marking the ones you don’t, and making notes of key terms you should remember. The PDF version is interactive and linked directly to the individual lessons on Smarty PANCE.

Smarty PANCE is not sponsored or endorsed by, or affiliated with, the NCCPA or the Physician Assistant Education Association (PAEA). All trademarks are the property of their respective owners.

The post Podcast Episode 99: Ten PANCE, PANRE, and Rotation Review Questions appeared first on The Audio PANCE and PANRE.

]]>
Welcome to episode 99 of the Audio PANCE and PANRE Physician Assistant/Associate Board Review Podcast. Join me as I cover ten PANCE, PANRE, and EOR™ review questions from the Smarty PANCE Instagram/Facebook page and the smartypance. Welcome to episode 99 of the Audio PANCE and PANRE Physician Assistant/Associate Board Review Podcast.
Join me as I cover ten PANCE, PANRE, and EOR™ review questions from the Smarty PANCE Instagram/Facebook page and the smartypance.com board review website.
Special from today’s episode:

* Join the Smarty PANCE Member’s Community
* Check out our all-new End of Curriculum™ (EOC) Exam Course (still in development)
* Follow Smarty PANCE and The Daily PANCE Blueprint on Instagram
* Follow Smarty PANCE and The Daily PANCE Blueprint on Facebook

Below you will find an interactive exam to complement today’s podcast.
The Audio PANCE/PANRE and EOR PA Board Review Podcast
I hope you enjoy this free audio component to the examination portion of this site. The full board review course includes over 2,000 interactive board review questions and is available to all members of Smarty PANCE.

* You can download and listen to past FREE episodes here, on iTunes, Spotify, Google Podcasts, Stitcher, and most podcasting apps.
* You can listen to the latest episode, take an interactive quiz, and download more resources below.

Listen Carefully Then Take the Practice Exam
If you can’t see the audio player, click here to listen to the full episode.
Podcast Episode 99: Ten PANCE/PANRE and EOR Topic Blueprint Questions
1. A 42-year-old male on lithium presents with polyuria, nocturia, and polydipsia. Laboratory findings are remarkable for slightly elevated sodium. Which of the following is the most likely diagnosis?
A. Neurogenic diabetes insipidus
B. Nephrogenic diabetes insipidus
C. Type 2 diabetes mellitus
D. SIADH
E. Adrenal insufficiency
2. A 50-year-old female presents with poor appetite, low energy, poor concentration, and feelings of hopelessness on most days for the past 3 years. She denies suicidal ideation. She has never had a past manic or hypomanic episode. Which of the following is the best treatment option?
A. Haloperidol
B. Fluoxetine
C. Lorazepam
D. Amitriptyline
E. Risperidone
3. An 85-year-old male with a history of chronic kidney disease presents to the ER with muscle cramps. Laboratory studies reveal potassium of 7.8 mEq/L. EKG reveals peaked T waves. Which of the following is the best initial med to give?
A. Insulin
B. Albuterol
C. Furosemide
D. Sodium bicarbonate
E. Calcium gluconate
4. A 28-year-old male with sickle cell disease presents to the ER with chest pain, dyspnea, and a cough for the past day. Vitals are remarkable for SpO2 91% and T 102.2F.]]>
The Physician Assistant Life | Smarty PANCE full 25:48 505
Podcast Episode 98: Ten PANCE, PANRE, and Rotation Review Questions http://podcast.thepalife.com/podcast-episode-98/ Tue, 24 May 2022 15:07:00 +0000 http://podcast.thepalife.com/?p=495 http://podcast.thepalife.com/podcast-episode-98/#respond http://podcast.thepalife.com/podcast-episode-98/feed/ 0 <p>Welcome to episode 98 of the Audio PANCE and PANRE Physician Assistant/Associate Board Review Podcast. Join me as I cover ten PANCE, PANRE, and EOR™ review questions from the Smarty PANCE Instagram/Facebook page and the smartypance.com board review website. Special from today’s episode: Join the Smarty PANCE Member’s Community Check out our all-new End of […]</p> <p>The post <a rel="nofollow" href="http://podcast.thepalife.com/podcast-episode-98/">Podcast Episode 98: Ten PANCE, PANRE, and Rotation Review Questions</a> appeared first on <a rel="nofollow" href="http://podcast.thepalife.com">The Audio PANCE and PANRE</a>.</p>
The Audio PANCE and PANRE Episode 98 Ten PANCE, PANRE, and Rotation Review Questions

Welcome to episode 98 of the Audio PANCE and PANRE Physician Assistant/Associate Board Review Podcast.

Join me as I cover ten PANCE, PANRE, and EOR™ review questions from the Smarty PANCE Instagram/Facebook page and the smartypance.com board review website.

Special from today’s episode:

Below you will find an interactive exam to complement today’s podcast.

The Audio PANCE/PANRE and EOR PA Board Review Podcast

I hope you enjoy this free audio component to the examination portion of this site. The full board review course includes over 2,000 interactive board review questions and is available to all members of Smarty PANCE.

Listen Carefully Then Take the Practice Exam

If you can’t see the audio player, click here to listen to the full episode.

Podcast Episode 98: Ten PANCE/PANRE and EOR Topic Blueprint Questions

1. An 81-year-old female presents to the ER with acute onset of low back pain. She complains of bowel dysfunction and loss of sensation over her inner thighs. Physical exam reveals decreased lower extremity reflexes. Which of the following is the best diagnostic test for the likely diagnosis?

A. Lumbar radiographs
B. MRI of the lumbosacral spine
C. CSF fluid analysis
D. Scoliosis studies
E. None of the above

Click here to see the answer

The answer is B. MRI of the lumbosacral spine

Cauda equina syndrome is a surgical emergency caused by severe stenosis in the lumbar spine (often due to acute disc herniation).

Clinical manifestations include bowel/bladder dysfunction, decreased lower extremity reflexes, sciatica, saddle anesthesia (loss of sensation over the perineum, buttock, medial aspect of thighs), and decreased anal sphincter tone. Symptoms can develop acutely or chronically.

An emergent MRI of the lumbosacral spine is the preferred diagnostic test. Surgery is the preferred treatment.

Click here to view the Smarty PANCE lesson

Smarty PANCE Content Blueprint Review:

Covered under ⇒ PANCE Blueprint MusculoskeletalSpinal DisordersCauda equina syndrome

Also covered as part of the Emergency Medicine PAEA EOR topic list

2. Which of the following is the most common cause of Cushing syndrome?

A. Iatrogenic
B. Bronchogenic carcinoma
C. Pituitary adenoma
D. Adrenal adenoma
E. Obesity

Click here to see the answer

The answer is A. Iatrogenic

Cushing syndrome is a condition defined by too much cortisol. The most common cause of Cushing syndrome is exogenous steroid therapy (i.e., medical providers prescribing steroids). Other causes include a pituitary ACTH-secreting adenoma, adrenal tumors, and ectopic ACTH production from neoplasms. Classic clinical features include buffalo hump, moon-facies, purple striae, easy bruising, weight gain, depression, and weakness.

Laboratory tests will be remarkable for elevated cortisol. Late night-salivary cortisol or 24-hour urinary free cortisol is typically the initial screening test. A low-dose dexamethasone test can be done. Referral to an endocrinologist is usually done at this point (or even prior).

Click here to view the Smarty PANCE lesson

Smarty PANCE Content Blueprint Review:

Covered under ⇒ PANCE Blueprint EndocrinologyAdrenal DisordersCushing’s syndrome

Also covered as part of the Internal Medicine EOR, Family Medicine EOR, and Emergency Medicine PAEA EOR topic list

3. A 73-year-old female presents with facial flushing, wheezing, and watery diarrhea for a few months. Her symptoms are worsened by certain foods. Which of the following is the best initial diagnostic study to order for the suspected diagnosis?

A. 5-HIAA in urine
B. Serum ACE levels
C. Chest radiograph
D. Upper endoscopy
E. Bronchoscopy

Click here to see the answer

The answer is A. 5-HIAA in urine

Carcinoid tumors are uncommon tumors that originate from neuroendocrine cells and secrete serotonin. The most common site is the appendix, but they can also be found in the lungs, kidney, etc.

Carcinoid syndromes develop 10% of the time; clinical features include flushing, sweating, wheezing, and watery diarrhea. These symptoms are due to the release of serotonin and can be precipitated by foods high in tyramine or ethanol.

The best initial diagnostic study is a 24-hour measurement of urinary excretion of 5-HIAA (Note: 5-HIAA is a degradation product from 5HT). Surgical resection is the definitive treatment of choice.

Click here to view the Smarty PANCE lesson

Smarty PANCE Content Blueprint Review:

Covered under ⇒ PANCE Blueprint PulmonaryPulmonary NeoplasmsCarcinoid tumors

Also covered as part of the Internal Medicine PAEA EOR topic list

4. Which of the following is the most common cause of Mallory-Weiss Syndrome?

A. Iatrogenic
B. Vomiting
C. Abdominal trauma
D. Hiatal hernia
E. Corrosive ingestion

Click here to see the answer

The answer is B. Vomiting

Mallory-Weiss Syndrome (MWS) is defined by a mucosal tear at or below the gastroesophageal junction. MWS is one of the most common causes of an upper GI bleed. It is most commonly due to forceful vomiting. It is classically associated with alcoholism (binge drinking), but really anything that can cause vomiting can lead to a tear. Other causes include iatrogenic, trauma, hiatal hernia, and corrosive ingestion. Diagnosis is made via upper endoscopy. Intervention is rarely necessary, but if needed, the patient can undergo surgery or embolization. Do not confuse this with Boerhaave syndrome, which is an actual perforation of the esophagus!

Click here to view the Smarty PANCE lesson

Smarty PANCE Content Blueprint Review:

Covered under ⇒ PANCE Blueprint GI and NutritionEsophageal DisordersMallory Weiss tear

Also covered as part of the Internal Medicine EOR and Emergency Medicine PAEA EOR topic list

5. A 51-year-old male presents to the clinic complaining of “band-like pressure” around his head. He has been more stressed lately. Which of the following is the most likely diagnosis?

A. Cluster headache
B. Migraine with aura
C. Sinus headache
D. Tension headache
E. Medication overuse headache

Click here to see the answer

The answer is D. Tension headache

A tension-type headache (TTH) is the most common headache. It usually presents with steady, aching, “band-like” pain that circles the entire head. There may also be tightness in the posterior neck muscles. Precipitants include anxiety, depression, and stress.

You should treat any underlying depression and/or anxiety. NSAIDs and acetaminophen are the first-line abortive treatment options for mild-moderate TTHs. For long-term prophylactic treatment, amitriptyline is preferred.

Click here to view the Smarty PANCE lesson

Smarty PANCE Content Blueprint Review:

Covered under ⇒ PANCE Blueprint NeurologyHeadachesTension headache

Also covered as part of the Internal Medicine PAEA EOR topic list

6. Which of the following physical exam findings would you expect in a patient with mitral regurgitation?

A. Holosystolic blowing murmur best heard at the apex with radiation to the axilla
B. Diastolic blowing murmur at the left upper sternal border
C. Systolic ejection crescendo-decrescendo murmur at upper right sternal border
D. Mid-systolic harsh murmur best heard at the left upper sternal border
E. None of the above

Click here to see the answer

The answer is A. Holosystolic blowing murmur best heard at the apex with radiation to the axilla

Mitral regurgitation (MR) is characterized by a blowing, holosystolic murmur at the apex with radiation to the axilla. It can be acute or chronic. Acute causes include endocarditis and papillary muscle rupture; chronic causes include mitral valve prolapse (MVP), rheumatic fever, cardiomyopathy. The most common cause of MR is MVP.

Remember, regurgitation murmurs will often have blowing sounds, while stenotic murmurs are harsh and rumbling. Diagnosis should be made with an echocardiogram. Treatment (repair vs. replace vs. medical therapy) depends on many factors.

Click here to view the Smarty PANCE lesson

Smarty PANCE Content Blueprint Review:

Covered under ⇒ PANCE Blueprint CardiologyValvular DisordersMitral regurgitation

Also covered as part of the Emergency Medicine EOR topic list

7. Which of the following is not an expected clinical manifestation of polyarteritis nodosa?

A. Kidney failure
B. Livedo reticularis
C. pANCA positive
D. Pulmonary fibrosis
E. Hypertension

Click here to see the answer

The answer is D. Pulmonary fibrosis

Polyarteritis nodosa is a systemic vasculitis of medium-sized vessels (specifically involving the nervous system and GI tract) that leads to thrombi and microaneurysms. It is associated with hepatitis B, HIV, and drug reactions. Clinical features include fever, hypertension, livedo reticularis, abdominal pain, and arthralgias. Remember patients may be positive for pANCA, but are ANCA negative.

A definitive diagnosis is made via biopsy of involved tissue (or mesenteric angiography). What sets polyarteritis nodosa apart from other vasculitides is that it spares the lungs. Treatment includes high-dose steroids. The prognosis is not really good.

Click here to view the Smarty PANCE lesson

Smarty PANCE Content Blueprint Review:

Covered under ⇒ PANCE Blueprint MusculoskeletalRheumatologic DisordersPolymyalgia rheumatica

Also covered as part of the Internal Medicine PAEA EOR topic list

8. An 8-year-old female presents to the clinic with her dad. She is complaining of left ear pain and pruritus for the past few days. The physical exam is remarkable for drainage and pain on tragal pressure. The tympanic membrane can’t be visualized. Which of the following is the best treatment option?

A. Oral amoxicillin
B. Neomycin/polymyxin B/hydrocortisone drops
C. Oral cefepime
D. Oral steroids
E. Ciprofloxacin/dexamethasone drops

Click here to see the answer

The answer is E. Ciprofloxacin/dexamethasone drops

The patient has otitis externa (aka swimmer’s ear), which is inflammation of the external auditory canal. The most common bacterial cause is Pseudomonas aeruginosa (~40%). Risk factors include water exposure, trauma, ear devices, and dermatologic conditions. Clinical features include ear pain, pruritus, auricular discharge, pain/tenderness when pressure is applied to the tragus, and hearing loss.

Treatment is topical ear antibiotic/steroid drops. Medication options include neomycin/polymyxin B /hydrocortisone and ciprofloxacin/dexamethasone. **Remember that aminoglycosides are ototoxic and should be avoided when the tympanic membrane can’t be visualized!**

Click here to view the Smarty PANCE lesson

Smarty PANCE Content Blueprint Review:

Covered under ⇒ PANCE Blueprint EENTEar DisordersExternal earOtitis externa

Also covered as part of the Pediatric EOR, Family Medicine EOR, and Emergency Medicine PAEA EOR topic list

9. Which of the following correctly depicts the typical clinical features of Parkinson’s disease?

A. Pill-rolling tremor, bradykinesia, rigidity, masked facies
B. Loss of declarative episodic memory, apraxia, olfactory dysfunction
C. Visual hallucinations, REM sleep behavior disorder, gait issues
D. Hyperorality, apathy, socially inappropriate behavior
E. None of the above

Click here to see the answer

The answer is A. Pill-rolling tremor, bradykinesia, rigidity, masked facies

Parkinson’s disease is a neurodegenerative disease. Its 3 cardinal features are tremor (“pill-rolling” at rest), bradykinesia, and rigidity. Tremor is the presenting feature in most patients. Other clinical features include hypomimia (masked facies), speech impairment, mood disorders (depression or anxiety), dysphagia, shuffling gait, stooped posture, sleep issues, cognitive dysfunction, and autonomic dysfunction.

It is mainly a clinical diagnosis. When patients respond to dopaminergic drugs, this supports the diagnosis. The mainstay of treatment is levodopa or dopamine agonists (pramipexole, ropinirole). For mild disease, monoamine oxidase type B inhibitors or amantadine can be given.

Click here to view the Smarty PANCE lesson

Smarty PANCE Content Blueprint Review:

Covered under ⇒ PANCE Blueprint NeurologyMovement DisordersParkinson’s disease

Also covered as part of the Internal Medicine EOR and Family Medicine PAEA EOR topic list

10. A 72-year-old male presents with an abnormal change in bowel habits and fatigue. The physical exam is remarkable for pale conjunctiva and a palpable abdominal mass. His hemoglobin is 10 mg/dL. Which of the following is likely to be elevated?

A. Alpha-fetoprotein
B. CEA
C. CA-125
D. CA 19-9
E. AAT1

Click here to see the answer

The answer is B. CEA

The patient has colorectal cancer (CRC), which is the third most common cause of cancer death in the U.S. The USPSTF suggests screening at age 45 for individuals with an average risk. Patients may present with alarming “red flag” symptoms, a bowel obstruction, or CRC may just be found via routine screening. The most common symptom is a change in bowel habits (~70%). Other symptoms include rectal bleeding, iron deficiency anemia, and pain.

Initial diagnostic tests include colonoscopy, CBC, fecal occult blood testing, and tumor markers like carcinoembryonic antigen (CEA). CEA is more often used for surveillance and not screening though (since its sensitivity for CRC is only 46%).

Click here to view the Smarty PANCE lesson

Smarty PANCE Content Blueprint Review:

Covered under ⇒ PANCE Blueprint GI and NutritionGastrointestinal System NeoplasmsColon cancer

Also covered as part of the Internal Medicine EOR, Family Medicine EOR, General Surgery PAEA EOR topic list

Looking for all the podcast episodes?

This FREE podcast series is limited to every other episode, you can download and enjoy the complete audio series by becoming a Smarty PANCE member.

I will be releasing new episodes every few weeks. Smarty PANCE is now discounted, so sign up now before it’s too late!

Additional resources and links from the show

This Podcast is available on iOS and Android

Download the Interactive Content Blueprint Checklist

Interactive Content Blueprint for the 2022-2023 PANCE

Follow this link to download your FREE copy of the PANCE/PANRE/EOR™ Content Blueprint Checklists

Print it up and start crossing out the topics you understand, marking the ones you don’t, and making notes of key terms you should remember. The PDF version is interactive and linked directly to the individual lessons on Smarty PANCE.

Smarty PANCE is not sponsored or endorsed by, or affiliated with, the NCCPA or the Physician Assistant Education Association (PAEA). All trademarks are the property of their respective owners.

The post Podcast Episode 98: Ten PANCE, PANRE, and Rotation Review Questions appeared first on The Audio PANCE and PANRE.

]]>
Welcome to episode 98 of the Audio PANCE and PANRE Physician Assistant/Associate Board Review Podcast. Join me as I cover ten PANCE, PANRE, and EOR™ review questions from the Smarty PANCE Instagram/Facebook page and the smartypance.



Welcome to episode 98 of the Audio PANCE and PANRE Physician Assistant/Associate Board Review Podcast.



Join me as I cover ten PANCE, PANRE, and EOR™ review questions from the Smarty PANCE Instagram/Facebook page and the smartypance.com board review website.



Special from today’s episode:



* Join the Smarty PANCE Member’s Community* Check out our all-new End of Curriculum™ (EOC) Exam Course (still in development)* Follow Smarty PANCE and The Daily PANCE Blueprint on Instagram* Follow Smarty PANCE and The Daily PANCE Blueprint on Facebook



Below you will find an interactive exam to complement today’s podcast.



The Audio PANCE/PANRE and EOR PA Board Review Podcast



I hope you enjoy this free audio component to the examination portion of this site. The full board review course includes over 2,000 interactive board review questions and is available to all members of Smarty PANCE.



* You can download and listen to past FREE episodes here, on iTunes, Spotify, Google Podcasts, Stitcher, and most podcasting apps.* You can listen to the latest episode, take an interactive quiz, and download more resources below.



Listen Carefully Then Take the Practice Exam





If you can’t see the audio player, click here to listen to the full episode.



Podcast Episode 98: Ten PANCE/PANRE and EOR Topic Blueprint Questions



1. An 81-year-old female presents to the ER with acute onset of low back pain. She complains of bowel dysfunction and loss of sensation over her inner thighs. Physical exam reveals decreased lower extremity reflexes. Which of the following is the best diagnostic test for the likely diagnosis?



A. Lumbar radiographsB. MRI of the lumbosacral spineC. CSF fluid analysisD. Scoliosis studiesE. None of the above





2. Which of the following is the most common cause of Cushing syndrome?



A. IatrogenicB. Bronchogenic carcinomaC. Pituitary adenomaD. Adrenal adenomaE. Obesity





3. A 73-year-old female presents with facial flushing, wheezing, and watery diarrhea for a few months. Her symptoms are worsened by certain foods. Which of the following is the best initial diagnostic study to order for the suspected diagnosis?



A. 5-HIAA in urineB. Serum ACE levelsC. Chest radiographD. Upper endoscopyE. Bronchoscopy
]]>
The Physician Assistant Life | Smarty PANCE full 17:16 495
Podcast Episode 97: Murmurs Made Incredibly Easy – Ten PANCE Murmur Questions http://podcast.thepalife.com/podcast-episode-97-ten-murmur-questions/ Wed, 27 Apr 2022 11:00:00 +0000 http://podcast.thepalife.com/?p=484 http://podcast.thepalife.com/podcast-episode-97-ten-murmur-questions/#respond http://podcast.thepalife.com/podcast-episode-97-ten-murmur-questions/feed/ 0 <p>Welcome to episode 97 of the Audio PANCE and PANRE PA board review podcast. Today is a bonus episode rounding out this fabulous five-part podcast series with Joe Gilboy PA-C, all about cardiac murmurs. In this week’s episode, we continue our discussion of cardiac murmurs with ten PANCE and PANRE murmur questions. We’ll cover the […]</p> <p>The post <a rel="nofollow" href="http://podcast.thepalife.com/podcast-episode-97-ten-murmur-questions/">Podcast Episode 97: Murmurs Made Incredibly Easy – Ten PANCE Murmur Questions</a> appeared first on <a rel="nofollow" href="http://podcast.thepalife.com">The Audio PANCE and PANRE</a>.</p>

Welcome to episode 97 of the Audio PANCE and PANRE PA board review podcast.

Today is a bonus episode rounding out this fabulous five-part podcast series with Joe Gilboy PA-C, all about cardiac murmurs. In this week’s episode, we continue our discussion of cardiac murmurs with ten PANCE and PANRE murmur questions.

We’ll cover the ins and outs of all the NCCPA content blueprint valvular disorders and learn how to identify and differentiate them from one another.

If you haven’t already, make sure to listen to our previous podcast episodes where we covered tricuspid stenosis, aortic valve murmurs, mitral valve murmurs, pulmonic valve murmurs, and HOCM and MVP.

Podcast Episode 97: Murmurs Made Incredibly Easy – 10 PANCE and PANRE Murmur Questions

Below is a transcription of this podcast episode edited for clarity.

[00:00:00] Welcome back, everybody. This is Joe Gilboy. I work with Stephen Pasquini over there at Smarty PANCE. And today is the most dreaded podcast of all because you know what I’m going to do. That’s right. I’m going to ask cardiac murmur questions.

And I know it’s the most dreaded thing in the world. Let’s do this together because what I want to try to do here is make sense of it all. So, let’s just kind of recap some basic rules before we start going down this thing. You know, the previous lectures, what have I been barking about? What’s Joe been saying? First – inspiration, right? Expiration left.

[00:00:40] Inspiration, right? Expiration left. So, with inspiration, the right-sided murmurs sound louder. So that’s the tricuspid and the pulmonic valve regardless of if it’s stenosis or regurgitation. Expiration – left. What am I saying? Everything on the left side sounds louder with expiration, whether it’s aortic or mitral – your call.

[00:01:01] It can be stenotic or regurgitation, it doesn’t matter, but it’ll sound louder. What is squatting? Squatting is a party. So, what are you doing? You’re bringing it all the blood flow back home. And so, if I bring all the blood flow back home, just from a laminar flow physics point of view, I bring more blood flow back home to the heart, more blood flow over a valve specifically with a diseased disease valve, it sounds louder.

[00:01:29] So, right off the bat, squatting will do what? It’s going to make all my murmurs sound louder. That’s the whole point to decrease venous return. In other words, take blood flow away from the heart. What do I do? Well, those are going to be Valsalva and standing. So, what am I doing when I do Valsalva and standing?

[00:01:56] Taking blood flow away from the heart. So, what’s going to happen to all my murmurs when I stand and perform Valsalva? It takes blood flow away from the heart. Exactly. And then hand grip. What did handgrip do? So, in school, what’d you learn about hand grip? Well, it increases afterload, right? So, what they’re really saying to you is this, and this is how I want you to visualize it.

[00:02:18] When I do hand grip, what I’m really doing is this. You’re right. I’m increasing the afterload. But you must stop and think this one out for a second, folks. What’s really in the heart? Which valve must fight afterload?

[00:02:39] Say you increase the peripheral vascular resistance for whatever reason. So, you increase the afterload, who fights it? And your answer will be… oh, wait for a second, Joe, that’s the aortic valve in the left ventricle. Exactly. So, when I perform handgrip, who am I really challenging? Oh, I’m challenging the aortic valve and everything behind it.

[00:03:00] Exactly. So that’s the point I need inside your head? Then remember our last podcast was about our low-volume lovers. Who were our low-volume lovers? Who were the special needs murmurs that really like low volume? They’re like, hey, I like low volume, not high volume. I like low volume. Who was that?

[00:03:19] That was HOCM and mitral valve prolapse. What did they both like? They both like low volume. So, they’re the opposite of all my regular murmurs. They go in the opposite direction. All right, everybody got that? I am not going to give you a list of answers. So, what I’m going to do is I want you to listen to what I’m going to say. I’m going to say the test question twice.

[00:03:47] And then I want you to pause the podcast. Just put it on pause and think about your answer before I explain it. All right. Is everybody with me? Because that’s the best thing to do. All right, here we go.

Murmur Question #1:

[00:04:11] You have a 76-year-old gentleman that presents to your emergency room and comes in complaining of shortness of breath and chest pain on physical exam. You notice an upper sternal murmur that sounds louder with squatting and goes away with hand grip—one more time. Upper sternal border, sounds louder with squatting, and goes away with hand grip. Who am I?

[00:04:45] So upper sternal border. This will be my aortic and pulmonic area. Well, it goes away with handgrip. So, who’s that going to be? So, what happens with hand grip? I increase my afterload. Which valve fights the afterload? The aortic valve. It’s not pulmonic because the pulmonic valve is not fighting the afterload; it is fighting the lung. So, this has to be aortic stenosis, which makes sense for the syncope and the shortness of breath. So aortic stenosis is going to do what? Well, squatting, that’s a party. Of course, it will sound louder. We could actually throw expiration in here as well, couldn’t we? We sure could, but it’s the handgrip. That differentiates it, doesn’t it? Because the handgrip made the murmur go away. And what did the handgrip do? I keep telling you I want you to view handgrip like sitting on your aortic valve, and you can barely open up squeak, squeak, squeak, squeak.

[00:05:47] You can barely open up. Now imagine sitting on that valve. Can it open up now? And you’re like, no, it goes away. Exactly. And that’s the point I’m trying to make. The answer is aortic stenosis.

Murmur Question # 2

[00:06:21] I have a 45-year-old gentleman who comes in with shortness of breath and chest pain. The murmur sounds louder with inspiration. It’s located at the upper sternal border. And I am also noticing that the patient has right ventricular hypertrophy and right bundle branch block on EKG. Who am I?

[00:06:44] Upper sternal border. Well, who are we talking about? The aortic valve or the pulmonic valve. Inspiration → right and expiration → left. So, this murmur sounds louder with inspiration, so I know that this is on the right side. Then it’s got to be the pulmonic valve. And now they’re telling me I’ve got right ventricular hypertrophy and right bundle branch block. Then blood must be backing up into the right ventricle.

[00:07:10] In other words, something must be backing up, which means the valve must be stenotic. Oh, I see your point. It’s stenotic, and the right ventricle is going to hypertrophy. Remember if you take right ventricular hypertrophy and let it go unopposed. You stretch, and you stretch, and you stretch, and you stretch, and you dilate, and you stretch, and you dilate and stretch that wall.

[00:07:28] What are you going to get? The wires are going to get busted. That’s right bundle branch block. For those of you thinking outside the box, and you’re telling me, Joe, if I dilate my left ventricle and just stretch it and stretch and a bust, the wires, I get left bundle branch? Exactly. No, that’s exactly what you’re going to get. So the answer to the question is pulmonic stenosis.

Murmur Question # 3

[00:07:45] You have a young 32-year-old female who recently immigrated to the United States from Indonesia and is currently working and has now been complaining of increasing shortness of breath at work. On exam, you hear a murmur at the left sternal border. It sounds louder with expiration and appears to be diastolic in nature. Who am I?

[00:08:38] So again, left sternal border. So that’s just about everybody but the pulmonic valve. The murmur sounds louder with expiration, so you know it will be on the left (louder with inspiration → right and louder with expiration → left). So, based on these parameters, I know I’m dealing with either the aortic valve or the mitral valve, and I am a diastolic murmur. So now you have to stop and think. So, this seems to be in the neighborhood of the mitral area, and it’s a diastolic murmur.

[00:09:00] So it can’t be mitral regurgitation. It has mitral stenosis, and that’s the correct answer. And then, of course, as I always say, you’ll see the test questions, do this all the time. They’ll talk about people who immigrated to the United States. And what they’re really saying to you is this. So, this lady most likely has some form of rheumatic fever.

[00:09:16] Maybe she had strep when she was a younger child and just wasn’t diagnosed. So, what they’re really sharing with you is this. As soon as I see the word immigrant, what they are really saying is this: Listen, this person is at high risk for TB. All right. That’s the one thing that we kind of worry about.

[00:09:29] The second thing is that their care is not as structured as ours. In other words, they slipped through the cracks. Could this patient have had a previous strep throat and had rheumatic fever, which led her to the mitral stenosis? Of course, she could.

[00:09:44] And that was the whole point to it. But again, it is expiration → left, left sternal border. Just about everybody is at the LSB, so that doesn’t really help. But it was on the left side. So, I know this is either going to be aortic or mitral. And then you told me it was diastolic, so this will fit mitral stenosis.

[00:10:06] So, just trying to get you to look at these murmurs differently. That’s all just more from a laminar flow physics point of view.

Murmur Question #4

[00:10:15] Next question. I have a 67-year-old female who comes in with an upper sternal border murmur. This murmur increases in intensity when she is squatting, and with handgrip, it decreases intensity with Valsalva and standing.

[00:10:41] One more time. This murmur increases in intensity with squatting and handgrip and decreases in intensity with Valsalva and standing. Who am I?

[00:10:54] So let’s think about this upper sternal border. So, what are we talking about? Pulmonic or aortic. They didn’t tell me what side right or left did they? However, they told me that this murmur increases in intensity with squatting – that’s everybody. So that’s not helping me, but they said it increases in intensity with hand grip. Stop. No stop right here.

[00:11:06] Handgrip, who’s that challenging? Pulmonic or aortic? Oh, that’s aortic, Joe. I see your point. That makes so much sense. It’s not pulmonic. So, this is going to be either aortic stenosis or aortic regurgitation.

[00:11:28] Well, let me think about this. You’re telling me that this murmur sounds louder with handgrip, and wait a second, it’s not shutting it off. It’s making it worse. This means that the valve is wide open. Oh, then this must be aortic regurgitation, and that’s the correct answer. And of course, you know, my low volume maneuvers – Valsalva and standing, of course, will make it go away. The answer is aortic regurgitation.

[00:11:51] So does everybody see this? I’m just trying to get you ready for the PANCE. But does every see how we’re looking at this now? This is what I keep hammering away at, approach this not from a memorization point of view but a laminar flow physics point. Stop memorizing!

[00:12:07] Start trying to make sense of this because once you make sense of this, it will flow so much easier for you. Ignore those things you memorized in PA school, those little four squares, the graphs, etc.. I’ve seen it all in my 30 years of teaching. You can’t teach me one more trick. And I laugh when I’m sitting here in my studio saying this to you because I know students better than students know students. And I get it, but I’m just trying to teach you what will help you the most on the boards.

Murmur Question #5

[00:12:36] Next question. I have a patient who is a 72-year-old gentleman with hypertension, diabetes, and coronary artery disease, who also smokes. He comes in complaining of increasing shortness of breath and bilateral leg swelling. He’s got a murmur noted on physical exam in the upper sternal border. The murmur sounds louder with inspiration. Squatting makes it sound louder. Standing and Valsalva make it go away. And this murmur appears to be diastolic in nature. Who am I?

[00:13:31] Upper sternal border (pulmonic and aortic area) and louder with inspiration. So, I’m on the right (inspiration → right and expiration → left). This has got to be pulmonic. So, it’s going to be pulmonic stenosis or pulmonic regurgitation. He’s a smoker. So, he’s got some pulmonary hypertension. I get it. And this is a diastolic murmur. So, what pulmonary valve disorder is diastolic?

[00:13:51] You’re going to go, oh, this has got to be pulmonic regurgitation. Exactly and wait a second. Oh, that makes so much sense. All that pulmonary hypertension, with COPD and stuff like that. Oh, that makes so much sense. And then he’s going to have to fight it. And, of course, everything is going to back up into the right ventricle and then go to his legs. The answer is pulmonic regurgitation.

Murmur Question #6

[00:14:27] Next question. I have a 37-year-old female with a previous history of mitral valve prolapse.

[00:14:33] However, recently, she’s been coming in with increasing shortness of breath and was diagnosed with atrial fib. Physical exam shows a murmur that increases with squatting and sounds louder with handgrip. And this murmur increases in intensity in the left lateral decubitus position. This murmur also decreases in intensity with Valsalva and standing. Who am I?

[00:15:10] One more time. The murmur increases intensity with squatting. Also, hand grip, also left lateral decubitus position, and decreases in intensity with Valsalva and standing. Who am I?

[00:15:31] All right. So, let’s think about this, she had a previous history of mitral valve prolapse. I got it. But now she’s got increasing shortness of breath and a fib – oh something went wrong.

[00:15:41] Do people with mitral valve prolapse get a fib? No, not at all. And so now this murmur is going to increase in intensity with squatting so that’s not helping me, but the hand grip, it increases with hand grip. So, wait a second. Let me think about this. So, there I am handgrip sitting on the aortic valve, which means the left ventricle has got to fill up with more blood and the left ventricle is going to contract with more blood.

[00:16:07] And then that mitral valve that should stay closed is actually blowing right on through. And if I put them in that left lateral decubitus position, that’s bringing the mitral valve to the surface. So let me think about this for a second, Joe. Hold on here. So, you’re telling me that the left lateral decubitus position brings the mitral valve to the right?

[00:16:31] So whether it be mitral stenosis or mitral regurgitation, that’s the maneuver, it’ll bring it closest to the chest. So, you hear it better. And we could argue expiration in this question, but this clue wasn’t given to me here. And does everybody see the most common cause for mitral regurgitation is actually mitral valve prolapse that goes untreated?

[00:16:48] And that’s how we get it. And then, of course, they start irritating the left atria, which is how they get the AFib. But now, does everybody understand how the handgrip made it worse? Again let’s think about it. Your hand gripping. What are you doing? Increasing the pressure behind the aortic valve.

[00:17:05] What’s the left ventricle going to do? Remember the Frank-Starling thing? What did Frank say? When you increase afterload, you’re going to increase preload. What’s the left ventricle going to do? Fill up with more blood. And there’s that mitral regurgitation valve – come and go as you please. Now that left ventricle goes to contract, what’s it going to do?

[00:17:20] Oh, it will push more blood flow against my regurgitant mitral valve. Exactly. And that’s the correct answer – mitral regurgitation.

Murmur Question #7

[00:17:31] Next question. I have this 67-year-old gentleman who comes in complaining of right-sided heart failure. He’s also got some right atrial enlargement, and he also has bilateral lower extremity edema. During the exam you note a mid-diastolic murmur at the left lower sternal border that increases in intensity with inspiration, sounds louder with squatting, and goes away with standing and Valsalva. Who am I?

[00:18:09] Again, mid-diastolic murmur, left lower sternal border, increasing with intensity with squatting and inspiration, goes away with Valsalva and standing. Who am I?

[00:18:30] Let’s think about it. So, what did we get in the question? Well, we got the word inspiration. So, what do we know? Oh, that’s on the right. And then we said left sternal border.

[00:18:39] So who’s that? Everybody. But it’s not the pulmonic valve or the aortic valve. So, who are we talking about if it’s on the right and not the pulmonic valve? The tricuspid valve. And then you told me that it was mid-diastolic. So, wait a second. The tricuspid valve, what’s the diastolic murmur associated with the tricuspid valve? Is this regurgitation or stenosis? Oh, this is stenosis. The answer is tricuspid stenosis.

[00:18:59] What are your low volume maneuvers? Standing and Valsalva. Usually, with these maneuvers, all of the murmurs will go away. The low volume lovers are murmurs that sound louder with low volume. That’s HOCM and mitral valve prolapse.

Murmur Question #8

[00:19:13] Next question. I have a murmur that is holosystolic in nature that is located on the left midsternal border and sounds louder with squatting and inspiration. This murmur also goes away with Valsalva and standing. Who am I?

[00:19:40] One more time. I am a holosystolic blowing murmur. I’m located at the left midsternal border. I sound louder with inspiration and squatting, and I go away with standing and Valsalva. Who am I? So holosystolic means, it’s probably most likely some type of regurgitation and now you’re telling me that it’s on the left midsternal border that’s everybody. But the key piece of information you gave me is inspiration, which means this is right. Which means it’s going to be the tricuspid valve. Now you’re telling me it’s holosystolic, so that can’t be tricuspid stenosis because that is diastolic. So, this is going to be tricuspid regurgitation. The answer is tricuspid regurgitation.  

[00:20:29] Then there’s that thing about the Carvallo’s sign. The increased murmur intensity with inspiration. And then sometimes we see that pulsatile liver. Because everything’s backing up on the right side, so, all that blood flow is going to the liver, and it’s causing it to kind of shake in a way. Sometimes we call it pulsatile.

So again, is everybody starting to understand this?

  • Inspiration → right and expiration → left
  • Squatting is not going to help you much
  • And now you have to ask, where’s the location of this murmur? And remember, when they say left, lower sternal border, they’re saying, hey, this is everybody but the pulmonic and the aortic valve, which are located in the upper sternal border.

They’re not going to be nice to you and say, oh, it’s right or left because that gives you too much information.

So again, we’re just going to pull back, and we’re going to use Joe’s basic rules.

Murmur Question #9

[00:21:17] Next question. You have a healthy 24-year-old female sitting on your exam table who comes in with mild lightheaded and dizziness with very little exertion. On physical exam, you notice that she has a murmur when she performs standing and Valsalva; however, this murmur also goes away with squatting. The rest of her physical exams is essentially unremarkable.

[00:21:57] And when you have her perform handgrip, the murmur disappears. Who am I?

[00:22:05] Remember this murmur sounded louder with standing and Valsalva. It went away with squatting, and it went away with handgrip. So, wait a second. You’re going; it went away with squatting? Isn’t squatting a party, Joe, doesn’t that make everybody sound louder?

[00:22:22] It does. Wait a second. I’m back to my special needs murmurs, the ones that like low volume. So, this is either HOCM or mitral valve prolapse. Exactly! So, let me take this one step further. So, you’re telling me if I squat, what do I do again? I’m pushing all the blood flow back home. So, you’re taking the left ventricle and filling it with blood.

[00:22:47] So if I have HOCM, then all that blood will do what? It’s going to push the walls apart. But wait a second, the murmur is going to go away. That makes sense. Wait a second. It went away with hand grip as well? So let me think about this again. Hold on. Handgrip. What’d you do? Oh, that’s right, I increased my afterload. Joe said basically you’re just sitting here on the aorta. What did Frank say? You increase afterload; you increase preload. So, what’s the left ventricle going to do to fight that afterload? Fill it with more blood. And what are the walls going to do? Push apart. That’s HOCM.

You see what you have to remember is that HOCM is not a valve problem. Say to yourself, HOCM is not a valve problem. And if you didn’t get it the first two times, let’s say it a third time, HOCM is not a valve problem. It’s a wall problem. The left ventricle walls are hitting. And so now, does everybody understand, as they’re at rest, what do you want to do to make the murmur louder? You know you want the walls to hit. Just lightly tap, and you want them to hit. So, what’s in between the walls? Blood. So, what makes it sound louder? Oh, standing and Valsalva. Put blood in there, and what will we do with the walls? Push them apart. So how do you put blood in the left ventricle? Oh, I could squat, and I could do hand grip. Exactly. And that’s the whole point I’m trying to make about HOCM. The answer is HOCM.

Murmur Question #10

[00:24:10] Last question. You have a young, healthy 31-year-old female who comes into your clinic, and she’s been complaining of progressive, mild shortness of breath and palpitations. On physical exam, you notice this murmur increases in intensity with Valsalva and standing. You also notice that this murmur goes away with squatting and handgrip. You also note that this murmur sounds louder in mid-systole. Who am I?

[00:25:04] So, let’s think about the information we were given. We were given information that this murmur sounds louder with Valsalva and standing (our low volume maneuvers).

[00:25:13] We also know that this murmur goes away with squatting and handgrip (our high volume maneuvers, and then they give us this mid-systole. So who am I?

[00:25:25] You’re telling me that squatting makes it go away, which means it’s quite the opposite of what we would expect, which means it’s got to be a low volume lover, which is correct because it sounds louder with Valsalva and standing. Also, this murmur is a mid-systolic murmur.

[00:25:43] So wait a second, when I look at my “low volume lover” murmurs, you have to remember there’s really only one low volume murmur. That’s mitral valve prolapse because you remember HOCM has nothing to do with the valves. It has everything to do with the walls. So, you’re like, oh, they’re telling me that this thing sounds louder mid-systole.

[00:26:05] One more time, when is the left ventricular pressure highest? Mid systole. Then there is that redundant mitral valve that has prolapsed, and the valve can’t hold. And then what does it do? The blood blows right on through. And that’s the correct answer – this is mitral valve prolapse.

[00:26:24] All right, listen, I need you to replay this podcast until you get tired of me talking, OK?

[00:26:32] It’s like, oh my gosh, someone put Joe out of his misery, right? Tell him to stop talking to me. And I want you to keep replaying this and replaying this. But most importantly, stop memorizing!

[00:26:46] So let’s go over it again just as a recap.

  • Upper sternal borders, who are they talking about? Aortic and pulmonic.
  • Where’s everybody else? The left lower sternal border.
  • So left lower sternal border is not helping you out much.
  • But if I say upper sternal border, I’m either pulmonic or aortic. If it’s on the right upper sternal border, that’s aortic. If it’s on the left upper sternal border, that’s pulmonic.
  • Louder with inspiration → right and louder with expiration → left.
  • What squatting? Party maneuver, who parties? Everybody, everyone’s going to sound louder.
  • Who are your low-volume maneuvers? That’s standing and Valsalva. So, you have less blood flow in the heart, and most murmurs sound quieter.
  • Who are your special needs murmurs? What were the two murmurs or two areas that like low volume? They like less blood in the ventricles. That is mitral valve prolapse, which is a valve problem, and HOCM, which is really a wall problem.
  • Then inspiration → right and expiration → left. How do you take the mitral valve and put it close to the chest? The left lateral decubitus position. That’s what I do. So, the left lateral decubitus position is best, whether it be mitral stenosis or mitral regurgitation.
  • And one more time. What’s hand grip? Handgrip is me sitting on the aortic valve. If I have aortic stenosis, I can barely open up, and you sit on the aortic valve, and is it going to be able to open up? Heck no! But if I have aortic regurgitation and now all this blood flow is going back against the aortic valve. Is there anything to stop it? No. And remember if I have a normal aortic valve and perform hand grip, what will I do? Remember what Frank Starling’s curve said, when I increase afterload, I increase preload, so what’s the left ventricle going to do? It’s going to fill it with more blood.
  • So if I have HOCM, what are you doing to the walls? Well, I’m filling it with more blood, but what am I doing with the walls? Pushing them apart.
[00:28:32] Hopefully, I’ve demystified the heart murmur issues for you, and I hope this makes more sense. Like I said, just replay this podcast episde.

Remember what I’ve been trying to teach about inspiration and expiration and squatting and handgrip and all these maneuvers we can do to make the murmur sound louder and disappear. And if you go with that approach, I promise you, no matter how the PANCE comes at you with a heart murmur question – You’re going to nail it.

[00:28:57] If you go at it from the memorization point of view, you will be in a boatload of trouble. Because all the buzzwords are gone and all those kinds of stuff. So listen, it’s been an honor. And it’s always a pleasure to speak with you out there in podcast land. And so right now, I got pixie.

[00:29:13] My rescue dog whose lying on my bed, sitting here, and she’s giving me that look like she wants to go outside for a walk. So it’s time to take my rescue dog out for a walk. All the best, and please, as you’re getting ready for the PANCE, like I said, in the previous podcast, make sure you take care of yourself, take care of your health and take care of your brain.

[00:29:32] All the best, Joe.

This podcast is available on every device. You can download and listen to past FREE episodes here, on iTunes, Spotify, Google Podcasts, Stitcher, Amazon Music, and all podcasting apps.

Download the Interactive Content Blueprint Checklist

Interactive Content Blueprint for the 2022-2023 PANCE

Follow this link to download your FREE copy of the PANCE/PANRE/EOR Content Blueprint Checklists.

Print it up and start crossing out the topics you understand, marking the ones you don’t, and making notes of key terms you should remember. The PDF version is interactive and linked directly to the individual lessons on Smarty PANCE.

The post Podcast Episode 97: Murmurs Made Incredibly Easy – Ten PANCE Murmur Questions appeared first on The Audio PANCE and PANRE.

]]>
Welcome to episode 97 of the Audio PANCE and PANRE PA board review podcast. Today is a bonus episode rounding out this fabulous five-part podcast series with Joe Gilboy PA-C, all about cardiac murmurs. In this week’s episode,



Welcome to episode 97 of the Audio PANCE and PANRE PA board review podcast.



Today is a bonus episode rounding out this fabulous five-part podcast series with Joe Gilboy PA-C, all about cardiac murmurs. In this week’s episode, we continue our discussion of cardiac murmurs with ten PANCE and PANRE murmur questions.



We’ll cover the ins and outs of all the NCCPA content blueprint valvular disorders and learn how to identify and differentiate them from one another.



If you haven’t already, make sure to listen to our previous podcast episodes where we covered tricuspid stenosis, aortic valve murmurs, mitral valve murmurs, pulmonic valve murmurs, and HOCM and MVP.



Podcast Episode 97: Murmurs Made Incredibly Easy – 10 PANCE and PANRE Murmur Questions



Below is a transcription of this podcast episode edited for clarity.



* You can download and listen to past FREE episodes here, on iTunes, Spotify, Google Podcasts, Stitcher, Amazon Music, and most podcasting apps.* You can listen to this latest episode and access even more resources below.



[00:00:00] Welcome back, everybody. This is Joe Gilboy. I work with Stephen Pasquini over there at Smarty PANCE. And today is the most dreaded podcast of all because you know what I’m going to do. That’s right. I’m going to ask cardiac murmur questions.



And I know it’s the most dreaded thing in the world. Let’s do this together because what I want to try to do here is make sense of it all. So, let’s just kind of recap some basic rules before we start going down this thing. You know, the previous lectures, what have I been barking about? What’s Joe been saying? First – inspiration, right? Expiration left.



[00:00:40] Inspiration, right? Expiration left. So, with inspiration, the right-sided murmurs sound louder. So that’s the tricuspid and the pulmonic valve regardless of if it’s stenosis or regurgitation.]]>
The Physician Assistant Life | Smarty PANCE full 29:51 484
Podcast Episode 96: Murmurs Made Incredibly Easy (Part 5 of 5) – MVP and HOCM http://podcast.thepalife.com/podcast-episode-96-murmurs-made-incredibly-easy-part-5-of-5-mvp-and-hocm/ Mon, 04 Apr 2022 09:00:00 +0000 http://podcast.thepalife.com/?p=479 http://podcast.thepalife.com/podcast-episode-96-murmurs-made-incredibly-easy-part-5-of-5-mvp-and-hocm/#respond http://podcast.thepalife.com/podcast-episode-96-murmurs-made-incredibly-easy-part-5-of-5-mvp-and-hocm/feed/ 0 <p>Welcome to episode 96 of the Audio PANCE and PANRE PA board review podcast. Today is part five of this fabulous five-part series with Joe Gilboy PA-C, all about cardiac murmurs. In this week’s episode of the Audio PANCE and PANRE podcast, we continue our discussion of cardiac murmurs with a focus on Mitral Valve […]</p> <p>The post <a rel="nofollow" href="http://podcast.thepalife.com/podcast-episode-96-murmurs-made-incredibly-easy-part-5-of-5-mvp-and-hocm/">Podcast Episode 96: Murmurs Made Incredibly Easy (Part 5 of 5) – MVP and HOCM</a> appeared first on <a rel="nofollow" href="http://podcast.thepalife.com">The Audio PANCE and PANRE</a>.</p>
Podcast Episode 96 - Murmurs Made Incredibly Easy - Mitral Valve Prolapse (MVP) and Hypertrophic Obstructive Cardiomyopathy (HOCM)

Welcome to episode 96 of the Audio PANCE and PANRE PA board review podcast.

Today is part five of this fabulous five-part series with Joe Gilboy PA-C, all about cardiac murmurs. In this week’s episode of the Audio PANCE and PANRE podcast, we continue our discussion of cardiac murmurs with a focus on Mitral Valve Prolapse (MPV) and Hypertrophic Obstructive Cardiomyopathy (HOCM).

We’ll cover the ins and outs of these two NCCPA content blueprint murmurs and learn how to identify and differentiate them from other types of murmurs.

If you haven’t already, make sure to listen to our previous podcast episodes where we covered tricuspid stenosis, aortic valve murmurs, mitral valve murmurs, and pulmonic valve murmurs.

HOCM and MVP (a brief introduction)

Blausen 0166 Cardiomyopathy Hypertrophic

Hypertrophic Obstructive Cardiomyopathy (HOCM) is a cardiac abnormality that leads to the muscle in the wall of the heart growing and thickening to the point that it blocks blood flow exiting the heart.

The condition can be mild or severe, and it can lead to a variety of symptoms, including shortness of breath, chest pain, and irregular heartbeat. Complications may include heart failure, an irregular heartbeat, and sudden cardiac death.

HOCM is a hereditary condition, and it is usually diagnosed in adulthood. There is no cure for HOCM, but treatments are available to manage the symptoms and help reduce the risk of complications. With proper care, people with HOCM can live long and healthy lives.

*Hypertrophic cardiomyopathy is covered under the PANCE cardiology content blueprint -> cardiomyopathy -> hypertrophic cardiomyopathy

HOCM is also covered as part of the PAEA EOR pediatric rotation -> cardiovascular topic list -> hypertrophic cardiomyopathy

Mitral Valve Prolapse

Mitral Valve Prolapse (MPV) is a condition in which the leaflets of the mitral valve bulge or prolapse back into the left atrium during systole. This may cause blood to flow backward into the left atrium, leading to a heart murmur.

In some cases, MPV may also cause symptoms such as fatigue, dizziness, chest pain, and shortness of breath.

While MPV is usually benign, it can occasionally lead to serious complications such as heart failure or stroke.

Treatment for MPV typically involves lifestyle modification and management of symptoms. In severe cases, surgery may be necessary to repair or replace the mitral valve.

*Miral valve prolapse is covered under the PANCE cardiology content blueprint -> valvular disorders -> mitral valve prolapse

Podcast Episode 96: Murmurs Made Incredibly Easy (Part 5 of 5) – MVP and HOCM

Below is a transcription of this podcast episode edited for clarity.

Welcome back, everybody out there in the podcast world. This is Joe Gilboy, and I work with Stephen Pasquini at Smarty PANCE.

Today is part five of our five-part series covering heart murmurs – one of the most dreaded subjects in PA land.

Today we are going to cover what I call the low-volume lovers.

And who are the murmurs that like low blood volumes? In other words’ low blood volumes make these murmurs sound louder? That is Hypertrophic Obstructive Cardiomyopathy (HOCM) and mitral valve prolapse.

Hypertrophic Obstructive Cardiomyopathy (HOCM)

So, what is happening with HOCM? Let us go back and view this from a pathophysiology point of view. So what do you have with HOCM?

I have this young adult, and he is going to start exercising. So what is the left ventricle going to do on a typical day during exercise? You will stress out the left ventricle, which can lead to hypertrophy. 

I want everybody to look in the space you are in right now. Maybe you are in a room. Perhaps you are in a car. I want you to look at the volume of this room or car and look at the wall.

Now I want you to imagine that you start working out. You see, the wall will get thicker.

Now, what happens when those walls are thicker, my muscles get thicker. Thus, what is going to happen next? My stroke volume will go up, my cardiac output will increase, and my resting heart rate will decrease. And that is a nice normal day. That is the way it is supposed to work.

This is why aerobic exercise is so good. That’s what your Apple Watch or Fitbit is telling you. They are going, “Hey, we hit the target zone. You’re stressing out your left ventricle.” Your left ventricle will hypertrophy, which is a normal response to exercise.

Your wall will get thicker, but the volume of the room you are sitting in stays the same. Let this marinate for a second. I stress out my left ventricle, and it’s going to hypertrophy. The wall is going to get thicker, with more healthy tissue. My stroke volume will go up, my cardiac output will go up, my resting heart rate will go down, and my exercise tolerance will improve. That’s a nice normal day.

Hi, I have HOCM by genetics, no fault of my own. I was just born with this genetically. I will stress out my left ventricle, except instead of hypertrophying out, with HOCM, I am going to hypertrophy in. 

Okay, please stop and think about what I am saying. Now. See the room you are sitting in, that wall. It is coming in. Not out, but in. Look at the volume. You have in your room now. What have you done? You have decreased the volume (the size) of your room.

Now, since I have less volume, what is the last thing you want to take away from me? Volume? So what drug is contraindicated in HOCM? Diuretics!

The last thing I want to do is have these walls hit with low volume and a high pulse. That’s when bad things can happen. This is the VTACH and the VFIB that you see these young athletes with HOCM die from.

So, these walls get thick and come in and make my room smaller. So, the last thing I want to do is take volume away, and the last thing I want to do is give them a diuretic.

What is a lifestyle thing I can tell my patients with HCOM to do? Drink more water! This will increase the volume, and the walls to be pushed apart. Does everybody see that? 

Okay, my patient with HCOM is sitting at the edge of the bed at rest. He is not exercising, and he does not have an elevated pulse and low volume – that’s that nightmare scenario we were talking about where a patient with HOCM is exercising, and they have low volume. They are sweating, and their pulse is high. Now the walls hit, and this is when the bad things happen.

So, I have this patient sitting at the edge of the bed in an exam room at rest. Remember, HOCM is not a valvular issue. No valve is not functioning correctly here. These are walls that are enlarged (hypertrophied).

Now I want to hear the walls hit each other while my patient with HOCM is at rest. That’s all I want. I want the walls to hit. What is in the way between those two walls? Blood!

If I want the walls to hit, I have to get the blood out. I can do this with my low-volume maneuvers. What are my low-volume maneuvers? Standing and Valsalva. 

When I have HOCM, if I perform standing and Valsalva (the low volume maneuvers), I can get the blood out of the way so walls can get closer and hit. So, what makes the murmur of HOCM sound louder? Standing and Valsalva.

How do I push the walls of the ventricle apart? I can add more blood. How do I add more blood? If I squat! That is right; squatting is a party. If I go in there and squat and all this blood comes running back home, what will I do with the walls of the left ventricle? I am going to push them apart.

So, this is the opposite of all those murmurs I learned about! This is why I call HOCM and MVP the low volume lovers – because they like low volume. 

Handgrip and HOCM

Let us take it one step further. Let’s do that one maneuver everybody hates out there in the podcast world: Handgrip.

Okay, think about it. Stop memorizing. Let’s make sense of this.

So, we perform handgrip. What am I doing? I am sitting on the aortic valve. You sat on the aortic valve and added more pressure to the valve by doing this. You have added more afterload.

What is the left ventricle going to do? Well, since I have more afterload, I need more preload. So, my ventricles are going to have to fill up more. With the increased volume, the walls will move apart from each other.

So, you are telling me that when I squat and put all that blood into the left ventricle, it will push the walls apart? Furthermore, when I do handgrip, it will also cause those walls to move apart because I must increase my preload and increase the volume. Exactly!

So, when I perform handgrip and have HOCM, the murmur goes away.

HOCM summary

  • HOCM It is not a valvular issue.
  • To make the murmur of HOCM louder, you want the ventricular walls to hit, which requires lowering the volume of blood between the ventricles.
  • In contrast, for my patient’s health, the last thing I want to have is low volume, so I do not want my patient to sweat. I do not want to give him a diuretic and lower the volume even more.
  • And what is his nightmare situation for patients with HOCM? Low volume and high pulse!
  • What can we give patients with HOCM that will keep their pulse low? We can give them a beta-blocker.
  • Remember, those walls are disorganized, and the tissue is stiff. So, I want to relax the tissue. What is an excellent smooth most relaxer? Calcium channel blockers!
  • And that is exactly how we treat HOCM with beta-blockade to lower the pulse and calcium channel blockers to relax the stiff tissue.

This is everything you need to know about HOCM: It is not a valvular issue. To make HOCM louder, you want the volume out so the walls can hit. What are the low-volume maneuvers? Standing and Valsalva. How do you put volume (aka blood) in the room? Standing and handgrip? Exactly. That is precisely how this works.

Mitral Valve Prolapse

The next murmur that likes low volume is the murmur of mitral valve prolapse. In comparison to HOCM, this one takes a bit more thought.

To demonstrate mitral valve prolapse put your two hands together like you are making the letter A. 

Okay, see how your fingertips are touching each other. This is how the mitral valve usually closes.

Now point your thumbs in the middle and view your thumbs as the chordae tendineae. They are pulling your fingers down. That is how it works on an average day.

To demonstrate mitral valve prolapse, take your right hand and place it where your metacarpals are – again, kind of like the letter A.

Do you see how your fingers are over the top of the other ones? They do not match up perfectly. This is what we sometimes call a redundant valve. In other words, there is extra tissue, and things are not lining up.

When the left atrium contracts on a normal day, the mitral valve comes down, and everything works as expected. However, we will run into trouble with MVP when the left ventricular pressure is highest. And when is the left ventricular pressure at its highest? Oh, that is right, I memorized that during PA school. It is highest during mid systole. That’s why we get a mid-systolic click – because the valve cannot hold the pressure below it during mid-systole.

That is because the connection is not very good. It is loose. So, the valve pops open during mid-systole, and we call that a mid-systolic click.

How can I crank that pressure up in the left ventricle? Oh, I could exercise. Exactly! This is why we often hear about people with mitral valve prolapse having palpitations with exercise.

Okay. I want the murmur of MVP to sound louder. Think about this. Let us go back to where my hands were in the example above. Remember how my fingers were coming together to form the letter A? Except when I have mitral valve prolapse, my right hand is touching my metacarpals, and I have these extra fingers on top of each other, creating a redundant valve.

So, I want this valve to collapse in on itself. I want it to fall in on itself.

What is down in the left ventricle? Blood. Now we need to get the blood out of there so the valve can fall down on itself. We will need to call in our low-volume maneuvers to do this.

What are our low-volume maneuvers? Standing and Valsalva! Standing and Valsalva are low volume maneuvers and will make the murmur associated with mitral valve prolapse sound louder. 

Now, if I put blood back in the ventricle, can that redundant valve fall down on itself? No, it cannot. So, squatting is a high-volume maneuver that puts blood back into the left ventricle, and it is not going to make the murmur of MVP sound louder because the valve is not going to fall in on itself. However, if I take the blood away, there is nothing down there in the left ventricle, and the valve will fall back down. So mitral valve prolapse is also a murmur that gets louder with low volume maneuvers.

Recap

  • What are your low-volume maneuvers? Standing and Valsalva!
  • With low volumes and MVP, the valve will fall back down on itself.
  • With high volumes and MVP, what am I going to hear? I am going to hear a mid-systolic click.
  • Who is my patient going to be? MVP is more common in females, so we will likely be presented with a young female with palpitations during exercise.
  • What are you going to write off as a young female with palpitations? Anxiety. These patients are often misdiagnosed with anxiety. What will the patient say to you that will be the clue that this is not anxiety? The patient may talk about the palpitations with exercise. Remember, with exercise, the left ventricle is full of blood, and there is a lot of pressure in the left ventricle. During mid systole, when the volume is the highest, you will hear a mid-systolic click
  • What if your patient is sitting at the edge of the bed in the clinic resting and not exercising? How can you hear the murmur of MVP – this valve that does not connect very well? I want to make the valve fall back down on itself. What is below the mitral valve? The left ventricle. What is in the left ventricle? Blood that needs to get out! What can I do to decrease the volume of blood in the heart, make the valve fall back down on itself, and increase the sound of the murmur at rest? Standing and Valsalva!

Handgrip and Mitral Valve Prolapse

There is just one more thing we need to cover. The ultimate apex question. Are you ready? Handgrip! With handgrip and HOCM, I could see how by increasing the volume of blood in the heart, the ventricle walls would get pushed apart, and the murmur goes away. Now we will cover handgrip and mitral valve prolapse.

Okay, handgrip, so what did I say? I am sitting on the aorta. Okay. So, what is the left ventricle going to do? I just increased my afterload. This means I also increased my preload.

So what did you do? You just put more blood in your left ventricle! Now you need to push that blood out, and you are pushing out more blood. The mitral valve is prolapsed and does not have a good connection. Is it going to be able to hold? No. So, will I hear the murmur of mitral valve prolapse if I do handgrip? Yes, the murmur of MVP will increase with handgrip.

How do I differentiate the low-volume lovers (MVP and HOCM)? By having the patient perform handgrip. With HOCM, when handgrip is applied, you push the walls of the ventricles apart. With mitral valve prolapse and handgrip, you increase left ventricular pressure, and what happens to the mid systolic click? Oh, it is still there, and if ff anything is louder. Because what did I do? I increased the pressure in my left ventricle.

Come back to this podcast and replay it again when you have a moment. This will make so much sense if you follow my physics, especially the laminar flow physics part.

Remember that HOCM and MVP are low volume lovers – meaning they like low volume – that is when the murmurs sound louder. They are the opposite of mitral stenosis, mitral regurgitation, aortic stenosis, and aortic regurgitation. Those usual players like more blood against the diseased valve.

Remember, in HOCM; it is not a valve issue; it is a wall issue. Mitral valve prolapse is a valve issue, but remember that the valve does not connect very well. It is still there. It works well, but the connection does not hold tight.

What’s next?

Next week we are going to start doing murmur questions.

We have covered aortic stenosis and aortic regurgitation, we went over mitral stenosis and mitral regurgitation, we covered pulmonic stenosis and pulmonic regurgitation, we have learned about tricuspid stenosis and tricuspid regurgitation, and in this episode, we covered what I call exceptions to the rule – which is HOCM, and mitral valve prolapse.

What is next? Test questions – where I will present these murmurs to you in a very ambiguous way, and you will have to figure out which murmur it is.

Take care of yourselves, and stay safe. I will see you soon.

Take care, Joe.

This podcast is available on every device. You can download and listen to past FREE episodes here, on iTunes, Spotify, Google Podcasts, Stitcher, Amazon Music, and all podcasting apps.

Download the Interactive Content Blueprint Checklist

Interactive Content Blueprint for the 2022-2023 PANCE

Follow this link to download your FREE copy of the PANCE/PANRE/EOR Content Blueprint Checklists.

Print it up and start crossing out the topics you understand, marking the ones you don’t, and making notes of key terms you should remember. The PDF version is interactive and linked directly to the individual lessons on Smarty PANCE.

The post Podcast Episode 96: Murmurs Made Incredibly Easy (Part 5 of 5) – MVP and HOCM appeared first on The Audio PANCE and PANRE.

]]>
Welcome to episode 96 of the Audio PANCE and PANRE PA board review podcast. Today is part five of this fabulous five-part series with Joe Gilboy PA-C, all about cardiac murmurs. In this week’s episode of the Audio PANCE and PANRE podcast,



Welcome to episode 96 of the Audio PANCE and PANRE PA board review podcast.



Today is part five of this fabulous five-part series with Joe Gilboy PA-C, all about cardiac murmurs. In this week’s episode of the Audio PANCE and PANRE podcast, we continue our discussion of cardiac murmurs with a focus on Mitral Valve Prolapse (MPV) and Hypertrophic Obstructive Cardiomyopathy (HOCM).



We’ll cover the ins and outs of these two NCCPA content blueprint murmurs and learn how to identify and differentiate them from other types of murmurs.



If you haven’t already, make sure to listen to our previous podcast episodes where we covered tricuspid stenosis, aortic valve murmurs, mitral valve murmurs, and pulmonic valve murmurs.



HOCM and MVP (a brief introduction)







Hypertrophic Obstructive Cardiomyopathy (HOCM) is a cardiac abnormality that leads to the muscle in the wall of the heart growing and thickening to the point that it blocks blood flow exiting the heart.



The condition can be mild or severe, and it can lead to a variety of symptoms, including shortness of breath, chest pain, and irregular heartbeat. Complications may include heart failure, an irregular heartbeat, and sudden cardiac death.



HOCM is a hereditary condition, and it is usually diagnosed in adulthood. There is no cure for HOCM, but treatments are available to manage the symptoms and help reduce the risk of complications. With proper care, people with HOCM can live long and healthy lives.



*Hypertrophic cardiomyopathy is covered under the PANCE cardiology content blueprint -> cardiomyopathy -> hypertrophic cardiomyopathy



HOCM is also covered as part of the PAEA EOR pediatric rotation -> cardiovascular topic list -> hypertrophic cardiomyopathy







Mitral Valve Prolapse (MPV) is a condition in which the leaflets of the mitral valve bulge or prolapse back into the left atrium during systole. This may cause blood to flow backward into the left atrium, leading to a heart murmur.



In some cases, MPV may also cause symptoms such as fatigue, dizziness, chest pain, and shortness of breath.



While MPV is usually benign, it can occasionally lead to serious complications such as heart failure or stroke.



Treatment for MPV typically involves lifestyle modification and...]]>
The Physician Assistant Life | Smarty PANCE full 21:24 479
Podcast Episode 95: Ten PANCE, PANRE, and Rotation Review Questions http://podcast.thepalife.com/podcast-episode-95/ Thu, 10 Mar 2022 17:36:24 +0000 http://podcast.thepalife.com/?p=474 http://podcast.thepalife.com/podcast-episode-95/#respond http://podcast.thepalife.com/podcast-episode-95/feed/ 0 <p>Welcome to episode 95 of the Audio PANCE and PANRE Physician Assistant/Associate Board Review Podcast. Join me as I cover ten PANCE, PANRE and EOR review questions from the Smarty PANCE Instagram/Facebook page and the smartypance.com board review website. Special from today’s episode: Follow Smarty PANCE and The Daily PANCE Blueprint on Instagram Follow Smarty […]</p> <p>The post <a rel="nofollow" href="http://podcast.thepalife.com/podcast-episode-95/">Podcast Episode 95: Ten PANCE, PANRE, and Rotation Review Questions</a> appeared first on <a rel="nofollow" href="http://podcast.thepalife.com">The Audio PANCE and PANRE</a>.</p>
Podcast Episode 95- Ten PANCE, PANRE, and Rotation Review Questions

Welcome to episode 95 of the Audio PANCE and PANRE Physician Assistant/Associate Board Review Podcast.

Join me as I cover ten PANCE, PANRE and EOR review questions from the Smarty PANCE Instagram/Facebook page and the smartypance.com board review website.

Special from today’s episode:

Below you will find an interactive exam to complement today’s podcast.

The Audio PANCE/PANRE and EOR PA Board Review Podcast

I hope you enjoy this free audio component to the examination portion of this site. The full board review course includes over 2,000 interactive board review questions and is available to all members of Smarty PANCE.

Listen Carefully Then Take the Practice Exam

If you can’t see the audio player, click here to listen to the full episode.

Podcast Episode 95: Ten PANCE/PANRE and EOR Topic Blueprint Questions

1. A 52-year-old patient is admitted with a lower gastrointestinal bleed. He is given 2 units of packed red blood cells. A few hours later the patient develops a fever but has no other symptoms or changes in vital signs. Lab studies reveal no significant changes. Which of the following is the most likely diagnosis?

A. Transfusion-associated circulatory overload
B. Acute immune-mediated hemolytic reaction
C. Transfusion-related acute lung injury
D. Febrile non-hemolytic transfusion reaction
E. None of the above

Click here to see the answer

The answer is D. Febrile non-hemolytic transfusion reaction

The most common transfusion reaction is a febrile non-hemolytic transfusion reaction (FNHTR).

Patients with FNHTR may present with a fever a few hours after receiving packed red blood cells (PRBCs), which is caused by the small number of white blood cells and cytokines that can be found in each unit of PRBCs. Treatment consists of giving acetaminophen and ruling out other causes of fever (e.g., central line infection, urinary tract infection, etc).

Incorrect Answers:

Transfusion-associated circulatory overload (Choice A) occurs when the volume of the transfused component causes hypervolemia.

Acute immune-mediated hemolytic reaction (Choice B) occurs when there are antibodies presented to blood donor antigens. Patients may present with hypotension, fever, or coagulopathic lab abnormalities.

Transfusion-related acute lung injury (Choice C) is due to antibodies reacting with antigens leading to the release of mediators that causes edema in the lungs. Symptoms include fever, hypoxia, and dyspnea.

VIEW BLUEPRINT LESSON

Smarty PANCE Content Blueprint Review:

Covered under ⇒ PANCE Blueprint Hematology ⇒ Immunologic disorders ⇒ Transfusion reaction

2. Which of the following is not a side effect of lithium?

A. Hyperparathyroidism
B. Hypothyroidism
C. Cognitive fog
D. Tremors
E. All are side effects

Click here to see the answer

The answer is E. All are side effects 

Lithium is the first-line treatment for bipolar disorder. It has many side effects that you should remember, such as hypothyroidismhyperparathyroidismnephrogenic diabetes insipiduscognitive fog, and tremors. Lithium levels should be monitored to avoid toxicity.

Remember, bipolar disorder is characterized by the symptoms DIG FASTDistractibility, Irritability, Grandiosity, Flight of ideas, Activity increased, Sleep deficits, and Talkativeness.

VIEW BLUEPRINT LESSON

Smarty PANCE Content Blueprint Review:

Covered under ⇒ PANCE Blueprint Psychiatry ⇒ Bipolar and related disorder

3. Which of the following best explains the pathophysiology of vasovagal syncope?

A. Increase in parasympathetic signals and withdrawal of sympathetic signals
B. Severe narrowing of the aortic valve
C. Drop in blood pressure upon standing due to inadequate peripheral vasoconstriction
D. Occlusion of the pulmonary artery leading to right ventricle dysfunction
E. Blood accumulation in the brain leading to compression of adjacent brain structures

Click here to see the answer

The answer is A. Increase in parasympathetic signals and withdrawal of sympathetic signals

Vasovagal syncope is the most common cause of syncope. Syncope is essentially transient loss of consciousness due to the lack of cerebral perfusion. Vasovagal syncope is usually triggered by something, such as emotion or stress.

A leading hypothesis of vasovagal syncope is that it is due to an increase in parasympathetic signals and withdrawal of sympathetic nervous signals.

Incorrect Answers:

A drop in blood pressure upon standing due to inadequate vasoconstriction describes neurogenic orthostatic hypotension (Choice C).

Occlusion of the pulmonary artery leading to RV dysfunction is describing a pulmonary embolism, which can lead to syncope (Choice D).

Blood accumulation in the brain leading to compression of brain structures is describing an intracranial hemorrhage (Choice E), which is a (rare) cause of syncope.

Narrowing of the aortic valve aka aortic stenosis (Choice B) can lead to syncope, but again, this is not vasovagal syncope.

VIEW BLUEPRINT LESSON

Smarty PANCE Content Blueprint Review:

Covered under ⇒ PANCE Blueprint Neurology ⇒ Vascular Disorders ⇒ Syncope

Also covered as part of the Internal Medicine EORFamily Medicine EOREmergency Medicine EORPediatric EOR, and General Surgery PAEA EOR topic list

4. A 71-year-old male smoker with prostate cancer presents to your clinic complaining of unilateral leg swelling and pain. Vital signs are unremarkable. On a physical exam, the entire leg is swollen with localized tenderness along the venous system. Which of the following is the next best step?

A. Order a d-dimer
B. Order a duplex ultrasound
C. Admit to the hospital immediately
D. Reassurance and follow-up in 6 months
E. Order a CT angiogram

Click here to see the answer

The answer is B. Order a duplex ultrasound

This patient needs an ultrasound because he has a high pre-test probability for thrombosis. Ordering a d-dimer, in this case, would be inappropriate. Lots of things (e.g., cancer, trauma, etc) can elevate d-dimer.

Well’s Score helps us determine a patient’s risk for DVT based on these criteria: active cancer, surgery or bedridden, calf swelling, collateral veins presence, entire leg swollen, localized tenderness along deep venous system, pitting edema, previous DVT, recent immobilization, & alternative diagnosis more likely.

The main idea: d-dimer should be ordered in low-risk patients, and a venous duplex US should be ordered for high-risk patients.

VIEW BLUEPRINT LESSON

Covered under ⇒ PANCE Blueprint Cardiology ⇒ Vascular Disease ⇒ Venous thrombosis

Also covered as part of the General Surgery PAEA EOR topic list

5. Which of the following individuals would be considered to have a positive purified protein derivative test at 6 millimeters of induration?

A. 31-year-old who was in contact with a person who has active TB
B. 65-year-old intravenous drug user
C. 53-year-old who lives in a prison
D. 90-year-old with hypertension
E. None of the above

Click here to see the answer

The answer is A. 31-year-old who was in contact with a person who has active TB

Tuberculosis (TB) is a worldwide health concern and is deadly. A purified protein derivative (PPD) test can be used to screen for TB. A PPD test is considered positive depending on the patient’s risk factors. Remember, we measure induration, NOT erythema. See below for a high-yield list of positive PPD tests (not exhaustive):

>5 mm: HIV, immunosuppressive condition, or recent contact with active TB case

>10 mm: IVDU, came from a country with a high TB prevalence (<5 y prior), health care worker, resident/employee in a high-risk congregate setting

>15 mm: everyone else

VIEW BLUEPRINT LESSON

Smarty PANCE Content Blueprint Review:

Covered under ⇒ PANCE Blueprint Pulmonary ⇒ Infectious Pulmonary Disorders ⇒ Tuberculosis

Also covered as part of the Internal Medicine EOREmergency Medicine EOR, and Family Medicine PAEA EOR topic list

6. A 48-year-old male presents with headaches, vision loss, enlarged jaw, and enlarged hands. Which of the following laboratory studies is the preferred initial diagnostic test?

A. Beta glycoprotein 2a
B. Growth hormone
C. Prolactin
D. Calcitonin
E. Insulin growth factor 1

Click here to see the answer

The answer is E. Insulin growth factor 1

The patient has acromegaly, which is an endocrine disorder characterized by excess secretion of growth hormone after epiphyseal closure. The most common cause is a pituitary adenoma. Clinical manifestations include coarsening of facial features, enlarged hands/feet, macrognathia, bitemporal hemianopsia, sleep apnea, and headaches.

Insulin growth factor-1 is the best initial diagnostic test to screen for acromegaly. A CT or MRI can help confirm the presence of a pituitary adenoma. A glucose suppression can also be ordered. Definitive treatment is transsphenoidal resection of the pituitary adenoma.

VIEW BLUEPRINT LESSON

Smarty PANCE Content Blueprint Review:

Covered under ⇒ PANCE Blueprint Endocrinology ⇒ Pituitary Disorders ⇒ Acromegaly/gigantism

Also covered as part of the Internal Medicine PAEA EOR topic list

7. A 14-year-old obese male presents to the ER complaining of dull pain in the right hip and thigh for the past few weeks. No history of trauma. On physical exam, the affected leg is held in an externally rotated position and is shorter than the left leg. What is the most likely diagnosis?

A. Legg Calve Perthes
B. Septic arthritis
C. Slipped capital femoral epiphysis
D. Transient synovitis
E. Pelvic stress fracture

Click here to see the answer

The answer is C. Slipped capital femoral epiphysis

The patient most likely has slipped capital femoral epiphysis (SCPE), which is defined as displacement of the capital femoral epiphysis from the femoral neck (through the physeal plate). The classic clinical presentation is hip pain in an obese, adolescent male. Remember, it can be bilateral (~20-40%)!

On physical exam, the affected leg may be externally rotated and shorter than the other leg. Cases are diagnosed via plain radiographs (AP view and lateral views of both hips). Once diagnosed with SCPE, most patients should be made non-weight-bearing and referred to an orthopedic surgeon.

VIEW BLUEPRINT LESSON

Smarty PANCE Content Blueprint Review:

Covered under ⇒ PANCE Blueprint Musculoskeletal ⇒ Lower extremity disorders ⇒ Disorders of the hip ⇒ Slipped capital femoral epiphysis

Also covered as part of the Pediatric Rotation PAEA EOR topic list

8. A 32-year-old male with sarcoidosis presents to the clinic complaining of shortness of breath, fatigue, and palpitations for a few months. His in-office EKG reveals QRS prolongation and some premature ventricular beats. A chest radiograph shows cephalization of the pulmonary vessels and cardiomegaly. Which of the following is the most likely diagnosis?

A. Cryptogenic organizing pneumonia
B. Hypertensive encephalopathy
C. Cardiac sarcoidosis
D. Acute coronary syndrome
E. Pneumoconiosis

Click here to see the answer

The answer is C. Cardiac sarcoidosis

Sarcoidosis is a multisystem granulomatous disorder; it impacts the lungs, eyes, lymph nodes, heart, skin, and nervous system. Cardiac sarcoidosis (CS) is often under-recognized and can occur as a clinical feature of sarcoidosis. Presenting symptoms may include palpitationssyncopefatigue, & dyspnea.

CS happens because granulomas infiltrate the heart, leading to conduction issues, tachyarrhythmias, cardiomyopathies, & heart failure. Holter monitoring and echocardiography are two helpful tests. The gold standard would be a heart biopsy (not routinely done because it is so invasive; cardiac MRI is preferred). Treatment is steroid therapy and anti-arrhythmic drugs.

Incorrect Answers:

Cryptogenic organizing pneumonia (Choice A) is a type of diffuse interstitial lung disease that typically presents with cough, fever, dyspnea, and malaise.

Hypertensive encephalopathy (Choice B) manifests as headaches, vomiting, confusion, and neurological symptoms. The patient has none of these.

Acute coronary syndrome (Choice D) is more likely to present with diaphoresis and crushing chest pain.

Pneumoconiosis (Choice E) is a fancy word that means the accumulation of dust (e.g., coal, asbestos, etc) within the lungs. This patient has absolutely nothing in his history that suggests this.

VIEW BLUEPRINT LESSON

Smarty PANCE Content Blueprint Review:

Covered under ⇒ PANCE Blueprint Pulmonary ⇒ Restrictive Pulmonary Disease ⇒ Sarcoidosis

Also covered as part of the Internal Medicine PAEA EOR topic list

9. A 64-year-old male presents with tremors in both of his hands. He says the tremors worsen with movement and caffeine. The tremors improve with alcohol use. Which of the following is the best initial treatment option for his likely diagnosis?

A. Dopamine agonist
B. Beta-blocker
C. Alpha antagonist
D. NMDA antagonist
E. None of the above

Click here to see the answer

The answer is B. Beta-blocker

The patient likely has an essential tremor (ET) which is the most common cause of action tremor in adults. It occurs bilaterally in both hands, commonly worsens with anxiety, and improves with alcohol use. The tremor becomes more noticeable when the hands are outstretched or during goal-directed movement (e.g., writing with a pencil).

ET is a clinical diagnosis. Treatment includes beta blockers (propranolol), anticonvulsants (primidone, gabapentin, topiramate) and benzodiazepines. Propranolol and primidone are the most effective treatment options. Remember there is often a strong genetic component (family history is present 30-70% of the time).

VIEW BLUEPRINT LESSON

Smarty PANCE Content Blueprint Review:

Covered under ⇒ PANCE Blueprint Neurology ⇒ Movement Disorders ⇒ Essential tremor

Also covered as part of the Family Medicine and Internal Medicine PAEA EOR topic list

10. Which of the following is the most common cause of infective lactational mastitis?

A. Escherichia coli
B. Staphylococcus aureus
C. Bacteroides species
D. Staphylococcus epidermidis

Click here to see the answer

The answer is B. Staphylococcus aureus 

Lactational mastitis presents as a red, painful, swollen breast usually during the first three months of breastfeeding. It commonly results from poor drainage. The most common cause is Staphylococcus aureus. If it develops over 24 hours, patients may also have flu-like symptoms and a fever — this is considered “infective” lactational mastitis.

Treatment includes cold compresses and complete emptying of the breasts. For infective lactational mastitis, antibiotics (e.g., clindamycincephalexindicloxacillin) should be given. Remember, patients should NOT stop breastfeeding.

VIEW BLUEPRINT LESSON

Smarty PANCE Content Blueprint Review:

Covered under ⇒ PANCE Blueprint Reproductive System ⇒ Breast Disorders ⇒ Mastitis 

Also covered as part of the Women’s Health EOR and Emergency Medicine PAEA EOR topic list

Looking for all the podcast episodes?

This FREE podcast series is limited to every other episode, you can download and enjoy the complete audio series by becoming a Smarty PANCE member.

I will be releasing new episodes every few weeks. Smarty PANCE is now discounted, so sign up now before it’s too late!

Resources and Links from the Show

This Podcast is available on iOS and Android

Download the Interactive Content Blueprint Checklist

Interactive Content Blueprint for the 2022-2023 PANCE

Follow this link to download your FREE copy of the PANCE/PANRE/EOR Content Blueprint Checklists

Print it up and start crossing out the topics you understand, marking the ones you don’t, and making notes of key terms you should remember. The PDF version is interactive and linked directly to the individual lessons on Smarty PANCE.

The post Podcast Episode 95: Ten PANCE, PANRE, and Rotation Review Questions appeared first on The Audio PANCE and PANRE.

]]>
Welcome to episode 95 of the Audio PANCE and PANRE Physician Assistant/Associate Board Review Podcast. Join me as I cover ten PANCE, PANRE and EOR review questions from the Smarty PANCE Instagram/Facebook page and the smartypance.



Welcome to episode 95 of the Audio PANCE and PANRE Physician Assistant/Associate Board Review Podcast.



Join me as I cover ten PANCE, PANRE and EOR review questions from the Smarty PANCE Instagram/Facebook page and the smartypance.com board review website.



Special from today’s episode:



* Follow Smarty PANCE and The Daily PANCE Blueprint on Instagram* Follow Smarty PANCE and The Daily PANCE Blueprint on Facebook



Below you will find an interactive exam to complement today’s podcast.



The Audio PANCE/PANRE and EOR PA Board Review Podcast



I hope you enjoy this free audio component to the examination portion of this site. The full board review course includes over 2,000 interactive board review questions and is available to all members of Smarty PANCE.



* You can download and listen to past FREE episodes here, on iTunes, Spotify, on Google Podcasts, Stitcher, and most podcasting apps.* You can listen to the latest episode, take an interactive quiz, and download more resources below.



Listen Carefully Then Take the Practice Exam





If you can’t see the audio player, click here to listen to the full episode.



Podcast Episode 95: Ten PANCE/PANRE and EOR Topic Blueprint Questions



1. A 52-year-old patient is admitted with a lower gastrointestinal bleed. He is given 2 units of packed red blood cells. A few hours later the patient develops a fever but has no other symptoms or changes in vital signs. Lab studies reveal no significant changes. Which of the following is the most likely diagnosis?



A. Transfusion-associated circulatory overloadB. Acute immune-mediated hemolytic reactionC. Transfusion-related acute lung injuryD. Febrile non-hemolytic transfusion reactionE. None of the above





2. Which of the following is not a side effect of lithium?



A. HyperparathyroidismB. HypothyroidismC. Cognitive fogD. TremorsE. All are side effects





3. Which of the following best explains the pathophysiology of vasovagal syncope?



A. Increase in parasympathetic signals and withdrawal of sympathetic signalsB. Severe narrowing of the aortic valveC. Drop in blood pressure upon standing due to inadequate peripheral vasoconstrictionD. Occlusion of the pulmonary artery leading to right ventricle dysfunctionE. Blood accumulation in the brain leading to compression of adjacent brain structures





4.]]>
The Physician Assistant Life | Smarty PANCE full 22:30 474
Podcast Episode 94: Murmurs Made Incredibly Easy (Part 4 of 5) – Tricuspid Stenosis http://podcast.thepalife.com/podcast-episode-94-murmurs-made-incredibly-easy-part-4-of-5-tricuspid-stenosis/ Wed, 16 Feb 2022 10:00:52 +0000 http://podcast.thepalife.com/?p=469 http://podcast.thepalife.com/podcast-episode-94-murmurs-made-incredibly-easy-part-4-of-5-tricuspid-stenosis/#respond http://podcast.thepalife.com/podcast-episode-94-murmurs-made-incredibly-easy-part-4-of-5-tricuspid-stenosis/feed/ 0 <p>Welcome to episode 94 of the Audio PANCE and PANRE PA board review podcast. Today is part four of this fabulous five-part series with Joe Gilboy PA-C, all about cardiac murmurs. In this week’s episode of the Audio PANCE and PANRE podcast, we continue our discussion of cardiac murmurs with a focus on the tricuspid […]</p> <p>The post <a rel="nofollow" href="http://podcast.thepalife.com/podcast-episode-94-murmurs-made-incredibly-easy-part-4-of-5-tricuspid-stenosis/">Podcast Episode 94: Murmurs Made Incredibly Easy (Part 4 of 5) – Tricuspid Stenosis</a> appeared first on <a rel="nofollow" href="http://podcast.thepalife.com">The Audio PANCE and PANRE</a>.</p>
Murmurs Made Incredibly Easy - Tricuspid Stenosis

Welcome to episode 94 of the Audio PANCE and PANRE PA board review podcast.

Today is part four of this fabulous five-part series with Joe Gilboy PA-C, all about cardiac murmurs. In this week’s episode of the Audio PANCE and PANRE podcast, we continue our discussion of cardiac murmurs with a focus on the tricuspid valve.

We’ll cover the ins and outs of tricuspid valve stenosis and learn how to identify it and differentiate it from other types of murmurs.

If you haven’t already, make sure to listen to our previous podcast episode where we covered aortic valve murmurs and mitral valve murmurs, and pulmonic valve murmurs.

The Tricuspid Valve

Tricuspid Valve - right atrioventricular valve

The tricuspid valve, or right atrioventricular valve, is a one-way valve that sits between the right atrium and right ventricle of the heart. It is essential for right ventricular filling and for preventing the backflow of blood from the right ventricle into the right atrium when the right ventricle contracts during systole.

When functioning properly, the tricuspid valve is a passive structure that opens and closes in response to the pressure of the blood flowing through the heart.

The right atrium receives deoxygenated blood from the superior and inferior vena cava. The tricuspid valve which separates the right atrium from the right ventricle opens during ventricular diastole, allowing the deoxygenated blood to flow from the right atrium into the right ventricle, and closes during ventricular systole preventing the backflow of blood from the right ventricle into the right atrium as the right ventricle contracts to pump blood into the lungs out through the pulmonary artery.

Tricuspid Valve Stenosis

Tricuspid stenosis is a narrowing of the tricuspid valve or one of its three leaflets.

If the tricuspid valve is narrowed or stenotic, it will not open properly during diastole, increasing the volume of blood in the right atrium. When the right ventricle contracts the stiffened tricuspid valve also fails to close completely and tricuspid regurgitation develops.

Nearly all cases are caused by rheumatic fever.

Podcast Episode 94: Murmurs Made Incredibly Easy (Part 4 of 5) – Tricuspid Valve Stenosis

Below is a transcription of this podcast episode edited for clarity.

Welcome everybody, this is Joe Gilboy PA-C and I work with Stephen Pasquini at Smarty PANCE. Today, we will continue this series on heart murmurs. Today’s topic is tricuspid stenosis.

So, sit back, listen to my voice, and try not to take notes. Don’t do anything other than just listen and follow my logic. I promise you, when this is all said and done, you’ll be looking at heart murmurs and more specifically, tricuspid stenosis in a vastly different light. I want you to view the tricuspid valve. Just think about it. There you are the right atria. The tricuspid valve opens during diastole and closes during systole.

There’s all your blood in the right ventricle, and it goes out through the pulmonary valve to get oxygenated out in the lungs.

So now, I want you to think about this for a second. I have a tricuspid valve that is stenotic. It’s stiff and hard to open.

When are you going to have problems opening this valve? During diastole? Or during systole?

You are going to have problems with this during diastole because that’s when the tricuspid valve is supposed to open up. Because it’s supposed to be closed during systole.

So, it’s a diastolic murmur, it’s considered a mid-diastolic murmur.

Now give this some more thought. Okay, so you’re in the right atria – Is it easier or harder to push through this stenotic valve?

Oh, it’s a lot harder. And so, where’s the blood flow going to back up to? Into the right atrium and now that right atrium, it’s going to have all this increased pressure going into it. So now what are you going to get? You are going to get right atrial enlargement!

Here is another question – If the right atrium is having a tough time pushing blood through the stenotic valve then where’s this extra blood going to back up to? The right side of the body. Exactly! So, I’m going to have JVD and I’m going to get pedal edema? You are going to get right-sided heart failure with tricuspid stenosis.

Can I ask another question? How’s your blood flow to your right ventricle? It is diminished. Well, then how’s your blood flow to the pulmonary area? That’s diminished! How’s your oxygenation? That’s also diminished. So how are you feeling? I’m short of breath. Exactly. And you are short of breath not because you’re in heart failure. You’re short of breath because you’re just not getting blood to oxygenate. Exactly.

So, tricuspid stenosis is on the right side. One more time – Inspiration right. Expiration left. Inspiration right. Expiration left that if you didn’t get it the first two times. Let’s try the third time. Inspiration right. Expiration left.

Hi, I’m tricuspid stenosis, which side of the heart am I on the right or left? I’m on the right. So, tricuspid stenosis is going to sound louder with inspiration. Exactly.

What’s the party maneuver where all the blood flow comes back home and everybody’s just partying as you get more blood flow back home – squatting. What else could make tricuspid stenosis sound louder? Squatting! Exactly!

Squatting is a high-volume maneuver and will make the murmur of tricuspid stenosis sound louder. 

What are my low-volume maneuvers? What are the maneuvers that take blood out of my heart? Standing in Valsalva!

So, what would make tricuspid stenosis go away? Standing and Valsalva are low volume maneuvers and will decrease the sound of tricuspid valve stenosis.

So, I have a right-sided heart murmur. It’s diastolic and is considered mid-diastolic. Where am I going to put my stethoscope?

You see, there’s only one thing on the right – that’s the aortic valve at the right second intercostal space, right that we all memorized in school. So, everybody’s on the left.

The left upper sternal space – that’d be pulmonic. But what is on the left – everybody else?

So hi, you’re the tricuspid valve, where are you? I’m on the left… Exactly! You’re going to be on the left at the lower sternal border.

So now I have this mid-diastolic murmur. Left sternal border that sounds louder with inspiration => goes away with expiration => left. It’s going to sound louder with squatting and is going to go away with standing and Valsalva – these are my low volume maneuvers.

And how did you get this? Usually, the most common cause of tricuspid stenosis is rheumatic fever, is it not?

So now, let’s pull back, and let’s look at this from 30,000 feet.

  • I’ve got a stenotic valve over there on the right side. Who is it? It’s the tricuspid valve.
  • Where’s the blood flow back going back up to? The right atria.
  • What’s the right atrium going to do? It’s going to hypertrophy and I’m going to get right atrial enlargement.
  • Where’s the blood going to go next? Outside to the body. And what do you get? Symptoms that look like right-sided heart failure.
  • Are you overloading your lungs? No. Did you get any CHF over there on the left? None.
  • How’s your oxygenation? It’s poor because everything’s backed up on the right. It’s not backed up on the left.
  • Inspiration right and expiration left? The tricuspid valve is on the right so tricuspid stenosis is going to sound louder with inspiration.
  • What else can make the murmur sound louder? I could party which happens when I squat – so the murmur of tricuspid stenosis gets louder with squatting.
  • What will make the murmur of tricuspid stenosis go away? My low-volume maneuvers standing, and Valsalva will decrease the murmurs of tricuspid stenosis. 
  • How did I get tricuspid stenosis? I got tricuspid stenosis through rheumatic fever.
  • Where am I going to put my stethoscope to listen to the murmur of tricuspid stenosis? Everybody’s on the left sternal border Joe, everybody’s here. The only thing that’s on the left upper sternal border is the pulmonic valve and the only thing on the right is the aortic valve. So, the murmur of tricuspid stenosis is going to be on the left lower sternal border
  • How do we treat tricuspid stenosis? We like to keep the volume low because we don’t want the blood to back up. Therefore, we use diuretics sometimes we’ll also add sodium restriction to a patient’s diet.
  • This is a stenotic valve so it’s harder to open. So, with tricuspid stenosis, we are going to get that opening snap, just like mitral stenosis

Closing thoughts

When you get to the boards (on your PANCE or PANRE) I want you to forget about brute memorization and all these buzzwords.

Just pull back and try to make sense of this from a laminar flow physics point of view.  If you do it this way, you’ll always do great.

In the real world when we hear a heart murmur what’s the first thing we do? We order an echocardiogram to figure it out. But what I’m really trying to get you ready for is your boards.

So, on the boards, they give you a patient with a murmur at the left sternal border murmur and they’re going to talk about how it gets louder with inspiration, which means okay, it’s going to be on the right.

Or they say the murmur gets louder with expiration so it’s going to be on the left.

I want you to start viewing it this way. Not the way that you memorized it in school with all the PowerPoints and all the other mnemonics. I’ve seen every mnemonic known to man in my 30 years of teaching, so I don’t want to go down that rabbit hole.

So now that we understand the physics hopefully this makes more sense and I hope this helps you better understand the murmur of tricuspid stenosis better.

And next week, what are we going to do guys? Oh is going to be tricuspid regurgitation. And don’t worry about the following week. Oh, that’s going to be a fun day. That’s when I start asking you test questions and we start trying to pull this all together.

So once again, it’s an honor. It’s an absolute pleasure to speak to all of you. Be safe out there, be healthy, and most importantly take care of yourself. I will talk to you soon.

Joe Gilboy PA-C


This episode was transcribed for your reading pleasure by Stephen Pasquini PA-C

Why is it all in there

This podcast is available on every device.

You can download and listen to past FREE episodes here, on iTunes, Spotify, Google Podcasts, Stitcher, Amazon Music, and all podcasting apps.

Download the Interactive Content Blueprint Checklist

Interactive Content Blueprint for the 2022-2023 PANCE

Follow this link to download your FREE copy of the PANCE/PANRE/EOR Content Blueprint Checklists.

Print it up and start crossing out the topics you understand, marking the ones you don’t, and making notes of key terms you should remember. The PDF version is interactive and linked directly to the individual lessons on Smarty PANCE.

The post Podcast Episode 94: Murmurs Made Incredibly Easy (Part 4 of 5) – Tricuspid Stenosis appeared first on The Audio PANCE and PANRE.

]]>
Welcome to episode 94 of the Audio PANCE and PANRE PA board review podcast. Today is part four of this fabulous five-part series with Joe Gilboy PA-C, all about cardiac murmurs. In this week’s episode of the Audio PANCE and PANRE podcast,



Welcome to episode 94 of the Audio PANCE and PANRE PA board review podcast.



Today is part four of this fabulous five-part series with Joe Gilboy PA-C, all about cardiac murmurs. In this week’s episode of the Audio PANCE and PANRE podcast, we continue our discussion of cardiac murmurs with a focus on the tricuspid valve.



We’ll cover the ins and outs of tricuspid valve stenosis and learn how to identify it and differentiate it from other types of murmurs.



If you haven’t already, make sure to listen to our previous podcast episode where we covered aortic valve murmurs and mitral valve murmurs, and pulmonic valve murmurs.



The Tricuspid Valve







The tricuspid valve, or right atrioventricular valve, is a one-way valve that sits between the right atrium and right ventricle of the heart. It is essential for right ventricular filling and for preventing the backflow of blood from the right ventricle into the right atrium when the right ventricle contracts during systole.



When functioning properly, the tricuspid valve is a passive structure that opens and closes in response to the pressure of the blood flowing through the heart.



The right atrium receives deoxygenated blood from the superior and inferior vena cava. The tricuspid valve which separates the right atrium from the right ventricle opens during ventricular diastole, allowing the deoxygenated blood to flow from the right atrium into the right ventricle, and closes during ventricular systole preventing the backflow of blood from the right ventricle into the right atrium as the right ventricle contracts to pump blood into the lungs out through the pulmonary artery.



Tricuspid Valve Stenosis



Tricuspid stenosis is a narrowing of the tricuspid valve or one of its three leaflets.



If the tricuspid valve is narrowed or stenotic, it will not open properly during diastole, increasing the volume of blood in the right atrium. When the right ventricle contracts the stiffened tricuspid valve also fails to close completely and tricuspid regurgitation develops.



Nearly all cases are caused by rheumatic fever.



Podcast Episode 94: Murmurs Made Incredibly Easy (Part 4 of 5) – Tricuspid Valve Stenosis





Below is a transcription of this podcast episode edited for clarity.



* You can download and listen to past FREE episodes here, on iTunes, Spotify, Go...]]>
The Physician Assistant Life | Smarty PANCE full 18:57 469
Podcast Episode 93: Murmurs Made Incredibly Easy (Part 3 of 5) – Pulmonary Valve Stenosis and Regurgitation http://podcast.thepalife.com/podcast-episode-93-murmurs-made-incredibly-easy-part-3-of-5-pulmonary-valve-stenosis-and-regurgitation/ Wed, 19 Jan 2022 10:00:05 +0000 http://podcast.thepalife.com/?p=458 http://podcast.thepalife.com/podcast-episode-93-murmurs-made-incredibly-easy-part-3-of-5-pulmonary-valve-stenosis-and-regurgitation/#respond http://podcast.thepalife.com/podcast-episode-93-murmurs-made-incredibly-easy-part-3-of-5-pulmonary-valve-stenosis-and-regurgitation/feed/ 0 <p>Welcome to episode 93 of the Audio PANCE and PANRE physician assistant/associate board review podcast. Today is part three of this extraordinary five-part series with Joe Gilboy PA-C, all about cardiac murmurs. In this week’s episode of the Audio PANCE and PANRE podcast, we continue our discussion of cardiac murmurs with a focus on the […]</p> <p>The post <a rel="nofollow" href="http://podcast.thepalife.com/podcast-episode-93-murmurs-made-incredibly-easy-part-3-of-5-pulmonary-valve-stenosis-and-regurgitation/">Podcast Episode 93: Murmurs Made Incredibly Easy (Part 3 of 5) – Pulmonary Valve Stenosis and Regurgitation</a> appeared first on <a rel="nofollow" href="http://podcast.thepalife.com">The Audio PANCE and PANRE</a>.</p>
Podcast Episode 93 Murmurs Made Incredibly Easy (Part 3 of 5) – Pulmonary Valve Stenosis and Regurgitation - Smarty PANCE

Welcome to episode 93 of the Audio PANCE and PANRE physician assistant/associate board review podcast.

Today is part three of this extraordinary five-part series with Joe Gilboy PA-C, all about cardiac murmurs. In this week’s episode of the Audio PANCE and PANRE podcast, we continue our discussion of cardiac murmurs with a focus on the pulmonic valve.

We’ll talk about the different types of murmurs (stenosis and regurgitation) that can occur with the pulmonic valve and how to differentiate them from other types of murmurs. If you haven’t already, make sure to listen to our previous podcast episode where we covered the aortic valve murmurs and mitral valve murmurs.

The Pulmonic Valve

Pulmonic valve stenosis (PVS) and pulmonic valve regurgitation (PVR) are two common heart valve diseases. PVS is a narrowing of the pulmonic valve, while PVR is leakage of blood back through the pulmonic valve. Both conditions can lead to significant heart problems if left untreated.

Pulmonic valve stenosis

PVS is the most common congenital heart disease, affecting approximately 1 in 1000 people. It can lead to right heart failure and/or pulmonary hypertension (high blood pressure in the lung arteries). PVS is caused by an abnormally thickened pulmonic valve or localized stenosis which means that it has a narrowing of the valve. Pulmonic valve stenosis has several different causes including:

  • Congenital heart disease – this means that the abnormal valve was present from birth but is often undiagnosed until adulthood. For example, children with Down’s syndrome or Noonan syndrome are more likely to have an undiagnosed congenital heart defect such as PVS. It commonly is a component of tetralogy of Fallot.
  • Cocaine use in pregnancy – research has shown that there is a link between the use of cocaine in pregnant women and children with PVS, particularly if it is used close to the time of conception or later stages of pregnancy.
  • Infection during pregnancy can cause an inflammatory response which may lead to heart problems for the baby.
  • Autoimmune disease – PVS is sometimes associated with other autoimmune diseases, such as lupus.

Symptoms of PVS can vary depending on the type of stenosis that is present and how much it has advanced. Mild cases may not cause any symptoms at all; however, it is common for patients to experience breathlessness, fatigue, chest pain, and palpitations.

Heart sounds include increased splitting of S2 and a harsh crescendo-decrescendo ejection murmur heard best at the left parasternal 2nd or 4th intercostal space when the patient leans forward; the murmur grows louder immediately with Valsalva release and with inspiration.

People with PVS may need to take some medication to ease their symptoms. For example, diuretics can help remove excess fluid; beta-blockers reduce the workload on the heart and therefore lower blood pressure and heart rate, and long-acting calcium channel blockers can reduce the force of contraction of the heart’s left ventricle.

Pulmonary valve replacement is an effective treatment for severe PVS, which involves surgically implanting a new pulmonary valve to replace the defective one. Because the valve only affects the blood flow through one part of the heart, this surgery can often be carried out using minimally invasive techniques.

For milder cases of PVS that do not cause symptoms then medication or exercise may be prescribed instead. Regular exercise helps to strengthen the left ventricle (the main pumping chamber) which is important for people with PVS. For more information, view our lesson on pulmonary stenosis.

Pulmonic valve regurgitation

Pulmonic valve regurgitation is leakage of blood back through the pulmonic valve.

It can lead to right-sided heart failure and/or pulmonary hypertension (high blood pressure in the lung arteries). This means that it has a leakage of the valve.

It can be caused by any condition that impairs cardiac function, including pulmonary hypertension (the most common cause), chronic obstructive pulmonary disease (COPD), left ventricular hypertrophy (LVH), and heart failure (HF).

Although 5-8% of the general population has PVR, it is more common in COPD and other respiratory diseases, such as bronchiectasis.

Pulmonary valve regurgitation is an important determinant of functional class and quality of life among patients with chronic obstructive pulmonary disease (COPD), who have a higher incidence of this valvular disease than the general population. It appears to be more frequent than traditionally thought and its impact on patient survival is greater than previously estimated.

Heart sounds when PR is due to pulmonary hypertension include a high-pitched, early diastolic decrescendo murmur that begins with P2 and ends before S1 and that radiates toward the mid-right sternal edge; it is heard best at the left upper sternal border while the patient holds the breath at end-expiration and sits upright.

The murmur of PR without pulmonary hypertension is shorter, lower-pitched, and begins after P2.

Treatment is directed at the cause; valve replacement is usually not needed.

Podcast Episode 93: Murmurs Made Incredibly Easy (Part 3 of 5) – Pulmonary Valve Stenosis and Regurgitation

Below is a transcription of this podcast episode edited for clarity.

Click here if you are unable to view the audio player.

Welcome back, everybody. This is Joe Gilboy PA-C, and I work with Stephen Pasquini PA-C at Smarty PANCE. Today, we will cover the dreaded two words that all PA students hate. What are these dreaded words?

Heart murmurs

We’ve covered the aortic valve. We’ve covered the mitral valve. Who’s the next one up?

The pulmonic valve

Okay. So, let’s erase everything we memorized in school.

Most of you are chuckling right now and saying Joe, it’s already erased, right? I’m sure it is.

So, let’s do it my way. I want you to pull back and forget everything you already know.

What’s the pulmonary valve supposed to do on a typical day? Let’s think about this. The right ventricle contracts and the pulmonic valve is supposed to open up. Then, that deoxygenated blood goes out to the pulmonary vasculature.

And then, on a normal day, the pulmonic valve closes during diastole to keep the blood out in the pulmonary area, and then that deoxygenated blood becomes oxygenated.

So, that diagram that you see in the books where the blood is blue, deoxygenated, then becomes oxygenated and red. And then it comes back to the pulmonary vein and back to the left atria, left ventricle, and up to the rest of the body, it goes. So that’s a nice normal day.

And where is the pulmonic valve located? Well, there’s only one valve on the right. That’s the aortic valve. Everybody else is on the left. So, where is the pulmonic valve? The left upper sternal border.

Pulmonary Stenosis

Okay, I want you to stop and just pull back and think, just think, and stop memorizing.

So, I have this stenotic pulmonary valve, and it’s hard to open up.

When am I supposed to open up? During systole. Does a stenotic pulmonic valve open up easily during systole? No, it has a hard time. So, when are you having difficulty with this valve? During systole. Because during diastole, it’s supposed to close. That’s what it wants to do. It wants to stay closed.

So, I don’t have a problem during diastole. I have a problem with systole. So, pulmonary stenosis is going to be a systolic ejection murmur.

So, here’s my question to you. Where’s the blood flow supposed to go? It’s supposed to go to the pulmonary area. Alright, stop and think about this for a second.

I’ve got a lack of blood flow to my pulmonary area. Lack of it. How is your oxygenation now? It’s pretty poor. So how are you going to present? I will be short of breath.

Imagine I go to walk up a flight of steps.  I’m walking down the aisle in Costco. I’m completely tanked out. I am short of breath – I lack so much oxygenation I become syncopal; maybe I lack so much oxygen to my heart I have angina. You’ve got a lack of oxygenated blood because the right ventricle can’t get that blood past that valve.

And now, let’s pull back. Remember from the last podcast that S1 is the date between the mitral and the tricuspid valve, and S2is the date with the aorta and pulmonary valve.

Hi, I’m pulmonary stenosis, and I’m taking longer to open up. How’s your S2?

Well, I’m kind of late to the date. Yes, you are because you’re taking longer to open up. So how is your S2 in pulmonic stenosis? It’s split!

Do you now see what the split S2 means? Either I’m aortic stenosis or pulmonic stenosis – someone’s late to the date because it’s taking longer to open up.

Now that I know I have a split S2, I need more information to determine if this is aortic or pulmonic stenosis. But imagine (on the exam) they give you a split S2, and I’m on the right side – that’s aortic.

Now imagine having a split S2, and I’m on the left? Oh, that’s pulmonic. And now give this some thought. Okay, you’ve got this stenotic pulmonic valve, and I can’t get blood out. So, of course, I’m short of breath.

Can I ask you a question? Just give this some thought. Ready?

Hi, you have a stenotic pulmonic valve. Where’s the blood flow back up to? The right ventricle!

OK, so you’re the right ventricle. What are you going to do with all this extra blood? I’m going to dilate, and I’m going to get right ventricular hypertrophy.

And now imagine, if you can, that right ventricle gets bigger and bigger and bigger – stretching, stretching, stretching, stretching, stretching, stretching your right ventricle wall. Can I ask you a question? Can I bust those wires in the right ventricular wall? Yes, I can!

Welcome to right bundle branch block. So, could I get a right bundle branch block with pulmonary stenosis? Yes, I can.

  • I get right ventricular hypertrophy.
  • I get right axis deviation on EKG. That makes sense.
  • And then I’m going to bust that wall wide open. And we’re going to bust the wires from the hypertrophy, and I get right bundle branch block.

And that’s precisely what we see with pulmonary stenosis.

So, pulmonary stenosis.

  • I see the shortness of breath, angina, and I can see myself having syncope.
  • I’ve got a split S2 because I’m late for the date
  • I have a systolic ejection murmur, and it’s going to be on the left upper sternal border, not the right upper sternal border (as in aortic stenosis).

Imagine you are given a question, and the patient has a systolic ejection murmur on the left side of the sternal border. There is a split S2, and the patient is short of breath. Maybe they are lacking so much blood flow they get syncopal.

What’s the diagnosis? You have all the information you need. It’s pulmonic stenosis!

Now let’s make the murmur sound louder.

Okay, back to our first podcast on murmurs. What’s the maneuver that makes all the blood flow go back home? Squatting, squatting is a party. So, if I bring more blood flow back home, just laminar flow physics, the more blood flow I have against the murmur, the louder it sounds – so squatting. That’s a party, and squatting makes the murmur of pulmonic stenosis sound louder.

Remember this: Inspiration, right = Louder and Expiration, left = louder. Inspiration right => Expiration => left

The pulmonic valve is on the right side, so inspiration will make the murmur of pulmonic stenosis sound louder, and the murmur of pulmonic stenosis would go away with expiration.

What are low-volume maneuvers? Standing and Valsalva. So, if I stand or Valsalva, the murmur of pulmonic stenosis goes away.  

So, let’s recap once again:

  • I have a systolic ejection murmur left upper sternal border.
  • Blood flow goes back to the right ventricle, and it dilates. I get right ventricular hypertrophy.
  • I’m going to get right bundle branch block because I stretched the wires.
  • Inspiration is going to make it sound louder. Expiration is going to make it go away.
  • Standing and Valsalva are low-volume maneuvers, and it goes away.
  • Handgrip challenges the aortic valve and does not affect the murmur of pulmonary stenosis.

Pulmonary regurgitation

Okay, so let’s just stop and think about this for a second.

Regurgitation – come and get goes you please, I could care less, my doors are always open. I’ve got regurgitation, and I have an open door, and I can come and go as I please.

So now think about it. During systole, should I be open? Yeah. But diastole, am I supposed to be closed? Oh, yes, I am.

But remember, in pulmonary regurgitation, you’re coming and going as you please. You’re supposed to be closed, but you’re not. So, this is a diastolic murmur, and it’s going to go from the left upper sternal border down to the apex. That’s just physics.

Can I ask you a question? That blood flow, where was it? Well, Joe, it was out in the lungs. Exactly. And so now what are you doing? I’m taking blood flow away from our lungs.

Is anybody late to the date? No, not at all. So that’s going to be okay.

So, it will be a diastolic murmur, and it will decrease. It will be loud in the beginning, and then, of course, as the right ventricle fills up, it’s going to sound less. So, it’s going to be considered a diastolic, decrescendo murmur.

But now I want you to stop and think about this because this is where a lot of the test questions come from.

Okay, so I have to blow through the pulmonic valve, right? Yeah. Okay, so let me think about this.

So, I have to push so hard push against the pulmonic valve that it blows open. So, I could see how right ventricular hypertrophy could cause that.

What does your right ventricle push against? It’s the pulmonary vascular.

I’m going to get pulmonary hypertension. See it? Who’s going to push hard against pulmonary hypertension? You’re like, oh, it’s the pulmonary valve on the right side. Is it built for high blood pressures? Yes, or no? And the answer is no.

So, you tell me if I get right ventricular hypertrophy due to pulmonary hypertension, I may blow my pulmonic valve? That’s precisely what’s going to occur.

What’s the most common cause of pulmonary regurgitation? Pulmonary hypertension because my right ventricle will hypertrophy, and I have to push harder. Exactly. The most common cause of pulmonary regurgitation is pulmonary hypertension.

Now, we can get to this argument of what causes pulmonary hypertension. CHF is a common cause.

Everybody’s on Adderall. What’s one of the side effects of Adderall? Pulmonary hypertension. And then here you are the pulmonary valve fighting, Adderall, and you’re like, Dude, that’s pulmonary hypertension out there. I have to push harder. Then you get right ventricular hypertrophy. And then what could I eventually get blowing through the valve? Pulmonary regurgitation!

What are the most common things that we see in people on Adderall? Pulmonary hypertension and pulmonary regurgitation.

So, when I start to fill in that right ventricular, and it hypertrophies due to all that pulmonary hypertension, I will push hard against your pulmonary valve. I can push so hard that I push right through it. Then what do you get? Pulmonary regulation.

Pulmonic valve disorders recap

Who was pulmonary stenosis? That was a systolic ejection murmur left upper sternal border, right ventricular hypertrophy, right bundle branch block.

This could be congenital. We see with tetralogy of Fallot.

And then we saw pulmonary regurgitation. What was that? It was a diastolic crescendo/decrescendo murmur. As the right ventricle filled up, it started tailing off, henceforth the word decrescendo.

Remember, what caused me to blow through my pulmonary valve? The right ventricle got so big that it just blew through because the pressure was so high. And remember, I’m the pulmonary valve. I’m not built for high blood pressure.

What was my right ventricle fighting the whole time? Pulmonary Hypertension. So, who was that? Well, that could be the Adderall. That could be the Fen-Phen. And then here’s the point. I don’t want you to forget.

Can I ask you a question? Who’s the vasoconstrictor everywhere?

What is nicotine? Nicotine is a drug. What is it? A vasoconstrictor? Where does it vasoconstrict? Everywhere! Who has the highest incidence of hypertension? Smokers!

What is the most common cause of coronary disease? Vasoconstricting and smoking.

What is a risk factor for peripheral vascular disease? Smoking.

I lack blood flow to my fingers and now my toes so much that they lack blood flow. They lack blood flow so much that they can’t grow. What’s clubbing, smoking! And now who saw the vasoconstriction? First, folks?

Your lungs? Yeah. What do you think you have as a smoker with COPD, and you’re going to go, oh, wait a second, Joe? I have pulmonary hypertension! Entirely correct.

Wait a second, Joe; then you’re telling me I get right into ventricular hypertrophy smoking? That’s exactly right. Then I can stretch my walls and get right bundle branch block? 100% correct. Then I can blow through my tricuspid and pulmonic valve with my pulmonary hypertension. Exactly.

What is the most common cause for pulmonic regurgitation or tricuspid regurgitation? Regurgitation blew through because the right ventricle got so big? Hmm. Smoking, COPD! Now, does everybody see the bigger picture?

So, imagine the exam question where a patient presents with a murmur at the right upper sternal border?  There’s only one person up there at the right upper sternal border, and that’s the aortic valve.

Now, if they tell you the murmur is at the left sternal border, that’s everybody else. Now I need some more information.

  • Is it systolic or diastolic?
  • Does it get better with inspiration => right or expiration => left?
  • Squatting is a party, so that’s just everybody and anybody.
  • Handgrip challenges the aortic valve.
  • Valsalva and squatting are low-volume maneuvers, and that makes everything go away.

Try to make sense of this, and if you do, you will always answer heart murmur questions the best way.

I still remember being in PA school at Duke, and I still remember memorizing all the buzzwords and all this stuff. And I remember my first year in the ER. I couldn’t tell you what murmur I was hearing on the physical exam.

And now that I have stepped back and taken the time to learn the pathophysiology, I can appreciate how the cardiologist calls this a two, or three, or four, five, or six systolic ejection murmur. I really can understand where they’re coming from. I hope one day that you too can appreciate that as well.

So once again, it’s been an honor and, as always, a pleasure to speak to everybody out there in the PA podcast world.

Please be healthy, please be good, and whatever you guys do. Try to make sense of things.

Take care, Joe.

Transcribed for your reading enjoyment by https://otter.ai and Stephen Pasquini PA-C

Resources and links from the show

This podcast is available on every device.

You can download and listen to past FREE episodes here, on iTunes, Spotify, Google Podcasts, Stitcher, Amazon Music, and all podcasting apps.

Download the Interactive Content Blueprint Checklist

Interactive Content Blueprint for the 2022-2023 PANCE

Follow this link to download your FREE copy of the PANCE/PANRE/EOR Content Blueprint Checklists.

Print it up and start crossing out the topics you understand, marking the ones you don’t, and making notes of key terms you should remember. The PDF version is interactive and linked directly to the individual lessons on Smarty PANCE.

Image attribution: LadyofHats, Public domain, via Wikimedia Commons

The post Podcast Episode 93: Murmurs Made Incredibly Easy (Part 3 of 5) – Pulmonary Valve Stenosis and Regurgitation appeared first on The Audio PANCE and PANRE.

]]> Welcome to episode 93 of the Audio PANCE and PANRE physician assistant/associate board review podcast. Today is part three of this extraordinary five-part series with Joe Gilboy PA-C, all about cardiac murmurs.



Welcome to episode 93 of the Audio PANCE and PANRE physician assistant/associate board review podcast.



Today is part three of this extraordinary five-part series with Joe Gilboy PA-C, all about cardiac murmurs. In this week’s episode of the Audio PANCE and PANRE podcast, we continue our discussion of cardiac murmurs with a focus on the pulmonic valve.



We’ll talk about the different types of murmurs (stenosis and regurgitation) that can occur with the pulmonic valve and how to differentiate them from other types of murmurs. If you haven’t already, make sure to listen to our previous podcast episode where we covered the aortic valve murmurs and mitral valve murmurs.



The Pulmonic Valve



Pulmonic valve stenosis (PVS) and pulmonic valve regurgitation (PVR) are two common heart valve diseases. PVS is a narrowing of the pulmonic valve, while PVR is leakage of blood back through the pulmonic valve. Both conditions can lead to significant heart problems if left untreated.



Pulmonic valve stenosis



PVS is the most common congenital heart disease, affecting approximately 1 in 1000 people. It can lead to right heart failure and/or pulmonary hypertension (high blood pressure in the lung arteries). PVS is caused by an abnormally thickened pulmonic valve or localized stenosis which means that it has a narrowing of the valve. Pulmonic valve stenosis has several different causes including:



* Congenital heart disease – this means that the abnormal valve was present from birth but is often undiagnosed until adulthood. For example, children with Down’s syndrome or Noonan syndrome are more likely to have an undiagnosed congenital heart defect such as PVS. It commonly is a component of tetralogy of Fallot.* Cocaine use in pregnancy – research has shown that there is a link between the use of cocaine in pregnant women and children with PVS, particularly if it is used close to the time of conception or later stages of pregnancy.* Infection during pregnancy can cause an inflammatory response which may lead to heart problems for the baby.* Autoimmune disease – PVS is sometimes associated with other autoimmune diseases, such as lupus.



Symptoms of PVS can vary depending on the type of stenosis that is present and how much it has advanced. Mild cases may not cause any symptoms at all; however, it is common for patients to experience breathlessness, fatigue, chest pain, and palpitations.



Heart sounds include increased splitting of S2 and a harsh crescendo-decrescendo ejection murmur heard best at the left parasternal 2nd or 4th intercostal space when the patient leans forward; the murmur grows louder immediately with Valsalva release and with inspiration.



People with PVS may need to take some medication to ease their symptoms. For example, diuretics can help remove excess fluid; beta-blockers reduce the workload on the heart and therefore lower blood pressure and heart rate, and long-acting calcium channel blockers can reduce the force of contraction of the heart’s left ventricle.
]]>
The Physician Assistant Life | Smarty PANCE full 30:53 458 Podcast Episode 92: Murmurs Made Incredibly Easy (Part 2 of 5) – Mitral Stenosis and Regurgitation http://podcast.thepalife.com/podcast-episode-92-murmurs-made-incredibly-easy-part-2-of-5-mitral-stenosis-and-regurgitation/ Wed, 08 Dec 2021 14:41:00 +0000 http://podcast.thepalife.com/?p=439 http://podcast.thepalife.com/podcast-episode-92-murmurs-made-incredibly-easy-part-2-of-5-mitral-stenosis-and-regurgitation/#respond http://podcast.thepalife.com/podcast-episode-92-murmurs-made-incredibly-easy-part-2-of-5-mitral-stenosis-and-regurgitation/feed/ 0 <p>The Audio PANCE/ PANRE Podcast Episode 92. Cardiac Murmurs Made Incredibly Easy Part 2 of 5. Mitral valve stenosis & regurgitation with Joe Gilboy PA-C.</p> <p>The post <a rel="nofollow" href="http://podcast.thepalife.com/podcast-episode-92-murmurs-made-incredibly-easy-part-2-of-5-mitral-stenosis-and-regurgitation/">Podcast Episode 92: Murmurs Made Incredibly Easy (Part 2 of 5) – Mitral Stenosis and Regurgitation</a> appeared first on <a rel="nofollow" href="http://podcast.thepalife.com">The Audio PANCE and PANRE</a>.</p> Welcome to episode 92 of the Audio PANCE and PANRE physician assistant/associate board review podcast.

Today is part two of this extraordinary five-part series with Joe Gilboy PA-C, all about cardiac murmurs. In this week’s episode of the Audio PANCE and PANRE podcast, we continue our discussion of cardiac murmurs with a focus on the mitral valve.

We’ll talk about the different types of murmurs (stenosis and regurgitation) that can occur with this valve and how to differentiate them from other types of murmurs. If you haven’t already, make sure to listen to our previous podcast episode where we covered the aortic valve murmurs.

The Mitral Valve

The mitral valve is located between the left atrium and the left ventricle in the heart. It consists of two leaflets (or cusps) and is responsible for preventing blood from flowing back into the atrium when the ventricle contracts.

The mitral valve can have two types of murmurs: stenosis and regurgitation.

  • Mitral stenosis is a narrowing (or constriction) of the valve opening, which reduces the amount of blood that can flow through the valve. This type of murmur is of low pitch, rumbling in character, and best heard at the apex with the patient in the left lateral position.
  • Mitral regurgitation is a leakage (or backflow) of blood from the ventricle into the atrium, caused by weakened or damaged valve leaflets. This type of murmur is a holosystolic (pansystolic) murmur, heard best at the apex with the diaphragm of the stethoscope when the patient is in the left lateral decubitus position.

In addition to auscultation, you can also look for certain signs and symptoms that may indicate mitral stenosis or regurgitation. For example, if a patient presents with chest pain (angina pectoris), this could be from decreased oxygen supply due to poor cardiac output in cases of significant stenosis. On the other hand, if a patient presents with an irregular heart rate (arrhythmia), this could be from increased electrical conduction velocity in cases of significant regurgitation.

Also, keep in mind that mitral valve disease can also occur secondary to rheumatic fever or endocarditis, so you may need to consider these diagnoses if a patient has any of the aforementioned signs and symptoms.

Below is a transcription of this podcast episode edited for clarity.

Welcome, everybody. This is Joe Gilboy, and I work with Stephen Pasquini at Smarty PANCE.

Today we’re back to the heart murmur podcast, and we just finished the aortic valve. The next one up is the mitral valve.

Let’s just look at this my way. In school, you memorize a bunch of facts, then someone asks a question about mitral stenosis or mitral regurgitation, and you are completely lost. It’s time to change that. Now, we are going to do it my way. Are you ready? Here we go.

Let’s look at the mitral valve on a normal day. What’s the mitral valve doing?

  • There is my left atria. It’s contracting during diastole, my mitral valve opens up, and the blood flow goes to the left ventricle.
  • Then during systole, the mitral valve will close, the left ventricle will squeeze, and the aortic valve opens.

Mitral Valve Stenosis

Let’s start with the first heart murmur, mitral stenosis. Think about it. Slow down and give this some thought.

I’ve got the stenotic mitral valve. See it? It can barely open. It’s like, man; you’ve got to push hard to open this door. So, you are the left atrium, and you’re pushing down on this stenotic mitral valve. It’s like, man; this is hard to open up! I have to push harder. Yes, you do!

Wait for a second; then I’m going to put more blood flow back against my left atrium? Completely correct. So, my left atrium might hypertrophy? Yes, that’s correct!

So, you’re telling me I might get left atrial hypertrophy with mitral stenosis? Exactly!

Now, think about it. There you are in the left atrium pushing against the mitral valve during stenosis that’s supposed to open during diastole. You’re just taking longer to open up. You see, in school, what you memorize is an opening snap. Remember that ridiculous graph where you saw it had S1, S2 on all those bar graphs? You thought you were reading a musical sheet. Do you remember that thing? Or you’re like oh, S1 S2 ……….!? You were looking at it thinking, ” I have no fricking idea what I’m looking at.” Let’s erase that.

Think about this for a second; if the left atrium is pushing on a stenotic valve, it’s going to have trouble during diastole. Because during systole, it has to be closed. It’s going to close without a problem.  The problem is opening it up. You can see the left atrium hypertrophy. Then blood flow may back up into the lungs and causes pulmonary hypertension.

Oh, wait for a second, I get pulmonary hypertension? Yes, you do!

Then maybe I have some fluid backed up in the lungs, and I might have a little shortness of breath and CHF. Exactly! Yes, that’s exactly how they’re going to present.

So, I have this diastolic murmur. It’s a diastolic ejection murmur to open that valve up. So, it’s not going to be an accentuated S1. But that’s not what they’re going to say on your boards, are they? No, that’s just a buzzword.

What they’re going to say is, “I have this diastolic rumbling murmur.” Rumbling means I’m trying to open up, and I’m going to radiate down towards the apex. That just makes logical sense.

So, I have this diastolic murmur radiating to the apex. Let’s take a closer look at the mitral valve. Remember, your heart doesn’t sit flat. It sits tilted and towards the back. So, I need this mitral valve to get towards the front. Okay, well, then tilt yourself to the left and lay down. Oh! That’s that left lateral to decubitus position they keep talking about. I’m going to bring it closer to the front.

Oh, that makes sense. I totally see that. So, I have a diastolic murmur that radiates to the apex. I can hear it better if I lay down on the left lateral decubitus position because that will bring the valve closer to the chest. Let’s make it sound louder now.

Okay, one more time back to how we were in the beginning with the aortic valve. Okay, what’s your party maneuver? Oh, I want to party, and I want to bring all the blood flow back home. How do you party? Oh, I squat. Squatting is a party. So, what are you going to do if I squat? How’s mitral stenosis sound? Louder! I get more blood flow pushing against the stenotic valve.

If I perform low-volume maneuvers, which are standing and Valsalva, I get less blood to the heart. Hi, standing and Valsalva, what’s going to happen to mitral valve stenosis. Yes, it’s going to go away!

Inspiration right (louder), and expiration left (louder). Repeat: inspiration right and expiration left. Hi, I have mitral stenosis! Oh, he’s on the left. What’s going to make him sound louder? Expiration? Yes, exactly!

Now, does everyone see this? So, you see, they’re not going to give you that “opening snapbuzzword.  All the buzzwords on the PANCE are gone, guys, so just throw them away. What they’re going to give you is some person with a heart murmur, who maybe has a little CHF, has a diastolic murmur, and that’s it. That’s all they’re going to give you now. What you have to think is diastolic: left side, going back to the lungs = stenosis. That makes perfect sense. And that’s mitral stenosis. So, the “open snap,” those buzzwords are gone.

Let’s review one more time:

  1. How do I make mitral stenosis sound louder? Squatting.
  2. Inspiration (right) and expiration (left) => If the murmur is on the left (the mitral valve is on the left) => expiration makes it sound louder.
  3. Standing and Valsalva are low volume maneuvers and the murmur of mitral stenosis goes away.
  4. How do I hear mitral stenosis better? I need to put the murmur towards the chest. Well, the heart is lying towards the back, so I need to lay it towards the front so I can hear the murmur better. The lateral decubitus position repositions the mitral valve towards the front. That makes so much sense.

There you go, that’s mitral stenosis.

Mitral valve regurgitation

Then there’s mitral valve regurgitation. I want you to view mitral regurgitation as “come and go as you please.”

This is a valve that won’t close. Repeat, “come and go as you please.” Let’s review the mitral valve again. “Hi, I’m diastole, I’m the left atria, and I’m coming through a valve that doesn’t care. Do you have any problems? No, I’m supposed to be open. So, I’m coming through during diastole not a problem.”

Now, let’s compare that to systole: “Hi, I’m systole and that mitral valve is supposed to be closed.” But is it going to be closed? Nooo. In systole, blood is coming and going as it pleases. So, wait for a second, then I’m going to be a systolic murmur. 100% correct. And are you having any resistance? Zero.

That’s why we call it a holosystolic murmur. Understand? And now I’m going from the apex at the bottom of the heart, and I’m radiating to the axilla. So, you’re going back up to the left atrium? Exactly.

So how is your blood flow out to the rest of the world? Not that good. Not good at all. So how are you feeling? I’m feeling a bit lightheaded (syncope) and short of breath.

Now let’s go one step further. Remember that LAD, the Windowmaker? Okay, if you look at the left anterior descending artery, it bifurcates into the left anterior descending and septal branches. Both these guys feed the chordae tendineae. It’s that octopus-looking muscle that pulls the mitral valve down. So now think about this for a second. Just give us some thought. Ready?

You just had a LAD MI. You’re in the hospital. The cardiologist has just put down a stent. You’re on the heparin drip. You are getting ready to go home, maybe on Xarelto or Eliquis. Right.

So, there you are, on a blood thinner. Can I ask you a question? Just give it a thought. You know, your chordae tendineae? You know, during the MI, remember that? Yes, your lacking blood flow to the left ventricle, weren’t you? Yes. How was your blood flow to your chordae tendineae? Yes, it was diminished.

Let’s review. The patient is in the hospital and on an IV beta-blocker (you want to rest the heart). Well, they’re going home and back to activities like walking through Costco or Walgreens, or CVS. He’s no longer on a beta-blocker, folks. His heart rates back up.

Is this chordae tendineae ready for normal heart rate? Yes, or no? No, not at all. He’s on the injured reserve list as well. And guess what? His chordae tendineae is not going to hold very well. He says, “Oh my god, I’m so weak. I can’t hold the mitral valve down”. Which means it is going to open back up.

That’s the test question you will see about post the MI patient who comes in two to four weeks later. The patient who’s got a new-onset murmur. What does he have? He’s got mitral regurgitation because his chordae tendineae got hypoxic during the MI, and now he’s got the murmur. So that’s a question that you see on Rosh Review, Hippo, Smarty PANCE, UWorld, and Kaplan. You’ll see this question rear its head, but the root of what they’re really after is something else. They want you to understand that the chordae tendineae itself was actually lacking blood flow during the LAD or ischemic event. So, it’s weakened. As soon as the patient goes to stress it out, it’s not going to work very well. It’s going to give way, and the chordae tendineae will rupture. What you have is mitral valve regurgitation. Does everybody see this?

So, as a recap, let’s look at this again.

  • What’s the mitral valve supposed to do? Well, I suppose you’d say, “to open up during diastole and close during systole.
  • I see that I’ve got mitral stenosis.” What’s your problem? Well, my problem is during diastole. It’s trying to open the thing up. Because during systole, don’t worry, it will close. It’s stenotic. It wants to stay closed. So, my problem is during diastole. They will not use the words “diastolic opening and snap.” But they may use the word “rumbling murmur.”
  • Where’s the blood flow backing up to? The left atria.
  • What’s the left atria going to do? Hypertrophy. What are you going to get? Left atrial hypertrophy.
  • Where’s the blood flow going to back up to? It’s going back up to the lungs. What do you have? Pulmonary hypertension.
  • What do you get? CHF.

This is the test question you’re going to get. An immigrant comes in with a diastolic murmur and shortness of breath. We don’t have the best testing for strep. Remember rheumatic fever? They are in their 30s and 40, and they’ve had rheumatic fever in the past, with mitral stenosis, and now the fluid is backing up to the lungs. That’s why they have shortness of breath.

  • Now, how do you make that murmur sound louder? Inspiration right, expiration left => expiration!
  • Squatting, it’s a party, and the murmur gets louder! Standing and Valsalva, it goes away!
  • Is your problem during diastole with mitral regurgitation? It’s like, come and go as you please. You know, let’s open and come on in. What’s your problem during systole because you should close… what he’s doing? You see, I don’t care. I can come and go as you please. So, my problem is going to be during systole.
  • So mitral regurgitation is going to be a holosystolic murmur.
  • And then, make it sound louder. Squatting is a party. Inspiration, expiration. Expiration: standing Valsalva or low volume maneuvers. It goes away. Exactly!

And that’s the point I’m trying to make about the mitral valve. Does everybody see that now?

When you get the test questions, stop and think. You’re going to laugh because one of the things I tell all my students to do all the time is to try to make sense of it. See, what you guys do is memorize these things, and you run into your, what I call your data closet. I call it a closet full of data.

You get into this data closet, and what happens is you get lost in the closet. You say to yourself, “Okay, diastolic murmur, axilla blah, blah, blah, what did I memorize? Let me write down my little mnemonic. Yes, yes. Oh, man, I can’t Oh, man.” Next, you’re just panicking. And you just choose A, B, C, or D.

You say to yourself, “oh my god, I think I got the question wrong.” Exactly.

So next time, just try making sense of it.

So, we did the aortic valve, and we did the mitral valve. So, we did everything on the left. Next week, we are going to the right folks. We’re going to go to the pulmonic valve and the tricuspid valve.

So, one more time, guys. Inspiration right, expiration left. Inspiration right, expiration left. To make the right side of your heart murmur sound better, what are you going to do? Inspiration, so pulmonic and tricuspid sound louder with inspiration.

Expiration left. Who sounds better on the left? Aortic and mitral!

It’s been an honor and a pleasure talking with you today. I hope this makes more sense. Next week we’ll do the pulmonic valve. Till then, be good to yourself and take care.

Joe Gilboy PA-C

This podcast is available on every device.

You can download and listen to past FREE episodes here, on iTunes, Spotify, Google Podcasts, Stitcher, Amazon Music, and all podcasting apps.

Download the Interactive Content Blueprint Checklist

Follow this link to download your FREE copy of the PANCE/PANRE/EOR Content Blueprint Checklists.

Print it up and start crossing out the topics you understand, marking the ones you don’t, and making notes of key terms you should remember. The PDF version is interactive and linked directly to the individual lessons on Smarty PANCE.

Featured Image attribution (aortic stenosis and regurgitation): Blausen.com staff (2014). “Medical gallery of Blausen Medical 2014“. WikiJournal of Medicine 1 (2). DOI:10.15347/wjm/2014.010ISSN 2002-4436. Crop by Stephen Pasquini PA-C, CC BY 3.0

The post Podcast Episode 92: Murmurs Made Incredibly Easy (Part 2 of 5) – Mitral Stenosis and Regurgitation appeared first on The Audio PANCE and PANRE.

]]>
The Audio PANCE/ PANRE Podcast Episode 92. Cardiac Murmurs Made Incredibly Easy Part 2 of 5. Mitral valve stenosis & regurgitation with Joe Gilboy PA-C. Welcome to episode 92 of the Audio PANCE and PANRE physician assistant/associate board review podcast.







Today is part two of this extraordinary five-part series with Joe Gilboy PA-C, all about cardiac murmurs. In this week’s episode of the Audio PANCE and PANRE podcast, we continue our discussion of cardiac murmurs with a focus on the mitral valve.



We’ll talk about the different types of murmurs (stenosis and regurgitation) that can occur with this valve and how to differentiate them from other types of murmurs. If you haven’t already, make sure to listen to our previous podcast episode where we covered the aortic valve murmurs.



The Mitral Valve



The mitral valve is located between the left atrium and the left ventricle in the heart. It consists of two leaflets (or cusps) and is responsible for preventing blood from flowing back into the atrium when the ventricle contracts.



The mitral valve can have two types of murmurs: stenosis and regurgitation.



* Mitral stenosis is a narrowing (or constriction) of the valve opening, which reduces the amount of blood that can flow through the valve. This type of murmur is of low pitch, rumbling in character, and best heard at the apex with the patient in the left lateral position.* Mitral regurgitation is a leakage (or backflow) of blood from the ventricle into the atrium, caused by weakened or damaged valve leaflets. This type of murmur is a holosystolic (pansystolic) murmur, heard best at the apex with the diaphragm of the stethoscope when the patient is in the left lateral decubitus position.



In addition to auscultation, you can also look for certain signs and symptoms that may indicate mitral stenosis or regurgitation. For example, if a patient presents with chest pain (angina pectoris), this could be from decreased oxygen supply due to poor cardiac output in cases of significant stenosis. On the other hand, if a patient presents with an irregular heart rate (arrhythmia), this could be from increased electrical conduction velocity in cases of significant regurgitation.



Also, keep in mind that mitral valve disease can also occur secondary to rheumatic fever or endocarditis, so you may need to consider these diagnoses if a patient has any of the aforementioned signs and symptoms.



Below is a transcription of this podcast episode edited for clarity.



* You can download and listen to past FREE episodes here, on iTunes, Spotify, Google Podcasts, Stitcher,]]>
The Physician Assistant Life | Smarty PANCE full 19:54 439
Podcast Episode 91: Cardiac Murmurs Made Easy Part 1 of 5 – Aortic Stenosis and Regurgitation http://podcast.thepalife.com/podcast-episode-91-cardiac-murmurs-made-easy-part-1-of-5-aortic-stenosis-and-regurgitation/ Wed, 01 Dec 2021 10:30:00 +0000 http://podcast.thepalife.com/?p=433 http://podcast.thepalife.com/podcast-episode-91-cardiac-murmurs-made-easy-part-1-of-5-aortic-stenosis-and-regurgitation/#respond http://podcast.thepalife.com/podcast-episode-91-cardiac-murmurs-made-easy-part-1-of-5-aortic-stenosis-and-regurgitation/feed/ 0 <p>Welcome to episode 91 of the Audio PANCE and PANRE Physician Assistant/Associate Board Review Podcast. Today is part one of an extraordinary five-part series with Joe Gilboy PA-C, all about cardiac murmurs. It’s time to throw away everything you have ever learned that made you hate cardiac murmurs (you know those crazy line diagrams) and […]</p> <p>The post <a rel="nofollow" href="http://podcast.thepalife.com/podcast-episode-91-cardiac-murmurs-made-easy-part-1-of-5-aortic-stenosis-and-regurgitation/">Podcast Episode 91: Cardiac Murmurs Made Easy Part 1 of 5 – Aortic Stenosis and Regurgitation</a> appeared first on <a rel="nofollow" href="http://podcast.thepalife.com">The Audio PANCE and PANRE</a>.</p>

Welcome to episode 91 of the Audio PANCE and PANRE Physician Assistant/Associate Board Review Podcast.

Today is part one of an extraordinary five-part series with Joe Gilboy PA-C, all about cardiac murmurs.

It’s time to throw away everything you have ever learned that made you hate cardiac murmurs (you know those crazy line diagrams) and let Joe hard-wire your brain for success.

I promise you that by the end of this podcast, you will no longer be afraid of murmurs, but you will welcome them into your brain with a whole heart, open arms, and the need to share this podcast with all your classmates.

Below is a transcription of this podcast episode slightly edited for clarity.

Welcome, everybody. This is Joe Gilboy PA-C, and today is part one of a five-part series on the two most dreaded words that every pa student and recert PA hates. Are you ready for the two words?

Heart murmurs!

Today we’re going to start talking about heart murmurs, and we are going to break this down into a five-part series:

  1. The first part is going to be on the aortic valve (today’s episode)
  2. The second part will cover the mitral valve
  3. The third will be all about the pulmonic valve
  4. The fourth part will be about the tricuspid valve
  5. And then, in the fifth part of this series, we’re going to wrap this all together

We’re going to cover each valve one at a time. We will look at it and try to really make sense of it all.

Because I know what you did in PA school, you had that little diagram that you made, you know, systolic or diastolic murmurs. You’ve memorized certain things. And then you get to that test question, and you’re completely clueless on it, you’re like, “I have no idea what they just said, Joe, I have no idea.”

And then you’re back to scrambling, looking for some word that’s going to trigger you. And that’s the key point- all the trigger words, all those high-value trigger words, they’re gone. That’s what the creators of the PANCE did about two to three years ago. They took all the trigger words out.

Key point:

  • Don’t rely on keywords (trigger words) for your PANCE because they’re gone!

So now, let’s look at this differently. You did it your way. How did it work out? Probably not so well. So, guess what, we’re going to do it my way. And that’s what I want to do here.

Let’s start with the aortic valve.

So, think about it for a second. I want you to picture the aortic valve.

Opening during systole, closing during diastole. That’s really what it’s supposed to do.

Go ahead and picture that in your head. And for those who are in a safe place, close your eyes because it makes more sense.

The valve is going to be open during systole and is going to be closed during diastole.

Now, who gets fed right after the aortic valve? Well, that’d be the coronary arteries.

If I’m lacking blood flow to my coronary arteries, how’s that feel? I’ve got chest pain.

What’s the second thing that gets fed after the aortic valve? Well, that’d be my internal and external carotid arteries.

If I lack blood flow to my internal or external carotids, how will I feel? Lightheaded with syncope and dizziness.

Where does blood flow third? That would be my left subclavian artery, and if I lack blood flow, then I get left arm pain. And then, of course, everything down south after that.

Aortic Stenosis

Okay, so let’s start with the first one—aortic stenosis.

So, think about this for a second. Stop memorizing. Start thinking. I have this stenotic aortic valve, and it can’t open up. So that’s my question to you. When are you going to have problems with this valve? During systole or diastole?

It is supposed to open during systole. And it won’t. It’s too stenotic.

The aortic valve is supposed to be closed during diastole, and it will close. Oh, then I see your point, then that is a systolic murmur. Right? Aortic stenosis is a systolic murmur!

Where’s the aortic valve located? The right second intercostal space. And where does it radiate to? The neck.

I want you to understand something. Who’s got to work harder to push against this stenotic aortic valve. Who’s got to work harder to open the door? It’s the left ventricle.

Okay, you’re the body. Are you going to take this, or are you going to fight back? I’m going to fight back a little bit, Joe.

So, what’s going to happen to the left ventricle? The left ventricle is going to hypertrophy.

And then go and stretch and stretch and stretch and stretch and stretch and stretch and stretch that left ventricle and you’re going to rupture the wires in there. And you’re going to get left bundle branch block.

And for those of you thinking, ahead of the game, will right ventricular hypertrophy, unopposed cause right bundle branch block? The answer is yes.

Back to aortic stenosis – I see the left ventricular hypertrophy because it’s backing up. I can even see the left bundle branch block. That makes sense.

So, aortic stenosis is a systolic ejection murmur. Oh, I see it because it’s supposed to be open during systole. But it’s a tight valve, and it won’t open up. I get it!

Let’s go one step further. It’s the most commonly acquired valvular stenosis, and usually, it’s congenital or from a bicuspid valve. But that’s not what they’re going to ask. That’s too easy. So, how would aortic stenosis present? That’s the right question.

Let’s say you have a stenotic aortic valve; it’s not opening during systole. “Hi, I have aortic stenosis, and I’m walking through Costco.”

So, you are walking through Costco, looking at the Christmas decorations or whatever they have. And how is your valve? Is it opening up? Yes, or no? No, it’s not Joe. And are you demanding more of it? You’re walking through Costco. How’s the blood flow to your coronary arteries? Oh, it’s diminished. Exactly.

Then I get chest pain with exertion – exactly. I could be walking up steps – and that’s the test question you see rear its ugly head – it’s going about the person with chest pain who’s walking up steps or exerting himself.

And then how’s your blood flow to your head? I get lightheaded and dizzy because of a lack of blood flow there. Get it? Do you see this picture?

I have this murmur in the right second intercostal space. It’s going to be systolic. The person is going to complain of chest pain and lightheadedness, and dizziness with exertion.

I can see the left ventricular hypertrophy. I can even see the left bundle branch block. Oh, that all fits.

So, they have chest pain. What do you give them? What do you give to people with chest pain? You give nitroglycerin.

And now the question is this. Well, what’s nitroglycerin? It’s a vasodilator. Where does it vasodilate? Everywhere. That’s right. I always love how people treat nitroglycerin like it’s intelligent, but it’s not. Nitroglycerin is going to vasodilate everywhere – including the left ventricle.

Wait for a second, hold on here. You just vasodilated my left ventricle, and I can’t get through the valve. Then the blood flow to my valve is not increased, but it’s decreased. And now you have less blood flow through that stenotic aortic valve. How’s your blood through the coronary arteries and in your internal and external carotids? It is incredibly diminished.

And what happens to these patients with aortic stenosis who are treated with nitroglycerine? They get syncope. See it now? It makes sense, doesn’t it?

Now, I have this systolic ejection murmur. It’s going to radiate to my neck because that’s how it’s pointing, and it’s going to be in that right second intercostal space.

Inspiration, expiration, squatting, standing, and handgrip

And now comes the questions about inspiration, expiration, squatting, standing, and handgrip! You’re like, I just can’t believe you’re going there, Joe. Yes, we’re going there. But we’re doing it my way, which is a lot more fun.

So now let’s think about this. I want you to imagine that S1 is a date with the mitral and tricuspid valve.

So, there they are, a cute little couple, the mitral and tricuspid valve come together, and they’re the S1. And now I want you to imagine S2 as a date between the aortic and pulmonic valve so S2 is the combination of the two sounds, and S1 is a combination of the two sounds.

Key points:

  • S1 is a combination of the mitral and tricuspid valve
  • S2 is a combination of the pulmonic and aortic valve

Okay. Imagine I have aortic stenosis – does my valve take shorter or longer to open up? It’s taking longer to open up.

Your pulmonic valve opened on time, but your aortic valve is late to the date!

That’s right. You took longer to get ready to open up. So, guess what happens to my S2 – It becomes split.

So, you see, a split means that someone’s late to the date. Now I need more information to figure out whether it’s pulmonic stenosis or aortic stenosis.

Key point:

  • Both aortic and pulmonic stenosis causes a split S2
  • If I have a split S2, someone is late to the date
  • Is it pulmonic or aortic? I need more information, but someone’s late to the date.

Do you remember that ridiculous bar graph that you saw in PA school? You’re like, I have no idea what I’m looking at! What they’re trying to show you is the timing of the valves. Someone is late to the date.

So, I want you to erase what you memorized in PA school. If I have a split S2, someone’s late to the date. Is it pulmonic or aortic? I need more information, but someone’s late to the date.

  • So now, do you understand the left ventricle hypertrophy?
  • Do you understand the left bundle branch block?
  • Do you see the systolic ejection murmur at the right intercostal space?
  • Do you see the split S2?

Okay. One more step. Can we all agree just from a laminar flow physics point of view that the more blood flow I put across the valve, the louder it sounds?

Think about this for a second. The more blood flow I put across the valve, the louder it sounds.

So, to make aortic stenosis sound louder? What do I need to do? Put more blood flow across the valve!

Key point:

  • To make the murmur of aortic stenosis louder, you need to send more blood flow across the valve.

Okay, so there is a party, and everybody’s showing up – Your mom, your dad, your cousins, your second cousins, the neighbors, the neighbors, friends. I mean, everybody’s showing up. Everybody’s coming back to the heart. Who’s the party maneuver? Squatting – squatting is a party!

So, what are you doing? I’m shoving all the blood flow back home. Who showed up? The entire neighborhood – and my cousins and my second cousins everybody’s showing up. So, when I squat, you know what I get? More blood flow in the heart. Guess which murmur sounds louder? All of them – mitral stenosis, mitral regurgitation, tricuspid stenosis, tricuspid regurgitation, pulmonic stenosis, pulmonic regurgitation, aortic stenosis, and aortic regurgitation – all sound louder with more blood flow back home. Who’s the maneuver? Squatting!

So, squatting really doesn’t help me much because everybody sounds louder.

Key point:

  • Think of squatting as a party – everyone comes home and brings more blood flow to your heart. This increases the sound of all the murmurs.

And then who’s your low volume maneuvers? Who makes blood flow go away from the heart? Well, that would be standing – when you stand up, that takes blood flow away from the heart. So, all the blood flows to your legs. How are all your murmurs? They go away. So, standing is a lack of blood flow, and all the murmurs are diminished.

Key point:

  • Standing is the opposite of squatting and makes all the blood flow go to your legs (away from the heart), and all the murmurs decrease in sound. 

And then Valsalva. When you grunt, what you’re doing is Valsalva or if you have a bowel movement – what you’re doing is you’re squeezing the inferior and superior vena cava, and you’re stopping blood flow from going to the right side of the heart.

You’ve got less blood flow in the heart. So, do you have any blood flow to put on my murmurs? No, it went away. So, standing and Valsalva make murmurs go away.

Squatting sounds louder. Standing and Valsalva – these are my low volume maneuvers, and the murmur goes away.

Key point:

  • Valsalva squeezes the inferior and superior vena cava reducing blood flow into the right side of the heart. So, you have less blood (low volume) and a decreased murmur.

Inspiration and Expiration

Inspiration right. Expiration left. Inspiration right. Expiration left – and for the third time. Inspiration right expiration left – to make right-sided heart murmur sound louder, you breathe in (inspiration). What do you do to make left-sided heart murmurs sound louder? Expiration!

When you inhale, you breathe air in – imagine your inferior and superior vena cava are pushing blood into your right side. So, you get more blood flow on to the right – increasing the laminar flow on the right and increasing the sound of right heart murmurs.

When you exhale and blow air out, you are decreasing that pressure in the chest. So now the blood on the left can get out easier – increasing the laminar flow across the valve and increasing the sound of the murmur on the left.

So, one more time, inspiration right and expiration left. Hi, I’m aortic stenosis – am I on the left or the right? I’m on the left.

So, what’s it going to make it louder? Expiration!

Key point:

  • With expiration, you blow air out, and you are decreasing the pressure in the chest. So now, the blood on the left can get out easier – increasing the laminar flow across the valve and increasing the sound of aortic stenosis.

Hand Grip

You learned that handgrip increases peripheral vascular resistance.

Okay. Let’s translate that to more layperson terms. When I increase my peripheral vascular resistance, I’m sitting on your aortic valve. What is your only exit valve? Yes, that’s right – it’s the aortic valve. So, if I increase your peripheral vascular resistance AKA, I’m pushing all the blood against your aortic valve AKA I’m sitting on your aortic valve.

So, I want you to get this, I want this visualization of handgrip and sitting on the aortic valve. So, I’m sitting on your aortic valve, and we’re just sitting on it. Got it?

Grab your hands and squeeze them – see how your muscles are tight? Can you visualize the blood flow getting harder to come out? So, you’re pushing it against the exit valve, which is your aortic valve. And now I want you to visualize this as sitting on the valve.

Okay, hi, I’m aortic stenosis, and I can barely open up – squeak, squeak, squeak, squeak, squeak. I can barely open the valve. I’m stenotic, and you did what? You did hand grip. You just sat on my aortic valve, and I can barely open up. Do you think I can open it up anymore?

The answer is no. Exactly. So, guess what murmur is in the right second intercostal space and goes away with handgrip? Aortic stenosis!

Key point:

  • Imagine handgrip as sitting on an already stenotic aortic valve – blood can’t get out because the valve can barely open up. And what happens to the murmur? It goes away with handgrip.

Aortic Stenosis Recap:

  • It’s a systolic ejection murmur. Makes sense? Right?
  • Second intercostal space – that’s where it’s located.
  • Most commonly congenital bicuspid. That makes sense – typically, the aortic valve has three cusps (tricuspid aortic valve), but some people are born with an aortic valve that has two cusps (bicuspid aortic valve).
  • I can’t get the flow out, so when I exert myself, I get chest pain, shortness of breath, and dizziness. That makes sense.
  • Give me nitroglycerin, and you dilate my left ventricle. Well, we’re going to fall down and go good night because less blood flow is going through to the aortic valve.
  • How’s your S2? Oh, I’m late to the date, and you’re S2 is split.
  • How’s your left ventricle? It’s going to hypertrophy because I have to fight the aortic valve.
  • And wait a second. I can stretch the muscles to a point where I break the wires. What do we get? We get left bundle branch block.
  • Inspiration right => expiration left. Expiration and squatting are a party. Everyone show’s up, and the murmur sounds louder. Standing and Valsalva. These are low volume moves, and the murmur is quieter.
  • Handgrip – I’m sitting on the valve. Blood can’t get out, so the murmur goes away.

And then, of course, how do we diagnose it? Echocardiogram!

The aortic valve is the one valve that requires an emergent replacement.  So, when somebody has aortic stenosis, what is the end game? It’s going to be an aortic valve replacement.

Aortic Regurgitation

Now, let’s discuss aortic regurgitation. Okay, with all these new tools, it’s going to be easy.

With aortic regurgitation, the valve is just flapping in the breeze, come and go as you please. I could care less – come and go as you please.

Are you going to have problems with this valve during systole? Yes, or no? No, because the valve is supposed to be open during systole.

So, when do you have problems with the regurgitating aortic valve? During diastole, when the valve is supposed to be closed!

Key point:

  • Aortic regurgitation is a diastolic murmur that will be noticeable when the valve is supposed to be closed during diastole.

It’s going to travel from my right-second intercostal space down to my apex.

Anything split? Is anybody late? No, not at all.

So, aortic regurgitation will be a diastolic murmur, and there is a wide arterial pulse pressure. They call this a water-hammer pulse. To understand this, imagine your pulse as a series of train cars. So, imagine a train, and there’s your aortic valve—pulse, pulse, pulse, pulse, pulse, pulse, pulse. But imagine if you’ve got aortic regurgitation – What do you do?

Well, I threw a pulse out there. But did you hold it out there? No, because my valve was open. So, what happened to some of that blood that you threw out there? It sneaks back home!

They call that wide arterial pulse pressure. I want you to view the arteries like this PVC pipe. You threw blood out there, didn’t you? So, it kept it full. But during diastole, I didn’t close the door. So, it all rushed back home, and the pipe collapsed. Exactly. That’s what we call a wide arterial pulse pressure, and this is what’s called a water hammer pulse.

So, I have this diastolic murmur. Right second intercostal space. Radiates to the apex. Okay. Let’s make it sound louder. What do you want to do? Oh, let’s squat – that’s a party!

To make it go away. I could stand or Valsalva. These are my low-volume maneuvers. Does inspiration or expiration make it louder? This is on the left, so expiration makes it louder.

Handgrip

So, I’m sitting on your aorta valve, and what’s the problem? You can come and go as you please. So, what will happen when we perform handgrip with aortic regurgitation is that blood flow is going to go against your valve, and it’s wide open. Blood is going to go right through it!

So now, does everybody see this question rearing its ugly head?

Instead of going for the knee-jerk answers that you’ve memorized (I call this the data closet in a student’s head), try to make sense of the question instead of going for your data closet. Think laminar flow physics.

  • So now, do you see the difference between aortic stenosis and aortic regurgitation?
  • Do you understand inspiration vs. expiration?
  • Do you see that squatting is a party? It’s not going to help with anything (squatting doesn’t do squat – it doesn’t help me delineate the cause of the murmur).
  • Inspiration, right, and expiration left.
  • My low volume maneuvers are Valsalva and standing.
  • Who challenges my aortic valve? Handgrip! And if I have aortic stenosis, it goes away, but if I have aortic regurgitation, the blood goes right through it.

And now, do you see the test question? They’re not going to tell you if it’s systolic or diastolic. That’s too easy. They’re going to tell you about some murmur in the right second intercostal space that either gets louder with handgrip (aortic regurgitation) or goes away with handgrip (aortic stenosis).

And now you will get the answer right every time!

Closing

Now go back and re-listen to this podcast episode a couple of times. I always tell people if they get a chance to listen to this podcast, do yourself a favor and find a quiet room and turn off all the lights.

And then just listen. Keep your eyes closed and visualize this in your head… and that’s just money!

Resources and links from the show

This podcast is available on every device

You can download and listen to past FREE episodes here, on iTunes, Spotify, on Google Podcasts, Stitcher, Amazon Music, and most podcasting apps.

Download the Interactive Content Blueprint Checklist

Follow this link to download your FREE copy of the PANCE/PANRE/EOR Content Blueprint Checklists

Print it up and start crossing out the topics you understand, marking the ones you don’t, and making notes of key terms you should remember. The PDF version is interactive and linked directly to the individual lessons on Smarty PANCE.

Photo Credit: Bruce Blaus, CC BY 3.0 https://creativecommons.org/licenses/by/3.0, via Wikimedia Commons

The post Podcast Episode 91: Cardiac Murmurs Made Easy Part 1 of 5 – Aortic Stenosis and Regurgitation appeared first on The Audio PANCE and PANRE.

]]>
Welcome to episode 91 of the Audio PANCE and PANRE Physician Assistant/Associate Board Review Podcast. Today is part one of an extraordinary five-part series with Joe Gilboy PA-C, all about cardiac murmurs.



Welcome to episode 91 of the Audio PANCE and PANRE Physician Assistant/Associate Board Review Podcast.



Today is part one of an extraordinary five-part series with Joe Gilboy PA-C, all about cardiac murmurs.



It’s time to throw away everything you have ever learned that made you hate cardiac murmurs (you know those crazy line diagrams) and let Joe hard-wire your brain for success.



I promise you that by the end of this podcast, you will no longer be afraid of murmurs, but you will welcome them into your brain with a whole heart, open arms, and the need to share this podcast with all your classmates.



Below is a transcription of this podcast episode slightly edited for clarity.



* You can download and listen to past FREE episodes here, on iTunes, Spotify, on Google Podcasts, Stitcher, Amazon Music, and most podcasting apps.* You can listen to the latest episode and access even more resources below.





Welcome, everybody. This is Joe Gilboy PA-C, and today is part one of a five-part series on the two most dreaded words that every pa student and recert PA hates. Are you ready for the two words?



Heart murmurs!



Today we’re going to start talking about heart murmurs, and we are going to break this down into a five-part series:



* The first part is going to be on the aortic valve (today’s episode)* The second part will cover the mitral valve* The third will be all about the pulmonic valve* The fourth part will be about the tricuspid valve* And then, in the fifth part of this series, we’re going to wrap this all together



We’re going to cover each valve one at a time. We will look at it and try to really make sense of it all.



Because I know what you did in PA school, you had that little diagram that you made, you know, systolic or diastolic murmurs. You’ve memorized certain things. And then you get to that test question, and you’re completely clueless on it, you’re like, “I have no idea what they just said, Joe, I have no idea.”



And then you’re back to scrambling, looking for some word that’s going to trigger you. And that’s the key point- all the trigger words, all those high-value trigger words, they’re gone. That’s what the creators of the PANCE did about two to three years ago. They took all the trigger words out.



Key point:



* Don’t rely on keywords (trigger words) for your PANCE because they’re gone!



So now, let’s look at this differently. You did it your way.]]>
The Physician Assistant Life | Smarty PANCE full 30:36 433
Podcast Episode 90: Ten PANCE, PANRE, and Rotation Review Questions http://podcast.thepalife.com/podcast-episode-90/ Wed, 03 Nov 2021 09:27:00 +0000 http://podcast.thepalife.com/?p=428 http://podcast.thepalife.com/podcast-episode-90/#respond http://podcast.thepalife.com/podcast-episode-90/feed/ 0 <p>Welcome to episode 90 of the Audio PANCE and PANRE Physician Assistant/Associate Board Review Podcast. Join me as I cover ten PANCE, PANRE and EOR review questions from the Smarty PANCE Instagram/Facebook page and the smartypance.com board review website. Special from today’s episode: Follow Smarty PANCE and The Daily PANCE Blueprint on Instagram Follow Smarty […]</p> <p>The post <a rel="nofollow" href="http://podcast.thepalife.com/podcast-episode-90/">Podcast Episode 90: Ten PANCE, PANRE, and Rotation Review Questions</a> appeared first on <a rel="nofollow" href="http://podcast.thepalife.com">The Audio PANCE and PANRE</a>.</p>

Welcome to episode 90 of the Audio PANCE and PANRE Physician Assistant/Associate Board Review Podcast.

Join me as I cover ten PANCE, PANRE and EOR review questions from the Smarty PANCE Instagram/Facebook page and the smartypance.com board review website.

Special from today’s episode:

Below you will find an interactive exam to complement today’s podcast.

The Audio PANCE/PANRE and EOR PA Board Review Podcast

I hope you enjoy this free audio component to the examination portion of this site. The full board review course includes over 2,000 interactive board review questions and is available to all members of Smarty PANCE.

Listen Carefully Then Take the Practice Exam

If you can’t see the audio player, click here to listen to the full episode.

Podcast Episode 90: Ten PANCE/PANRE and EOR Blueprint Questions

1. A 32-year-old male presents with acute ascending symmetrical paralysis and diminished reflexes in his bilateral lower extremities. He has a history of bloody diarrhea a few days ago. What did he likely eat that led to his symptoms?

A. Uncooked rice

B. Mayonnaise

C. Uncooked chicken

D. Soft unpasteurized cheese

E. None of the above

Click here to see the answer

C. Uncooked Chicken

This patient has Guillain-Barré Syndrome (GBS), an acute immune-mediated polyneuropathic disorder. Clinical features include ascending symmetrical paralysis, diminished tendon reflexes, and respiratory muscle weakness.

GBS is typically precipitated by an infection. Campylobacter jejuni infection is the most common precipitant of GBS. It is commonly found in uncooked poultry.

Cerebrospinal fluid analysis helps confirm the diagnosis. Treatment includes plasmapheresis and IV immune globulin.

View lesson: Guillain-Barré Syndrome

2. Which of the following disorders is characterized by a scaly, rough erythematous patch that appears on sun-exposed areas?

A. Seborrheic keratosis

B. Keratosis pilaris

C. Actinic keratosis

D. Lichen planus

E. Pityriasis rosea

Click here to see the answer

Answer: C. Actinic keratosis

Actinic keratosis (AK) is a skin lesion that results from atypical keratinocyte proliferation. Risk factors include UV radiation, fair skin, and sunburns.

AK can present in various ways, but classically you will see an erythematous, scaly macule, papule, or plaque. A shave or punch biopsy can confirm the diagnosis. However, AK is often diagnosed clinically.

Treatment depends on the location and extent of the condition. Options include cryotherapy, fluorouracil, and imiquimod. It is important to remember that AK can develop into SCC.

View Lesson: Actinic keratosis (AK)

3. A 42-year-old male is admitted for typical pneumonia. He is given IV azithromycin and ceftriaxone. His QT interval becomes prolonged and he develops a polymorphic ventricular tachycardia. Which of the following is the most appropriate pharmacological management?

A. Intravenous calcium gluconate

B. Intravenous magnesium sulfate

C. Intramuscular epinephrine

D. Oral amiodarone

E. Oral procainamide

Click here to see the answer

B. Intravenous magnesium sulfate

The patient developed torsades de pointes (Tdp), which is a specific type of polymorphic ventricular tachycardia in patients with a long QT interval. It is a ventricular dysrhythmia characterized by rapid (>100 bpm), irregular, wide QRS complexes that vary in size and shape. Diagnosis is made via EKG.

Examples of meds that prolong the QT interval include antiarrhythmics (amiodarone, sotalol), antimicrobials (fluoroquinolones, macrolides), antidepressants (fluoxetine, sertraline), and antipsychotics (haloperidol, quetiapine).

Treatment is usually IV magnesium sulfate.

View lesson: torsades de pointes (Tdp)

4. Which of the following is true about an indirect inguinal hernia?

A. It does not usually enter the scrotum

B. It is caused by a patent processus vaginalis

C. The strangulation risk is lower than a direct inguinal hernia

D. It is the least common type of inguinal hernia overall

E. It is medial to the inferior epigastric vessels

Click here to see the answer

Answer: B. It is caused by a patent processus vaginalis

A hernia is a protrusion or projection of an organ through the body wall that typically contains it. An indirect inguinal hernia is specifically a protrusion of the abdominal organ into a patent processus vaginalis that extends into the inguinal canal.

Indirect inguinal hernias are the most common type of inguinal hernia. An indirect inguinal hernia is found lateral to the inferior epigastric vessels and it usually enters the scrotum. The strangulation risk is actually higher in indirect inguinal hernias. Definitive treatment is surgery.

View lesson: Indirect inguinal hernias

5. Which of the following is the most common cause of death in hemochromatosis?

A. Hepatocellular carcinoma

B. Diabetic-related complications

C. Acute respiratory failure

D. Left ventricular failure

E. Pancreatic cancer

Click here to see the answer

Answer: A. Hepatocellular carcinoma

Hemochromatosis is an autosomal recessive disorder defined by excessive iron deposition in the liver, pancreas, and heart. It is often caused by a mutation in the HFE gene. Clinical features include skin hyperpigmentation, weakness, hepatomegaly, liver function abnormality, and diabetes mellitus.

Lab findings will show elevated serum iron, elevated ferritin, elevated transferrin saturation percentage, and decreased transferrin. Diagnostic tests include genetic testing and liver biopsy (gold standard, not always needed). Treatment consists of life-long phlebotomy. The most common cause of death in these patients is hepatocellular carcinoma.

View lesson: Hemochromatosis

6. A 15-year-old female with no medical history presents to the ER with altered mental status. She is tachycardic. Labs reveal a glucose level of 900 mg/dL and elevated beta-hydroxybutyric acid. Which of the following acid-base issues would you expect to see?

A. Normal anion gap metabolic acidosis

B. High anion gap metabolic acidosis

C. Metabolic alkalosis

D. Respiratory alkalosis

E. Respiratory acidosis

Click here to see the answer

Answer: B. High anion gap metabolic acidosis

The patient most likely has diabetic ketoacidosis (DKA), a life-threatening emergency defined by insulin insufficiency, along with an elevation in counter-regulatory hormones. Patients in DKA will usually have a high-anion gap metabolic acidosis (HAGMA).

An anion gap is calculated by subtracting anions (HCO3- and Cl-) from a cation (Na+). It is normally 8 to 12. The gap is elevated in DKA because unmeasured anions (ketones) are giving off H+, which interacts with bicarbonate. Bicarbonate becomes depleted, thus increasing the gap.

Other causes of HAGMA include acute kidney injury, lactic acidosis, and toxins.

View lesson: Acid-Base Disorders

7. Which of the following is the first-line treatment for nasal polyps in patients with chronic rhinosinusitis?

A. Anti-histamines

B. Intranasal steroids

C. Leukotriene inhibitors

D. Beta-agonists

E. None of the above

Click here to see the answer

Answer: B. Intranasal steroids

Nasal polyps are grayish masses filled with inflammatory mediators in the nasal cavity or sinuses. They are associated with aspirin sensitivity, chronic rhinosinusitis, asthma, allergies, and cystic fibrosis. Symptoms include increased nasal drainage, congestion, anosmia.

Nasal examination via nasal endoscopy can help confirm the diagnosis. The first-line treatment for nasal polyps is typically nasal corticosteroid spray. Surgery may be needed if the nasal polyp obstructs the cavity/airway.

View lesson: Nasal polyps

8. A 52-year-old female with a history of mitral valve regurgitation presents with exertional dyspnea. On physical exam, you notice abdominal swelling and lower extremity edema. You suspect pulmonary hypertension (PH). Which of the following is the gold standard for diagnosing PH?

A. Right heart catheterization

B. Chest radiograph

C. Computed tomography angiography

D. Echocardiogram

E. Polysomnography

Click here to see the answer

Answer: A. Right heart catheterization

Pulmonary hypertension (PH) is defined by a mean pulmonary arterial pressure > 25 mmHg. The World Health Organization lists 5 broad etiologies of PH: pulmonary arterial hypertension (Group 1), left heart disease (Group 2), lung disease (Group 3), chronic thromboembolism (Group 4), and unknown causes (Group 5).

Symptoms are usually related to right ventricle failure and include “body congestion” signs: pitting edema, abdominal swelling, hepatomegaly, etc. A transthoracic echocardiogram is the best initial diagnostic test. A right heart catheterization is the gold standard for diagnosing PH.

View lesson: Pulmonary hypertension (PH)

9. A 72-year-old female smoker with a history of atrial fibrillation presents to the ER with severe abdominal pain for a few hours. Vitals are unremarkable. Physical exam reveals nothing significant. Which of the following is the gold standard diagnostic modality for the most likely diagnosis?

A. Computer tomography scan

B. Right upper quadrant ultrasound

C. Abdominal radiographs

D. Mesenteric angiography

E. None of the above

Click here to see the answer

Answer: D. Mesenteric angiography

The patient most likely has acute mesenteric ischemia, which is the sudden onset of small intestinal hypoperfusion. The most common cause is emboli from the heart (usually from atrial fibrillation). Other causes include thrombosis and atherosclerotic disease. The most common artery occluded is the superior mesenteric artery.

Clinical features include acute onset of abdominal pain out of proportion of exam findings, abdominal distention, and elevated lactate. A good initial diagnostic test for a stable patient is an abdominal CT scan. The gold standard test is mesenteric angiography. Treatment is surgical revascularization and antibiotics.

View lesson: acute mesenteric ischemia

10. Orbital cellulitis most commonly occurs when an infection spreads into the orbit from which sinus?

A. Maxillary sinus

B. Frontal sinus

C. Sphenoid sinus

D. Ethmoid sinus

E. Temporal sinus

Click here to see the answer

Answer: D. Ethmoid sinus

Orbital cellulitis is an ophthalmic emergency, as it can lead to vision loss. Clinical features include diplopia, decreased vision, pain during eye movement, proptosis, fever, and an erythematous, edematous eyelid. The most common bacterial culprits are Staphylococcus aureus and Streptococcus species. The ethmoid sinus is the most common origin of infection (90%).

The best diagnostic test is a high-resolution CT scan. Treatment is intravenous antibiotics, like IV vancomycin and ceftriaxone.

View lesson: Orbital cellulitis

Looking for all the podcast episodes?

This FREE podcast series is limited to every other episode, you can download and enjoy the complete audio series by becoming a Smarty PANCE member.

I will be releasing new episodes every few weeks. Smarty PANCE is now discounted, so sign up now before it’s too late!

Resources and Links from the Show

This Podcast is available on iOS and Android

itunes_logo-1

Download the Interactive Content Blueprint Checklist

Follow this link to download your FREE copy of the PANCE/PANRE/EOR Content Blueprint Checklists

Print it up and start crossing out the topics you understand, marking the ones you don’t, and making notes of key terms you should remember. The PDF version is interactive and linked directly to the individual lessons on Smarty PANCE.

The post Podcast Episode 90: Ten PANCE, PANRE, and Rotation Review Questions appeared first on The Audio PANCE and PANRE.

]]>
Welcome to episode 90 of the Audio PANCE and PANRE Physician Assistant/Associate Board Review Podcast. Join me as I cover ten PANCE, PANRE and EOR review questions from the Smarty PANCE Instagram/Facebook page and the smartypance.



Welcome to episode 90 of the Audio PANCE and PANRE Physician Assistant/Associate Board Review Podcast.



Join me as I cover ten PANCE, PANRE and EOR review questions from the Smarty PANCE Instagram/Facebook page and the smartypance.com board review website.



Special from today’s episode:



* Follow Smarty PANCE and The Daily PANCE Blueprint on Instagram* Follow Smarty PANCE and The Daily PANCE Blueprint on Facebook



Below you will find an interactive exam to complement today’s podcast.



The Audio PANCE/PANRE and EOR PA Board Review Podcast



I hope you enjoy this free audio component to the examination portion of this site. The full board review course includes over 2,000 interactive board review questions and is available to all members of Smarty PANCE.



* You can download and listen to past FREE episodes here, on iTunes, Spotify, on Google Podcasts, Stitcher, and most podcasting apps.* You can listen to the latest episode, take an interactive quiz, and download more resources below.



Listen Carefully Then Take the Practice Exam





If you can’t see the audio player, click here to listen to the full episode.



Podcast Episode 90: Ten PANCE/PANRE and EOR Blueprint Questions



1. A 32-year-old male presents with acute ascending symmetrical paralysis and diminished reflexes in his bilateral lower extremities. He has a history of bloody diarrhea a few days ago. What did he likely eat that led to his symptoms?



A. Uncooked rice



B. Mayonnaise



C. Uncooked chicken



D. Soft unpasteurized cheese



E. None of the above






2. Which of the following disorders is characterized by a scaly, rough erythematous patch that appears on sun-exposed areas?



A. Seborrheic keratosis



B. Keratosis pilaris



C. Actinic keratosis



D. Lichen planus



E. Pityriasis rosea


]]>
The Physician Assistant Life | Smarty PANCE full 17:18 428
Podcast Episode 89: Ten PANCE, PANRE, and Rotation Review Questions http://podcast.thepalife.com/podcast-episode-89-ten-pance-panre-and-rotation-review-questions/ Tue, 21 Sep 2021 09:00:00 +0000 http://podcast.thepalife.com/?p=421 http://podcast.thepalife.com/podcast-episode-89-ten-pance-panre-and-rotation-review-questions/#respond http://podcast.thepalife.com/podcast-episode-89-ten-pance-panre-and-rotation-review-questions/feed/ 0 <p>Welcome to episode 89 of the Audio PANCE and PANRE PA Board Review Podcast. Join me as I cover ten PANCE, PANRE and EOR review questions from the Smarty PANCE Instagram/Facebook page and the smartypance.com board review website.</p> <p>The post <a rel="nofollow" href="http://podcast.thepalife.com/podcast-episode-89-ten-pance-panre-and-rotation-review-questions/">Podcast Episode 89: Ten PANCE, PANRE, and Rotation Review Questions</a> appeared first on <a rel="nofollow" href="http://podcast.thepalife.com">The Audio PANCE and PANRE</a>.</p>
Episode 89 The Audio PANCE and PANRE Board Review Podcast

Welcome to episode 89 of the Audio PANCE and PANRE PA Board Review Podcast.

Join me as I cover ten PANCE, PANRE and EOR review questions from the Smarty PANCE Instagram/Facebook page and the smartypance.com board review website.

Special from today’s episode:

Below you will find an interactive exam to complement today’s podcast.

The Audio PANCE/PANRE and EOR PA Board Review Podcast

I hope you enjoy this free audio component to the examination portion of this site. The full board review course includes over 2,000 interactive board review questions and is available to all members of Smarty PANCE.

Listen Carefully Then Take the Practice Exam

If you can’t see the audio player, click here to listen to the full episode.

Podcast Episode 89: Ten PANCE/PANRE and EOR Blueprint Questions

1. The definition of amblyopia is . . .

A. Congenital cataracts noted at birth

B. Retinal detachment seen in premature children

C. Irregular pupillary size

D. Increased distance between the medial and lateral canthus

E. Subnormal visual acuity in one or both eyes despite correction of refractive error

Click here to see the answer

Answer: E. Subnormal visual acuity in one or both eyes despite correction of refractive error

Amblyopia, also called lazy eye, is a disorder of sight in which the brain fails to process inputs from one eye and over time favors the other eye. It results in decreased vision in an eye that otherwise typically appears normal

  • Amblyopia is not correctable by refractive means
  • Amblyopia occurs in early childhood when nerve pathways between the brain and an eye aren’t properly stimulated
  • It may be caused by strabismus (crossed eye); uremia; or toxins, such as alcohol, tobacco, lead, and other toxic substances
  • Symptoms include a wandering eye, eyes that may not appear to work together, poor depth perception, blurred vision, or double vision. Both eyes may be affected

DX: Screening to detect amblyopia in all children younger than five years of age

  • Screening includes vision risk assessment at all health maintenance visits and vision screening at age three, four, and five years of age

TX: Treatment includes correction of refraction error as well as forced use of the amblyopic eye by patching the better eye

  • Some children cannot tolerate the patch, in which case the good eye is blurred with glasses or drops (penalization therapy) to stimulate proper visual development of the more severely affected eye
  • It is more resistant to treatment at an older age; thus, children should be treated early

View lesson: Amblyopia

A 45-year old woman being managed for ulcerative colitis, developed abdominal pain, vomiting, diarrhea, the passage of blood and mucus per rectum, and fever. On examination, she was pale, febrile (temp: 102.2 C), moderately dehydrated, heart rate: 124bpm. There was abdominal distention and tenderness, bowel sounds were hypoactive. Lab results showed Hb: 9g/dl, WBC: 14 x 109/L, elevated CRP. Stool was negative for C. difficile. HIV status was negative. Abdominal radiograph showed dilated transverse colon of about 11 cm. What is the most likely diagnosis of this patient?

A. Hirschsprung’s disease

B. Cytomegalovirus colitis

C. Toxic megacolon

D. Kaposi’s sarcoma

Click here to see the answer

Answer: C. Toxic megacolon

The hallmarks of toxic megacolon (toxic colitis) are nonobstructive colonic dilatation (>6 cm) and signs of systemic toxicity. It occurs following complications from causes of colitis e.g. ulcerative colitis as is the case in the index patient.

Incorrect Answers:

  • Hirschsprung’s disease (choice A) presents with chronic constipation. Patients are not usually toxic except when intestinal perforation occurs.
  • Cytomegalovirus colitis and Kaposi’s sarcoma (choice B and D) Occurs in immunocompromised persons.

View lesson: Toxic megacolon

3. Which of the following is not transmitted by blood?

A. Hepatitis A

B. Hepatitis B

C. Hepatitis C

D. Hepatitis D

Click here to see the answer

Answer: A. Hepatitis A

Hepatitis A is transmitted via the fecal-oral route

Incorrect Answers:

  • Hepatitis B can be transmitted via blood and blood products through close living quarters/playground play as a toddler, vertical transmission, infected unscreened blood, needle stick injury, etc.
  • Hepatitis D is transmitted via blood.
  • Hepatitis C is transmitted via blood and blood products through Intravenous drug misuse, unscreened blood products, needle stick injury, vertical transmission etc

View lesson: Acute and Chronic Hepatitis

4. When is PKU testing performed?

A. 24 weeks gestation

B. 24 to 48 h after birth

C. 48 to 72 h after birth

D. at the first well-child visit

Click here to see the answer

Answer: B. 24 to 48 h after birth

In the US and many developed countries, all neonates are screened for PKU 24 to 48 h after birth. Tandem mass spectrometry is the method of choice. It has a low false-positive rate, and excellent accuracy and precision.

View lesson: Phenylketonuria

5. A 20-year-old female with a history of type 1 diabetes presents to the clinic complaining of weight loss, increased flatulence, foul-smelling stools, and a pruritic rash. You notice multiple papules and vesicles on the extensor surfaces of the elbows. Which of the following is most likely to be positive?

A. Anti-cyclic citrullinated peptide antibody

B. Anti-dsDNA antibody

C. Anti-tissue transglutaminase antibody

D. Anti-topoisomerase antibody

E. Anti-beta 2 glycoprotein 1antibody

Click here to see the answer

Answer: C. Anti-tissue transglutaminase antibody

The patient has Celiac disease, which is an immune disorder characterized by sensitivity to gluten. It is more common in patients with Type 1 diabetes, autoimmune thyroiditis, Trisomy 21, and Turner syndrome.

Clinical features include diarrhea with bulky, foul-smelling floating stool, flatulence, and weight loss. Patients may have dermatitis herpetiformis, which is a rash located on the elbows, forearms, and knees.

Laboratory studies to order include anti-tissue transglutaminase antibodies and IgA anti-endomysial antibodies. The best diagnostic test is an upper endoscopy with a small bowel biopsy. Patients must avoid food with gluten, which includes barley, rye, oats, and wheat.

Incorrect Answers:

  • Anti–cyclic citrullinated peptide antibodies (choice A) are highly specific to rheumatoid arthritis.
  • Anti-dsDNA antibodies (choice B) are a specific marker for systemic lupus erythematosus.
  • Anti-topoisomerase antibodies (choice D) are specific to diffuse scleroderma.
  • anti-beta 2 glycoprotein 1 antibodies (choice E) are specific to antiphospholipid syndrome.

View lesson: Celiac disease

6. Which of the following is the most common cause of postpartum hemorrhage?

A. Severe vaginal laceration

B. Uterine atony

C. Hematoma

D. Retained products of conception

E. Placental abruption

Click here to see the answer

Answer: B. Uterine atony

Postpartum hemorrhage is an obstetric emergency. It is defined as an estimated blood loss ≥ 500 mL during vaginal delivery or ≥ 1000 in a cesarean section. The most common cause is uterine atony.

Uterine atony presents as a soft, boggy uterus after delivery and should initially be managed with fundal massage and IV oxytocin.

In general, the main focus of managing postpartum hemorrhage should be (1) resuscitation and management of hypovolemic shock and (2) identifying and treating the underlying cause.

View lesson: Postpartum hemorrhage

7. A 37-year-old female with no past medical history is brought in by EMS. She is hypotensive. On physical exam, you notice muffled heart sounds and jugular vein distention. What is the most appropriate initial treatment for this condition?

A. Emergent needle decompression

B. IV beta-blockers

C. Pericardiocentesis

D. Thrombolytics

E. None of the above

Click here to see the answer

Answer: C. Pericardiocentesis

The patient is experiencing cardiac tamponade, which is defined as fluid accumulation in the pericardial sac. The most common cause of cardiac tamponade is trauma. Other causes include malignancy, uremia, infection, and iatrogenic.

Patients may have Beck’s Triad – hypotension, muffled heart sounds, and jugular vein distention. They may also have pulsus paradoxus (large decrease in systolic blood pressure on inspiration).

The best diagnostic test is an echocardiogram. The treatment of cardiac tamponade is pericardiocentesis. An open surgical approach (i.e., thoracotomy) is typically needed for traumatic cases though.

Incorrect Answers:

  • Emergent needle decompression (choice A) is the treatment of choice for a tension pneumothorax, which is when air rapidly accumulates in the pleural space. It typically presents as shortness of breath.
  • IV beta-blockers (choice B) would be the management of an aortic dissection, not cardiac tamponade. An aortic dissection presents with sharp chest pain, pulse deficits, and hypotension.
  • Thrombolytics (choice D) would be the preferred treatment for an unstable patient with a massive pulmonary embolism.

View lesson: Cardiac Tamponade

8. Which of the following should be avoided in patients who have an acute kidney injury?

A. NSAIDs

B. Aminoglycosides

C. Macrolides

D. Acetaminophen

E. Both A and B

Click here to see the answer

Answer: E. Both A and B

Acute kidney injury (AKI) is defined by an acute decline in renal function. You will see an increase in serum creatinine with possible oliguria or anuria.

AKI can be broken up into 3 main etiologies: pre-renal, intrinsic, and post-renal. The most common cause of acute kidney injury overall is reduced perfusion. Prolonged pre-renal AKI can lead to ischemic acute tubular necrosis.

Nephrotoxic agents, such as NSAIDs, vancomycin, aminoglycosides, and ACEI/ARBs, should be avoided. Treatment is managing the underlying cause, plus correcting acid-base & electrolyte issues.

View lesson: Acute Renal Failure

9. A 38-year-old male presents to the ER with acute onset of dyspnea, dizziness, chest pain, and diaphoresis, He has had 4 episodes similar in the past. Vital signs are remarkable for mild tachycardia (103 bpm). Lab and imaging studies are insignificant. Which of the following is the best long-term medical treatment for the likely diagnosis?

A. Benzodiazepines

B. Selective serotonin reuptake inhibitors

C. Atypical antipsychotics

D. Calcium channel blockers

E. Melatonin

Click here to see the answer

Answer: B. Selective serotonin reuptake inhibitors

The patient most likely has panic disorder. The DSM V criteria for panic disorder include the following:

  • 1 or more panic attack that reoccurs without warning and at least 4 or more symptoms
  • Must be worried about having another attack or change behaviors

Symptoms of a panic attack include diaphoresis, chest pain, lightheadedness, shortness of breath, palpitations, sense of doom, or fear of dying. The treatment of panic disorder is selective serotonin reuptake inhibitors. Patients may benefit from taking hydroxyzine or benzodiazepines on a short-term, as-needed basis.

View lesson: Panic Disorder

10. Which of the following best represents values you would expect in a patient with SIADH?

A. High serum sodium, high serum osmolality, low urine osmolality

B. High serum sodium, high serum osmolality, high urine osmolality

C. Low serum sodium, low serum osmolality, low urine osmolality

D. Low serum sodium, low serum osmolality, high urine osmolality

E. None of the above

Click here to see the answer

Answer: D. Low serum sodium, low serum osmolality, high urine osmolality

SIADH or syndrome of inappropriate antidiuretic hormone secretion is characterized by the excessive release of ADH. ADH increases the permeability of the distal convoluted tubule and collecting ducts, increasing the reabsorption of water. This decreases plasma osmolality, increases urine osmolality, and leads to hyponatremia.

Causes include intrathoracic diseases (pneumonia, TB, acute respiratory failure, lung cancer), neurological issues (stroke, hemorrhage, tumors), and drugs (NSAIDs, opioids, ecstasy, and SSRIs). The goal is to determine and manage the underlying cause. Fluid restriction is often used as well.

View lesson: Syndrome of inappropriate antidiuretic hormone secretion

Looking for all the podcast episodes?

This FREE podcast series is limited to every other episode, you can download and enjoy the complete audio series by becoming a Smarty PANCE member.

I will be releasing new episodes every few weeks. Smarty PANCE is now discounted, so sign up now before it’s too late!

Resources and Links from the Show

This Podcast is available on iOS and Android

itunes_logo-1

Download the Interactive Content Blueprint Checklist

Follow this link to download your FREE copy of the PANCE/PANRE/EOR Content Blueprint Checklists

Print it up and start crossing out the topics you understand, marking the ones you don’t, and making notes of key terms you should remember. The PDF version is interactive and linked directly to the individual lessons on Smarty PANCE.

The post Podcast Episode 89: Ten PANCE, PANRE, and Rotation Review Questions appeared first on The Audio PANCE and PANRE.

]]>
Welcome to episode 89 of the Audio PANCE and PANRE PA Board Review Podcast. Join me as I cover ten PANCE, PANRE and EOR review questions from the Smarty PANCE Instagram/Facebook page and the smartypance.com board review website.



Welcome to episode 89 of the Audio PANCE and PANRE PA Board Review Podcast.



Join me as I cover ten PANCE, PANRE and EOR review questions from the Smarty PANCE Instagram/Facebook page and the smartypance.com board review website.



Special from today’s episode:



* Follow Smarty PANCE and The Daily PANCE Blueprint on Instagram* Follow Smarty PANCE and The Daily PANCE Blueprint on Facebook



Below you will find an interactive exam to complement today’s podcast.



The Audio PANCE/PANRE and EOR PA Board Review Podcast



I hope you enjoy this free audio component to the examination portion of this site. The full board review course includes over 2,000 interactive board review questions and is available to all members of Smarty PANCE.



* You can download and listen to past FREE episodes here, on iTunes, Spotify, on Google Podcasts, Stitcher, and most podcasting apps.* You can listen to the latest episode, take an interactive quiz, and download more resources below.



Listen Carefully Then Take the Practice Exam





If you can’t see the audio player, click here to listen to the full episode.



Podcast Episode 89: Ten PANCE/PANRE and EOR Blueprint Questions



1. The definition of amblyopia is . . .



A. Congenital cataracts noted at birth



B. Retinal detachment seen in premature children



C. Irregular pupillary size



D. Increased distance between the medial and lateral canthus



E. Subnormal visual acuity in one or both eyes despite correction of refractive error






A 45-year old woman being managed for ulcerative colitis, developed abdominal pain, vomiting, diarrhea, the passage of blood and mucus per rectum, and fever. On examination, she was pale, febrile (temp: 102.2 C), moderately dehydrated, heart rate: 124bpm. There was abdominal distention and tenderness, bowel sounds were hypoactive. Lab results showed Hb: 9g/dl, WBC: 14 x 109/L, elevated CRP. Stool was negative for C. difficile. HIV status was negative. Abdominal radiograph showed dilated transverse colon of about 11 cm. What is the most likely diagnosis of this patient?



A. Hirschsprung’s disease



B.]]>
The Physician Assistant Life | Smarty PANCE full 17:01 421
How to Be a Better PA Part 2: Medical Staff, Administrators, and Supervisors http://podcast.thepalife.com/how-to-be-a-better-pa-part-2-medical-staff-administrators-supervisors/ Tue, 24 Aug 2021 07:01:00 +0000 http://podcast.thepalife.com/?p=418 http://podcast.thepalife.com/how-to-be-a-better-pa-part-2-medical-staff-administrators-supervisors/#respond http://podcast.thepalife.com/how-to-be-a-better-pa-part-2-medical-staff-administrators-supervisors/feed/ 0 <p>This is part two of a special three-part podcast series by Joe Gilboy PA-C on becoming a better PA. Part one was all about building better relationships with our nurses and ancillary staff. In part two we cover essential strategies on how to stay off the medical staff radar and get the medical staff, administrators, […]</p> <p>The post <a rel="nofollow" href="http://podcast.thepalife.com/how-to-be-a-better-pa-part-2-medical-staff-administrators-supervisors/">How to Be a Better PA Part 2: Medical Staff, Administrators, and Supervisors</a> appeared first on <a rel="nofollow" href="http://podcast.thepalife.com">The Audio PANCE and PANRE</a>.</p>

This is part two of a special three-part podcast series by Joe Gilboy PA-C on becoming a better PA.

Part one was all about building better relationships with our nurses and ancillary staff.

In part two we cover essential strategies on how to stay off the medical staff radar and get the medical staff, administrators, and supervisors on your side.

Let’s jump right into this special episode of The Audio PANCE and PANRE Podcast.

You can listen to the podcast below and read the summarized (and edited version) of the transcript or listen in your podcast player of choice.

The Audio PANE/PANRE Podcast Episode 86: How to be a Better PA (Part 2 of 3)

You can also click here to listen to or download this episode.

Episode Transcript and Summary

This episode was recorded by Joe Gilboy PA-C and edited for clarity and readability by Stephen Pasquini PA-C.

Welcome, everybody. This is Joe Gilboy, and this is a podcast to have a series of three podcasts on how to become a better PA.

Today, we’re going to talk about medical staff, our administrators, and our supervisors, whether a nursing supervisor, PA supervisor, or possibly physician supervisor.

What I will be teaching you here in this podcast is how to interact with them and how to deal with them. And most importantly, how to have you make a better reputation for yourself and how to make you a better PA.

  • First, we’re going to talk about the medical staff, which is a big group of people.
  • Second, we need to learn how to deal with administrators. These can also be the office managers at the clinics where you work.
  • And then your supervisors, these are the people above you, whether a nurse, a PA, or a doc.
  • Lastly, we will covering how to deal with (or leave) a toxic job

So, let’s talk about all these three people we’re going to have to interact with.

How to be a Better PA with Medical Staff

Let me explain who medical staff really are, not whom you think there are, but I’m going to show you who they really are.

I’ve been a physician assistant in the emergency room for 35 years. You would think that after 35 years, I’ve developed some tough skin. But there’s one group of people that I fear the most, and you will begin to fear them as well. Who are they? The medical staff!

Everybody has a boss. For example, you have a boss, and your boss has a boss, they have a boss, everybody has a boss. But there’s one person who’s at the very top of this pyramid. That’s medical staff.

So, when medical staff asked for something, when do you get it done? Now? Yes, right now, like yesterday was too late.

So, the medical staff has asked for your ACLs card, PALS card, NCCPA card, or DEA number. They want a TB test, a flu shot, or they want your antibody titer for chickenpox.

When are we getting it done?

Now! No, literally right now. Because you see, the last people you want to be on the radar is medical staff.

Let me share with you who medical staff really is.

What’s their job?

Are you ready for this? I hope you’re sitting down?

Their job is to fire you.

Yeah, you heard me. They want to find a problem. That’s their job.

So, when they start asking for things, they’re looking for a problem. So, let’s just say a young physician assistant is working in urgent care that we will call Betty.

So, here’s Betty, the new PA working at the urgent care. Medical staff wants something, and she’s like, well, you know, I’ll get to it tomorrow. I have a birthday party to go to this weekend. And you know, and then it’s my parents, wedding anniversary, I’m going to have that. And then my boyfriend’s taking me to this. And you know, I’ll get back to it when I get back in town. And as this time has gone by, Tick-tock, tick-tock. She’s getting these new emails in her inbox from the medical staff. They’ve sent one, they’ve sent two, they sent three. Now they send one to her, and they’re saying, hey, if you don’t get this done in 10 days, you’re on suspension!

Guess who gets the seat in that email? Your boss.

Let me explain to you how this plays itself out. You see, at the end of the year, when you’re up there for that yearly evaluation, they’re going to say, “Hey, so how’s our new physician assistant? How’s she doing?”

Well, they’re going to talk to a group of people. They’re going to talk to the nurses. They’re going to talk to the physicians. And guess who else they are going to talk to, folks? That’s right, the medical staff.

“Are there any dings with this person? Have they no gotten things done on time?”

Now think about what I’m saying hard.

So, imagine you’re the boss. And there’s these two PAs that show up. One PA gets everything done on time, and the medical staff doesn’t even know they exist.

They’re like, yeah, this person gets his TB test and flu shot and antibody titers done on time.

There’s this other PA. Well, we had him on suspension for a week because he wouldn’t get a TB test. And you’re the boss, and it’s time for her yearly evaluation. Whom do you want to give the raise to?

Yeah, let that bounce around in your head for a second.

Whom do you want to give the raise to? The person that medical staff has no idea that even exists in the department because they’re getting everything done on time or a PA that they had to put on suspension for ten days?

Now, do you get it, guys?

You see, I hate to say this, but getting along with the medical staff is not only the way to preserve your job. But it’s also the way to make more money.

Because when it’s time for that yearly evaluation, your boss is going to look at you like, “Hey, you show up to work on time, you get along with the nurses, and you get along with the medical staff.” They want to keep you.

That’s when you can turn the screw and say, okay, well, to keep me, I would like $10 more an hour, or more vacation time or whatever it is that you’re trying to angle at. That’s how this works, folks.

I get these new grads who get out of school. They tell their supervising physician that they want to make $80 an hour, right? And let’s just say the going rate is $65. And the doc will say, well, wait a sec, you know, I start new PAs off at $65. They’re going to go well; I’m a recent grad, and I deserve this. I deserve that. Or I went to some fancy, dancy school.

Now, can you understand that at the end of the day, your boss does not care where you went to school? It means absolutely nothing. That’s just a piece of paper. What’s going to mean everything to your boss?

Can you see 20 patients, not kill anybody, get along with the nurses, get along with the staff? And most importantly, doesn’t get on the medical staff’s radar.

This is about your reputation.

You want to be the PA that medical staff doesn’t even know exist.

You’re like, so when does this girl get her TB or guy get a TB test, always on time, gets his flu shot on time gets his antibodies done on time. I don’t even know his DEA numbers expire because he turns it in on time. That’s who you want to be.

So never, ever, ever get on medical staff’s radar.

I am going to give you a little inside story. I got an email. This was about nine o’clock in the morning. I got an email from medical staff, and they wanted antibody titers. They’re like, “you know, hey, Joe, we need your antibody titer, Hep B titer, measles titer, and chickenpox titer.”

So that was nine o’clock in the morning. I was in the walk-in clinic at noon, getting my antibodies, titer tests. And then the lady looked at goes I said, Yeah, there’s something about a DPT shot, and she goes, you know, our records down, we can’t pick them up. I said, just give them to give it to me. And lo and behold, they did my antibody titer. And they gave me a DPT shot. And believe it or not, guess who got two DPT shots last year? Yeah, that’s me.

The medical staff emailed me back that afternoon. I emailed them back within four hours. Everything’s been done. And they emailed me back saying, “Hey, thank you for getting things done so expeditiously.”

Does everybody understand my boss is looking at me like the medical staff doesn’t even know you exist? They’re kind of worried because you got two DPT shots. I’m like, you know, I got twice the amount of antibodies. I’m not really worried about it.

So now, at the end of the day, this is what I want to share with you. Don’t ever get on the radar. They want something – you get it done. Right. Now, if you do that, guys, he’ll do great.

How to be a Better PA with Administrators

So now the next group we need people we need to talk about is the administrators.

So, when the administrators come downstairs to your hospital, urgent care, walk-in clinics, surgery center, wherever it is, be polite, and be cordial. Be respectful to them. Say hey, it’s great to see you, how have you been you know, it’s you know, I hope all is well with you and sounds great.

So, if you’re cordial and polite with them and respectful, they’ll remember you. And then when that yearly evaluation comes up, the administrator can go to your boss (who’s your boss’s boss), and they go, hey, you know, I met one of your PAs the day ago, great guy. Very nice. He was very polite and very cordial, and very respectful to me. You’ve got a good kid on board.

And now your boss is going to go, “wait a second. I need to keep this new grad, maybe young pa, on board. That’s what you want to do.

So, in other words, be polite, be cordial. Be respectful to your administrators, and remember, they’re the ones that make policies.

I think it is kind of funny that everyone argues so much about who’s in charge of making policies. At the end of the day, guys, it all boils down to policies. That’s what you must play under.

These are the rules. They’re not the referees. You must play by these rules. You can get mad at the referee, but he’s just enforcing the rule.

So, think of administrators as the person who is just enforcing the policy. So, when you see the administrators, be polite, be cordial, be respectful.

How to be a Better PA with Supervisors

The last group of people that we need to be aware of is our supervisors. So, whether this is a nursing supervisor, PA supervisor, or physician supervisor, they are your supervisor. And I keep telling everybody this again: be respectful, kind, cordial, and very workable.

You want to be the PA that everyone wants to work with. You want to be the PA that when you walk into the OR suite, or you walk into the clinic, or you walk into the surgery center or walk into, you know, medical office or you walk into the ER, you’re going to be that PA.

“Oh my gosh, it’s going to be a great day. Look who’s here.” So, when the nursing supervisor comes up and asks you a question, again, be polite, cordial, and respectful. The same thing with your peers. Most people have a PA supervisor, right?

Just be polite and cordial with them. Your MD supervisors, you want to be that pa that does everything?

No, listen to me. You see, you want to be the PA that the MD is like, oh, god, this person’s work with me today!

I know they’ll pick up the extra patients. They’ll pick up the pace. They’ll do better in the OR. They’re much more respectful. They’re much more cordial. This is the person I want to work with, that guy or that like that girl, that’s who I want to work with, it’s them.

And this is what you need. Because you see at the end of the day, guys, what I’m really trying to share with you. And I’m trying to tie this all in with you. This is about reputation.

You see, guys, at the end of the day, no one cares where you went to school. I mean, I know it’s a congratulation from whatever university or PA program you graduate from. Congratulations.

Okay. But no one cares. No one will care about your GPA. No one will care about your PANCE or your PANRE score. They’ll mean absolutely zero.

What will mean everything, your reputation?

What kind of person is this individual like to work with? Are they good? Are they bad? Do they get along with the staff? Do they get along with medical staff? Do they get along with the administrators? Do they get along with the supervisors?

That’s what’s going to count, folks. Because at the end of the day, let’s just say you leave your job and go to the next job. Who do you think they’re going to call?

Yeah, they’re going to call one of your supervisors. And what are they going to say? They’re not going to say where you went to school. They are not going to say what your GPA was, your PANCE or PANRE score was?

They’re going to say what kind of person you are to work with, which is based upon your reputation.

This is what I keep trying to hammer home is. So how does everyone understand where I’m trying to with this

In other words, the medical staff wants something. When am I getting it done? Now?

How am I going to be to the administrators when they come downstairs? I’m not going to be afraid of them. I’ll walk up to them and shake their hand. I’m polite cordial respectful.

My supervisors, what am I going to be? Polite, cordial, and respectful.

I’m going to be that workable PA? Hey, can you go see these extra four patients? Do you mind staying an hour later? I’d love to. I’d love to stay 10 hours later. Thank you. Can I have another?

That’s who you want to be. Not the person who is like, “well, I need to leave a half-hour early, and can I get my hour lunch? No, I want a two-hour lunch. You know, I just don’t feel like coming in today because I just I’m not feeling it. You know, my chakra’s not in line. You know, I’m just not feeling it.”

Yeah, this is not who you want to be. This is not who they want to hire.

They want the person they can rely upon. They want the person that they can trust the reputation on, and this is what I’m trying to share with you.

The Toxic Job

One other little piece here because I know many of you guys are going to ask this question. Hey, Joe. I’ve got this really toxic supervisor. I got this toxic nursing supervisor, I’ve got this toxic PA supervisor, or I got this toxic physician supervisor, so just say you’ve got a toxic person in your crew, right?

And trust me, I’ve been here, guys; I have Pandora’s box full of bad memories here. I just want to keep this box closed. So, I’m on your side here.

So, what do you do with the toxic nursing supervisor?

Two things. So, whether it be a toxic nursing supervisor toxic, physician assistant supervisor, toxic physician supervisor? So what do I do?

Okay, so this is the way you get out of this. This is how you thread the needle.

One, you go up to them directly. So, let’s just say you have a toxic nursing supervisor, right? So, you go up to whoever this nursing supervisor is and say, “listen, I find some of the things you inappropriate, I don’t find this working for the best interest of the department.”

And remember, that’s a buzz word, best interests of the department or the office, the surgery center, or the ER, this isn’t serving the best interests of our department.

“So, if you don’t mind, I would appreciate it, if you change your policy or attitude towards me or to the group of people.”

So, the first thing you do is you confront you lay down your complaint.

You also followed it up with an email.

Dear nurse supervisor or PA supervisor, or MD supervisor. The other day, we spoke about (x,y,x). Thank you for your time consideration. Sincerely, Joe.

You have verbally stated this to them. And now you have an email, which is very important.

Now, let’s say that they continue to be toxic. Now, what do you do?

Everybody has a boss. Now, you go to their boss’s boss, except this time, you’ve got documentation.

And you say, “so, I presented this to my nursing or physician supervisor, and this is what I presented to them, and they continue this behavior.”

I’ve been at some toxic jobs. I just look back, and I find it amazing that I ever survived. So, what do I recommend when people are in a toxic job?

You’ve completed the steps above. You’ve politely confronted them and verbalized this in an email, and you went to a higher source, and nothing got done.

So, when you leave this toxic job, how do you do it?

You be polite, be respectful, be cordial.

“This is my two-month resignation. Thank you for your time consideration.

You don’t even bring it up. You just exit.

Okay. So, can we all agree that most of us sleep 6-8 hours a day, which means we’re awake for 16, right? Eight to 10 hours is going to be at work.

My wife’s a writer. So, she’s off doing her writer stuff, right? I might see her for a couple of hours. I don’t see her that much.

But my nurses, are you kidding me? I see them eight to 10 hours a day. So, you see, the people you work with are your family. You’re going to spend more time with these people than your own family.

If you have a miserable family at work, you will take home a miserable attitude and then take it out against the people you love the most.

So, these toxic jobs that you sometimes get yourself into. Get out. Get out as soon as you can!

This will literally ruin your soul. And then you’ll take it home to the people you love the most.

“But I’m making so much money.”

I see this all the time; I hear this from all my students all the time.

“But Joe I was making such good money.”

One of the things you learn about life as you get older, it’s not always about money. I agree that money helps. It helps pay off your student loan, mortgage, car payments, bills, etc. I get it, I’ve got the same things.

But at the end of the day, I’m happy with the family I have in my ER. I get along great with everybody. I get along with the medical staff. I get along with the administrators, and I get along to supervisor. I get along with almost everybody. It’s my family.

And so now, when I come home, I’m not in a bad mood. I don’t have any toxic stew on me. So, I can spend time with my wife and my daughter, and my dogs. And I’m just Joe. Nothing came home with me.

Your Reputation Matters

And this is what I keep trying to share with you guys. This is how you develop a good, healthy reputation as a PA.

And now if you go to the next job, what’s your boss going to say?

Oh, this guy is great. Or this girl is great. I don’t want to give her up. She gets along with everybody. You’re the luckiest person in the world to get her. Get him.

That’s what you want to do.

So, listen, guys, it’s been enjoyable. I hope this podcast has helped you out. And this is podcast two or three. The third and the final podcast is getting ready to come up. And at that podcast. We’re going to wrap the first two podcasts up. And what are we going to talk about? Your reputation.

Take care, guys.

Joe Gilboy PA-C

The post How to Be a Better PA Part 2: Medical Staff, Administrators, and Supervisors appeared first on The Audio PANCE and PANRE.

]]>
This is part two of a special three-part podcast series by Joe Gilboy PA-C on becoming a better PA. Part one was all about building better relationships with our nurses and ancillary staff. In part two we cover essential strategies on how to stay off t...



This is part two of a special three-part podcast series by Joe Gilboy PA-C on becoming a better PA.



Part one was all about building better relationships with our nurses and ancillary staff.



In part two we cover essential strategies on how to stay off the medical staff radar and get the medical staff, administrators, and supervisors on your side.



Let’s jump right into this special episode of The Audio PANCE and PANRE Podcast.



You can listen to the podcast below and read the summarized (and edited version) of the transcript or listen in your podcast player of choice.



The Audio PANE/PANRE Podcast Episode 86: How to be a Better PA (Part 2 of 3)





You can also click here to listen to or download this episode.



Episode Transcript and Summary



This episode was recorded by Joe Gilboy PA-C and edited for clarity and readability by Stephen Pasquini PA-C.



Welcome, everybody. This is Joe Gilboy, and this is a podcast to have a series of three podcasts on how to become a better PA.



Today, we’re going to talk about medical staff, our administrators, and our supervisors, whether a nursing supervisor, PA supervisor, or possibly physician supervisor.



What I will be teaching you here in this podcast is how to interact with them and how to deal with them. And most importantly, how to have you make a better reputation for yourself and how to make you a better PA.



* First, we’re going to talk about the medical staff, which is a big group of people. * Second, we need to learn how to deal with administrators. These can also be the office managers at the clinics where you work.* And then your supervisors, these are the people above you, whether a nurse, a PA, or a doc.* Lastly, we will covering how to deal with (or leave) a toxic job



So, let’s talk about all these three people we’re going to have to interact with.



How to be a Better PA with Medical Staff







Let me explain who medical staff really are, not whom you think there are, but I’m going to show you who they really are.



I’ve been a physician assistant in the emergency room for 35 years. You would think that after 35 years, I’ve developed some tough skin. But there’s one group of people that I fear the most, and you will begin to fear them as well. Who are they? The medical staff!



Everybody has a boss. For example, you have a boss, and your boss has a boss, they have a boss, everybody has a boss. But there’s one person who’s at the very top of this pyramid. That’s medical staff.



So, when medical staff asked for something, when do you get it done? Now? Yes, right now, like yesterday was too late.



So, the medical staff has asked for your ACLs card, PALS card, NCCPA card, or DEA number. They want a TB test, a flu shot,]]>
The Physician Assistant Life | Smarty PANCE full 30:55 418
Podcast Episode 87: Ten Internal Medicine EOR Questions http://podcast.thepalife.com/podcast-episode-87-ten-internal-medicine-eor-questions/ Mon, 14 Jun 2021 09:00:00 +0000 http://podcast.thepalife.com/?p=413 http://podcast.thepalife.com/podcast-episode-87-ten-internal-medicine-eor-questions/#respond http://podcast.thepalife.com/podcast-episode-87-ten-internal-medicine-eor-questions/feed/ 0 <p>Welcome to episode 87 of the Audio PANCE and PANRE PA Board Review Podcast. Join me as I cover ten internal medicine rotation EOR content blueprint questions from the Smarty PANCE physician assistant board and rotation review website. Special from today’s episode: Check out the updated Smarty PANCE Internal Medicine Rotation (EOR) Review Course Download […]</p> <p>The post <a rel="nofollow" href="http://podcast.thepalife.com/podcast-episode-87-ten-internal-medicine-eor-questions/">Podcast Episode 87: Ten Internal Medicine EOR Questions</a> appeared first on <a rel="nofollow" href="http://podcast.thepalife.com">The Audio PANCE and PANRE</a>.</p>
Podcast Episode 87 - Ten Internal Medicine EOR Questions

Welcome to episode 87 of the Audio PANCE and PANRE PA Board Review Podcast.

Join me as I cover ten internal medicine rotation EOR content blueprint questions from the Smarty PANCE physician assistant board and rotation review website.

Special from today’s episode:

Below you will find an interactive exam to complement the podcast.

The Audio PANCE/PANRE and EOR PA Board Review Podcast

I hope you enjoy this free audio component to the examination portion of this site. The full board review course includes over 2,000 interactive board review questions and is available to all members of Smarty PANCE.

Listen Carefully Then Take The Practice Exam

If you can’t see the audio player click here to listen to the full episode.

Podcast Episode 87: Ten Internal Medicine EOR Pulmonology Questions

The following questions are linked to PAEA Content Blueprint lessons from the Smarty PANCE and PANRE Board Review Website. If you are a member, you will be able to login and view this interactive video lesson.

1. A 67-year-old man with a long history of constipation presents with steady left lower quadrant pain. Physical exam reveals low-grade fever, mid abdominal distention, and lower left quadrant tenderness. Stool guaiac is negative. An absolute neutrophilic leukocytosis and a shift to the left are noted on the CBC. Which of the following is the most accurate test for this patient’s condition?

A Barium enema
B Colonoscopy
C CT scan
D. Sigmoidoscopy

Click here to see the answer

Answer: C CT scan

Diverticulitis presents with left lower quadrant abdominal pain, systemic symptoms (such as fever), nausea, vomiting, and leukocytosis on lab values. It typically occurs in a patient with a history of diverticulosis. The most accurate test for diverticulitis is a CT scan. Colonoscopy is contraindicated as it could cause rupture. Further management is keeping the patient NPO and administering ciprofloxacin, metronidazole, and IV fluids. Patients who can not tolerate oral antibiotics should be started on IV antibiotics. (Review topic: Diverticular disease)

  • Barium enema is contraindicated in diverticulitis as it could cause irritation. A barium enema is the most accurate test in diverticulosis.
  • Colonoscopy and sigmoidoscopy are contraindicated in diverticulitis as they could cause rupture. Colonoscopy is an accurate test for diverticulosis.

2. A 42-year-old homeless man presents to the emergency department with fever, painful muscle spasms in his arms and legs, and difficulty eating because of painful spasms in his jaw muscles. Until a week ago, he was wandering around the city looking for food and work and taking shelter in a commercial construction site. He reports not having seen a medical professional in more than 15 years. Examination of his feet reveals shoes with holes in the soles and a small, puncture-type wound on the bottom of the right foot. It is surrounded by erythema and somewhat tender to touch. The patient is uncertain what he may have stepped on. X-ray is negative for any radiopaque foreign body. In addition to hospital admission, which of the following is the first-line therapy for this patient?

A Tetanus immune globulin and tetanus toxoid
B Tetanus immune globulin and metronidazole
C Tetanus toxoid and penicillin
D Tetanus immune globulin, tetanus toxoid, and metronidazole

Click here to see the answer

The answer is D Tetanus immune globulin, tetanus toxoid, and metronidazole

Clostridium tetani infection is a vaccine-preventable disease that results in approximately 50 cases per year in the United States. Even with modern medical resources, 20% to 25% of patients with generalized tetanus die. Treatment includes airway protection, benzodiazepines for muscle spasm, tetanus immune globulin immediately, and three doses of tetanus toxoid given by the standard schedule. Metronidazole or penicillin is also administered to destroy the organism and prevent toxin production. (Review topic: Tetanus)

3. A 21-year-old male presents to the ED with increasing dyspnea and pleuritic chest pain of sudden onset after getting hit in the left side of the chest during a bar fight. Examination reveals moderate respiratory distress with an absence of breath sounds and hyperresonance to percussion on the left, with tracheal deviation to the right. Which of the following is the most appropriate next step?

A Order a V/Q scan
B Order a chest x-ray
C Administer a sclerosing agent
D Insert a large-bore needle into the left 2nd ICS stat

Click here to see the answer

The answer is D Insert a large-bore needle into the left 2nd ICS stat

Simple aspiration by insertion of a needle into the involved side will decompress the tension pneumothorax until a chest tube can be inserted. (Review topic: Pneumothorax)

  • A V/Q scan is indicated in suspected cases of pulmonary embolism.
  • Patients in respiratory distress and evidence of a tension pneumothorax, such as tracheal deviation, should have treatment initiated without waiting on a chest x-ray to be taken.
  • Pleurodesis by the administration of a sclerosing agent is indicated in the treatment of recurrent, not traumatic, pneumothorax.

4. A 55-year-old woman with a history of ulcerative colitis presents to the emergency department with a severe flare. The patient reports numerous bloody loose stools and has been febrile for two days. Vital signs are T 102.0 HR 98 BP 131/86 RR 17 Sat 100%. The abdominal exam is notable for markedly distended abdomen with tympani and tenderness to palpation without guarding or rebound. A CT scan shows a markedly dilated descending and sigmoid colon with no perforations. What is the next best step in management for this patient?

A Oral prednisone
B IV hydrocortisone
C Rectal 5-ASA
D IV Metoclopramide
E IV Ondansetron

Click here to see the answer

The answer is B IV hydrocortisone

This patient is presenting with toxic megacolon secondary to ulcerative colitis (UC). The first-line treatment for patients with toxic megacolon from UC is IV glucocorticoids to reduce inflammation and the need for surgical intervention. (Review topic: Ulcerative colitis)

  • Oral prednisone is not appropriate, as patients with toxic megacolon require NGT suction and NPO.
  • While rectal 5-ASA is a treatment option for moderate colitis, but this patient has severe colitis.
  • Metoclopramide is a Dopamine-2 receptor antagonist used as a prokinetic agent in individuals with a variety of gastric motility diseases. It is also an antiemetic (via 5-HT3 antagonist activity) and has further prokinetic activity by acting as an agonist on 5-HT4 receptors.
  • Ondansetron is a Serotonin-3 receptor antagonist used as an antiemetic in patients with postoperative nausea or nausea due to chemotherapy.

5. A 21-year-old male with hematemesis. He is brought by his girlfriend who reports that he and his buddies have been out drinking every night last week in celebration of his 21st birthday. He reports having vomited each night, but tonight when he started vomiting, he noticed that there was streaking of blood. Concerned, he decided to come to the emergency department. Which of the following best describes the most likely diagnosis?

A Dilated submucosal esophageal veins
B Gastric mucosal erosion
C Mucosal tear at the gastroesophageal junction
D Transmural distal esophagus tear
E Transmural erosion of the gastric wall

Click here to see the answer

The answer is C Mucosal tear at the gastroesophageal junction

This patient is presenting with blood in his vomitus after forceful vomiting suggesting a diagnosis of a Mallory-Weiss tear. A Mallory-Weiss tear occurs secondary to a mucosal laceration at the gastroesophageal junction. (Review topic: Mallory-Weiss tear)

  • Dilated submucosal esophageal veins describes esophageal varices. Though variceal bleeds could lead to bloody vomitus, this is a less likely diagnosis given this patient’s young age and lack of a past medical history of cirrhosis. There is no mention of ascites in the case and the normal PT/PTT suggests against a diagnosis of cirrhosis.
  • Gastric mucosal erosion can occur due to decreased prostaglandin production which occurs in gastritis and peptic ulcer disease and presents with gnawing/burning epigastric pain, and can be associated with nausea and vomiting. This can occur with NSAID use or with alcohol abuse.
  • Transmural distal esophagus tear describes Boerhaave syndrome which presents after violent retching with similar symptoms to a Mallory-Weiss tear but with the additional findings of subcutaneous emphysema and odynophagia. This is a surgical emergency and is a progression from a simple Mallory-Weiss tear which presents with only bleeding.
  • Transmural erosion of the gastric wall describes a perforated gastric ulcer which can cause bleeding from the left gastric artery, epigastric pain, and unstable vitals. It is less likely to present after violent vomiting. A perforated peptic ulcer could present with severe abdominal pain and free air under the diaphragm.

6. A 25-year old female presents to the clinic with an 8-month history of intermittent crampy lower abdominal pain, with the passage of loose stools 4 times a day. Pain is usually worse during her menstrual period and is relieved by defecation. She also feels bloated. She says she has been undergoing work-related stress for months. There are no ALARM symptoms and there are no significant findings on physical examination. Which of the following is associated with this patient’s condition?

A Alvarado score
B Ranson criteria
C Rome criteria
D Revised Jones criteria

Click here to see the answer

The answer is C Rome criteria

The diagnosis of IBS can be made using the Rome criteria if patients have no red flag findings, such as rectal bleeding, weight loss, and fever, or other findings that might suggest another etiology. Patients with one of these red flag findings require further imaging studies and/or colonoscopy. (Review topic: Irritable bowel syndrome)

  • The Alvarado score is used in making a diagnosis of acute appendicitis
  • Ranson criteria Are used in assessing the severity of acute pancreatitis
  • Revised Jones criteria are used in diagnosing rheumatic fever

7. A 56-year-old woman came to the clinic complaining of a lump protruding from her anal opening. It was initially reducible, but it now irreducible. There is associated pain and itching. She also noticed bright red blood on her stool when she defecates. There is an associated history of chronic constipation. Examination of the perianal area revealed skin tags and a tender perianal mass covered with mucosa. Inspection of the anal mucosa showed no fissure. What is the grade of the condition?

A Grade I
B Grade II
C Grade III
D Grade IV

Click here to see the answer

Answer: D Grade IV

This is a Grade IV permanently prolapsed hemorrhoid. (Review topic: Hemorrhoids)

Incorrect Answers:

  • Grade I is bleed only no prolapse
  • Grade II is prolapsed but reduces spontaneously
  • Grade III is prolapsed and has to be manually reduced

8. A 45-year-old male presents with complaints of heartburn, belching, and epigastric pain for the past six months. He reports that symptoms occur within an hour of eating a meal and are aggravated by drinking coffee, eating fatty foods, and lying down. He has tried eating smaller meals and avoiding spicy food to no avail. He denies vomiting, difficulty swallowing, recent weight loss, or changes in stool color. His temperature is 98.9 °F, blood pressure is 147/82 mmHg, pulse is 86/min, respirations are 18/min, and BMI is 32 kg/m^2. His abdomen is soft, non-tender, and bowel sounds are auscultated in all quadrants. His laboratories are unremarkable and his fecal occult blood test (FOBT) is negative. What is the next best step in this patient’s management?

A 24-hour pH monitoring
B Endoscopy
C Metoclopramide
D Ranitidine
E. Omeprazole

Click here to see the answer

The answer is E. Omeprazole

GERD is a common complaint at primary care offices. It classically presents with heartburn, epigastric pain, and a sour taste that occurs within an hour of consuming a meal. PPIs are the initial medical management, except in the case of alarm symptoms. Any patient with symptoms of GERD accompanied by dysphagia, recurrent vomiting, weight loss, hematemesis, anemia, melena, or age > 50 should undergo endoscopy as these are considered high risk for the presence of an upper gastrointestinal malignancy. (Review topic: Gastroesophageal reflux disease)

Incorrect answers

  • 24-hour pH monitoring is considered the gold standard in the diagnosis of GERD. However GERD can usually be diagnosed clinically, and 24-hour pH monitoring should only be employed in order to confirm the diagnosis in atypical presentations or in patients refractory to PPI therapy
  • Endoscopy for GERD is indicated if patients have alarm symptoms. This patient is under the age of 50 and lacks alarm symptoms; therefore, endoscopy is not indicated.
  • Metoclopramide is a prokinetic agent that is often used to treat gastroparesis. While it has utility in managing GERD, it is not a first-line medical agent.
  • Ranitidine is a histamine-2 receptor antagonist that is indicated if patients have failed PPI therapy. Ranitidine products were found to be contaminated with NDMA, a probable human carcinogen that may be linked to many types of cancer, including bladder cancer, colon cancer, and prostate cancer.

9. A 65-year old man who is being managed for lung cancer on the ward makes a complaint of a 2-day history of the passage of nonbloody watery stool up to 4 times per day, anorexia, cramping abdominal pain, and fever. Meanwhile, he had a 10-day course of antibiotics 4 weeks ago on account of a lung infection. Which of the following is the most likely cause of his diarrhea:

A Salmonella
B Rotavirus
C Clostridium difficile
D E. coli

Click here to see the answer

The answer is C Clostridium difficile

Clostridium difficile colitis results from a disturbance of the normal bacterial flora of the colon, colonization by C. difficile, and the release of toxins that cause mucosal inflammation. Antibiotic therapy is the key factor that alters the colonic flora. (Review topic: Gastroenteritis)

Incorrect answers

  • Salmonella is a cause of diarrhea following food poisoning.
  • Rotavirus is a common cause of diarrhea in children. Less common in adults. Doesn’t occur as a result of recent use of antibiotics.
  • E. coli is a cause of diarrhea following food poisoning.

10. A 37-year-old male with a history of daily NSAID use complains of epigastric pain, nausea, and vomiting, all worsened by eating. On physical examination, he is tender to palpation in the epigastrium. He admits to drinking approximately two beers per day. He was prescribed a course of ranitidine followed by omeprazole after his symptoms did not resolve. He was referred for endoscopy, with findings consistent with a gastric ulcer. Biopsy with silver staining is positive for H-Pylori. Which of the following is the most effective regimen for the treatment of this condition?

A Omeprazole, metronidazole, tetracycline, bismuth
B Omeprazole, penicillin, famotidine
C Amoxicillin, clarithromycin
D Pantoprazole and levofloxacin
E Pantoprazole

Click here to see the answer

The answer is C. Omeprazole, metronidazole, tetracycline, bismuth

This patient presents with H. pylori gastritis complicated by a peptic ulcer. Treatment for H. pylori is with triple or quadruple therapy. Omeprazole, metronidazole, tetracycline, and bismuth are appropriate quadruple therapy. (Review topic: Esophagitis)

Incorrect answers

  • Omeprazole, penicillin, famotidine is not a treatment regimen for H. pylori infection
  • Although amoxicillin and clarithromycin is an appropriate antibiotic regimen, this combination requires a PPI such as omeprazole
  • Pantoprazole and levofloxacin are not a treatment regimen for H. pylori; the combination requires the addition of amoxicillin for effective triple therapy.

Looking for all the podcast episodes?

This FREE podcast series is limited to every other episode, you can download and enjoy the complete audio series by becoming a Smarty PANCE member.

I will be releasing new episodes every few weeks. Smarty PANCE is now discounted, so sign up now before it’s too late!

Resources and Links From The Show

This Podcast is also available on iOS and Android

itunes_logo-1

Download the Interactive Content Blueprint Checklist

Follow this link to download your FREE copy of the PANCE/PANRE/EOR Content Blueprint Checklists

Print it up and start crossing out the topics you understand, marking the ones you don’t, and making notes of key terms you should remember. The PDF version is interactive and linked directly to the individual lessons on Smarty PANCE.

The post Podcast Episode 87: Ten Internal Medicine EOR Questions appeared first on The Audio PANCE and PANRE.

]]>
Welcome to episode 87 of the Audio PANCE and PANRE PA Board Review Podcast. Join me as I cover ten internal medicine rotation EOR content blueprint questions from the Smarty PANCE physician assistant board and rotation review website.



Welcome to episode 87 of the Audio PANCE and PANRE PA Board Review Podcast.



Join me as I cover ten internal medicine rotation EOR content blueprint questions from the Smarty PANCE physician assistant board and rotation review website.



Special from today’s episode:



* Check out the updated Smarty PANCE Internal Medicine Rotation (EOR) Review Course* Download your Free Trello Smarty PANCE PAEA Internal Medicine EOR Tracking Template* Members can take the all-new Internal Medicine Rotation Practice Exam* View the interactive Smarty PANCE Internal Medicine Rotation (EOR) Topic List



Below you will find an interactive exam to complement the podcast.



The Audio PANCE/PANRE and EOR PA Board Review Podcast



I hope you enjoy this free audio component to the examination portion of this site. The full board review course includes over 2,000 interactive board review questions and is available to all members of Smarty PANCE.



* You can download and listen to past FREE episodes here, on iTunes, Spotify, on Google Play Music or Stitcher.* You can listen to the latest episode, take an interactive quiz, and download more resources below.



Listen Carefully Then Take The Practice Exam





If you can’t see the audio player click here to listen to the full episode.



Podcast Episode 87: Ten Internal Medicine EOR Pulmonology Questions



The following questions are linked to PAEA Content Blueprint lessons from the Smarty PANCE and PANRE Board Review Website. If you are a member, you will be able to login and view this interactive video lesson.



1. A 67-year-old man with a long history of constipation presents with steady left lower quadrant pain. Physical exam reveals low-grade fever, mid abdominal distention, and lower left quadrant tenderness.]]>
The Physician Assistant Life | Smarty PANCE full 22:25 413
Podcast Episode 86 – How to Be a Better PA Part 1: Nurses, Lab Techs, and X-Ray Technicians http://podcast.thepalife.com/podcast-episode-86-how-to-be-a-better-pa-part-1-nurses-lab-techs-and-x-ray-technicians/ Mon, 09 Nov 2020 13:00:00 +0000 http://podcast.thepalife.com/?p=404 http://podcast.thepalife.com/podcast-episode-86-how-to-be-a-better-pa-part-1-nurses-lab-techs-and-x-ray-technicians/#respond http://podcast.thepalife.com/podcast-episode-86-how-to-be-a-better-pa-part-1-nurses-lab-techs-and-x-ray-technicians/feed/ 0 <p>This is part one of a special three-part podcast series by Joe Gilboy PA-C on becoming a better PA. Part one is all about building amazing relationships with our nurses and ancillary staff. You may be thinking to yourself, “Hey Stephen, I thought this podcast was about the PANCE and PANRE.” “How does talking about […]</p> <p>The post <a rel="nofollow" href="http://podcast.thepalife.com/podcast-episode-86-how-to-be-a-better-pa-part-1-nurses-lab-techs-and-x-ray-technicians/">Podcast Episode 86 – How to Be a Better PA Part 1: Nurses, Lab Techs, and X-Ray Technicians</a> appeared first on <a rel="nofollow" href="http://podcast.thepalife.com">The Audio PANCE and PANRE</a>.</p>
How to Be a Better PA Part 1 - Nurses, Lab Techs, and X-Ray Technicians

This is part one of a special three-part podcast series by Joe Gilboy PA-C on becoming a better PA.

Part one is all about building amazing relationships with our nurses and ancillary staff.

You may be thinking to yourself, “Hey Stephen, I thought this podcast was about the PANCE and PANRE.” “How does talking about nurses and ancillary make me a better PA or help me pass my boards?”

If you asked this question, then you have come to the right place.

Because as anybody who has ever worked as part of a healthcare team can tell you, the key to your success as a PA has little to do with you and a lot more to do with how you treat other people.

So how do you become a better PA?

Let’s jump right into this unique episode of The Audio PANCE and PANRE Podcast. You can listen to the podcast below and read the summarized (and edited version) of the transcript or listen in your podcast player of choice.

The Audio PANE/PANRE Podcast Episode 86: How to be a Better PA (Part 1 of 3)

You can also click here to listen to or download this episode.

Episode Transcript and Summary

This episode was recorded by Joe Gilboy PA-C and edited for clarity and readability by Stephen Pasquini PA-C.

I’ve been a PA in the emergency department for 35 years. And you can imagine, I’ve had a lot of interaction with nurses and ancillary staff. I want to pass on some wisdom and guidelines to help you deal with the nurses and ancillary staff.

This will help you become the best PA you can become and help you immensely in your career and, most importantly, with your reputation.

Become Best Friends with the Nurses

Make Friends with The Nurses - How to be a better PA Part 1

The first group of people we’re going to talk about is the nurses. And this is a big one, folks, my God, how big is this!

So, to all my new graduates out there, I really need you to listen to me, stop what you’re doing, and let this bounce around in your head for a second. As a new graduate, your nurses will know more than you. That’s right, you heard me, they will know more than you.

So, let’s say you’re working at an urgent care family practice clinic or working in a neurology clinic. There you are seeing your patients, and the nurse will know more than you on how this patient needs to be worked up, which labs or imaging tests to order, and what the doctor likes.

So, what I always tell my new graduates to do, and what you need to do, is become best friends with the nurses because sometimes (a lot of times), they’re going to know more than you!

As time goes on and as you become more experienced and more seasoned, yes, you will know more than the nurses, which is just a byproduct of time and experience. I can’t fast forward time, but I can tell you that the nurses will know more than you initially, so follow them.

They’re going to point you in the right direction. They will say, “Hey, you know Dr. Smith likes it this way, or Dr. Jones likes it that way or, whoever the person that you’re working with, this is how I like to work with them” Follow them, and if they like you, they will lead you in the right direction. 

Make the Nurses Part of Your Team

The second thing you need to understand about the nurses is that they like to be part of the team. They like to feel like they’re part of the team and know that you recognize their experience and expertise when working up the patient.

The worst thing you can ever say to a nurse is this: “I’m the PA; you’re the nurse; you must do what I say.”

As soon as you say this, there are two things you should do:

  1. Sign your organ donation card
  2. Pick up the phone and call The Legacy Group, you know, that’s the organ donation team that we have here in California, and they’re going to come and harvest your organs, and you will be dead.

And that’s what’s going to happen. Don’t ever say that to the nurses!

What should you say instead?

Ask for Input from the Nurses

What you should say to the nurses goes something like this: “hey, so what’s your input?”

It’s that easy!

Honor Input from the Nurses

There will come a time, and it’s going to happen to you, where the nurse is going to want to order something that you may not 100% agree with. What should you do then?

If it’s not doing any harm to the patient or it is something that’s no far outside the standard of care, your interaction should go something like this: “Okay, I see your point, I agree with you, let’s add this lab test (or this other study) that we may want to do.”

Now let’s say the recommendation made by the nurse is way outside the standard of care. For example, let’s say you have someone with an ankle sprain and your nurse wants to order a CAT scan of the head. And you’re like, “okay, this is not going to work out well.”

So now, at that point, what’s the best way to disagree with nurses, but do it politely?

You can say this: “I hear you loud and clear. However, I think that what’s going to serve our patient best is not a CT of his head, but let’s x-ray his ankle.”

“I hear you!” This is the magic word. This is what they need to know: That you hear them, you respect them, and that they are part of the team. 

Embrace “The Doughnut Theory”

How to be a better PA - Embrace the doughnut theory

The doughnut theory works like this: You bring doughnuts to your clinic or hospital department once a week, then watch what happens!

That $7 worth of doughnuts is going to change everything!

The nurses are going to be like, “who brought the doughnuts?” “Oh look, you know the new PA Allison over here she brought the doughnuts, and we love Alison.”

Suddenly, PA Allison is a superstar! Like, oh my gosh, we love Alison! And every Wednesday, they’re going to be looking at the door like, hey, did Alison bring the doughnuts?

Feed the nurses! I know this sounds crazy, but feed your nurses, and they will love you!

Another thing that you should bring the nurses once a month is pizza. I know this sounds crazy. Again, you’re like, what’s Joe talking about? But trust me when I tell you that you should buy the staff pizza once a month. Pizza is a universal food. I even think aliens from other galaxies like pizza!

We are Family . . .  All the Nurses, MAs, and Me!

The nurses need to feel like you are part of the community. You’re part of the family. Because what this is about is family.

They did a recent study in the ER about how much time we spend with the nurses. Do you realize that I spend more time with my nurses than I do with my wife?

This is a well-documented fact. Because anybody who’s been married knows what I’m talking about. When you come home, what’s going on? You’re doing this, and your spouse’s doing that. They’re picking up the kids, and you’re going to Costco. You’re over here, and they’re over there.

But to have quality time with your spouse? That’s exceedingly rare!

How many times are you two feet from your nurse? Discussing issues? My nurse is my spouse!

I tell everybody I have twenty-seven work spouses at my job. I’ll be talking to my wife on my cell phone, and the nurses show up, I will hang up on my wife to talk to the nurse.

Their Opinion Matters

At the end of the year, when you show up for that evaluation with your boss, do you know who she’s talking to? Yes, that’s right, the nurses!

Your supervisor will ask, “So what’s Allison like to work with?”

And if you have implemented the above tactics correctly, the nurses go, “Man, we love Allison! she brings doughnuts, she brings pizza, she talks to us very respectfully, if we disagree, she’ll listen and talk to us.”

You need to treat them with respect and treat them like family members. Because that’s what they are. If you feed them, treat them like family, and treat them with respect. Oh, wow. Watch what happens.

Your nurses will look at you like, “Oh, man, Allison’s here today!” And this is what you want to be the PA that everybody’s excited to see when you walk through the clinic door, the OR door, the emergency room door, the urgent care, or wherever it is that you’re working. It’s like, “Oh, my gosh, Alison’s here today.” “All right, this is a great shift!” And when you showed up, you brought doughnuts.

And that is the sweet spot. That is where your reputation really starts to grow.

When you apply to a new job, what do they always ask for? Your background and experience! And when they call your boss, it’s about your reputation.

One of the questions they always ask is, “What is Allison like?” “Oh, Allison’s great, and the nurses love her!” And that is a slam dunk! Once they hear that, their next thought is, “Oh, wow, we want this person!”

Think about it for a second. Really pull back and give this some thought. Imagine you’re a physician hiring a PA. Imagine that the other previous employer tells you, “Well, you know, he’s had some run-ins with the nurses.” And then you call the other applicant who’s applying for the same job, and their supervisor is like, “Oh, he’s great, and the nurses love him.” Which PA are you going to hire?

Exactly, the one that has a good relationship with the nurses!

What I’m really trying to encapsulate here is this:

  1. Always be respectful to the nurses.
  2. Understand that sometimes the nurses will know more than you, and that’s OK.
  3. Feed the nurses doughnuts and pizza
  4. Treat them like a family member – like your mom, dad, brother, or sister.

And you will see how well this works! Make no mistake, in my 35 years as an ER PA, I’ve seen so many PhDs who’ve been disrespectful to nurses, and what has happened to them? It’s not good!

Remember, at the end of the year, this is what your boss is looking at. And then if you have a good relationship with the nurses, guess who gets to ask for more money? That would be you. And guess whose reputation just grew? That would be you.

Love Your Lab Techs

Love Your Lab Technicians

The next group of people you need to be nice to and get to know very well are your lab techs.

As you go through your PA career, make sure you become very friendly and are exceedingly kind to your lab techs. Let me tell you why.

The lab techs have an important job that often goes unseen. They come, they draw the blood, do the cultures, etc. etc. They do much of their work behind the scenes. You don’t see it. You see the results.

When you’re nice to the lab techs, and you treat them with love and respect, one of these days, you’re going to have trouble ordering a specific test. There you are poking around your EMR, maybe you are in Epic, and you can’t figure out what to order.

Then lo and behold, you call the lab to ask their advice, and they’re going to say, “Hey, you know, this is Allison. And we’re good friends with Allison, and she’s nice to us, right?”

Then suddenly, when you call down to the lab and say, “Hey, guys, how do you put in the order for this specific test?” And what’s going to happen? They’ll walk you right through it, and they’re going, “Hey, no problem, we’ll take care of it!” “And you know what, Allison, we’ll put in the order for you? No big deal.”

When you are friendly with the lab techs, they’ll draw extra tubes of blood, saying, “Hey, you know, just in case Allison forgot to ask to order this, we’ll have it done for her.”

Or say you’re working at a family practice clinic, a neurology clinic, or an OBGYN clinic. And there you are, it’s 4:30 in the afternoon, and you want to get off at five. You just sent down a urinalysis and a pregnancy test, which usually takes about 45 minutes to an hour to result.

And there you are, waiting to meet friends at 5:20, and it’s 4:30 pm, and you are thinking, “How am I going to get this done?”

Well, if you’re friendly with the lab techs, and you call down and say, “Hey guys, is there any way we can rush this pregnancy test and urinalysis?” And they say, “Oh, for you, Allison, anything!”

And there it is!

So, in other words, they will help you get things done on time and make you a better PA.

The lab techs are some of the most important people you’ll meet in the hospital, not hospital administrators.

So, as you go through your PA career, get to know your lab techs. Be friendly with them, get to know their first names, and take time to understand how they do things.

X-Ray Technicians

The last group of people we’re going to talk about is our X-ray department, including our X-ray techs, our ultrasound techs, CT techs, and even the radiologists.

When you’re working your patients up, you must wait for your labs, you must wait for your nurses to do their work, and then you’re waiting for the X-ray tech or CT tech, or the ultrasound tech to get that imaging study done. And what I’ve learned in my 35 years is that the radiology department can slow you up the most.

I agree; sometimes, the lab can slow you up. But what you’ll find out nine times out of ten, when you get to the end of the day, what was really the thing that took the longest? Your radiology imaging test.

So please get to know your X-ray techs or your CT techs, and get to know the girls or the guys in the ultrasound department, get to know them on a first-name basis.

Because soon, the day will come that you’re working in the ER, the urgent care, family practice, or in the surgical suite, and you need the X-ray tech to come and perform a chest X-ray or bring in the CRM fluoroscope. You need this done quickly because you’re in a rush, or the patient is in critical condition.

Or maybe you’re reducing a fracture, and you want to see what the post-reduction looks like before you even put in the splint.

This is when knowing the folks in the radiology department are going to serve your best interests.

So, let’s say you’re ordering a post-reduction study on a wrist fracture. So, there you are, you have reduced the fracture, and before you place the patient in the splint, you need the post-reduction study.

Now, since I’m good friends with the X-ray techs, I can give them a call and say, “Hey can you guys come over here and take a post-reduction x-ray really quick?”

And they’re going to be like, “No problem, Allison, we’ll come over and take it because it’s you, you’re the one that’s nice to us, don’t worry, we’ll put the order in for you.”

And then everything goes nice and smooth.

One of these days, it will be 4:30 in the afternoon, and you’ve got a patient with a finger fracture. Your patient is the fifth one in the row, which means you’ll be there for the next hour.

But you’ve got to go to your kid’s soccer game in an hour. And now you’re like, “Man, I wish there’s some way I could get this x-ray done quickly.” This is when knowing the x-ray techs becomes valuable.

Be Nice to Everybody

How to be a better PA - Be Nice to Everybody

Be nice to everybody!

This entire podcast episode is about building your reputation. This is about you as a PA. If you follow this advice, your colleagues will start viewing you as the nice PA that’s always respectful, knows everyone by their first name, and is always polite. And as a byproduct, everyone will be more than happy to do something nice for you.

It’s a human behavior thing; it’s not a medical thing; it’s a human thing. When human beings are nice to you, you’re more likely to be nice back and do favors for them in return.

That’s exactly what this boils down to. And I’m trying to make your job more efficient. Most importantly, I’m trying to get you to work on a better reputation, and this is what’s going to carry the most weight because every job you apply for, what do they ask for? Your references!

And what are they going to do? They’re going to call your previous employers. What is their conversation going to be based upon? It will have nothing to do with where you went to PA school. It will have absolutely nothing to do with your PANCE scores. It’ll have everything to do with your reputation.

Podcast two, which is coming up next, will be all about medical staff, administrators, and office managers and how to handle them. And that one is a big one, folks. So be your best, do your best. And try to build a great relationship with everybody you work with!

I’ll see you soon for podcast number two.

– Joe Gilboy PA-C

The post Podcast Episode 86 – How to Be a Better PA Part 1: Nurses, Lab Techs, and X-Ray Technicians appeared first on The Audio PANCE and PANRE.

]]>
This is part one of a special three-part podcast series by Joe Gilboy PA-C on becoming a better PA. Part one is all about building amazing relationships with our nurses and ancillary staff. You may be thinking to yourself, “Hey Stephen,



This is part one of a special three-part podcast series by Joe Gilboy PA-C on becoming a better PA.



Part one is all about building amazing relationships with our nurses and ancillary staff.



You may be thinking to yourself, “Hey Stephen, I thought this podcast was about the PANCE and PANRE.” “How does talking about nurses and ancillary make me a better PA or help me pass my boards?”



If you asked this question, then you have come to the right place.



Because as anybody who has ever worked as part of a healthcare team can tell you, the key to your success as a PA has little to do with you and a lot more to do with how you treat other people.



So how do you become a better PA?



Let’s jump right into this unique episode of The Audio PANCE and PANRE Podcast. You can listen to the podcast below and read the summarized (and edited version) of the transcript or listen in your podcast player of choice.



The Audio PANE/PANRE Podcast Episode 86: How to be a Better PA (Part 1 of 3)





You can also click here to listen to or download this episode.



Episode Transcript and Summary



This episode was recorded by Joe Gilboy PA-C and edited for clarity and readability by Stephen Pasquini PA-C.



I’ve been a PA in the emergency department for 35 years. And you can imagine, I’ve had a lot of interaction with nurses and ancillary staff. I want to pass on some wisdom and guidelines to help you deal with the nurses and ancillary staff.



This will help you become the best PA you can become and help you immensely in your career and, most importantly, with your reputation.



Become Best Friends with the Nurses







The first group of people we’re going to talk about is the nurses. And this is a big one, folks, my God, how big is this!



So, to all my new graduates out there, I really need you to listen to me, stop what you’re doing, and let this bounce around in your head for a second. As a new graduate, your nurses will know more than you. That’s right, you heard me, they will know more than you.



So, let’s say you’re working at an urgent care family practice clinic or working in a neurology clinic. There you are seeing your patients, and the nurse will know more than you on how this patient needs to be worked up, which labs or imaging tests to order, and what the doctor likes.



So, what I always tell my new graduates to do, and what you need to do, is become best friends with the nurses because sometimes (a lot of times), they’re going to know more than you!



As time goes on and as you become more experienced and more seasoned, yes, you will know more than the nurses, which is just a byproduct of time and experience. I can’t fast forward time, but I can tell you that the nurses will know more than you initially, so follow them.



They’re going to point you in the right direction. They will say, “Hey, you know Dr. Smith likes it this way, or Dr. Jones likes it that way or,]]>
The Physician Assistant Life | Smarty PANCE full 25:56 404
Podcast Episode 85: Ten Mixed Multiple Choice PANCE and PANRE Board Review Questions http://podcast.thepalife.com/episode-85/ Mon, 14 Sep 2020 14:49:26 +0000 http://podcast.thepalife.com/?p=394 http://podcast.thepalife.com/episode-85/#respond http://podcast.thepalife.com/episode-85/feed/ 0 <p>Welcome to episode 85 of the Audio PANCE and PANRE PA Board Review Podcast. Join me as I cover ten PANCE and PANRE Board review questions from the SMARTYPANCE course content following the NCCPA™ content blueprint (download the FREE cheat sheet). Special from today’s episode: Download your copy of the Free Trello Smarty PANCE NCCPA™ Blueprint […]</p> <p>The post <a rel="nofollow" href="http://podcast.thepalife.com/episode-85/">Podcast Episode 85: Ten Mixed Multiple Choice PANCE and PANRE Board Review Questions</a> appeared first on <a rel="nofollow" href="http://podcast.thepalife.com">The Audio PANCE and PANRE</a>.</p> Podcast Episode 85_ The Audio PANCE and PANRE PA Board Review PodcastWelcome to episode 85 of the Audio PANCE and PANRE PA Board Review Podcast.

Join me as I cover ten PANCE and PANRE Board review questions from the SMARTYPANCE course content following the NCCPA™ content blueprint (download the FREE cheat sheet).

Special from today’s episode:

This week we will be covering ten general board review questions based on the NCCPA PANCE and PANRE Content Blueprints. 

Below you will find an interactive exam to complement the podcast.

Below you will find an interactive exam to complement the podcast.

The Audio PANCE/PANRE and EOR PA Board Review Podcast

I hope you enjoy this free audio component to the examination portion of this site. The full board review course includes over 2,000 interactive board review questions and is available to all members of the PANCE and PANRE Academy and Smarty PANCE.

Listen Carefully Then Take The Practice Exam

If you can’t see the audio player click here to listen to the full episode.

Podcast Episode 85: Ten MIXED PANCE and PANRE Board Review Questions

The following questions are linked to NCCPA Content Blueprint lessons from the Smarty PANCE and PANRE Board Review Website. If you are a member, you will be able to log in and view this interactive video lesson.

1. A 14-hour-old boy has failed to pass stool and is vomiting greenish fluid. He was born at 40 weeks gestation with no complications during delivery or pregnancy. His temperature is 97.6°F (36.4°C), blood pressure is 64/34 mmHg, pulse is 140/min, respirations are 33/min, and oxygen saturation is 98% on room air. The child is currently breastfeeding and appears irritable. Physical exam is notable for a distended and non-tender abdomen. The rectal exam is unremarkable. An abdominal radiograph demonstrates distended loops of bowel. What is the most likely diagnosis?

  1. Cystic fibrosis
  2. Hirschsprung disease
  3. Jejunal atresia
  4. Pyloric stenosis
  5. Tracheoesophageal fistula

Click here to see the answer

Answer: A. Cystic fibrosis

Cystic fibrosis (CF) commonly presents with meconium ileus, characterized by bilious vomiting, distended loops of bowel on radiography, and failure to pass meconium. CF is an autosomal recessive disease and is common in Caucasians. Neonates with CF commonly present with failure to thrive, respiratory compromise, and meconium ileus. In meconium ileus, abnormally thick meconium results in bowel obstruction, perforation, or volvulus. CF is diagnosed by the sweat chloride test or genetic testing. Radiography in ileus can demonstrate distended loops of bowel from the obstruction.

  • Hirschsprung disease would present with abdominal pain with chronic constipation. There would be an absence of stool in the rectal vault on rectal exam and a positive squirt sign on physical exam (expulsion of stool/flatus on rectal exam).
  • Jejunal atresia is characterized by the “triple bubble” sign on abdominal radiographs as well as bilious vomiting.
  • Pyloric stenosis presents with non-bilious and projectile vomiting with a palpable mass on abdominal exam. This diagnosis can be confirmed with an ultrasound. The patient should first be rehydrated and have their electrolytes repleted prior to imaging.
  • Tracheoesophageal fistula presents with coughing and choking with feeding. This breastfeeding and asymptomatic child in terms of his respiratory status suggests against this.

Review NCCPA Blueprint Topic: Cystic fibrosis

2. A 34-year-old female presents to her ophthalmologist with 6 hours of blurry vision in her right eye. She reports severe pain with eye movement that has not been relieved with NSAIDs. She feels that she has been generally healthy all her life, although she does note one week of right arm weakness during the previous year that resolved without treatment. On exam, she has no noted ocular secretions. On her fundal exam, you note blurred borders on the optic disc. Which of the following additional findings is likely to be present in this patient?

  1. Polycythemia
  2. Elevated IgG in the cerebrospinal fluid (CSF)
  3. Multiple areas of periventricular hyperintensity on T2 FLAIR imaging
  4. Subdural hematoma
  5. Small, punctate hypointensities on gradient-echo MR

Click here to see the answer

The answer is B. Elevated IgG in the cerebrospinal fluid (CSF)

This patient’s clinical presentation is consistent with multiple sclerosis (MS), which is associated with an elevated IgG in the CSF.

MS is associated with oligoclonal bands on CSF evaluation in 80% of cases. The bands result from IgG overproduction, likely as a result of the autoimmune processes that occur in MS. Patients with MS frequently present first with symptoms of optic neuritis, which include loss of vision or blurry vision (usually unilateral), with severe pain with eye movement. The prognosis of MS differs drastically from patient to patient, but most commonly has a relapsing-remitting course.

Incorrect Answers:

  • Mononuclear pleocytosis, not polycythemia, is associated with MS.
  • MS plaque is hyperintense, not hypointense, on T2 FLAIR.
  • MS is not associated with subdural hematomas. In addition, a subdural hematoma would be unusual in this young patient without a history of trauma.
  • Small, punctate hypointensities on gradient-echo MRI is typical of diffuse axonal injury, which would be unlikely in this patient without a history of trauma.

Review NCCPA Blueprint Topic: Multiple sclerosis (ReelDx + Lecture)

3. A 43-year-old man is brought to the emergency department after having a seizure. His wife states that the patient has been struggling with alcohol abuse and has recently decided to “quit once and for all”. Physical exam is notable for a malnourished patient responsive to verbal stimuli. He has moderate extremity weakness, occasional palpitations, and brisk deep tendon reflexes (DTRs). EKG demonstrates normal sinus rhythm and a prolonged QT interval. What nutritional deficiency most likely contributed to these findings?

  1. Potassium
  2. Calcium
  3. Folate
  4. Magnesium
  5. Vitamin D

Click here to see the answer

The answer is D. Magnesium

This patient in this vignette demonstrates a constellation of findings related to the complications of hypomagnesemia. Although alcohol withdrawal may present with seizures, the constellation of brisk DTRs, weakness, and prolonged QT suggest hypomagnesemia. The treatment for this patient will be multi-faceted, with magnesium repletion a necessity.

Hypomagnesemia (<1.5 mg/dL) is a potentially serious condition that may be difficult to recognize due to non-specific manifestations. Mild signs of hypomagnesemia include: generalized weakness, fatigue, nausea and vomiting. As deficiency progresses patients may complain of numbness, cramping, and dysphagia. Physical exam findings may reveal increased DTRs and fasciculations. If hypomagnesemia is acute in onset and/or severe, patients may develop altered mental status, seizures, or cardiac conduction abnormalities. Low magnesium levels also have the potential to affect other electrolyte levels. In patients with hypomagnesemia it is not uncommon to see hypokalemia (K+ conductance changes increasing renal losses) and hypocalcemia (due to PTH resistance).

Incorrect Answers:

  • This patient is likely hypokalemic secondary to hypomagnesemia. In fact, hypokalemia may contribute to the EKG findings. If the patient is solely hypokalemic, without any other electrolyte disturbances, DTRs will be decreased rather than increased.
  • A patient with hypocalcemia may present in the same fashion as this patient (altered mental status, seizures, increased DTRs, and prolonged QT). While hypomagnesemia can lead to hypocalcemia, it is important to understand that a deficiency in magnesium itself can independently cause neuromuscular abnormalities. Seeing as this patient is malnourished and a chronic alcoholic, magnesium deficiency is the likely etiology.
  • Folate deficiency is common in chronic alcoholics. It typically presents with macrocytic anemia without neurological complications
  • Vitamin D deficiency is often asymptomatic but may present with signs of osteomalacia in adults and rickets in children.

Review NCCPA Blueprint Topic: Fluid and Electrolyte Disorders (PEARLS)

4. A 17-year-old female presents to her physician’s office after noticing a round lump in her left breast 2 months ago. She reports that the lump seemed to enlarge and became tender just preceding her last 2 menses. It is otherwise painless, and the patient denies any discharge or skin changes. She has no past medical history but her grandmother, age 72, was just diagnosed with invasive ductal carcinoma of the breast. The patient is an avid softball player at her high school and denies alcohol, smoking, or illicit drug use. On exam, the breasts appear symmetric and normal. A 3-cm round, mobile mass is palpated in the upper outer quadrant of the left breast. There is slight tenderness to deep palpation of the mass. There is no axillary lymphadenopathy on either side. Which of the following is the most likely outcome of this patient’s condition?

  1. This mass will likely require excision
  2. This mass will decrease in size if the patient starts oral contraceptives
  3. This mass slightly increases this patient’s risk of breast cancer in the future
  4. This mass will most likely decrease in size or disappear over time
  5. If this mass grows rapidly to greater than 5 cm, radiation and chemotherapy are indicated

Click here to see the answer

The answer is D. This mass will most likely decrease in size or disappear over time

This patient is a young female with a round, mobile mass that seems to respond to hormonal fluctuations, most likely a fibroadenoma, a benign mass common in reproductive-aged women. In adolescents, the majority of lesions will diminish or completely resolve over time, so only reassurance and observation are required. 

Classic fibroadenomas are relatively small (2-3 cm in size), in the upper outer quadrant of one or both breasts, and feel rubbery to palpation. They are generally painless but may become tender around the time of menses, as in this patient. There should be no skin changes, breast drainage, or lymphadenopathy. For an adolescent patient without concerning features or strong family history of premenopausal breast cancer in multiple first-degree relatives, there is no increased risk of malignancy from fibroadenomas. Fat necrosis is also a possible diagnosis in this softball player, as many patients do not recall a specific inciting trauma. However, given that the mass has been present for 2 months and undergoes hormonal changes, fibroadenoma is still the most likely diagnosis. Either way, the mass should eventually resolve and ultrasound is only indicated if the patient is older or there is persistence or change in the mass (Illustration A). Ultrasound is preferred in young women due to high breast density and the radiosensitivity of their tissues, but if the patient is over 35, mammography may also be performed. It would show a “popcorn” appearance (Illustration B). Upon confirmation of the diagnosis, either by imaging findings or core needle biopsy, surgical excision can be performed only if the patient experiences significant symptoms. Otherwise, routine follow-up is sufficient.

Incorrect Answers:

  • Excision is usually not required for fibroadenomas given that they are not associated with increased risk of malignancy. If a patient feels the symptoms are too bothersome, surgical resection can be performed. If the diagnosis is called into question due to persistence or growth and there is concern for malignancy, excision may be required after imaging and biopsy are performed.
  • Oral contraceptives containing estrogen may increase, not decrease, the size of the fibroadenoma. Since these tumors are hormonally receptive, they often grow and become tender around the time of menses, during pregnancy, and with estrogen administration (as in an OCP). For this reason, they also typically regress in menopause.
  • Breast cancer risk is not increased by fibroadenomas, especially ones that are simple (no skin changes, drainage, lymphadenopathy, etc.), as in this patient. This patient’s family history of postmenopausal breast cancer in a second degree relative does put her at minimally higher risk of breast cancer, but this is unrelated to her fibroadenoma.
  • A rapid growth to over 5 cm suggests that the mass is actually not a fibroadenoma but a phyllodes tumor (Illustration C). This is a rare neoplasm that can be benign or malignant, and although most phyllodes tumors occur in older women, they have been reported in patients as young as 10 years old. They are known for their rapid increase in size within weeks and any such growth should be evaluated with ultrasound and core needle biopsy (and mammography if the patient is over 35). Given the malignant potential, wide local excision is the standard of care. Chemotherapy and radiation are generally not indicated as effectiveness is unclear.

Review NCCPA Blueprint Topic: Fibroadenoma

5. A 26-year-old monogamous female presents with cyclic pelvic pain that has been increasing over the last 6 months. She complains of significant dysmenorrhea and dyspareunia. She uses condoms for birth control. On physical examination, her uterus is retroverted and non-mobile, and she has a palpable adnexal mass on the left side. Her serum pregnancy test is negative. Which of the following is the most likely diagnosis?

  1. Ovarian cancer
  2. Endometriosis
  3. Functional ovarian cyst
  4. Pelvic inflammatory disease

Click here to see the answer

The answer is B. Endometriosis

With endometriosis, the uterus is often fixed and retroflexed in the pelvis. The palpable mass is an endometrioma or “chocolate cyst”. The patient with endometriosis also often has dysmenorrhea, dyspareunia, and dyschezia.

Incorrect answers:

  • It is important to consider ovarian cancer in a patient with a pelvic mass however, ovarian cancer usually occurs in older women over age 55 and patients are often asymptomatic until the disease is more advanced
  • Functional ovarian cysts occur from ovulation and usually are not symptomatic.
  • With PID the patient will have abdominal tenderness, adnexal tenderness, cervical motion tenderness, and elevated temperature.

Review NCCPA Blueprint Topic: Endometriosis (Lecture)

6.  A healthy 29-year-old woman at 30-weeks gestational age has gained 35lbs since becoming pregnant. She complains of several weeks of bilateral numbness and tingling of her palms, thumbs, index, and middle fingers that is worse at night. She also notes weakness gripping objects at the office. Which nerve is most likely affected?

  1. Median nerve
  2. Ulnar nerve
  3. Radial nerve
  4. Axillary nerve
  5. Anterior interosseous nerve

Click here to see the answer

The answer is A. Median nerve

This clinical presentation is consistent with carpal tunnel syndrome (CTS), the most common entrapment neuropathy, caused by compression of the median nerve as it travels under the flexor retinaculum. During pregnancy, increased edema causes a narrowing of the carpal tunnel, which predisposes the entrapment of the median nerve. The median nerve is responsible for sensory and motor distribution of the thumb, index, middle and radial half of the ring finger. Hallmark symptoms of CTS include numbness and paresthesias in the median nerve distribution. Weakness and atrophy of the thenar muscles may be evident if CTS is left untreated.

Incorrect Answers:

  • The ulnar nerve supplies sensory innervation to the 5th digit and the medial half of the 4th digit and motor innervation of the forearm flexors, and several intrinsic muscles of the hand. The ulnar nerve is most commonly injured at the elbow as it courses adjacent to the humeral medial epicondyle or it may be compressed between the two heads of the flexor carpi ulnaris muscles (cubital tunnel syndrome).
  • The radial nerve supplies the medial, lateral, long heads of the triceps brachii, 12 muscles in the posterior compartment of the forearm and the associated joints and overlying skin. Radial nerve injury may result from spiral fracture of the midshaft of the humerus and result in “wrist drop”
  • The axillary nerve supplies the deltoid, teres minor, and the long head of the triceps brachii. Axillary nerve injury may result from dislocation of the head of the humerus
  • The anterior interosseous nerve branch of the median nerve supplies the deep muscles on the anterior of the forearm. This includes the flexor pollicis longus, pronator quadratus, and the radial half of flexor digitorum profundus

Review NCCPA Blueprint Topic: Carpal Tunnel Syndrome

7. A 55-year-old female comes to the ED complaining of moderate right eye pain, headache, and acute onset of blurry vision, which she describes as colored halos around lights. She was watching a movie at home with her husband about an hour ago when the pain began. On the physical exam of her right eye, her pupil is mid-dilated and unresponsive to light. Her right eyeball is firm to pressure. Intraocular pressure (IOP) measured with a tonometer is elevated at 36mmHg. Which of the following is the most appropriate emergency treatment?

  1. Timolol ophthalmic solution
  2. Epinephrine ophthalmic solution
  3. Laser peripheral iridotomy
  4. Anti-cholinergic ophthalmic solution
  5. NSAID ophthalmic solution

Click here to see the answer

Answer: A. Timolol ophthalmic solution

Acute angle-closure glaucoma, also known as narrow-angle glaucoma, presents with sudden onset blurry vision, hardened eyeball, and increased IOP. First-line emergency treatment includes alpha 2 selective adrenergic agonists, beta-blockers, or carbonic anhydrase inhibitors. Acute angle-closure glaucoma is caused by relative pupillary block of aqueous humor as it flows from the posterior to the anterior chamber through the iris-lens channel (the canal of Schlemm). Sudden attacks are more likely to occur when the pupil is partially dilated, for example, being in a darkened room such as a movie theater, or when eye drops are taken that dilate the pupil. If not treated immediately, it can damage the optic nerve and result in permanent vision loss within hours. Risk factors include certain medications (dilating drops, anticholinergic, antidepressants).

Incorrect Answers:

  • Epinephrine would cause pupil dilation which would worsen acute angle glaucoma.
  • Laser peripheral iridotomy is an appropriate and definitive treatment for acute angle glaucoma in the post-acute phase of treatment once IOP is controlled.
  • Anticholinergics would cause pupil dilation which would worsen acute angle glaucoma
  • NSAID ophthalmic solution can theoretically worsen acute angle glaucoma because of its anti-prostaglandin effect.

Review NCCPA Blueprint Topic: Glaucoma (Lecture)

8. A 68-year-old woman presents to your office for her annual check-up. Her vitals are HR 85, T 98.8 F, RR 16, BP 125/70. She has a history of smoking 1 pack a day for 35 years but states she quit five years ago. She had her last pap smear at age 64 and states all of her pap smears have been normal. She had her last colonoscopy at age 62, which was also normal. Which is the following is the next best test for this patient?

  1. Abdominal ultrasound
  2. Chest CT scan
  3. Pap smear
  4. Colonoscopy
  5. Chest radiograph

Click here to see the answer

The answer is B: Chest CT scan

The patient presents between the ages of 55 and 80 and has quit smoking within the last 15 years. She should undergo an annual low dose chest CT scan for lung cancer screening.

Cigarette smoking is the leading cause of preventable death in the United States and significantly contributes to deaths from cancer along with cardiovascular and pulmonary diseases. Smoking not only harms adults but also results in the deaths of about 1,000 infants annually. The USPSTF report on the guidelines for lung cancer screening states that age, total cumulative exposure to tobacco smoke, and years since quitting smoking are the most important risk factors for lung cancer. They report that annual screening for lung cancer with low-dose CT in a defined population of high-risk persons can prevent a substantial number of lung cancer–related deaths as evidenced by large randomized controlled trials.

Incorrect Answers:

  • An abdominal ultrasound is recommended for men between the ages of 65 and 75 who have ever smoked, but this recommendation does not hold for women
  • The patient does not require a pap smear because she is older than 65 and does not have a history of cervical or endometrial malignancy, and all of her previous pap smears have been negative.
  • The patient should next get her colonoscopy at age 72 since her last colonoscopy was at age 62 and was normal
  • The patient does not have any acute respiratory or pulmonary problems and therefore does not need a chest radiograph

Review NCCPA Blueprint Topic: Substance-related and addictive disorders (ReelDx)

9. A 22-year-old man presents to the emergency room complaining of pain upon urination and a watery discharge from his penis. It started a few days ago and has been getting progressively worse. His temperature is 98.0°F (36.7°C), blood pressure is 122/74 mmHg, pulse is 83/min, respirations are 14/min, and oxygen saturation is 98% on room air. Physical exam is notable for a tender urethra with discharge. Gram stain of the discharge is negative for bacteria but shows many neutrophils. Which of the following is the most likely infectious etiology of this patient’s symptoms?

  1. Chlamydia trachomatis
  2. Escherichia coli
  3. Neisseria gonorrhoeae
  4. Staphylococcus saprophyticus
  5. Trichomonas vaginalis

Click here to see the answer

The answer is A. Chlamydia trachomatis

This patient is presenting with a tender urethra with a discharge and a negative Gram stain suggesting a diagnosis of urethritis from Chlamydia trachomatis.

Urethritis in men presents with dysuria and urethral discharge with numerous neutrophils on urethral swab. Urethritis in men is most commonly caused by 2 categories of sexually transmitted infections. In gonococcal urethritis, patients present with purulent discharge and gram-negative diplococci on Gram stain, with N. gonorrhoeae as the offending agent. In nongonococcal urethritis, patients present with watery discharge which shows no bacteria on Gram stain (it is intracellular). Chlamydia is the most common offending agent in nongonococcal urethritis and is the most common cause of urethritis overall. Azithromycin is a good choice for the treatment of nongonococcal urethritis because it covers Chlamydia and other common causative organisms. It is often given with ceftriaxone which offers double coverage for N. gonorrhoeae.

Incorrect Answers

  • Escherichia coli is the most common cause of cystitis which presents with dysuria and leukocytes, bacteria, and nitrites on urinalysis. TMP-SMX is a first-line agent for the treatment of uncomplicated cystitis.
  • Neisseria gonorrhoeae is the most common cause of gonococcal urethritis which presents with a purulent urethral discharge and gram-negative diplococci on Gram stain. Ceftriaxone (a single injection) is the most common treatment for gonococcal urethritis. Ceftriaxone would be appropriate if the patient presented with purulent urethral discharge and a supportive Gram stain. Patients treated for gonococcal urethritis are generally treated for a Chlamydia trachomatis infection since there is a high rate of co-infection.
  • Staphylococcus saprophyticus is a common cause of UTI in young and sexually active women. It is still less common a cause of a UTI when compared to Escherichia coli.
  • Trichomonas vaginalis is a less common cause of male urethritis. It can present with pruritus but with less specific symptoms when compared to women with this condition. If the patient did not respond to antibiotics, this diagnosis could be suspected. Metronidazole is the treatment of choice.

Review NCCPA Blueprint Topic: Chlamydia (Lecture)

10. A 6-year-old boy is admitted with a one-week history of diarrhea, which was sometimes bloody and originally began after a birthday party. He has become lethargic and has not been eating or drinking. His vital signs are as follows: T 38.5 C, HR 135, BP 82/54. Physical examination is significant for petechiae on his legs and diffuse abdominal tenderness to palpation. Lab-work shows BUN 72 mg/dL, creatinine 8.1 mg/dL, and platelet count < 10,000. PT and PTT are within normal limits. Which of the following would be expected on a peripheral blood smear?

  1. Rouleaux formation
  2. Fragmented red blood cells
  3. Spur cells
  4. Giant platelets
  5. No abnormalities

Click here to see the answer

The answer is B. Fragmented red blood cells

The boy in this vignette most likely has hemolytic uremic syndrome (HUS), which is characterized by microangiopathic hemolytic anemia with schistocytes.

HUS usually occurs in children and is caused by an E. coli 0157:H7 infection. The classic presentation follows an acute diarrheal illness. HUS is characterized by microangiopathic hemolytic anemia, thrombocytopenia, and renal failure. The presentation is similar to thrombotic thrombocytopenic purpura (TTP) but without the fever and neurologic symptoms. The key differentiating factor, in this case, is diarrhea + elevated BUN. Keep in mind that in HUS uremia is elevated to a greater extent than TTP. On the other hand, TTP presents with more neurologic signs, and will not be preceded by diarrhea on the PANCE exam.

Incorrect Answers:

  • Rouleaux (stacked RBCs) are seen most notably in multiple myeloma. As well as many other hyperviscosity syndromes. This finding is highly non-specific.
  • Spur cells are seen with liver disease.
  • Giant platelets are seen in idiopathic thrombocytopenic purpura (ITP) as well as Bernard-Soulier syndrome.
  • Schistocytes would be expected in HUS.

Review NCCPA Blueprint Topic: Infectious Diarrhea (ReelDx + Lecture)

Looking for all the podcast episodes?

This FREE podcast series is limited to every other episode, you can download and enjoy the complete audio series by becoming a Smarty PANCE member.

I will be releasing new episodes every few weeks. Smarty PANCE is now discounted, so sign up now before it’s too late!

Resources and Links From The Show

This Podcast is also available on iOS and Android

  1. iTunes: The Audio PANCE and PANRE Podcast iTunes
  2. Spotify: The Audio PANCE and PANRE on Spotify
  3. Stitcher Radio: The Audio PANCE and PANRE Podcast Stitcher
  4. Google Play: The Audio PANCE and PANRE Podcast Google Play

itunes_logo-1

Download The Content Blueprint Checklist

Follow this link to download your FREE copy of the PANCE Content Blueprint Checklist

Print it up and start crossing out the topics you understand, marking the ones you don’t, and making notes of key terms you should remember. The PDF version is interactive and linked directly to the individual lessons on Smarty PANCE.

2020 Interactive NCCPA PANCE Content Blueprint

Download for PANCE Download for PANRE

The post Podcast Episode 85: Ten Mixed Multiple Choice PANCE and PANRE Board Review Questions appeared first on The Audio PANCE and PANRE.

]]>
Welcome to episode 85 of the Audio PANCE and PANRE PA Board Review Podcast. Join me as I cover ten PANCE and PANRE Board review questions from the SMARTYPANCE course content following the NCCPA™ content blueprint (download the FREE cheat sheet). Welcome to episode 85 of the Audio PANCE and PANRE PA Board Review Podcast.
Join me as I cover ten PANCE and PANRE Board review questions from the SMARTYPANCE course content following the NCCPA™ content blueprint (download the FREE cheat sheet).
Special from today’s episode:

* Download your copy of the Free Trello Smarty PANCE NCCPA™ Blueprint Study Plan
* Make the Audio PANCE and PANRE an Alexa Skill

This week we will be covering ten general board review questions based on the NCCPA PANCE and PANRE Content Blueprints. 
Below you will find an interactive exam to complement the podcast.
Below you will find an interactive exam to complement the podcast.
The Audio PANCE/PANRE and EOR PA Board Review Podcast
I hope you enjoy this free audio component to the examination portion of this site. The full board review course includes over 2,000 interactive board review questions and is available to all members of the PANCE and PANRE Academy and Smarty PANCE.

* You can download and listen to past FREE episodes here, on iTunes, on Google Play Music or Stitcher Radio.
* You can listen to the latest episode, take an interactive quiz, and download more resources below.

Listen Carefully Then Take The Practice Exam
If you can’t see the audio player click here to listen to the full episode.
Podcast Episode 85: Ten MIXED PANCE and PANRE Board Review Questions
The following questions are linked to NCCPA Content Blueprint lessons from the Smarty PANCE and PANRE Board Review Website. If you are a member, you will be able to log in and view this interactive video lesson.
1. A 14-hour-old boy has failed to pass stool and is vomiting greenish fluid. He was born at 40 weeks gestation with no complications during delivery or pregnancy. His temperature is 97.6°F (36.4°C), blood pressure is 64/34 mmHg, pulse is 140/min, respirations are 33/min, and oxygen saturation is 98% on room air. The child is currently breastfeeding and appears irritable. Physical exam is notable for a distended and non-tender abdomen.]]>
The Physician Assistant Life | Smarty PANCE full 23:14 394
Cardiac Pharmacology Part One: The Audio PANCE and PANRE Episode 84 http://podcast.thepalife.com/cardiac-pharmacology-part-one-the-audio-pance-and-panre-episode-84/ Tue, 11 Aug 2020 17:04:16 +0000 http://podcast.thepalife.com/?p=391 http://podcast.thepalife.com/cardiac-pharmacology-part-one-the-audio-pance-and-panre-episode-84/#respond http://podcast.thepalife.com/cardiac-pharmacology-part-one-the-audio-pance-and-panre-episode-84/feed/ 0 <p>Welcome to episode 84 of the Audio PANCE and PANRE PA Board Review Podcast Today we welcome back PA board review expert Joe Gilboy PA-C as he covers the exceptionally important topic of cardiac pharmacology. Below you will find a downloadable audio version of this podcast as well as links to resources mentioned in today’s episode. […]</p> <p>The post <a rel="nofollow" href="http://podcast.thepalife.com/cardiac-pharmacology-part-one-the-audio-pance-and-panre-episode-84/">Cardiac Pharmacology Part One: The Audio PANCE and PANRE Episode 84</a> appeared first on <a rel="nofollow" href="http://podcast.thepalife.com">The Audio PANCE and PANRE</a>.</p> Cardiac Pharmacology - The Audio PANCE and PANRE Board Review Podcast Episode 84Welcome to episode 84 of the Audio PANCE and PANRE PA Board Review Podcast

Today we welcome back PA board review expert Joe Gilboy PA-C as he covers the exceptionally important topic of cardiac pharmacology.

Below you will find a downloadable audio version of this podcast as well as links to resources mentioned in today’s episode. Members of Smarty PANCE can sign in and listen to the second half of Joe’s lecture as well as part two of this high-yield cardiac pharmacology lecture series by clicking here.

Not a Smarty PANCE member? You can gain instant access to this lecture as well as all the Smarty PANCE resources instantly by clicking here.

From today’s episode:

The Audio PANCE and PANRE Board Review Podcast Episode 84: Cardiac Pharmacology

I hope you enjoy this free audio component to the examination portion of this site. The full board review course includes over 2,000 interactive board review questions and is available to all members of the PANCE and PANRE Academy and Smarty PANCE.

If you can’t see the audio player click here to listen to the full episode.

Looking for all the podcast episodes?

This FREE podcast series is often limited to every other episode, you can download and enjoy the complete audio series by becoming a Smarty PANCE member.

Additional Resources

This Podcast is available on all platforms

Just search for “The Audio PANCE and PANRE” in your favorite podcasting program or use the following links.

Download The Content Blueprint Checklist

Follow this link to download your FREE copy of the PANCE Content Blueprint Checklist

Print it up and start crossing out the topics you understand, marking the ones you don’t, and making notes of key terms you should remember. The PDF version is interactive and linked directly to the individual lessons on Smarty PANCE.

2020 Interactive NCCPA PANCE Content Blueprint

Download for PANCE Download for PANRE

The post Cardiac Pharmacology Part One: The Audio PANCE and PANRE Episode 84 appeared first on The Audio PANCE and PANRE.

]]>
Welcome to episode 84 of the Audio PANCE and PANRE PA Board Review Podcast Today we welcome back PA board review expert Joe Gilboy PA-C as he covers the exceptionally important topic of cardiac pharmacology. Welcome to episode 84 of the Audio PANCE and PANRE PA Board Review Podcast
Today we welcome back PA board review expert Joe Gilboy PA-C as he covers the exceptionally important topic of cardiac pharmacology.
Below you will find a downloadable audio version of this podcast as well as links to resources mentioned in today’s episode. Members of Smarty PANCE can sign in and listen to the second half of Joe’s lecture as well as part two of this high-yield cardiac pharmacology lecture series by clicking here.
Not a Smarty PANCE member? You can gain instant access to this lecture as well as all the Smarty PANCE resources instantly by clicking here.
From today’s episode:

* To listen to the second half of this lecture as well as cardiac pharmacology part 2 with Joe Gilboy sign in to Smarty PANCE then click here.
* Listen to Joe’s previous podcast episodes:

* How to Answer The Hardest PANCE and PANRE Test Questions Part 1
* How to Answer Tricky PANCE and PANRE Test Questions Part 2
* How to Study for Your PANCE
* Podcast Episode 65: Hepatitis B Breakdown


* Content Blueprint PANCE/PANRE Review Course and EOR (Rotation) Courses

The Audio PANCE and PANRE Board Review Podcast Episode 84: Cardiac Pharmacology
I hope you enjoy this free audio component to the examination portion of this site. The full board review course includes over 2,000 interactive board review questions and is available to all members of the PANCE and PANRE Academy and Smarty PANCE.

* You can download and listen to past FREE episodes here, on iTunes, Spotify, Google Play Music, Stitcher, or just search “The Audio PANCE and PANRE” in your favorite podcast listening app.

If you can’t see the audio player full 49:57 391
How to Answer Tricky PANCE and PANRE Test Questions Part 2: Podcast Episode 83 http://podcast.thepalife.com/how-to-answer-tricky-pance-and-panre-test-questions-part-2-podcast-episode-83/ Tue, 09 Jun 2020 18:43:12 +0000 http://podcast.thepalife.com/?p=385 http://podcast.thepalife.com/how-to-answer-tricky-pance-and-panre-test-questions-part-2-podcast-episode-83/#respond http://podcast.thepalife.com/how-to-answer-tricky-pance-and-panre-test-questions-part-2-podcast-episode-83/feed/ 0 <p>In this second podcast of the series, Joe Gilboy PA-C discusses tips for answering tricky PANCE and PANRE test questions that make answering even the most difficult PANCE Board review questions a breeze</p> <p>The post <a rel="nofollow" href="http://podcast.thepalife.com/how-to-answer-tricky-pance-and-panre-test-questions-part-2-podcast-episode-83/">How to Answer Tricky PANCE and PANRE Test Questions Part 2: Podcast Episode 83</a> appeared first on <a rel="nofollow" href="http://podcast.thepalife.com">The Audio PANCE and PANRE</a>.</p> Welcome to episode 83 of the Audio PANCE and PANRE PA Board Review Podcast

Join me as we welcome Joe Gilboy PA-C back again to cover best practices that make answering even the toughest PANCE and PANRE Board review questions (and EOR exam questions) a breeze!

Below you will find interactive audio with clips covering the most important points. It’s a perfect companion to the podcast.

From today’s episode:

The Audio PANCE and PANRE Board Review Podcast Episode 83: How to Answer PANCE and PANRE Test Questions Part 2

I hope you enjoy this free audio component to the examination portion of this site. The full board review course includes over 2,000 interactive board review questions and is available to all members of the PANCE and PANRE Academy and Smarty PANCE.

If you can’t see the audio player click here to listen to the full episode.

Audio clips from today’s episode

Here are selected audio tips from today’s episode covering the most important points.

How to answer: “What is the most accurate test (what is the gold standard)?”

How to answer “What would you like to do next?” or “What is the next step?”

How to answer: “What’s the most likely answer/diagnosis – which one fits best?” (the simplest answer always wins)

In other words, if something carries 51% weight versus 49% weight, the 51% weight wins.

How to answer: “What is the most common etiology?”

How to answer: “What is second-line therapy?”

Looking for all the podcast episodes?

This FREE podcast series is often limited to every other episode, you can download and enjoy the complete audio series by becoming a Smarty PANCE member.

Additional Resources

This Podcast is also available on iOS and Android

  1. iTunes: The Audio PANCE and PANRE Podcast iTunes
  2. Stitcher Radio: The Audio PANCE and PANRE Podcast Stitcher
  3. Google Play: The Audio PANCE and PANRE Podcast Google Play

Download The Content Blueprint Checklist

Follow this link to download your FREE copy of the PANCE Content Blueprint Checklist

Print it up and start crossing out the topics you understand, marking the ones you don’t, and making notes of key terms you should remember. The PDF version is interactive and linked directly to the individual lessons on Smarty PANCE.

2020 Interactive NCCPA PANCE Content Blueprint

Download for PANCE Download for PANRE

The post How to Answer Tricky PANCE and PANRE Test Questions Part 2: Podcast Episode 83 appeared first on The Audio PANCE and PANRE.

]]> In this second podcast of the series, Joe Gilboy PA-C discusses tips for answering tricky PANCE and PANRE test questions that make answering even the most difficult PANCE Board review questions a breeze Welcome to episode 83 of the Audio PANCE and PANRE PA Board Review Podcast
Join me as we welcome Joe Gilboy PA-C back again to cover best practices that make answering even the toughest PANCE and PANRE Board review questions (and EOR exam questions) a breeze!
Below you will find interactive audio with clips covering the most important points. It’s a perfect companion to the podcast.
From today’s episode:

* Listen to Joe’s previous podcast episodes:

* How to Answer The Hardest PANCE and PANRE Test Questions: Podcast Episode 82
* How to Study for Your PANCE: Podcast Episode 79
* Podcast Episode 65: Hepatitis B Breakdown


* Start with high-quality PANCE practice questions in tutor mode
* Content Blueprint PANCE/PANRE Review Course and EOR (Rotation) Courses

The Audio PANCE and PANRE Board Review Podcast Episode 83: How to Answer PANCE and PANRE Test Questions Part 2
I hope you enjoy this free audio component to the examination portion of this site. The full board review course includes over 2,000 interactive board review questions and is available to all members of the PANCE and PANRE Academy and Smarty PANCE.

* You can download and listen to past FREE episodes here, on iTunes, on Google Play Music or Stitcher Radio.
* You can listen to the latest episode, take an interactive quiz, and download more resources below.

If you can’t see the audio player click here to listen to the full episode.
Audio clips from today’s episode
Here are selected audio tips from today’s episode covering the most important points.
How to answer: “What is the most accurate test (what is the gold standard)?”
How to answer “What would you like to do next?” or “What is the next step?”
How to answer: “What’s the most likely answer/diagnosis – which one fits best?” (the simplest answer always wins)
In other words, if something carries 51% weight versus 49% weight, the 51% weight wins.
How to answer: “What is the most common eti...]]>
The Physician Assistant Life | Smarty PANCE full 22:49 385 How to Answer The Hardest PANCE/PANRE Test Questions: Podcast Episode 82 http://podcast.thepalife.com/how-to-answer-the-hardest-pance-panre-test-questions-podcast-episode-82/ Tue, 31 Mar 2020 19:32:48 +0000 http://podcast.thepalife.com/?p=378 http://podcast.thepalife.com/how-to-answer-the-hardest-pance-panre-test-questions-podcast-episode-82/#respond http://podcast.thepalife.com/how-to-answer-the-hardest-pance-panre-test-questions-podcast-episode-82/feed/ 0 <p>In this podcast episode, Joe Gilboy PA-C discusses best practices for answering hard PANCE and PANRE test questions that make answering even the toughest NCCPA PANCE Board review questions a breeze</p> <p>The post <a rel="nofollow" href="http://podcast.thepalife.com/how-to-answer-the-hardest-pance-panre-test-questions-podcast-episode-82/">How to Answer The Hardest PANCE/PANRE Test Questions: Podcast Episode 82</a> appeared first on <a rel="nofollow" href="http://podcast.thepalife.com">The Audio PANCE and PANRE</a>.</p> How to Answer Hard PANCE and PANRE Test QuestionsWelcome to episode 82 of the Audio PANCE and PANRE PA Board Review Podcast

Join me as we welcome Joe Gilboy PA-C back again to cover best practices that make answering even the toughest PANCE and PANRE Board review questions (and EOR exam questions) a breeze!

Below you will find interactive audio breaking down each question. It’s a perfect companion to the podcast.

Special from today’s episode:

The Audio PANCE and PANRE Board Review Podcast Episode 82: How to Answer The Hardest PANCE/PANRE Test Questions

I hope you enjoy this free audio component to the examination portion of this site. The full board review course includes over 2,000 interactive board review questions and is available to all members of the PANCE and PANRE Academy and Smarty PANCE.

If you can’t see the audio player click here to listen to the full episode.

Questions we break down in this episode:

The following questions are linked to NCCPA Content Blueprint lessons from the Smarty PANCE and PANRE Board Review Website

Question 1: Developing good test-taking habits

1. 10-year-old boy presents to the emergency department complaining of sudden onset lightheadedness and chest discomfort, vital signs, 98.6, heart rate of 205, respiratory rate of 30 and 98% oxygen saturation, his EKG demonstrated sinus tachycardia with no p waves. What is the most likely diagnosis?

  1. A-fib
  2. A-flutter
  3. Atrioventricular reentry tachycardia
  4. V-tach

Steps to answering this question

Question 2: Choosing between two good answers

2. A 60-year-old woman with a history of hypertension, dyslipidemia, and coronary heart disease was sent to the emergency room from her primary care physician’s office for a heart rate of 40. She has no complaints, except for mild fatigue. Medications include metoprolol, atorvastatin, lisinopril, and baby aspirin. Her EKG reveals sinus bradycardia and her physical exam is normal. Which of the following is the most appropriate next step and management?

  1. Atropine
  2. Make a medication adjustment
  3. Schedule for a temporary pacemaker
  4. Watchful waiting

Steps to answering this question

Question 3: The long test question

3. A 32-year-old previously healthy man comes to ER with a four-hour history of palpitations. He denies chest pain, shortness of breath or history of similar palpitations. He does admit to heavy alcohol use in the past week drinking one pint of vodka and 24 packs of beer each day in the ED his vital signs of blood pressure 135/75, heart rate of 115, respiratory rate of 14, and oxygen saturation of 98% on room air. He’s got an irregularly irregular rhythm heard on auscultation and EKG shows atrial fibrillation. What is the next step in management?

  1. Chemical conversion
  2. Observation
  3. Rate control
  4. Synchronized cardioversion

Steps to answering this question:

Question 4: The short question with long answer choices

4. Which of the following patients should be classified as having unstable angina?

  1. A 51-year-old woman who has chest pain three days ago, but is now chest pain-free, is found to have a positive troponin with q waves in II, III, and aVF without ST-segment elevation
  2. A 55-year-old woman with a history of hypertension, but no prior cardiac disease who complains of one episode of chest pain that began while pushing her grocery cart and lasted 30 minutes
  3. A 65-year-old male with a known history of coronary artery disease who gets chest pain and shortness of breath every time he climbs the steps to his bedroom
  4. A 71-year-old man who underwent a coronary cardiac catheterization one month ago for early morning chest pain but shows minimal coronary artery disease with no fixed lesions presents with recurrent early morning chest pain that is relieved with nitroglycerin

Steps to answering this question:

Looking for all the podcast episodes?

This FREE podcast series is often limited to every other episode, you can download and enjoy the complete audio series by becoming a Smarty PANCE member.

Additional Resources

This Podcast is also available on iOS and Android

  1. iTunes: The Audio PANCE and PANRE Podcast iTunes
  2. Stitcher Radio: The Audio PANCE and PANRE Podcast Stitcher
  3. Google Play: The Audio PANCE and PANRE Podcast Google Play

itunes_logo-1

Download The Content Blueprint Checklist

Follow this link to download your FREE copy of the PANCE Content Blueprint Checklist

Print it up and start crossing out the topics you understand, marking the ones you don’t, and making notes of key terms you should remember. The PDF version is interactive and linked directly to the individual lessons on Smarty PANCE.

2020 Interactive NCCPA PANCE Content Blueprint

Download for PANCE Download for PANRE

The post How to Answer The Hardest PANCE/PANRE Test Questions: Podcast Episode 82 appeared first on The Audio PANCE and PANRE.

]]>
In this podcast episode, Joe Gilboy PA-C discusses best practices for answering hard PANCE and PANRE test questions that make answering even the toughest NCCPA PANCE Board review questions a breeze Welcome to episode 82 of the Audio PANCE and PANRE PA Board Review Podcast
Join me as we welcome Joe Gilboy PA-C back again to cover best practices that make answering even the toughest PANCE and PANRE Board review questions (and EOR exam questions) a breeze!
Below you will find interactive audio breaking down each question. It’s a perfect companion to the podcast.
Special from today’s episode:

* Listen to Joe’s previous podcast episodes:

* How to Study for Your PANCE: Podcast Episode 79
* Podcast Episode 65: Hepatitis B Breakdown


* Start with high-quality questions in tutor mode (untimed with hints on we have a TON on Smarty PANCE)
* Poem “lockdown“ by Brother Richard Hendrick
* Content Blueprint Review Course and EOR Courses

The Audio PANCE and PANRE Board Review Podcast Episode 82: How to Answer The Hardest PANCE/PANRE Test Questions
I hope you enjoy this free audio component to the examination portion of this site. The full board review course includes over 2,000 interactive board review questions and is available to all members of the PANCE and PANRE Academy and Smarty PANCE.

* You can download and listen to past FREE episodes here, on iTunes, on Google Play Music or Stitcher Radio.
* You can listen to the latest episode, take an interactive quiz, and download more resources below.

If you can’t see the audio player click here to listen to the full episode.
Questions we break down in this episode:
The following questions are linked to NCCPA Content Blueprint lessons from the Smarty PANCE and PANRE Board Review Website
Question 1: Developing good test-taking habits
1. 10-year-old boy presents to the emergency department complaining of sudden onset lightheadedness and chest discomfort, vital signs, 98.6, heart rate of 205, respiratory rate of 30 and 98% oxygen saturation, his EKG demonstrated sinus tachycardia with no p waves. What is the most likely diagnosis?

* A-fib
* A-flutter
]]>
The Physician Assistant Life | Smarty PANCE full 27:50 378
Podcast Episode 81: Internal Medicine EOR Cardiology Questions http://podcast.thepalife.com/podcast-episode-81-internal-medicine-eor-cardiology-questions/ Mon, 23 Mar 2020 15:15:19 +0000 http://podcast.thepalife.com/?p=375 http://podcast.thepalife.com/podcast-episode-81-internal-medicine-eor-cardiology-questions/#respond http://podcast.thepalife.com/podcast-episode-81-internal-medicine-eor-cardiology-questions/feed/ 0 <p>Welcome to episode 81 of the Audio PANCE and PANRE PA Board Review Podcast. Join me as I cover ten internal medicine rotation EOR content blueprint questions from the Smarty PANCE physician assistant board and rotation review website. Special from today’s episode: Check out the new Smarty PANCE Internal Medicine Rotation (EOR) Review Course Download […]</p> <p>The post <a rel="nofollow" href="http://podcast.thepalife.com/podcast-episode-81-internal-medicine-eor-cardiology-questions/">Podcast Episode 81: Internal Medicine EOR Cardiology Questions</a> appeared first on <a rel="nofollow" href="http://podcast.thepalife.com">The Audio PANCE and PANRE</a>.</p> Episode 81 episode 81 Ten PAEA Internal Medicine Rotation Exam (EOR) QuestionsWelcome to episode 81 of the Audio PANCE and PANRE PA Board Review Podcast.

Join me as I cover ten internal medicine rotation EOR content blueprint questions from the Smarty PANCE physician assistant board and rotation review website.

Special from today’s episode:

Below you will find an interactive exam to complement the podcast.

The Audio PANCE/PANRE and EOR PA Board Review Podcast

I hope you enjoy this free audio component to the examination portion of this site. The full board review course includes over 2,000 interactive board review questions and is available to all members of the PANCE and PANRE Academy and Smarty PANCE.

Listen Carefully Then Take The Practice Exam

If you can’t see the audio player click here to listen to the full episode.

Podcast Episode 81: Ten Internal Medicine EOR Cardiology Questions

The following questions are linked to PAEA Content Blueprint lessons from the Smarty PANCE and PANRE Board Review Website. If you are a member you will be able to log in and view this interactive video lesson.

1. 55-year-old woman is admitted to the hospital because she has shortness of breath and pain on both sides of her chest with deep breathing which has worsened over the past 5 days. In recent weeks, she has been feeling fatigued and has had low-grade fevers and night sweats, and was found to have a new cardiac murmur on examination. Her temperature is 38.1°C (100.6°F), pulse is 106/min, respirations are 26/min, and blood pressure is 136/88 mm Hg. She appears diaphoretic and is in mild respiratory distress. Cardiac auscultation reveals a faint systolic murmur heard over the lower left sternal border. Her neck veins are distended and abdominal examination shows hepatomegaly. Which of the following is the most likely cause of her clinical presentation?

A. Fat embolism
B. Infective endocarditis
C. Myocardial infarction
D. Rheumatic fever
E. Small cell lung cancer

Click here to see the answer

Answer: B, Infective endocarditis

Infective endocarditis (IE) is an infection of the endocardial surfaces of the heart, most commonly the heart valves. IE occurs when a microorganism begins to invade the heart valves causing an inflammatory reaction that damages the valve – sometimes leading to stenosis and sometimes leading to regurgitation. This patient developed a new cardiac murmur in recent weeks that is described as a faint systolic murmur heard over the lower left sternal border. This is most consistent with tricuspid regurgitation. She also has distended neck veins and hepatomegaly, both of which suggest right-sided heart failure which can result from tricuspid regurgitation. Small infectious emboli can break away from the primary lesion on the tricuspid valve and go into the pulmonary vasculature causing shortness of breath and pleuritic chest pain. Most cases of IE are caused by Staphylococcus aureus and Viridans streptococci, and diagnosis is confirmed by obtaining multiple blood cultures that show that there is continuous bacteremia.

  • Fat embolism to the pulmonary circulation almost always occurs with major trauma, including surgical procedures like intramedullary nailing of long bones. Fat emboli can occlude the microvasculature, triggering a systemic inflammatory response. The woman did not have recent trauma, and a fat embolism would not explain her cardiac murmur.
  • Clinical presentation of myocardial infarction most often includes substernal crushing chest pain, radiation of pain to the arms, left shoulder, back, neck, and jaw, as well as shortness of breath, anxiety, and fatigue. A myocardial infarction would not explain her developing a new cardiac murmur in recent weeks.
  • Rheumatic fever is an autoimmune inflammatory process that develops as a sequela of streptococcal infection. Clinical manifestations include polyarthritis, carditis, erythema marginatum, chorea, and subcutaneous nodules. Rheumatic fever can cause some of the symptoms seen in this case, but the acute onset makes this option less likely.
  • Although small cell lung cancer can present with pulmonary symptoms and low grade fevers, it wouldn’t easily explain her new cardiac murmur or the signs of right-sided heart failure.

Review PAEA EOR Blueprint Pearls With Links to Topic Lesson: Internal Medicine Rotation: Cardiovascular (PEARLS) ⇒ Endocarditis

2. A 54-year-old man comes to the emergency department following a four-day history of left-sided chest pain and shortness of breath. One week ago, he experienced upper respiratory symptoms along with myalgias and general fatigue. He has no known past medical history. He has not traveled outside the US. His temperature is 38.1°C (100.6°F), pulse is 104/min, respirations are 17/min, oxygen saturation is 94% on room air, and blood pressure is 100/72 mm Hg. Physical examination shows an ill-appearing man with bibasilar rales, jugular venous distention of 11 cm with no murmurs, rubs, or gallops, and 1+ bilateral pitting edema of the lower extremities. His labs are within normal limits and his blood cultures are negative. An echocardiogram shows an ejection fraction of 35%. Which of the following is the most likely cause of this patient’s condition?

A. Adenovirus
B. Coxsackie A virus
C. Staphylococcus aureus
D. Corynebacterium diphtheriae

Click here to see the answer

The answer is A. Adenovirus

Myocarditis in the United States is most commonly caused by a viral infection, typically adenovirus, coxsackie B, parvovirus B19, or others. It often presents with systemic symptoms such as fatigue and chest pain, but may cause sudden death.

  • Though coxsackie B virus is one of the most common causal pathogens of acute myocarditis, Coxsackie A causes herpangina, aseptic meningitis, and hand, foot, and mouth disease.
  • Staphylococcus aureus is a common cause of bacterial endocarditis, not myocarditis. Endocarditis is more likely to present with a new murmur and signs of distant emboli such as nail-bed hemorrhage and Janeway lesions.
  • Corynebacterium diphtheriae is the causal agent of diphtheria, which may involve myocarditis. However, the patient would then most likely have other symptoms of the disease, including pseudomembranous pharyngitis, lymphadenopathy, or an arrhythmia.

Review PAEA EOR Blueprint Pearls With Links to Topic Lesson: Internal Medicine Rotation: Cardiovascular (PEARLS)Myocarditis

3. A 42-year-old woman comes to the emergency department because of chest pain, dyspnea, and lightheadedness. She recently recovered from a presumed viral upper respiratory infection and has a 10-year history of systemic lupus erythematosus. Physical examination shows a decrease in systolic blood pressure by 20 mm Hg during inspiration. An ECG is shown here. Which of the following is the most likely diagnosis?

A. Aortic dissection
B. Tricuspid regurgitation
C. Pericardial effusion
D. Aortic stenosis
E. Mitral stenosis

Click here to see the answer

The answer is C. Pericardial effusion

Pericardial effusions are associated with a variety of causes, including autoimmune disorders and infectious pericarditis. If pericardial effusion leads to cardiac tamponade, patients may have pulsus paradoxus, or a decrease in systolic blood pressure during inspiration of more than 10 mm Hg. Certain ECG findings are characteristic for pericardial effusion. In particular, electrical alternans (shown by arrows) is highly specific for pericardial effusion (usually in association with cardiac tamponade) but not particularly sensitive. This pattern, characterized by beat-to-beat changes in the QRS axis in the limb and precordial leads, is caused by swinging of the heart within the accumulated pericardial fluid. Other common findings on ECG include sinus tachycardia and low QRS voltage.

  • Patients with aortic dissection classically present with a tearing or ripping pain in the chest or back and may have a significant variation in systolic blood pressure between arms (not across respirations).
  • The severity of tricuspid regurgitation varies widely, ranging from asymptomatic disease to symptoms of right-sided heart failure (hepatosplenomegaly, ascites, peripheral edema).
  • Patients with aortic stenosis, the most common cause of left ventricular outflow obstruction, typically present with exertional dizziness or angina. In end-stage disease, symptoms may progress to heart failure.
  • Mitral stenosis obstructs the flow of blood from the left atrium to the left ventricle and typically manifests as exertional dyspnea and decreased exercise tolerance.

Review PAEA EOR Blueprint Pearls With Links to Topic Lesson: Internal Medicine Rotation: Cardiovascular (PEARLS) ⇒ Pericarditis

4. A 57-year-old man comes to the emergency department because of intermittent, severe leg pain in both his calves for 2 weeks. He has a history of untreated high blood pressure, diabetes, and high cholesterol. For the past 3 years, the pain started after walking three blocks and only going away upon resting. In the past 2 weeks, he has had the same pain at rest. His temperature is 36.5°C (97.7°F), pulse is 78/min, respirations are 17/min, and blood pressure is 160/89 mm Hg. Examination shows both calves are atrophied and there is a paucity of hair, but no swelling or discoloration. Additionally, his lower calves are cool to the touch and dusky in appearance. Doppler ultrasound shows perfusion to both feet, and blood pressures of 35/20 mm Hg in the posterior tibial artery are obtained bilaterally. Which of the following is most appropriate next step in management?

A. Immediate high-dose statin therapy
B. Immediately consult vascular surgery
C. Intravenous alteplase
D. Pain control and discharge; refer to vascular surgery as an outpatient
E. Rest, elevate, and compress the affected leg

Click here to see the answer

The answer is B. Immediately consult vascular surgery

Peripheral vascular disease with intermittent claudication and signs of decreased perfusion should be examined using Doppler ultrasound and the ankle-brachial index (ABI). An ABI of <0.41 is grounds for the immediate surgical consultation.

  • Statin therapy has shown to be effective in treating intermittent claudication. However, this intervention is appropriate only when the patient is deemed to be stable, and the risk of limb ischemia has been addressed.
  • Alteplase (tPa) has been used to treat intermittent claudication, but its use is outside the scope of the emergency room. Treatments initiated in the emergency department are unfractionated heparin, aspirin, pain control, and positioning of the leg to maintain perfusion.
  • The patient has significant impairment of perfusion to his legs. Discharge and vascular surgery follow-up as an outpatient would be inappropriate and likely to result in morbidity or mortality.
  • While this leg should be rested, compression and elevation are the opposite of what is needed. These therapies will further decrease the blood flow to the leg and put the patient at increased risk of complications.

Review PAEA EOR Blueprint Pearls With Links to Topic Lesson: Internal Medicine Rotation: Cardiovascular (PEARLS)Peripheral vascular disease

5.  A 6-year-old boy comes to the pediatric clinic because of 4 weeks of fatigue and shortness of breath. His medical history is notable for poor follow-up with yearly check-ups, but his mother notes that he has been fairly healthy for the duration of his life, except for a “bad sore throat that needed antibiotics” about 6 months ago. On examination, the boy appears fatigued, but is appropriately oriented and responsive, and is generally non-toxic appearing. He is afebrile. Cardiac examination is notable for a widely split S2, a quiet S1, and point of maximum impulse displaced to the left. Which of the following is the next best step in the management of this patient?
Elimination tool

A. Digoxin
B. Long-term penicillin
C. Short-term course of clindamycin
D. Reassurance & routine care
E. Intramuscular ceftriaxone

Click here to see the answer

The answer is B. Long-term penicillin

Rheumatic heart disease is caused by autoimmune cross-reactivity following a Streptococcal infection. Development of antibody-mediated mitral valve damage is common. Long-term penicillin is suitable for acute treatment and prophylaxis from complications.

  • Digoxin is a purified cardiac glycoside that is typically used in the treatment of various heart conditions, such as atrial fibrillation, atrial flutter, and heart failure that cannot be controlled by other medication. It is not recommended for use in rheumatic fever.
  • Clindamycin is an antibiotic used to treat middle ear infections, bone or joint infections, pelvic inflammatory disease, strep throat, pneumonia, and endocarditis. Clindamycin is typically used as endocarditis prophylaxis for patients that are already receiving penicillin for secondary rheumatic fever prophylaxis.
  • This patient is suffering from rheumatic fever, which can lead to rheumatic heart disease. This can cause significant carditis which manifests as congestive heart failure.
  • Ceftriaxone is an antibiotic used to treat numerous bacterial infections, such as pneumonia, ear infections, skin infections, urinary tract infection, and meningitis. It is not used in the treatment of rheumatic fever.

Review PAEA EOR Blueprint Pearls With Links to Topic Lesson: Internal Medicine Rotation: Cardiovascular (PEARLS)Rheumatic heart disease

6. A 35-year-old woman, gravida 2, para 1, comes to the office because of her first prenatal visit at 12 weeks’ gestation. She has no current concerns, and her first child was born without complications. She has a history of long-standing untreated hypertension. Her temperature is 36.5°C (97.7°F), pulse is 78/min, respirations are 17/min, and blood pressure is 150/89 mm Hg. Which of the following medications would most likely be appropriate as initial treatment?

A. Atenolol
B. Hydrochlorothiazide
C. Labetalol
D. Lisinopril
E. Losartan

Click here to see the answer

The answer is C. Labetalol

Chronic hypertension in pregnancy can be treated with some medications and labetalol is recommended as a first-line therapy for treatment of hypertension during pregnancy. Angiotensin-converting enzymes (ACE) inhibitors and angiotensin II receptor blockers (ARBs) should be avoided.

  • Atenolol is a β-blocker medication, which decreases blood pressure by decreasing heart rate and stroke volume. It is contraindicated in pregnancy because it has been associated with fetal growth restriction. β-blockers considered safe in pregnancy are metoprolol and labetalol.
  • Hydrochlorothiazide is a thiazide diuretic which decreases blood pressure via volume depletion. Diuretics do not cause fetal malformations but are generally avoided in pregnancy, as they prevent the physiologic volume expansion seen in normal pregnancy. They may be used in states of volume-dependent hypertension, such as renal or cardiac disease.
  • Lisinopril is an angiotensin-converting enzyme (ACE) inhibitor which decreases blood pressure through inhibition of the renin-angiotensin-aldosterone system. ACE inhibitors and the closely related angiotensin-receptor blockers are contraindicated in pregnancy because of the risk of birth defects including renal dysgenesis and oligohydramnios.
  • Losartan is an angiotensin II receptor blocker (ARB) which decreases blood pressure through inhibition of the renin-angiotensin-aldosterone system. ARBs and the closely related angiotensin converting enzyme (ACE) inhibitors are contraindicated in pregnancy because of the risk of birth defects including renal dysgenesis and oligohydramnios.

Review PAEA EOR Blueprint Pearls With Links to Topic Lesson: Internal Medicine Rotation: Cardiovascular (PEARLS) ⇒ Hypertension

7. A 42-year-old man comes to the office for a routine check-up. Medical history includes diabetes mellitus and a long history of smoking. Family history includes coronary artery disease. Temperature is 36.5°C (97.7°F), pulse is 78/min, respirations are 17/min, and blood pressure is 160/89 mm Hg. A repeat blood pressure taken 2 days later shows 143/88 mm Hg. Which of the following is most likely the best initial therapy?

A. Enalapril
B. Furosemide
C. Hydrochlorothiazide
D. Metoprolol
E. Nifedipine

Click here to see the answer

Answer: A. Enalapril

Hypertension in the setting of diabetes should be treated, with a goal systolic blood pressure of <130 mm Hg. The best first choice of medication is an angiotensin-converting enzyme inhibitor (ACEi).

Incorrect Answers:

  • Furosemide is a loop diuretic that can be used to treat hypertension but it is not a first line agent. Unlike ACE inhibitors, loop diuretics have no renoprotective qualities and therefore are a worse initial choice in a hypertensive diabetic patient.
  • Hydrochlorothiazide is a thiazide diuretic and a first-line agent for the treatment of hypertension in non-diabetic patients. It is relatively contraindicated in diabetes because it has the potential adverse effect of hyperglycemia. It may also precipitate attacks of gout by increasing uric acid concentrations.
  • Metoprolol is a β-blocker that is used for the treatment of hypertension but is not a first line treatment. Also, β-blocker are contraindicated in asthma, as well as in diabetic patients due to its potential to mask signs of low blood sugar.
  • Nifedipine is a calcium channel blocker that is used in the treatment of hypertension but it is not a first line therapy. It has no renoprotective effects, and commonly causes peripheral edema as an unwanted adverse effect.

Review PAEA EOR Blueprint Pearls With Links to Topic Lesson: Internal Medicine Rotation: Cardiovascular (PEARLS)Hypertension

8. A 45-year-old obese Caucasian gentleman arrives at your clinic for a routine check-up after having some blood work done during a workplace health screening. He is found to have an LDL cholesterol level of 550 mg/dL. He states that his father and brother had high cholesterol and both died at a young age from a heart attack. He has a follow-up appointment with his cardiologist because of some occasional chest pain and abnormalities seen on his EKG. Additionally, you notice that he has well-demarcated yellow deposits around his eyes. He is started on high dose statin and his LDL at 12 weeks is 350 mg/dL. What is the next best step in this patient’s management?

A. Continue high dose statin, the patient’s LDL is at goal
B. Add niacin 100 mg three times daily
C. Add ezetimibe (Zetia) 10 mg daily
D. Add a PCSK9 inhibitor
E. Refer to a lipid specialist

Click here to see the answer

The answer is C: add ezetimibe 10 mg

If LDL-C is not at goal after 6-12 weeks the next best step for the treatment of familial hypercholesterolemia is to add ezetimibe 10 mg daily and check again in 6-12 weeks. If at that time the patient’s LDL is still not at goal (ideally < 150) refer to lipid specialist to consider adding a PCSK9 inhibitor.

Pearls

  • Familial hypercholesterolemia (FH) is the most common autosomal dominant genetic disease. The clinical syndrome (phenotype) is characterized by extremely elevated levels of low-density lipoprotein cholesterol (LDL-C) and a propensity to early-onset atherosclerotic cardiovascular disease. In general, homozygotes manifest the disease at a much earlier age than heterozygotes and the disease is more severe.
  • Homozygous FH patients are rare and have an estimated prevalence of approximately 1:300,000 to 1:400,000
  • Heterozygous FH is estimated to occur in 1 in 200 to 250 individuals in the United States.
  • It is estimated that about 7 percent of American adults have an untreated lipoprotein cholesterol ≥190 mg/dL but only 1.7 percent carry an FH mutation
  • Patients with undiagnosed homozygous familial hypercholesterolemia (FH) develop severe, premature, atherosclerotic cardiovascular disease and die before age 20 in many cases.
  • In patients with a negative or unknown family history, an untreated LDL-C level of ≥190 mg/dL (4.9 mmol/L) suggests FH. This value is greater than the 90th percentile for age and sex.

Diagnosis

  • The diagnosis of heterozygous familial hypercholesterolemia (FH) is made with genetic testing or clinical criteria. A causative mutation in the LDLR, APOB, or PCSK9 gene(s) secures this diagnosis
  • When genetic testing is not available or not felt to be necessary, you can use the Dutch Lipid Clinic Network criteria, which assigns points based on low-density lipoprotein cholesterol (LDL-C) levels, personal history of early atherosclerotic cardiovascular disease (ASCVD), family history of early ASCVD, or high cholesterol in a first-degree relative, and personal and physical examination finding

Treatment

  • Patients with homozygous familial hypercholesterolemia (FH) – intensive LDL-C lowering, which targets a minimal value of <150 mg/dL (3.9 mmol/L)
  • In addition to a high-dose statin (atorvastatin 80 mg daily or rosuvastatin 40 mg daily), most homozygous patients will require additional therapies such as ezetimibe, a PCSK9 inhibitor, or potentially LDL-C apheresis

Review PAEA EOR Blueprint Pearls With Links to Topic Lesson: Internal Medicine Rotation: Cardiovascular (PEARLS)Hyperlipidemia

9. A 65-year-old man comes to the emergency department because of progressive dyspnea, coughing, and orthopnea. The patient says that over the past 2 months he has been feeling fatigued with ordinary physical activity. His medical history is relevant for dyslipidemia, type II diabetes mellitus, and a posterior myocardial infarction 4 months ago. Auscultatory findings reveal a pansystolic murmur over the mitral area. His temperature is 37.5°C (99.5°F), pulse is 90/min, respirations are 17/min, blood pressure is 120/90 mm Hg, and pulse oximetry on room air shows an oxygen saturation of 95%.  This patient most likely has which of the following conditions?

A. Aortic stenosis
B. Aortic valve regurgitation
C. Mitral valve regurgitation
D. Mitral valve stenosis
E. Pulmonary stenosis

Click here to see the answer

The answer is C. Mitral valve regurgitation

Mitral valve regurgitation is defined as an incompetent closure of the mitral valve. Classically patients will present with a pansystolic heart murmur over the mitral area. Posterior myocardial infarction is the second most common cause of mitral valve regurgitation. Mitral valve regurgitation is characterized by an incompetent closure of the mitral valve. This condition causes retrograde blood flow into the left atrium during systole. The most common cause of mitral regurgitation is mitral valve prolapse, followed by a dysfunction of the posteromedial papillary muscle due to posterior myocardial infarction. Other causes may include, endocarditis or stretching of the mitral valve ring.

  • Aortic stenosis is the reduction of the valvular orifice (<2 cm) with left ventricular outflow obstruction. Patients with aortic stenosis present with chest pain, syncopal episodes, and dyspnea. Here, the patient has progressive dyspnea (NYHA II), and a pansystolic murmur (mitral area), which relates to mitral valve regurgitation
  • Aortic valve regurgitation (AOR) is described as an inefficient closure of the aortic valve leading to a retrograde blood flow into the left ventricle during diastole. Patients will present with a wide pulse pressure (Corrigan hammer pulse) or an Austin-Flint murmur (severe), none of which are present in this case.
  • Mitral valve stenosis (MVS) is the reduction of the mitral valve orifice (<2.5cm). The left atrium becomes dilated and hypertrophied because of increased work of the left atrium. Patients with MVS present with dyspnea, rust-colored sputum, atrial fibrillation, and a diastolic heart murmur (diastolic rumble and opening snap).
  • Pulmonary stenosis is an uncommon valvular lesion. It is commonly associated with congenital heart disease and carcinoid heart disease. Patients present a systolic ejection murmur in the left second intercostal space.

Review PAEA EOR Blueprint Pearls With Links to Topic Lesson: Internal Medicine Rotation: Cardiovascular (PEARLS)Valvular heart disease

10. A 43-year-old man with a history of rheumatic fever comes to the primary care clinic for a check up. Cardiac examination reveals a late systolic crescendo murmur with midsystolic click best heard over the apex and loudest just before S2. Which of the following maneuvers will cause an earlier onset of the click/murmur?

A. Inspiration
B. Rapid squatting
C. Standing
D. Hand grip
E. Left lateral cubital position

Click here to see the answer

The answer is C. Standing

The ‘click’ of mitral valve prolapse, caused by the tightening of the chordae tendinae, moves closer to S2 with increased preload. Increased preload causes the left ventricle to stretch, as a result, the chordae tendinae are stretched as well. This makes it harder for the mitral valve to prolapse until the ventricles shrink enough to allow the chordae tendinae to let the mitral valve prolapse. Since there is more blood in the ventricles, it takes them longer to pump it out and shrink to a point at which MVP can occur, hence it occurs later in systole. Thus, the click will get closer to S2 with increased preload. Hand grip, rapid squatting, and inspiration all increase preload. Standing decreases preload and will cause an earlier onset of the click.

  • The click of mitral valve prolapse is heard closer to S2 in the event of increased preload. Preload is increased by hand grip, squatting and inspiration.
  • Placing the patient in the left lateral cubitus position will not affect preload and will have no effect on the timing of the click in mitral valve prolapse.

Review PAEA EOR Blueprint Pearls With Links to Topic Lesson: Internal Medicine Rotation: Cardiovascular (PEARLS) ⇒ Heart murmurs

Looking for all the podcast episodes?

This FREE podcast series is limited to every other episode, you can download and enjoy the complete audio series by becoming a Smarty PANCE member.

I will be releasing new episodes every few weeks. Smarty PANCE is now discounted, so sign up now before it’s too late!

Resources and Links From The Show

This Podcast is also available on iOS and Android

  1. iTunes: The Audio PANCE and PANRE Podcast iTunes
  2. Stitcher Radio: The Audio PANCE and PANRE Podcast Stitcher
  3. Google Play: The Audio PANCE and PANRE Podcast Google Play

itunes_logo-1

Download The Content Blueprint Checklist

Follow this link to download your FREE copy of the PANCE Content Blueprint Checklist

Print it up and start crossing out the topics you understand, marking the ones you don’t, and making notes of key terms you should remember. The PDF version is interactive and linked directly to the individual lessons on Smarty PANCE.

2020 Interactive NCCPA PANCE Content Blueprint

Download for PANCE Download for PANRE

The post Podcast Episode 81: Internal Medicine EOR Cardiology Questions appeared first on The Audio PANCE and PANRE.

]]>
Welcome to episode 81 of the Audio PANCE and PANRE PA Board Review Podcast. Join me as I cover ten internal medicine rotation EOR content blueprint questions from the Smarty PANCE physician assistant board and rotation review website. Welcome to episode 81 of the Audio PANCE and PANRE PA Board Review Podcast.
Join me as I cover ten internal medicine rotation EOR content blueprint questions from the Smarty PANCE physician assistant board and rotation review website.
Special from today’s episode:

* Check out the new Smarty PANCE Internal Medicine Rotation (EOR) Review Course
* Download your Free Trello Smarty PANCE PAEA Internal Medicine EOR Tracking Template
* Take the FREE Internal Medicine Rotation Cardiovascular Practice Exam
* View the interactive Smarty PANCE Internal Medicine Rotation (EOR) Topic List

Below you will find an interactive exam to complement the podcast.
The Audio PANCE/PANRE and EOR PA Board Review Podcast
I hope you enjoy this free audio component to the examination portion of this site. The full board review course includes over 2,000 interactive board review questions and is available to all members of the PANCE and PANRE Academy and Smarty PANCE.

* You can download and listen to past FREE episodes here, on iTunes, on Google Play Music or Stitcher Radio.
* You can listen to the latest episode, take an interactive quiz and download more resources below.

Listen Carefully Then Take The Practice Exam
If you can’t see the audio player click here to listen to the full episode.
Podcast Episode 81: Ten Internal Medicine EOR Cardiology Questions
The following questions are linked to PAEA Content Blueprint lessons from the Smarty PANCE and PANRE Board Review Website. If you are a member you will be able to log in and view this interactive video lesson.
1. 55-year-old woman is admitted to the hospital because she has shortness of breath and pain on both sides of her chest with deep breathing which has worsened over the past 5 days. In recent weeks, she has been feeling fatigued and has had low-grade fevers and night sweats, and was found to have a new cardiac murmur on examination. Her temperature is 38.1°C (100.6°F), pulse is 106/min, respirations are 26/min, and blood pressure is 136/88 mm Hg. She appears diaphoretic and is in mild respiratory distress.]]>
The Physician Assistant Life | Smarty PANCE full 28:09 375
How to Study for Your PANCE: Podcast Episode 79 http://podcast.thepalife.com/how-to-study-for-your-pance-podcast-episode-79/ Mon, 02 Mar 2020 21:41:53 +0000 http://podcast.thepalife.com/?p=367 http://podcast.thepalife.com/how-to-study-for-your-pance-podcast-episode-79/#respond http://podcast.thepalife.com/how-to-study-for-your-pance-podcast-episode-79/feed/ 0 <p>Podcast Episode 79: How to Study for Your PANCE Welcome to episode 79 of the Audio PANCE and PANRE PA Board Review Podcast. In this episode 35-year PA veteran and PANCE/PANRE board review expert Joe Gilboy PA-C shares his top tips on how to best prepare for the Physician Assistant National Certification Exam Special Links […]</p> <p>The post <a rel="nofollow" href="http://podcast.thepalife.com/how-to-study-for-your-pance-podcast-episode-79/">How to Study for Your PANCE: Podcast Episode 79</a> appeared first on <a rel="nofollow" href="http://podcast.thepalife.com">The Audio PANCE and PANRE</a>.</p> Podcast Episode 79 How To Study For Your PANCE with Joe Gilboy PA-C

Podcast Episode 79: How to Study for Your PANCE

Welcome to episode 79 of the Audio PANCE and PANRE PA Board Review Podcast.

In this episode 35-year PA veteran and PANCE/PANRE board review expert Joe Gilboy PA-C shares his top tips on how to best prepare for the Physician Assistant National Certification Exam

Special Links From Today’s Episode:

The Audio PANCE and PANRE Physician Assistant Board Review Podcast

I hope you enjoy this free audio component to the examination portion of this site. The full board review includes over 2,000 interactive board review questions and is available to all members of Smarty PANCE.

Looking for all the podcast episodes?

This FREE series is limited to every other episode, you can download and enjoy the complete audio series by joining The PANCE and PANRE Exam Academy + SMARTYPANCE

I will be releasing new episodes every few weeks. Smarty PANCE is discounted, so sign up now.

Table of Contents and Key Points from this episode:

  1. Start Preparing One Year Before Your Exam Date
  2. Consider Forming a Study Group
  3. Print out the Blueprint
  4. Jot Down Quick Notes From Memory
  5. The Simple Answer Is Usually The Correct Answer
  6. In a 51/49 Split Choose The 51%
  7. Look For The Hook!
  8. Control Your X-Factors: I.E., Don’t Add Stress Where You Don’t Need It
  9. Practice Your Questions In Tutor Mode
  10. Correct Yourself in Real-Time
  11. Only Start Timing Yourself Towards The End
  12. Only Use High-Quality Questions
  13. Do 20-25 Questions Max Per Day
  14. The Best Textbooks for PANCE Prep
  15. The Best Websites for PANCE Prep
  16. Summary and Recommendations

Podcast Transcript and Timestamps

If you can’t see the audio player click here to listen to the full episode.

0:00

Welcome, PA, world. My name is Joe Gilboy, PA-C. I work with Stephen Pasquini with the smarty PANCE website.

Today’s podcast is going to be on how to prepare for the PANCE, which is a very big subject and a very important subject to a lot of you guys out there. So come on guys, let’s get this game roll, and let’s learn how to prepare for the PANCE.

0:33

Hello, PA podcast world. My name is Joe Gilbert. I’ve been a PA for about 35 years, with all my time being in the emergency room.

I’m a graduate of the Duke University PA program from 1984. I started helping students prepare for the PANCE and PANRE starting in 1990. So, as you can imagine, I’ve had a lot of students come through my hands. The answer is greater than 10,000. I’ve taught in several schools here in California I teach at several schools in Nevada. I also help in Tennessee. I help in Ohio, I help in Georgia, so I am around in a lot of different places, but for today, let’s get you ready to take the PANCE. So I know a lot of ways to get students prepared. So, let’s start.

1:14

So, today’s podcast is actually going to be on how to prepare for the PANCE. So before we get going into the very specifics, let’s look at so just come some basic guidelines as a student as you’re getting ready to take the PANCE. So, the first thing that we need to actually do is study! I now I know this sounds a little oxymoronic, but you’d be surprised how many students I’ve met along the way, do really well in PA school, you know, they do well in their grades they do well in their End of Rotation Exams, they feel like they don’t need to study. This couldn’t be farther from the truth.

1:44

Start Preparing One Year Before Your Exam Date

So usually what I tell most students to do is start preparing for this exam literally about a year before it comes. What you don’t understand and what’s coming is that when you get towards the end of your PA school, you’re going to be so burned out is just amazing how burned out you’ll be, and you’ll see that especially when you go out on rotations and come back for maybe your callback week, or maybe End of Rotation Exam or some kind of maybe specific lecture, and I can guarantee you on that first day back, when you’ve been gone on rotations, it’s like pulling teeth, you’re like, oh, I’m back in this room, again, that I’ve spent so much of my life, I don’t want to be here. And it’s like, your mind’s just not ready for it. So, what I tell my students to do, instead of trying to cram this all in at the end, play it out.

2:28

So basically, about a year before, start getting ready for this. So what I tell most people to do study. Now, one of the questions that commonly comes up is how much do I study? And at most, most people, I tell them, if you can get up to like maybe two hours or three hours a day, that’s great. Max is four. So you’ll see there’s a lot of studies out there that will show that if you take a human being and actually get them studying for more than about three hours, three and a half hours they do not retain and then things get a little frustrating. So two to three hours if you can, no more than four. And of course, you know, always give yourself a day off.

3:13

The other thing about studying is the worst place to study. Yeah, where are you going to study guys home? And so what you have to understand is that when you’re studying, you actually need stimulus free. So your cell phone is off. Now I know this is a big one for most of you young millennials out there; you’re going “what, I have to turn it off?” That’s right. Don’t worry; the world will keep revolving with the cell phone off. But one of the things they’ve done there’s a lot of studies out there showing that when a cell phone is on, literally 15% of your brain is waiting for that thing to go off. I need that 15% of the brain getting ready for the PANCE, and at the end of the day, guys, there’s no cell message or text messages that’s that important. They can always call 911. So again, just make sure your cell phone is off. This is a big thing to really keep in the back your head.

4:00

Consider Forming a Study Group

The other thing sometimes people like to work together and groups. Groups are great. Study groups work well; just the key thing to a study group is that as you’re in the group, you have to teach the other student, maybe about a subject. And you see when you’re a teacher, you have to know a subject twice, once for your own knowledge, but second to be able to present it that knowledge to another person. So study groups do work.

4:09

Print out the Blueprint

The other thing I tell people to do is print out the PANCE blueprint. It’s pretty extensive and pretty large if you’ve ever seen it, it’s a very large extensive blueprint. And what I tell people to do, so go the NCCPA website, print out the blueprint.

2020 Interactive NCCPA PANCE Content Blueprint

 

Download for PANCE Download for PANRE

4:33

Jot Down Quick Notes From Memory

Then what you do, it’s like, let’s say you get to your cardiovascular and let’s say it says mitral stenosis. So, what I tell my students to do is actually physically write down the word mitral stenosis, and then just jog down some quick notes about things that you memorize in school, and you’re like, I think I remember opening snap. I think I remember rheumatic fever. I think I remember somebody Jones, Jone’s criteria. Jones something, I remember and then stop. And then at that moment, what you do is you start reading up on other things about the Jones criteria. So, in other words, you write more notes to help you out and stuff like that. And this is one of the things that I tell a lot of students to do. Because believe it or not, when you write something down, you know, it’s twice, once for thought, second for motor. For example, just to show you how your brain works, and I know you did this in PA school, you wrote down a note, maybe like, on your one note on your computer, or maybe you wrote it down on a handout that you’re given. You took the test. There you were on the test going. What was it again? I wrote about Jone’s criteria? God, I can’t remember this question. I know I wrote it down somewhere, but I can’t remember. And as soon as the test is over, you walk outside, and you know exactly where it is on your notes. It is in the upper right-hand corner of circle than pink. Welcome to your Brain, you wrote it down once for thought and then second for motor. So when we do this, we actually locking it. I’m not saying that we need to write a dissertation on mitral stenosis, what I’m really saying to you is that just write down some quick notes and try to make this more kind of like thought friendly, like, hey, this is what I remember. And this is what makes sense. And then you read up on the Jones criteria like That’s right. Fevers, minor, even though I want to make it major and things like that. So that tends to work out really well with my students.

6:28

The Simple Answer Is Usually The Correct Answer

The other thing that we’re going to do, and I’ll try to help you guys out, is with the ambiguous questions. Who got into a test, you’re taking the test? And you were choosing between A and B. And you were like, I think it’s A I think it’s B thing? It’s A I think it’s B. And so, there was this ambiguous question, and you couldn’t figure out which was the right answer. So you felt like you’re in this what I call ambiguous 50/50 game. So when we play the ambiguous 50/50 game, there are a couple things you need to do. Just a little test-taking technique that works all the time. One, keep it simple. The simple answer is usually the correct answer one more time; the simple answer is usually the correct answer.

7:06

The Answer That Weighs Just a Little Bit More Than The Other is Your Answer (In a 51/49 Split Take the 51%)

If one answer outweighs the other, even if it’s to a 51/49 edge. In other words, 51% of the time is right here. 49% of the time, it’s right there. What do I take? The 51! So, it only takes 1% to push me over. So one more time. Keep it simple. And the one that weighs just a little bit more than the other? That’s your answer.

7:30

Look for the Hook!

Every now and then, what you’ll see is that student’s kind of miss like a little, like a little piece of the word up in the question. In other words, most likely, at least likely first line, second line. So again, as we go through test questions, we have to be really alert about all the various words they use. And remember at the end of the question is what I call the hook. So sometimes what students will do is they’ll read the last line of the question, so we’ll just say, you know, there’s a question, and it says, What’s the most likely treatment. So as you go to the question, you realize this is what it’s asking for is a treatment. So the whole time you read the question like, so what’s going to be the treatment for this patient? Maybe it’s a CHF, pneumonia, whatever it may be. So again, another test-taking technique that kind of works pretty well. Yeah, try saying that five times fast.

8:21

Control Your X-Factors: i.e., Don’t Add Stress Where You Don’t Need It

The other thing that I tell students to do, and hopefully this, this handout will also be up there on the smarty PANCE website on how to prepare the seven major points, which is what we’re going over is to control your x-factors. This is huge. Guys. If I had to put my 30 years of teaching students into a paragraph, oh, this is it. So when I say control your x-factors, what I’m saying to you is this is that as you get ready to take this PANCE, don’t change your emotional or social wellbeing at all. Now granted, you can’t prevent certain things from coming like a death in the family and illness, things like that. But don’t create chaos is what I always tell my students because PA school is chaotic enough; the last thing you need to do is create more chaos. So don’t try to change your relationships during PA school, just play them out. Don’t try to change your surroundings, don’t buy anything big. I mean, come on, guys, every single one of you guys is in debt. So, going out and buying a Tesla is not the smartest thing to do before the PANCE because now you’re kind of worried about how am I going to pay for this Tesla. And I always tell people, this is you know, in other words, don’t add stress to your life. This is what I’m really trying to get at. Try to make this as stress-free as possible.

9:34

Practice Your Questions in Tutor Mode

The last thing, the number seven-point that I’m always trying to make to my students, practice your questions in tutor mode. This is big. This is huge. And if all this thing in the podcast I hope remember is this point, you see guys; our brain is much more like a dog than you know. In other words, we need to be corrected in real-time. For example, there’s this world called neuroplasticity, just type it in enjoy the ride, guys. Okay, and it pretty much goes like this. you fire it, I wire it. That’s how your brain works. If you fire it, I’ll wire it. But if it’s fired the wrong way, you need to wire it the correct way, in real-time. For example, when your dog pees on the bed, what do you do? You scold him at that moment, but you don’t scold him three hours later. He’s like, going, what do you hit me for? I’m over here chewing my bone, and in other words, you have to correct them in real-time. As humans, we’re the same way.

10:30

Correct Yourself in Real-Time

We need to be corrected in real-time. And so, there’s a lot of studies that have come out here in the world of neuroplasticity and how our brain works like this. So, I kept telling my students to keep it in Tudor mode. Let’s say you get the question wrong. We’re back to mitral stenosis, and for whatever reason, you forget the left lateral decubitus position is the best way to listen to the murmur. So what you do is you stop at that moment. You read up on it, you correct yourself in real-time, and then you go back to the question again. So again, it’s correcting yourself in real-time. This is huge. You have no idea how big this is. But it’s always in Tudor mode

11:05

Only Start Timing Yourself Towards The End

Only towards the end, and I’m talking about the last month before you take the exam, you start timing yourself.

Only Use High-Quality Questions

The other thing to do high-quality questions, one more time, high-quality questions. One of the biggest mistakes students will make they’ll tell you like, yeah, I’m doing 150 questions today. Wrong. What you’re doing is you’re probably going through these questions so fast, you’re probably developing a lot of bad habits.

Do 20-25 Questions Max Per Day

So I tell students to do is 20-25 a day, Max, no, Max, but I want you to do them in a high-quality fashion. I want you to pick apart every single word on that question. I want you to go beyond the question. So they’re asking the question, you know, aortic stenosis. Like I got this. I know the left ventricular hypertrophy, I know It’s a split S2 to I want you to go past the answer. In other words, you got the answer. And you went through it nice and slow. You peeled it apart. It was ambiguous, and you chose 51 verses 49. Practice these good habits because come the day the test, these good habits will rear up and they will help you. Because one of the things I see all the time with my high-risk students, the ones that fail, it’s always the same story. They’re always telling me that they are doing anywhere between the one to 200 questions a day, and I’m like I need you to do 25 high-quality questions a day. In other words, practice good test habits. So when you get to the day of the test, this will help you out the most.

12:42

The Best Textbooks for PANCE Prep

So now, our next topic we’re going to talk about is what are the best textbooks to get ready for the PANCE. About five years ago, I went out and actually bought all the PANCE and PANRE review booklets. I’m a big fan. If I’m going to talk the talk, I’m going to walk the walk, and what I did is it went out to and took a look at all these booklets that you guys are being introduced to. And what I did is I came up with the list of the booklets that I thought was best. Let me frame this statement quite clearly. There is no great book one more time; there is no great book; you will be piecemealing this. They’re all good. Nobody’s great. Each one is good, but each one has its flaws. So there are a lot of good ones out there, but nobody’s Great. So what were the better booklets that I saw out there when I was kind of doing my little research on how to get ready for the PANCE? First, one I came by is the AAPA book. That’s a Comprehensive Review for the Certification and Recertification Examinations for Physician Assistants that you see. It’s good. It’s not great. Some of the information is pretty good, but as you read through it, it’s really hard to find the nugget. Go ahead, look up meningitis, and osteomyelitis. You’ll be spending minutes trying to find out what your most common cause, which IV anabolic do I use? Things like that. So it’s good. It’s not great. Then there’s the Physician Assistant Board Review: Certification and Recertification by James Van Rhee, which is also a good book, but it’s not great. The Davis PA Exam Review book is good. There’s a Kaplan medical PANCE review book that’s good. Lange has gotten really good. So Lange, if you go back in time, 10 years ago, Lange was definitely not the best book that out there. But they got new editors, and they’ve done much, much better and I can only tell you the book is very good and this is the one that’s the Lange Q&A Physician Assistant Examination, Seventh Edition by Rachel Carlson and Albert Simon. It’s gotten very, very good. So highly recommend that one as well.

Read my post: 

The 4 Best PANCE and PANRE Study Guides and Review Books

15:16

PANCE PREP PEARLS

PANCE PREP PEARLS V3A - BEST PA SCHOOL REVIEW BOOKSSo let’s stop right here right now. I can guarantee you, if you’re listening to this podcast, you have this booklet. Make no mistake. It’s a great book, but it’s a reference book, not a study book. One more time, PANCE Prep Pearls. Wonderful reference book, not a good study book. If you don’t believe me, I know you already did this. What did you do? And I’m not begrudging anybody out there in the podcast world, but as students would you do? Okay, so let’s just say you had a GI test on a Monday, and you had a pulmonary test on Wednesday, and you had an ENT test on Friday. What do you do? You pulled out your PANCE Prep Pearls. You looked at the subjects or the objectives you had to do and you just brutally memorize the stuff, did you not you brutally memorize all the stuff at the pulmonary exam, the ENT exam, so you really memorized it off the reference book. Then what happened? I don’t know. By Friday afternoon, I could walk up to you and say, Hey, what is the first-line treatment of choice for pneumonia? Like I have no idea. I don’t even know where I parked. So in other words, you memorized it. It’s in it’s out. So again, it’s a great reference book, but not a good study book. If I had to paint, the student who is going to fail the boards. How would I paint on this one? I know I have your attention now. So who are my high-risk students? Oh, let me tell you what they all have in common. One. They live off a PANCE Prep Pearls. No, that’s the only book they have. They live off a Rosh review. That’s the only database and questions they use, period. And then what ends up happening to them because they had all the stuff memorized. They get a concept question. Some type of concept question shows up, and oh my god, they’re flat-footed. They have no idea which way to go. Because you see if it doesn’t show up as a trigger word of something they memorize and PANCE Prep Pearls. They’re done. And we’ll get to talk about Rosh in a second here about why I think Rosh is good, but again, it’s not great. So again, PANCE Prep Pearls is a wonderful, wonderful reference book. I think it’s great. It’s wonderful. I think Dwayne Williams did a wonderful job on this. But to use it specifically to study for the PANCE. Oh no, not at all. You’re going to get yourself lost.

Cost: $34.70 on Amazon – Version 3 is now divided into two books: part A / part B

17:00

The Best Websites for PANCE Review (PA Easy, Kaplan, Rosh)

When it comes to the website question. There are some database questions out there that are pretty good, and again, I went out there bought them all and there are some good ones some of them can get pretty pricey there’s no doubt about they can get pretty expensive.

PA Easy

So the first one I came across was PA Easy. I like PA Easy; it is quite good on the databank questions with the Lange references was really good, and again, you can put it in tutor mode. And again, I really enjoyed that a lot.

www.paexamprep.com

Kaplan

Kaplan is good, alright, so let’s stop right here. I love Kaplan Don’t get me wrong, but Kaplan, you know, I will say with Kaplan, take it with a grain of salt. Do you ever see some of those questions on Kaplan? They are hard so if you score above 60% or Kaplan, you’re doing great. And I know as students that suddenly you take some you know, cardiovascular exam, you score 62% and you swear that you’re going to you know, not pass the PANCE. That’s not true. Kaplan is a very, very difficult exam bank, but again, just take it with a grain of salt and just understand that some of these questions can be very tough.

www.kaptest.com

Rosh Review

If you’re listening to this podcast, I’d say about 80% of you guys use Rosh Review. Rosh is good, but it’s not great. Let me explain to you why. Who out there in podcast world took a question on Rosh Review, you swore you got it, right? You’re like I know this, this is what I learned in school. And then you went to Rosh Review, and they said no. And then if you look to the right, there is this thing called peer comparison, and you’re gonna find out that wait a second 70% that people got the question wrong. So as a teacher, just let you know that peer comparison over there. So when you do peer comparison, one of the things you should do if you’re a good teacher, your test question should sit around 70/30. This is really what’s called the discretion of a question in the education world. In other words, if a question is presented correctly, 70% of your class should get it right, 30% should get it wrong. That’s pretty much the ballpark. That’s the sweet spot anything hire is obviously good, but anything lower, ends up being a context problem. So when you get below 70%, getting it right. This is context. This isn’t a knowledge base issue with the students. This is a context of the question and otherwise, the question is worded poorly or incorrectly? Who on Rosh, if you have seen peer comparisons below 70/30? Yeah, you’re all shaking your head. Aren’t you going? Yeah, I’ve seen it all the time, Joe. Exactly. That’s my whole point. The other problem with Rosh, some of those questions aren’t even on the blueprint. That’s right. Look it up. You’ll see some of these questions going, who’s this disease? And it’s not even on your blueprint? So again, it’s like, I always tell people stick to the blueprint. This is what you’re going to be held accountable for. You’re not going to be held accountable for some, you know, chemo drug from breast cancer and some crazy side effects. But knowing the breast guidelines and what kind of cell type breast cancer is. Yes, that will be there. But not this other craziness that sometimes you’ll see on Rosh. So again, Rosh, it’s good. It’s not great. Of course, I’m going to push out my personal bias, Smarty PANCE. I think it’s a great website.

www.roshreview.com

20:03

Smarty PANCE

It’s just a treasure trove of information that takes a while to get through, but it’s a treasure trove of information. Wonderful website. And it follows the blueprint. So what I tell people to do is open up your smarty PANCE. There’s your blueprint, right, so let’s just say we’re back to aortic stenosis or mitral stenosis. So there it is, on the smarty PANCE blueprint there it is, you’re like, Okay, I gotta know, mitral stenosis. You open it up there’s an opening snap. Maybe you use another reference to study, and maybe you use the Merck manual? Maybe you’re using Lang, but in other words, you’re using other references to pull this all together? I keep trying to tell you guys; this is piecemeal, nobody’s great. Everybody’s good. So now, with all the information that you have, what would I recommend? Oh, this is what I would tell people to do.

www.smartypance.com

Putting it all together

20:50

One, start studying a year before, keep the stress out of your life. What I’d start doing is start studying. Yes, I would buy smarty PANCE ASAP use the blueprint; there’s my blueprint. This is the information I will be held accountable for, then I’m I looked at other references about maybe a or cynicism or look at PANCE pro tips. Maybe here’s some stuff that I forgot that and maybe I’ll read up on Merck manual or maybe I’ll read up on Lang and I’m going to do my questions how tutor mode? Because what do I need to correct myself in real-time? And that’s exactly what you need to do. So again, tutor mode, correct myself from real-time have all these references, my cell phone is off. I got no stress in my life. And I’m starting to go at it. That’s how you do it guys. That’s the secret sauce. That’s how it goes about. You see, where people go wrong, is they start not studying till the very end, they start memorizing things. They start doing things in test mode, not tutor mode. They’re doing 100 questions a day, not high-quality questions. But if we could just do high-quality questions, start them out a year before, it’s in tutor mode. I got smarty PANCE up and on one side, I’m looking at my blueprint. I got her other resources. Maybe I’m going to spend the afternoon on aortic stenosis. That’s all I’m doing. Sounds good to me. And then the next day, maybe another murmur and start trying to make sense of it all. Not memorize, because you see guys, just as my last little parting comment to you see, guys, the reason the board scores have dropped, and they have nationwide, and for some of these schools, they have dropped like a rock.

22:40

What happened? Well, what they did is a couple of things. One, they took away the buzzwords. So all these little buzzwords that you were used to writing, you know, you know, frothy green discharge, you know, purulent those kinds of things, right? They’re all gone. And now what they did is they made some of these questions, kind of concept questions. Do you know the concept of this issue and that is what I’m really trying to get at and see if you see something and you’ve got a big broad look at it? Now you understand the concept. You didn’t memorize it off PANCE Prep Pearls. You kept to the blueprint. You didn’t go way off base on some crazy Rosh Review that had nothing to do with the blueprint. And you stuck to it. You stick to that and I guarantee you you’re going to do great guys. So in the end of the day, high-quality questions, tutor mode, stick to the blueprint. Keep the stress out of my life. You do that guys, nine times out of 10 you’re going to end up in a very very good area guys. You really will.

So I hope this helps you guys out there in the podcast world. It was really great talking to everybody and I will be talking to you guys real soon. Take care.

Resources and Links From The Show

This Podcast is also available on iOS and Android

  1. iTunes: The Audio PANCE and PANRE Podcast iTunes
  2. Stitcher Radio: The Audio PANCE and PANRE Podcast Stitcher
  3. Google Play: The Audio PANCE and PANRE Podcast Google Play

itunes_logo-1

Download The Content Blueprint Checklist

Follow this link to download your FREE copy of the Content Blueprint Checklist

Print it up and start crossing out the topics you understand, marking the ones you don’t and making notes of key terms you should remember. The PDF version is interactive and linked directly to the individual lessons on SMARTY PANCE.

2020 Interactive NCCPA PANCE Content Blueprint

 

Download for PANCE Download for PANRE

The post How to Study for Your PANCE: Podcast Episode 79 appeared first on The Audio PANCE and PANRE.

]]>
Podcast Episode 79: How to Study for Your PANCE Welcome to episode 79 of the Audio PANCE and PANRE PA Board Review Podcast. In this episode 35-year PA veteran and PANCE/PANRE board review expert Joe Gilboy PA-C shares his top tips on how to best prepar... In this episode 35-year PA veteran and PANCE/PANRE board review expert Joe Gilboy PA-C shares his top tips on how to best prepare for the Physician Assistant National Certification Exam The Physician Assistant Life | Smarty PANCE full clean 40:54 367
Podcast Episode 77: The Audio PANCE and PANRE Board Review Podcast http://podcast.thepalife.com/podcast-episode-77/ Tue, 10 Dec 2019 17:21:04 +0000 http://podcast.thepalife.com/?p=355 http://podcast.thepalife.com/podcast-episode-77/#respond http://podcast.thepalife.com/podcast-episode-77/feed/ 0 <p>Welcome to episode 77 of the Audio PANCE and PANRE PA Board Review Podcast. Join me as I cover ten PANCE and PANRE Board review questions from the Smarty PANCE course content following the NCCPA™ content blueprint (download the FREE cheat sheet). Special from today’s episode: Download your copy of the Free Trello Smarty PANCE […]</p> <p>The post <a rel="nofollow" href="http://podcast.thepalife.com/podcast-episode-77/">Podcast Episode 77: The Audio PANCE and PANRE Board Review Podcast</a> appeared first on <a rel="nofollow" href="http://podcast.thepalife.com">The Audio PANCE and PANRE</a>.</p>
Episode 77 The Audio PANCE and PANRE PA Board Review Podcast By The Physician Assistant Life

Welcome to episode 77 of the Audio PANCE and PANRE PA Board Review Podcast.

Join me as I cover ten PANCE and PANRE Board review questions from the Smarty PANCE course content following the NCCPA™ content blueprint (download the FREE cheat sheet).

Special from today’s episode:

This week we will be covering ten general board review questions based on the NCCPA PANCE and PANRE Content Blueprints. 

Below you will find an interactive exam to complement the podcast.

The Audio PANCE and PANRE Physician Assistant Board Review Podcast

I hope you enjoy this free audio component to the examination portion of this site. The full board review includes over 2,000 interactive board review questions and is available to all members of the PANCE and PANRE Academy and Smarty PANCE.

Listen Carefully Then Take The Practice Exam

If you can’t see the audio player click here to listen to the full episode.

Podcast Episode 77: Ten Question PANCE and PANRE Podcast Quiz

The following questions are linked to NCCPA Content Blueprint lessons from the Smarty PANCE and PANRE Board Review Website. If you are a member you will be able to log in and view this interactive video lesson.

1.  A 10-month-old girl was admitted to the hospital for cardiac catheterization. Her history included cyanosis noted at about 6 weeks of age, increasing over the last 7 months and becoming more severe with crying or physical activity. The chest x-ray demonstrates a “boot-shaped heart.” A presumptive diagnosis of tetralogy of Fallot (TOF) was made on admission. TOF has 4 components, which of the following below is NOT PART of the diagnosis?

A. Pulmonary valve stenosis
B. VSD
C. Overriding aorta
D. Right ventricular hypertrophy
E. ASD

Click here to see the answer

Answer: E, ASD

In 1888, Fallot described a congenital heart defect composed of four characteristics (a) large ventricular septal defect (VSD) (b) right ventricular outflow obstruction (pulmonary valve stenosis) (c) overriding aorta (d) right ventricular hypertrophy. The main characteristic of TOF is cyanosis. Hypercyanotic spells or “tet spells” are paroxysmal episodes in which the cyanosis acutely worsens. Crying, feeding, or defecating can bring on these episodes.

Review NCCPA Blueprint Topic: Tetralogy of Fallot

2. A 65-year-old man presents with pain in his right knee. He says he fell and “banged it up fairly bad” approximately 6 months ago but that it had since recovered spontaneously and provided no further trouble until now. On examination, his temperature is 37.5 °C and his blood pressure is 125/70 mm Hg. He has an inflamed, tender, swollen right knee. No other joints are affected. No other abnormalities are found on physical examination. A plain radiographic examination of the right knee reveals streaking of the surrounding soft tissue with calcium deposits (chondrocalcinosis). What is the definitive diagnostic test of choice for this patient’s disease

A. A plasma level
B. A random urine test
C. A 24-hour urine
D. A synovial fluid analysis
E. Gram stain plus culture and sensitivity

Click here to see the answer

The answer is D: A synovial fluid analysis

A definitive diagnosis of gout is made by demonstrating negatively birefringent, needle-shaped monosodium urate crystals under a polarizing microscope. Although an elevated serum uric acid concentration is often seen in acute gout, it is neither as sensitive nor as specific a test as the demonstration of uric acid crystals in the synovial fluid under a microscope.

Serum uric acid levels can be normal in patients with acute gouty arthritis. The diagnosis of septic arthritis can be ruled out by appropriate Gram stain and culture of the same specimen of synovial fluid obtained for examination with the polarizing microscope.

Review NCCPA Blueprint Topic: Gout and pseudogout (Lecture)

3. What is (are) the major difference(s) between polymyalgia rheumatica (PMR) and polymyositis?

A. Marked proximal muscle weakness in polymyositis
B. Marked proximal muscle tenderness in polymyositis
C. Elevated muscle enzymes such as creatine kinase (CK) in polymyositis
D. a, b, and c

Click here to see the answer

The answer is D: a, b, and c

The differences between Polymyalgia Rheumatica (PMR) and polymyositis on clinical examination are as follows:

  • There is marked weakness associated with proximal muscle pain in polymyositis
  • There is often marked muscle tenderness (versus joint pain in PMR) associated with the proximal muscle pain in polymyositis
  • Laboratory examination reveals elevated muscle enzymes only in polymyositis

Polymyositis Pearls

Polymyositis is an autoimmune-mediated inflammatory destruction of muscles leading to muscle weakness

  • Patients with polymyositis experience proximal symmetric (bilateral) muscle weakness
  • The shoulders and hips are the parts of the body most commonly affected by polymyositis
  • Early fatigue while walking and inability to rise from a seated position

Diagnosis

  • The three autoantibodies anti-Jo-1anti-SRP, and anti-Mi-2, are associated with polymyositis
  • Creatine kinase levels are increased in polymyositis
  • Electromyography can detect regions of dead muscle cells
  • Muscle biopsy can show endomysial inflammation and various stages of necrosis

Treatment

  • Initial treatment of polymyositis  involves suppressing the immune response with corticosteroids
  • Methotrexate is used for long-term immunosuppressive therapy in polymyositis

Review NCCPA Blueprint Topic: Polymyositis (ReelDx + Lecture)

4. Clinical features of botulism include all of the following except

A. Paresthesia
B. Dysphagia
C. Diplopia
D. Fixed/dilated pupils

Click here to see the answer

The answer is A: Paresthesia

Paresthesia is not a feature of botulism while dysphagia, diplopia, and fixed dilated pupils are.

Botulism Pearls

  • Caused by a neurotoxin elaborated by Clostridium botulinum a gram-positive bacillus, which is an anaerobic, spore-forming bacteria
  • Associated with home-canned food products and honey in infants (wait until babies are at least 12 months before introducing honey)
  • Presents with double visiondrooping of eyelids, inability to make facial expressions, and difficulty swallowing
  • Autonomic nervous system effects may cause dry mouth, postural hypotension, nausea, vomiting, and constipation
  • Can lead to complete flaccid paralysis which is deadly if it involves respiratory muscles
  • In infants, symptoms include constipation and generalized weakness, with weak crying, poor feeding, lethargy, and loss of head control (floppy baby syndrome)

Diagnosis

  • Diagnosis is by toxin assays
  • Sometimes electromyography

Treatment

  • Treatment is supportive
  • The greatest threat to life is respiratory impairment and its complications
  • IV botulinum immunoglobulin/heptavalent botulinum antitoxin
  • Correct canning and adequate heating of home-canned food before serving are essential
  • Canned foods showing evidence of spoilage and swollen or leaking cans should be discarded

Review NCCPA Blueprint Topic: Botulism

5. A 59-year-old male complains of “flashing lights behind my eye” followed by a sudden loss of vision, stating that it was “like a curtain across my eye.” He denies trauma. He takes Glucophage for his diabetes mellitus and atenolol for his hypertension. He has no other complaints. On the fundoscopic exam, the retina appears to be out of focus. Which of the following is the most likely diagnosis?

A. Central retinal vein occlusion
B. Retinal artery occlusion
C. Retinal detachment
D. Hyphema

Click here to see the answer

The answer is C: Retinal Detachment

Patients with retinal detachment frequently complain of flashes of light or floaters that occur during traction on the retina as it detaches. This is followed by loss of vision. In small detachments, the retina may appear out of focus, but with larger detachments, a retinal fold may be identified.

  • Central retinal vein and artery occlusion cause painless, variable loss of vision. Exam shows retinal hemorrhages in all quadrants and edema of the optic disk
  • Hyphema is usually associated with trauma and is a collection of blood in the anterior chamber

Retinal Detachment Pearls

  • Retinal detachment is a separation of the neurosensory retina from the underlying retinal pigment epithelium
  • Look for sudden increase or change in floaters along with curtain or veil across the visual field
  • Often is spontaneous, but may have an underlying cause – example recent cataract surgery
  • Myopia (nearsightedness) is a risk factor for the development of retinal detachment
  • Retinal detachment is usually unilateral
  • Retinal detachment usually presents with defects in the peripheral visual field

Diagnosis

  • Diagnosis is by fundoscopy – retinal detachment is visualized as crinkling of retinal tissue and changes in vessel direction
  • Ultrasonography may help determine the presence and type of retinal detachment if it cannot be seen with funduscopy

Treatment

  • Retinal detachment is an ophthalmologic emergency
  • Stay supine (lying face upward) with head turned towards the side of the detached retina
  • Pneumatic retinopexy is a procedure for the management of retinal detachment that involves cryoretinopexy followed by injection of an air bubble in the vitreous

Review NCCPA Blueprint Topic: Retinal detachment (Lecture)

6. The first dose of the combined vaccine of measles, mumps, and rubella (MMR) is usually given at age

A. 12 months
B. 6 weeks
C. Birth
D. 9 months

Click here to see the answer

The answer is A: The first dose of MMR is given at age 12 – 15 months and a second dose at age 4-6 years

Mumps Pearls

Mumps is a viral disease that is part of the paramyxovirus family. It presents with parotitis (painful parotid gland swelling), orchitis, or aseptic meningitis. It is transmitted through respiratory droplets and has an incubation period of 12-14 days

  • Prodrome of fevermalaise, and anorexia
  • Parotid enlargement (usually bilateral but not always synchronous) 24 h later
  • Swelling of submaxillary and submandibular glands
  • Orchitis (usually unilateral) with testicular enlargement two to three times normal size
  • Mumps is the most common cause of pancreatitis in children

Diagnosis

  • During an outbreak, a diagnosis can be made by determining recent exposure and parotitis. Usually, the disease is diagnosed on clinical grounds, and no confirmatory laboratory testing is needed
  • If there is uncertainty about the diagnosis, a test of saliva or blood may be carried out; a newer diagnostic confirmation, using real-time nested (PCR) technology, has also been developed
  • As with any inflammation of the salivary glands, the serum level of the enzyme amylase is often elevated
  • CSF demonstrates increased lymphocytes and decreased glucose 

Treatment

  • There is no available cure for mumps and treatment is supportive
  • Symptoms usually last for 7-10 days and patients are contagious for up to 9 days after onset
  • May need to provide scrotal support if painful or swollen testicle (as in case presentation)
  • MMR vaccine is given at 12-15 months then again at 4-6 years of age

Review NCCPA Blueprint Topic: Mumps

7. A 28-year old sub-fertile woman presents to you on account of dysmenorrhea, deep dyspareunia, dyschezia, and pelvic pain of a few months’ duration. Physical examination revealed nodularity of the uterosacral ligaments, tenderness in the pouch of Douglas, and a fixed retroverted uterus with positive cervical excitation tenderness. What is the most likely diagnosis?

A. Endometriosis
B. Pelvic inflammatory disease
C. Adenomyosis
D. Uterine leiomyoma

The diagnosis is generally made by

A. Detection of increased estrogen levels
B. Endometrial biopsy
C. Pelvic ultrasound
D. Laparoscopy
E. CT of the pelvis

Click here to see the answer

Answer: A, endometriosis, and D Laparoscopy

  • Endometriosis, which is the presence of benign endometrial tissue outside of the uterine cavity typically presents as described in this clinical vignette. Remember the “three d’s” – Dyspareunia, dyschezia (difficulty in defecating) and dysmenorrhea
  • Definitive diagnosis is made by laparoscopy and confirmed with a biopsy
  • Imaging tests (eg, ultrasonography, barium enema, IV urography, CT, MRI) are not specific or adequate for diagnosis. However, they sometimes show the extent of endometriosis and thus can be used to monitor the disorder once it is diagnosed.

Incorrect Answers:

  • Patients with PID may have similar symptoms with endometriosis, but also presents with fever and vaginal discharge
  • Patients with adenomyosis (a condition in which endometrial tissue exists within and grows into the uterine wall) present with uterine mass with or without pressure symptoms and menorrhagia
  • Uterine leiomyomas cause chronic painful bleeding and are common in women in their late thirties and early forties.

Review NCCPA Blueprint Topic: Endometriosis (Lecture)

8. A 45-year-old obese Caucasian gentleman arrives at your clinic for a routine check-up after having some blood work done during a workplace health screening. He is found to have an LDL cholesterol level of 550 mg/dL. He states that his father and brother had high cholesterol and both died at a young age from a heart attack. He has a follow-up appointment with his cardiologist because of some occasional chest pain and abnormalities seen on his EKG. Additionally, you notice that he has well-demarcated yellow deposits around his eyes. He is started on high dose statin and his LDL at 12 weeks is 350 mg/dL. What is the next best step in this patient’s management?

A. Continue high dose statin, the patient’s LDL is at goal
B. Add niacin 100 mg three times daily
C. Add ezetimibe (Zetia) 10 mg daily
D. Add a PCSK9 inhibitor
E. Refer to a lipid specialist

Click here to see the answer

The answer is C: add ezetimibe 10 mg

If LDL-C is not at goal after 6-12 weeks the next best step for the treatment of familial hypercholesterolemia is to add ezetimibe 10 mg daily and check again in 6-12 weeks. If at that time the patient’s LDL is still not at goal (ideally < 150) refer to lipid specialist to consider adding a PCSK9 inhibitor.

Pearls

  • Familial hypercholesterolemia (FH) is the most common autosomal dominant genetic disease. The clinical syndrome (phenotype) is characterized by extremely elevated levels of low-density lipoprotein cholesterol (LDL-C) and a propensity to early-onset atherosclerotic cardiovascular disease. In general, homozygotes manifest the disease at a much earlier age than heterozygotes and the disease is more severe.
  • Homozygous FH patients are rare and have an estimated prevalence of approximately 1:300,000 to 1:400,000
  • Heterozygous FH is estimated to occur in 1 in 200 to 250 individuals in the United States.
  • It is estimated that about 7 percent of American adults have an untreated lipoprotein cholesterol ≥190 mg/dL but only 1.7 percent carry an FH mutation
  • Patients with undiagnosed homozygous familial hypercholesterolemia (FH) develop severe, premature, atherosclerotic cardiovascular disease and die before age 20 in many cases.
  • In patients with a negative or unknown family history, an untreated LDL-C level of ≥190 mg/dL (4.9 mmol/L) suggests FH. This value is greater than the 90th percentile for age and sex.

Diagnosis

  • The diagnosis of heterozygous familial hypercholesterolemia (FH) is made with genetic testing or clinical criteria. A causative mutation in the LDLR, APOB, or PCSK9 gene(s) secures this diagnosis
  • When genetic testing is not available or not felt to be necessary, you can use the Dutch Lipid Clinic Network criteria, which assigns points based on low-density lipoprotein cholesterol (LDL-C) levels, personal history of early atherosclerotic cardiovascular disease (ASCVD), family history of early ASCVD, or high cholesterol in a first-degree relative, and personal and physical examination finding

Treatment

  • Patients with homozygous familial hypercholesterolemia (FH) – intensive LDL-C lowering, which targets a minimal value of <150 mg/dL (3.9 mmol/L)
  • In addition to a high-dose statin (atorvastatin 80 mg daily or rosuvastatin 40 mg daily), most homozygous patients will require additional therapies such as ezetimibe, a PCSK9 inhibitor, or potentially LDL-C apheresis

Review NCCPA Blueprint Topic: Hypercholesterolemia

9. What best describes the time that preeclampsia is commonly seen?

A. Before 18 weeks of pregnancy
B. After 18 weeks of pregnancy
C. After 16 weeks of pregnancy
D. After 20 weeks of pregnancy
E. Before 12 weeks of pregnancy

Click here to see the answer

The answer is D, after 20 weeks of pregnancy

Preeclampsia is a systemic disease characterized by hypertension that is accompanied by proteinuria. Preeclampsia usually begins after the 20th week of gestation; however, it can appear at any time during pregnancy. It occurs most frequently in the final trimester.

Pearls

  • Preeclampsia is a systemic disease characterized by hypertension that is accompanied by proteinuria after the 20th week of gestation.
  • If left untreated, preeclampsia can lead to serious, and even fatal, complications.
  • Risk factors include nulliparity, age younger than 19 or older than 35, obesity, multiple gestations, positive family history, pre-existing hypertension or renal disease, and diabetes mellitus.
  • Eclampsia is defined as the development of seizures in a woman with preeclampsia.

Diagnosis

  • Hypertension with proteinuria
  • Mild Preeclampsia
    • BP 140/90 – 160/110
    • Proteinuria – > 300 mg/24 hours or > +1 on dipstick
  • BP > 160/110
    • Proteinuria > 5g in 24 hours or no urine or 3 +on dipstick
    • ***HELLP SYNDROME – Hemolysis, elevated liver enzymes, and low platelets

Treatment

  • Delivery is the only cure for preeclampsia. The decision to induce depends on the stage of pregnancy and the severity of the disease
  • Patients with preeclampsia without severe symptoms are generally induced into labor after 37 weeks gestation in severe preeclampsia delivery is performed at 24-26 weeks
  • If less than 34 weeks antenatal steroids promote fetal lung development
  • Intravenous magnesium sulfate as seizure prophylaxis)

Review NCCPA Blueprint Topic: PANCE Blueprint Reproductive System (7%)Hypertension disorders in pregnancy

10. A 36-year-old male who is hospitalized because of severe injuries from a motor vehicle accident develops a rapid onset of profound dyspnea. The initial chest x-ray shows a normal heart size with diffuse bilateral infiltrates. A follow-up chest x-ray shows confluent bilateral infiltrates that spare the costophrenic angles. Which of the following is the best clinical intervention for this patient?

A. Tracheal intubation
B. Bilateral chest tube insertion
C. Type-specific packed cells
D. Colloid solutions
E. Provide supplemental oxygen

Click here to see the answer

The answer is A – Tracheal intubation

Tracheal intubation with the lowest level of PEEP is required to maintain the PaO2 above 60 mmHg or SaO2 above 90% in a patient with ARDS

Pearls

Acute respiratory distress syndrome (ARDS) is a type of respiratory failure characterized by fluid collecting in the lungs depriving organs of oxygen

  • The underlying abnormality in ARDS is ⇑ Permeability of alveolar-capillary membranes ⇒ development of protein-rich pulmonary edema (non-cardiogenic pulmonary edema)
  • ARDS can occur in those who are critically ill or who have significant injuries

Three clinical settings account for 75% of ARDS cases:

  • Sepsis syndrome (most common cause)
  • Severe multiple trauma
  • Aspiration of gastric contents (alcoholics), toxic inhalation, near-drowning

People with ARDS have severe shortness of breath and often are unable to breathe on their own without support from a ventilator

  • Occurring 12-24 hours after the precipitating event
  • Tachypnea, pink frothy sputum, crackles

Diagnosis

Chest radiograph shows air bronchograms and bilaterally fluffy infiltrate

  • Normal BNP, pulmonary wedge pressure, left ventricle function and echocardiogram

Treatment

  • Treatment involves identifying and managing underlying precipitation and secondary conditions
  • Tracheal intubation with the lowest level PEEP to maintain PaO2 >60 mmHg or SaO2 >90
  • ARDS is often fatal, the risk increases with age and severity of illness

Review NCCPA Blueprint Topic: PANCE Blueprint Pulmonary (10%) ⇒ Acute respiratory distress syndrome (Lecture)

[spoiler title=”C

Looking for all the podcast episodes?

This FREE series is limited to every other episode, you can download and enjoy the complete audio series by joining The PANCE and PANRE Exam Academy + SMARTYPANCE

I will be releasing new episodes every few weeks. The Academy is discounted, so sign up now.

Resources and Links From The Show

This Podcast is also available on iOS and Android

  1. iTunes: The Audio PANCE and PANRE Podcast iTunes
  2. Stitcher Radio: The Audio PANCE and PANRE Podcast Stitcher
  3. Google Play: The Audio PANCE and PANRE Podcast Google Play
itunes_logo-1

Download The Content Blueprint Checklist

Follow this link to download your FREE copy of the Content Blueprint Checklist

Print it up and start crossing out the topics you understand, marking the ones you don’t and making notes of key terms you should remember. The PDF version is interactive and linked directly to the individual lessons on SMARTY PANCE.

Download for PANCE Download for PANRE

The post Podcast Episode 77: The Audio PANCE and PANRE Board Review Podcast appeared first on The Audio PANCE and PANRE.

]]>
Welcome to episode 77 of the Audio PANCE and PANRE PA Board Review Podcast. Join me as I cover ten PANCE and PANRE Board review questions from the Smarty PANCE course content following the NCCPA™ content blueprint (download the FREE cheat sheet).



Welcome to episode 77 of the Audio PANCE and PANRE PA Board Review Podcast.



Join me as I cover ten PANCE and PANRE Board review questions from the Smarty PANCE course content following the NCCPA™ content blueprint (download the FREE cheat sheet).



Special from today’s episode:



* Download your copy of the Free Trello Smarty PANCE NCCPA™ Blueprint Study Plan* Follow The Daily PANCE Blueprint on Instagram* Follow The Daily PANCE Blueprint on Facebook



This week we will be covering ten general board review questions based on the NCCPA PANCE and PANRE Content Blueprints. 



Below you will find an interactive exam to complement the podcast.



The Audio PANCE and PANRE Physician Assistant Board Review Podcast



I hope you enjoy this free audio component to the examination portion of this site. The full board review includes over 2,000 interactive board review questions and is available to all members of the PANCE and PANRE Academy and Smarty PANCE.



* You can download and listen to past FREE episodes here, on iTunes, on Google Play Music or Stitcher Radio.* You can listen to the latest episode, take an interactive quiz and download your results below.



Listen Carefully Then Take The Practice Exam





If you can’t see the audio player click here to listen to the full episode.



Podcast Episode 77: Ten Question PANCE and PANRE Podcast Quiz



The following questions are linked to NCCPA Content Blueprint lessons from the Smarty PANCE and PANRE Board Review Website. If you are a member you will be able to log in and view this interactive video lesson.



1.  A 10-month-old girl was admitted to the hospital for cardiac catheterization. Her history included cyanosis noted at about 6 weeks of age...]]>
The Physician Assistant Life | Smarty PANCE full clean 30:27 355
Podcast Episode 75: Ten FREE PANCE and PANRE Audio Board Review Questions http://podcast.thepalife.com/podcast-episode-75-ten-free-pance-and-panre-audio-board-review-questions/ Thu, 03 Oct 2019 07:01:44 +0000 http://podcast.thepalife.com/?p=350 http://podcast.thepalife.com/podcast-episode-75-ten-free-pance-and-panre-audio-board-review-questions/#respond http://podcast.thepalife.com/podcast-episode-75-ten-free-pance-and-panre-audio-board-review-questions/feed/ 0 <p>Welcome to episode 75 of the Audio PANCE and PANRE PA Board Review Podcast. Join me as I cover ten PANCE and PANRE Board review questions from the SMARTYPANCE course content following the NCCPA™ content blueprint (download the FREE cheat sheet). This week we will be covering ten general board review questions based on the NCCPA PANCE and […]</p> <p>The post <a rel="nofollow" href="http://podcast.thepalife.com/podcast-episode-75-ten-free-pance-and-panre-audio-board-review-questions/">Podcast Episode 75: Ten FREE PANCE and PANRE Audio Board Review Questions</a> appeared first on <a rel="nofollow" href="http://podcast.thepalife.com">The Audio PANCE and PANRE</a>.</p>
Episode 75 The Audio PANCE and PANRE PA Board Review Podcast

Welcome to episode 75 of the Audio PANCE and PANRE PA Board Review Podcast.

Join me as I cover ten PANCE and PANRE Board review questions from the SMARTYPANCE course content following the NCCPA™ content blueprint (download the FREE cheat sheet).

This week we will be covering ten general board review questions based on the NCCPA PANCE and PANRE Content Blueprints. 

Below you will find an interactive exam to complement the podcast.

The Audio PANCE and PANRE Physician Assistant Board Review Podcast

I hope you enjoy this free audio component to the examination portion of this site. The full board review includes over 2,000 interactive board review questions and is available to all members of the PANCE and PANRE Academy and Smarty PANCE.

Listen Carefully Then Take The Practice Exam

If you can’t see the audio player click here to listen to the full episode.

Podcast Episode 75: Ten Question PANCE and PANRE Podcast Quiz

The following questions are linked to NCCPA Content Blueprint lessons from the Smarty PANCE and PANRE Board Review Website. If you are a member you will be able to log in and view this interactive video lesson.

1. A 5-year-old girl is brought to the emergency department after drinking a bottle of drain cleaner. It is unknown how much the child drank. She has a past medical history of Down syndrome and obesity. The patient’s vitals are unremarkable. Physical exam is notable for a child in no acute distress. She is tolerating her oral secretions and is interactive. Inspection of the oropharynx is unremarkable. Which of the following is the appropriate management of this patient?

  1. Dilute hydrochloric acid
  2. Endoscopy
  3. Intubation
  4. Observation
  5. Polyethylene glycol
Click here to see the answer

Answer: B. Endoscopy 

Ingestion of caustic fluids (acid or alkali) such as drain cleaner may lead to esophageal damage and stricture. Any patient presenting after caustic ingestion should have an endoscopy performed between 12 and 24 hours of presentation. Injury due to ingestion of alkaline fluids such as drain/oven cleaner occurs rapidly in the first minutes to hours and is characterized by liquefactive necrosis of the esophageal tissue. Subsequently, esophageal strictures form due to scarring of the affected tissue. Patients should immediately be resuscitated following the ABC’s (airway, breathing, and circulation). Subsequent management involves endoscopy (typically within 12 to 24 hours of ingestion) to assess the extent of the damage and plan any further treatment that may be needed.

Incorrect Answers:

  • Answer 1: Dilute hydrochloric acid or administration of any agent to titrate the ingestion is always contraindicated as this will lead to more tissue damage.
  • Answer 3: Intubation would be indicated if the patient was failing to protect their airway or if they had impending airway loss. This well-appearing patient has no airway concerns.
  • Answer 4: Observation is certainly a part of this patient’s care; however, she must also receive endoscopy to assess for any esophageal/GI tract injury.
  • Answer 5: Polyethylene glycol or whole bowel irrigation has limited indications. It may be used to pass bags of drugs if ingested by a packer/stuffer but would not aid in the management of caustic ingestion.

Review NCCPA Blueprint Topic: Ingestion of toxic substances or foreign bodies

2. A 27-year-old man presents to the emergency department after a motor vehicle collision. The patient was a front seat unrestrained driver in a head-on collision. The patient’s echocardiogram (ECG) is notable only for sinus tachycardia. His temperature is 99.5°F (37.5°C), blood pressure is 107/58 mmHg, pulse is 120/min, respirations are 17/min, and oxygen saturation is 98% on room air. The patient is given 2 liters of Ringer lactate solution and morphine. Initial workup demonstrates that the patient’s pulmonary capillary wedge pressure and troponins are elevated. The patient is currently complaining of chest pain. Physical exam is notable for an uncomfortable young man with bruising over his chest wall. Which of the following is the most likely diagnosis?

  1. Cardiac contusion
  2. Hemorrhage
  3. Myocardial infarction
  4. Pulmonary contusion
  5. Takotsubo cardiomyopathy
Click here to see the answer

Answer: A. Cardiac contusion

This patient is presenting after blunt chest trauma (which is common in motor vehicle accidents) with chest pain, elevated troponins, and an elevated pulmonary capillary wedge pressure suggesting a diagnosis of a cardiac contusion. A cardiac contusion is a blunt injury to the heart which can disrupt the mechanical and electrical function of the heart. There will typically be visible signs of chest trauma such as bruising and the patient will often complain of chest pain and dyspnea. The ECG can be unremarkable, demonstrate sinus tachycardia, or even demonstrate more severe dysrhythmias such as supraventricular tachycardia, atrial fibrillation, and a right bundle branch block. Initial cardiac troponins can be elevated. In the setting of any ECG abnormality or elevated troponins, patients with a suspected diagnosis of a cardiac contusion should be admitted to the hospital and observed until clinically stable with normalization of their troponins.

Incorrect Answers:

  • Answer 2: Hemorrhage would present with hypotension, tachycardia, and a decreased pulmonary capillary wedge pressure. Internal bleeding could be assessed with a FAST exam which assesses for pleural sliding, pericardial fluid and cardiac function, fluid in the hepatorenal and splenorenal recesses, and fluid surrounding the bladder.
  • Answer 3: Myocardial infarction presents with chest pain, shortness of breath, ST elevation on ECG and elevated cardiac troponins. This patient’s elevated troponins are likely secondary to blunt trauma to the heart as he has no risk factors for ischemic heart disease and is young.
  • Answer 4: Pulmonary contusion presents with chest pain, hypoxia, and patchy opacities on chest radiography which may not be initially apparent. Management for a pulmonary contusion is typically supportive in nature and to adequately control the patient’s pain.
  • Answer 5: Takotsubo cardiomyopathy presents with chest pain and dyspnea as well as ST elevation on ECG without reciprocal changes. Global hypokinesis of the heart can be seen on echocardiography and minor elevations in troponins can be present. The typical precipitating event for Takotsubo cardiomyopathy is an emotionally stressful life event and the patient is typically a woman in contrast to this male patient who has a mechanism of injury that supports a diagnosis of a cardiac contusion.

Review NCCPA Blueprint Topic: Chest/Rib Fractures and Trauma

3. A 33-year-old woman presents to her primary care PA for a wellness check-up. She states that recently she has been feeling well other than headaches that occur occasionally, which improve with ibuprofen and rest. She has a past medical history of hypertension and headaches and is currently taking hydrochlorothiazide. Her temperature is 99.2°F (37.3°C), blood pressure is 157/108 mmHg, pulse is 90/min, respirations are 14/min, and oxygen saturation is 98% on room air. Physical exam reveals a young woman who appears healthy. A normal S1 and S2 are auscultated on cardiac exam, and her lungs are clear with good air movement bilaterally. From her previous visit, it was determined that she has an elevated aldosterone and low renin level. Laboratory values are ordered as seen below.

Serum:
Na+: 139 mEq/L
Cl-: 100 mEq/L
K+: 3.7 mEq/L
HCO3-: 29 mEq/L
BUN: 20 mg/dL
Creatinine: 1.1 mg/dL

Which of the following is the most likely diagnosis?

  1. Benign essential hypertension
  2. Cushing syndrome
  3. Narrowing of the renal arteries
  4. Pheochromocytoma
  5. Primary hyperaldosteronism
Click here to see the answer

Answer: E. Primary hyperaldosteronism

This patient is presenting with hypertension refractory to a diuretic with a decreased potassium, a decreased renin level, and an increased aldosterone level suggesting a diagnosis of primary hyperaldosteronism.

Primary hyperaldosteronism occurs when the adrenal gland produces excess aldosterone. Aldosterone has the effect in the kidney of absorbing sodium (and thus water) and wasting potassium and hydrogen. Thus, hyperaldosteronism can lead to hypertension, hypokalemia, and a metabolic alkalosis. In primary hyperaldosteronism, the high blood pressure is detected by the kidney, and thus renin levels are decreased in the setting of an elevated aldosterone.

Incorrect Answers:

  • Answer 1: Benign essential hypertension typically occurs in overweight patients and has no clear underlying cause. It would not be associated with elevated aldosterone levels.
  • Answer 2: Cushing syndrome would be associated with weight gain, limb muscle atrophy, and striae on dermatological exam in the setting of a decreased renin and aldosterone.
  • Answer 3: Narrowing of the renal arteries (renal artery stenosis) would present with refractory hypertension in the setting of an elevated renin and aldosterone level.
  • Answer 4: Pheochromocytoma would present with severe episodic hypertension and headaches secondary to an adrenal mass that releases catecholamines.

Summary: Primary hyperaldosteronism presents with hypertension, a decreased renin and potassium level, a metabolic alkalosis, and an increased aldosterone level.

Review NCCPA Blueprint Topic: Secondary hypertension

4. A 24-year-old man is brought in to the emergency room after being retrieved by firefighters from a burning building. The patient is responding coherently to questions but reports pain secondary to a burn on his leg. He states he also has a headache and feels dizzy. His temperature is 98.5°F (36.9°C), blood pressure is 129/66 mmHg, pulse is 126/min, respirations are 14/min, and oxygen saturation is 99% on room air. Physical exam is notable for a confused young man with dry and flushed skin. The cardiopulmonary exam reveals a normal S1 and S2, as well as clear breath sounds bilaterally. The patient’s neurological exam is within normal limits. Towards the end of his exam, the patient begins vomiting. The dermatologic exam reveals a superficial burn covering 1% of the patient’s body over his right leg. Which of the following is the best next step in management for this patient?

  1. 100% oxygen
  2. CT scan of the head
  3. Hydroxocobalamin
  4. Normal saline
  5. Ondansetron
Click here to see the answer

Answer: A. 100% oxygen

This patient is presenting after being rescued from a fire with confusion, headache, nausea, vomiting, and flushed skin suggesting a diagnosis of carbon monoxide poisoning which should be treated with 100% oxygen.

Carbon monoxide exposure is common in fires and in patients who heat their house with an old-fashioned wood stove. Carbon monoxide binds to hemoglobin with a higher affinity than oxygen thus displacing it leading to symptoms. Carbon monoxide poisoning presents with confusion, headache, altered mental status, nausea, vomiting, and cherry-red skin. Pulse oximetry is often normal in these patients as the device detects hemoglobin bound to oxygen or carbon monoxide similarly. A carboxyhemoglobin level can be obtained to confirm the diagnosis in these patients; however, patients presenting with a clinical picture supportive of carbon monoxide poisoning should be treated with 100% (or hyperbaric) oxygen. Sequelae of carbon monoxide poisoning should be treated supportively, including dantrolene for increased muscle activity or benzodiazepines for seizure activity.

Incorrect Answers:

  • Answer 2: CT scan of the head would be indicated if a patient presented with head trauma followed by altered mental status, nausea, and vomiting. This patient’s neurological abnormalities, as well as his nausea and vomiting, can be explained by his carbon monoxide poisoning.
  • Answer 3: Hydroxocobalamin is the treatment of choice for cyanide poisoning which is also associated with exposure to fires. Patients will present with weakness, malaise, headache, dizziness, nausea, vomiting, and shortness of breath. Though this diagnosis is possible in this patient, a more likely diagnosis both epidemiologically and given his flushed skin is carbon monoxide poisoning. This patient will need empiric treatment for cyanide poisoning, however, a more dire intervention for the most likely diagnosis is putting the patient on oxygen to simultaneously and rapidly improve his respiratory status and treat his carbon monoxide poisoning.
  • Answer 4: Normal saline may be necessary to stabilize this patient’s blood pressure; however, his tachycardia is likely secondary to his pain and a more important initial intervention is stabilizing the patient’s respiratory status including administering oxygen for this patient’s carbon monoxide poisoning.
  • Answer 5: Ondansetron only treats this patient’s symptoms of nausea but does not address their respiratory status which should be treated promptly and prior to treating other concerns.

Summary: The treatment of carbon monoxide poisoning is 100% oxygen.

Review NCCPA Blueprint Topic: Burns (ReelDx)

5. A 42-year-old woman is brought to the emergency department after a motor vehicle accident. She complains of lower back pain and some minor abdominal pain. The patient has a past medical history of obesity and type II diabetes. Her current medications include atorvastatin, metformin, and glyburide. A FAST exam is performed in the trauma bay and does not reveal any signs of intra-abdominal bleeding. Her temperature is 98.2°F (36.8°C), blood pressure is 130/77 mmHg, pulse is 90/min, respirations are 16/min, and oxygen saturation is 99% on room air. Ultrasound findings are notable for multiple gallstones in the gallbladder. The patient is given naproxen. Which of the following is the best next step in management?

  1. CT scan of the abdomen
  2. NPO, IV fluids, and broad-spectrum antibiotics
  3. Perform a cholecystectomy this hospital visit
  4. Schedule an outpatient cholecystectomy
  5. Supportive therapy
Click here to see the answer

Answer: E. Supportive therapy

This patient is presenting with asymptomatic gallstones discovered incidentally. Asymptomatic gallstones do not need to be managed with a cholecystectomy. 

Acute cholecystitis classically presents with right upper quadrant abdominal pain that presents in a fat, fertile, female in her forties. Once the diagnosis is confirmed with ultrasound, “cooling off” of the gallbladder is necessary (keeping the patient NPO and IV fluids) followed by a cholecystectomy that hospital visit. However, if asymptomatic gallstones are discovered incidentally, there is no indication for cholecystectomy. These patients should be managed conservatively.

Incorrect Answers:

  • Answer 1: CT scan of the abdomen is unnecessary as there is no indication to perform a CT scan, in particular when the only finding in this assessment was asymptomatic gallstones on ultrasound. 
  • Answer 2: NPO, IV fluids, and broad spectrum antibiotics is the best initial management of ascending cholangitis. Patients with ascending cholangitis are acutely ill and present with a fever, jaundice, and right upper quadrant tenderness. They require immediate treatment as well as decompression (from interventional radiology), followed by ERCP/cholecystectomy. 
  • Answer 3: Performing a cholecystectomy this hospital visit is the appropriate management of acute cholecystitis. Acute cholecystitis would present with right upper quadrant abdominal pain in an overweight woman after eating a fatty meal. It is not indicated in the management of asymptomatic gallstones. 
  • Answer 4: Scheduling an outpatient cholecystectomy is not necessary as this patient is asymptomatic. Outpatient cholecystectomies are not typically performed in the management of acute cholecystitis as most patients do not comply with outpatient recommendations (avoiding triggers like fatty foods and alcohol) and end up returning to the hospital for another flare before their procedure. In certain compliant patients, it may be a viable option.

Review NCCPA Blueprint Topic: Cholelithiasis (ReelDx + Lecture)

6. A 60-year-old woman presents to the emergency department with dizziness. She states it started this morning when she woke up from bed and was severe causing her to vomit. The episode resolved in 1 minute. The patient has a past medical history of hypertension, diabetes, obesity, and atrial fibrillation treated with warfarin and metoprolol. She recently recovered from a cold a few days ago. Her temperature is 99.0°F (37.2°C), blood pressure is 174/99 mmHg, pulse is 115/min, respirations are 12/min, and oxygen saturation is 98% on room air. Physical exam is notable for a well-appearing woman. Her neurological exam including cranial nerves and gait is within normal limits. The patient is laid flat in the bed which causes an episode of dizziness with notable nystagmus and vomiting. She feels better after 1 minute. The patient’s ECG is within normal limits. Lab values are notable for an INR of 3.5. Which of the following is the most likely etiology of this patient’s symptoms?

  1. Canalithiasis
  2. Increased endolymph production
  3. Inflammation of the vestibular apparatus
  4. Inflammation of the vestibulocochlear apparatus
  5. Vertebrobasilar insufficiency
Click here to see the answer

Answer: A. Canalithiasis

This patient is presenting with intermittent, severe vertigo which is provoked by position changes which is most consistent with benign paroxysmal positional vertigo (BPPV). BPPV is commonly caused by canalithiasis.

Benign paroxysmal positional vertigo (BPPV) is a common form of peripheral vertigo that results from a dislodged piece of otolith (called otoconia when dislodged) causing disturbances in the semicircular canals. The presentation of BPPV involves sudden and episodic vertigo with head movements that lasts for seconds to minutes accompanied by nausea and vomiting. Physical exam will demonstrate a horizontal nystagmus with specific head postures (such as the Dix-Hallpike maneuver). Treatment involves repositioning exercises (such as the Epley maneuver) as well as meclizine or diphenhydramine for symptomatic control.

Incorrect Answers:

  • Answer 2: Increased endolymph production describes Meniere disease which presents with chronic symptoms including hearing loss and ear fullness and intermittent episodes of vertigo. Treatment involves diuretics and salt restriction.
  • Answers 3-4: Inflammation of the vestibular apparatus and the vestibulocochlear apparatus describes vestibular neuritis (sustained/persistent vertigo after a cold) and labyrinthitis (sustained/persistent vertigo and hearing loss after a cold), respectively. This condition will resolve on its own; however, symptoms can be treated with meclizine or diphenhydramine.
  • Answer 5: Vertebrobasilar insufficiency can present with syncope or if there is a hemorrhage/ischemia, may present in an elderly patient with multiple risk factors with sustained and severe vertigo that is sudden onset and associated with dysarthria and dystonia.

Summary: Benign paroxysmal positional vertigo is commonly caused by canalithiasis.

Review NCCPA Blueprint Topic: Vertigo (ReelDx + Lecture)

7. An 18-year-old male presents to his primary care provider with his parents for a sports physical. He was last seen in the clinic several months ago when he was diagnosed with attention deficit hyperactivity disorder (ADHD). He was started on methylphenidate at that time, and the patient now reports improvement in his ability to concentrate in school and at home. He hopes to play baseball in college and has begun lifting weights daily in preparation for baseball season. The patient reports that he eats a healthy diet to fuel his exercise regimen. His parents have no concerns and are pleased with the recent improvement in his grades. On physical exam, the patient has tall stature with average muscle mass for his age. He has no dysmorphic features. His chest has a normal appearance other than mild gynecomastia. The patient has sparse facial hair and a moderate amount of coarse pubic hair that extends across the pubis and spares the medial thighs. His testes are small and firm. Due to the latter, laboratory testing is performed and reveals the following:

  • Follicle-stimulating hormone (FSH): 42 mIU/mL (Reference range: 4-25 mIU/mL)
  • Luteinizing hormone (LH): 38 mIU/mL (Reference range: 6-23 mIU/mL)

Which of the following is the most likely etiology of this patient’s presentation?

  1. Anabolic steroid use
  2. CGG trinucleotide repeat disorder
  3. CTG trinucleotide repeat disorder
  4. Failure of neuronal migration
  5. Meiotic nondisjunction
Click here to see the answer

Answer: E. Meiotic nondisjunction

This patient presents with tall stature, gynecomastia, and small testes with elevated FSH and LH, which suggests a diagnosis of Klinefelter syndrome. Klinefelter syndrome is usually caused by meiotic nondisjunction that results in a 47,XXY genotype.

Klinefelter syndrome is the most common cause of primary hypogonadism. Patients with Klinefelter syndrome present with tall stature, neurocognitive difficulties (ADHD) and features of hypogonadism including gynecomastia, small testes, small phallus, hypospadias, underdeveloped secondary sex characteristics, and cryptorchidism. Patients without hypospadias or cryptorchidism are often not diagnosed until after puberty, when the symptoms of gynecomastia and small testes become more prominent. Because the hypogonadism in Klinefelter syndrome is caused by testicular fibrosis, laboratory results demonstrate a low testosterone and elevated FSH and LH.

Incorrect Answers:

  • Answer 1: Anabolic steroid use causes decreased levels of FSH and LH due to the suppression of GnRH release by the hypothalamus, which in turn suppresses FSH and LH release by the pituitary gland. Anabolic steroid use would not present with signs of hypogonadism.
  • Answer 2: The CGG trinucleotide repeat disorder characterizes Fragile X syndrome. Fragile X presents with macroorchidism rather than hypogonadism, and patients typically have dysmorphic features of a long, narrow face with large ears, prominent forehead, and prominent chin. Fragile X is the most common cause of inherited intellectual disability.
  • Answer 3: The CTG trinucleotide repeat disorder characterizes myotonic dystrophy. Although myotonic dystrophy presents with hypogonadism, patients would also present with symptoms of progressive weakness, such as facial weakness, dysphagia, or hand grip weakness.
  • Answer 4: Failure of neuronal migration characterizes Kallmann syndrome. Kallmann syndrome presents with the classic symptoms of loss of smell and hypogonadism, but patients with Kallmann syndrome have a low FSH and LH.

Summary: Klinefelter syndrome results in primary hypogonadism and presents with tall stature, gynecomastia, small testes, a small phallus, hypospadias, and cryptorchidism.

Review NCCPA Blueprint Topic: Hypogonadism

8. A 65-year-old man presents to the emergency department for sudden weakness. The patient states that he was at home enjoying his morning coffee when his symptoms began. He says that his left arm suddenly felt very odd and weak thus prompting him to come to the ED. The patient has a past medical history of diabetes, COPD, hypertension, anxiety, alcohol abuse, and PTSD. He recently fell off a horse while horseback riding but claims to not have experienced any significant injuries. He typically drinks 5-7 drinks per day and his last drink was yesterday afternoon. His current medications include insulin, metformin, atorvastatin, lisinopril, albuterol, and fluoxetine. His temperature is 99.5°F (37.5°C), blood pressure is 177/118 mmHg, pulse is 120/min, respirations are 18/min, and oxygen saturation is 93% on room air. On physical exam, you note an elderly man who is mildly confused. The cardiopulmonary exam demonstrates bilateral expiratory wheezes and a systolic murmur along the right upper sternal border that radiates to the carotids. Neurological exam reveals cranial nerves II-XII as grossly intact with finger-nose exam mildly abnormal on the left and heel-shin exam within normal limits. The patient has 5/5 strength in his right arm and 3/5 strength in his left arm. The patient struggles to manipulate objects such as a pen with his left hand. The patient is given a dose of diazepam and started on IV fluids. Which of the following is the most likely diagnosis in this patient?

  1. Berry aneurysm rupture
  2. Bridging vein tear
  3. Cerebellar bleeding
  4. Hypertensive encephalopathy
  5. Lacunar stroke
Click here to see the answer

Answer: E. Lacunar stroke

This patient is presenting with risk factors and symptoms suggestive of a diagnosis of a lacunar stroke. 

Lacunar strokes typically occur in patients with risk factors such as hypertension, diabetes, old age, and smoking. The basal ganglia, pons, and subcortical white matter are commonly affected. The pathophysiology occurs secondary to a small penetrating artery occlusion from hypertensive arteriolar sclerosis, lipohyalinosis, or microatheroma formation. Patients present with neurological deficits that can include pure motor hemiparesis, pure sensory stroke, ataxic hemiparesis, or dysarthria-clumsy hand syndrome.

Incorrect Answers:

  • Answer 1: Berry aneurysm rupture describes a subarachnoid hemorrhage that would present with a sudden onset, severe headache.
  • Answer 2: Bridging vein tear describes a subdural hematoma which could present with gradual neurological deficits and altered cognition in the setting of recent trauma in an elderly patient/alcoholic.
  • Answer 3: Cerebellar bleeding would present with ataxic gait and an abnormal finger-nose and heel-shin exam.
  • Answer 4: Hypertensive encephalopathy presents with general CNS dysfunction which can include headache, irritability, nausea/vomiting, disturbances of consciousness, and seizures.

Review NCCPA Blueprint Topic: Stroke (ReelDx + Lecture)

9. A 26-year-old woman presents to the emergency department with abdominal pain. She states that she was walking up the stairs at work when she felt sudden and severe abdominal pain followed by nausea and vomiting. Her past medical history is noncontributory and she is not currently taking any medications. Her temperature is 99.7°F (37.6°C), blood pressure is 122/78 mmHg, pulse is 120/min, respirations are 17/min, and oxygen saturation is 98% on room air. Physical exam is notable for an absence of abdominal tenderness, a left adnexal mass, and left adnexal tenderness. A transvaginal ultrasound demonstrates free fluid surrounding the ovary with edema and the presence of doppler flow. A urinary pregnancy test is negative. The patient’s symptoms persisted after ibuprofen and acetaminophen. Which of the following is the best next step in management?

  1. CT scan of the abdomen
  2. Laparoscopy
  3. Laparotomy
  4. MRI of the pelvis
  5. Observation and serial abdominal exams
Click here to see the answer

Answer: B. Laparoscopy

This patient is presenting with sudden onset abdominal pain, nausea, vomiting, and free fluid around the ovary with normal blood flow which is still concerning for ovarian torsion that should be managed with laparoscopy. Ovarian torsion occurs when the ovary twists around its blood supply causing ischemia and necrosis. Patients are typically young women who experience sudden/severe abdominal or vaginal pain, blood per vagina, a left adnexal mass with adnexal tenderness, and a negative urine pregnancy test. The best initial test is a transvaginal ultrasound with Doppler which can show blood flow to the ovary as well as ovarian enlargement and edema (signs of ischemia). Even in the setting of normal blood flow, torsion is possible as the ovary may twist and untwist. Patients presenting with symptoms concerning for ovarian torsion should be managed with laparoscopy to definitively treat and salvage the ovary.

Incorrect Answers:

  • Answer 1: CT scan of the abdomen would not be necessary as this patient has no abdominal tenderness and has physical exam and ultrasound findings concerning for ovarian torsion which should be managed surgically. 
  • Answer 3: Laparotomy is an open procedure that would be indicated if the patient suddenly decompensated and needed more aggressive management to stop acute bleeding. 
  • Answer 4: MRI of the pelvis would be a very accurate test for diagnosing pelvic pathology; however, it is not necessary in this patient who has a history and findings concerning for ovarian torsion. Performing an MRI would only delay treatment.
  • Answer 5: Observation and serial abdominal exams would be the appropriate management of a patient with abdominal pain with a clear etiology after a CT scan had ruled out any serious pathology. This patient’s likely ovarian torsion should not be observed.

Summary: Ovarian torsion should be treated with laparoscopy. 

Review NCCPA Blueprint Topic: Ovarian torsion

10. A 72-year-old man is brought into the emergency department by emergency medical services. He looks disheveled and states that he is homeless. He has bruising over his arms and legs and states that he does not have a regular source of nutrition. He denies prior medical conditions but states that he still smokes one pack of cigarettes per day. On exam, the patient’s vital signs are normal, but he appears extremely malnourished. His gums are swollen and bleeding and his tongue is unusually smooth. The hair on his arms is pinwheel-shaped. What is the most likely cause?

  1. Iron deficiency
  2. Vitamin B3 deficiency
  3. Vitamin C deficiency
  4. Vitamin B12 deficiency
  5. Vitamin K deficiency
Click here to see the answer

Answer: C. Vitamin C deficiency

An elderly, malnourished, cigarette-smoking male with swollen gums, bruising, and corkscrew hair is most likely suffering from vitamin C deficiency.

Though vitamin C deficiency, or scurvy, is uncommon in the developed world, cases in developing countries still exist. Typically, these cases occur in those who are very old or very young due to the inability to feed themselves properly. Since vitamin C, or ascorbic acid, is found in citrus fruits and green vegetables, deficiency in these foods or consumption of these foods with denatured vitamins (due to over-boiling) can result in deficiency. Those who smoke cigarettes have also been found to be more deficient in vitamin C.

Incorrect Answers:

  • Answer 1: Iron deficiency anemia would most commonly be asymptomatic, and if serious, would present with symptoms of fatigue and shortness of breath. It would not present with glossitis or corkscrew hair.
  • Answer 2: Vitamin B3, or niacin, can present with glossitis, but would typically also have the “3 D’s”: diarrhea, dermatitis, and dementia.
  • Answer 4: Vitamin B12 deficiency can also present with glossitis, but it would also have physical exam signs such as numbness and parasthesias of the extremities along with ataxia.
  • Answer 5: Vitamin K deficiency can present with bleeding and bruising, but it would not present with the other symptoms typically associated with scurvy.

Review NCCPA Blueprint Topic: Hypervitaminosis/hypovitaminosis

[spoiler title=”C

Looking for all the podcast episodes?

This FREE series is limited to every other episode, you can download and enjoy the complete audio series by joining The PANCE and PANRE Exam Academy + SMARTYPANCE

I will be releasing new episodes every few weeks. The Academy is discounted, so sign up now.

Resources and Links From The Show

This Podcast is also available on iTunes and Stitcher Radio for Android

  1. iTunes: The Audio PANCE and PANRE Podcast iTunes
  2. Stitcher Radio: The Audio PANCE and PANRE Podcast Stitcher
  3. Google Play: The Audio PANCE and PANRE Podcast Google Play
itunes_logo-1

Download The Content Blueprint Checklist

Follow this link to download your FREE copy of the Content Blueprint Checklist

Print it up and start crossing out the topics you understand, marking the ones you don’t and making notes of key terms you should remember. The PDF version is interactive and linked directly to the individual lessons on SMARTY PANCE.

Download for PANCE Download for PANRE

The post Podcast Episode 75: Ten FREE PANCE and PANRE Audio Board Review Questions appeared first on The Audio PANCE and PANRE.

]]>
Welcome to episode 75 of the Audio PANCE and PANRE PA Board Review Podcast. Join me as I cover ten PANCE and PANRE Board review questions from the SMARTYPANCE course content following the NCCPA™ content blueprint (download the FREE cheat sheet).



Welcome to episode 75 of the Audio PANCE and PANRE PA Board Review Podcast.



Join me as I cover ten PANCE and PANRE Board review questions from the SMARTYPANCE course content following the NCCPA™ content blueprint (download the FREE cheat sheet).



This week we will be covering ten general board review questions based on the NCCPA PANCE and PANRE Content Blueprints. 



Below you will find an interactive exam to complement the podcast.



The Audio PANCE and PANRE Physician Assistant Board Review Podcast



I hope you enjoy this free audio component to the examination portion of this site. The full board review includes over 2,000 interactive board review questions and is available to all members of the PANCE and PANRE Academy and Smarty PANCE.



* You can download and listen to past FREE episodes here, on iTunes, on Google Play Music or Stitcher Radio.* You can listen to the latest episode, take an interactive quiz and download your results below.



Listen Carefully Then Take The Practice Exam





If you can’t see the audio player click here to listen to the full episode.



Podcast Episode 75: Ten Question PANCE and PANRE Podcast Quiz



The following questions are linked to NCCPA Content Blueprint lessons from the Smarty PANCE and PANRE Board Review Website. If you are a member you will be able to log in and view this interactive video lesson.



1. A 5-year-old girl is brought to the emergency department after drinking a bottle of drain cleaner. It is unknown how much the child drank. She has a past medical history of Down syndrome and obesity. The patient’s vitals are unremarkable. Physical exam is notable for a child in no acute distress. She is tolerating her oral secretions and is interactive. Inspection of the oropharynx is unremarkable. Which of the following is the appropriate management of this patient?



* Dilute hydrochloric acid* Endoscopy* Intubation* Observation* Polyethylene glycol





2. A 27-year-old man presents to the emergency department after a motor vehicle collision.]]>
The Physician Assistant Life | Smarty PANCE full clean 35:46 350
Podcast Episode 73: Ten PANCE and PANRE Audio Board Review Questions http://podcast.thepalife.com/podcast-episode-73-ten-pance-and-panre-audio-board-review-questions/ Wed, 14 Aug 2019 10:23:41 +0000 http://podcast.thepalife.com/?p=344 http://podcast.thepalife.com/podcast-episode-73-ten-pance-and-panre-audio-board-review-questions/#respond http://podcast.thepalife.com/podcast-episode-73-ten-pance-and-panre-audio-board-review-questions/feed/ 0 <p>The Audio PANCE and PANRE Physician Assistant Board Review Podcast Welcome to episode 73 of the Audio PANCE and PANRE PA Board Review Podcast. Join me as I cover ten PANCE and PANRE Board review questions from the SMARTYPANCE course content following the NCCPA™ content blueprint (download the FREE cheat sheet). This week we will be […]</p> <p>The post <a rel="nofollow" href="http://podcast.thepalife.com/podcast-episode-73-ten-pance-and-panre-audio-board-review-questions/">Podcast Episode 73: Ten PANCE and PANRE Audio Board Review Questions</a> appeared first on <a rel="nofollow" href="http://podcast.thepalife.com">The Audio PANCE and PANRE</a>.</p> The Audio PANCE and PANRE Physician Assistant Board Review Podcast
Episode 73 The Audio PANCE and PANRE Physician Assistant Board Review Podcast

Welcome to episode 73 of the Audio PANCE and PANRE PA Board Review Podcast.

Join me as I cover ten PANCE and PANRE Board review questions from the SMARTYPANCE course content following the NCCPA™ content blueprint (download the FREE cheat sheet).

This week we will be covering ten general board review questions based on the NCCPA PANCE and PANRE Content Blueprints. 

Below you will find an interactive exam to complement the podcast.

I hope you enjoy this free audio component to the examination portion of this site. The full board review includes over 2,000 interactive board review questions and is available to all members of the PANCE and PANRE Academy and Smarty PANCE.

Listen Carefully Then Take The Practice Exam

If you can’t see the audio player click here to listen to the full episode.

Podcast Episode 73: Ten Question PANCE and PANRE Podcast Quiz

The following questions are linked to NCCPA Content Blueprint lessons from the Smarty PANCE and PANRE Board Review Website. If you are a member you will be able to log in and view this interactive video lesson.

1. A 45-year obese female (BMI=36.7 ) presents to your clinic with a random serum glucose level of 242.  You diagnose the patient with type II diabetes and place her on metformin 500 mg twice daily and ask that she check her glucose first thing in the morning and again late at night. The patient states that her glucose first thing in the morning is 424 and by the evening it is 96.  Which of the following tests would you like to order to confirm your diagnosis?

  1. ACTH level
  2. HgA1C
  3. 24 collection of VMA
  4. Dexamethasone suppression test
  5. Cortisol levels
Click here to see the answer

Answer: D. Dexamethasone suppression test

This patient is presenting with elevated glucose levels in the morning and normal levels at night. This is due to the dawn phenomenon or secreting too much cortisol first thing in the morning which is very common in patients with Cushing syndrome. The diagnostic test of choice for Cushing syndrome is a dexamethasone suppression test. While it is true that the cortisol levels will be elevated this does not diagnose Cushing’s syndrome.

Review NCCPA Blueprint Topic: Cushing’s syndrome

2. A 49-year-old male is admitted to the hospital with a diagnosis of pneumonia. He has had a cough and fever up to 101.2 °F for five days.  The patient states that his cough is productive and he is also complaining of sharp right-sided chest pain. On physical exam, you note diminished breath sounds on the right as well as dullness to percussion.  The chest x-ray demonstrates a right lower lobe infiltrate. Which of the following bacteria would be the most likely cause of this patient’s pneumonia?

  1. Mycoplasma
  2. Pneumococcal
  3. H. Flu
  4. Legionnaires 
  5. Chlamydia
Click here to see the answer

Answer: B. Pneumococcal

Pneumococcal pneumonia is the most common form of pneumonia in an adult that is greater than 40. It usually presents as lobar pneumonia with dullness to percussion and diminished breath sounds. There is also a vaccination available. Mycoplasma is the most common form of pneumonia in young adults and those less than 40.

Review NCCPA Blueprint Topic: Bacterial pneumonia (ReelDx)

3. A 21-year-old male is brought into the emergency room by his fraternity brothers. He appears confused and is agitated. He has a temperature of 100.5 F, horizontal nystagmus, muscular rigidity and at times becomes incredibly combative. His laboratory evaluation shows a CPK level of 8000 and his blood alcohol level is 0. Which of the following drugs most likely explains this patient’s symptoms?

  1. Methamphetamines
  2. LSD
  3. PCP
  4. Ecstasy 
  5. Marijuana
Click here to see the answer

Answer: C. PCP

PCP can cause extreme agitation as well as fever, muscular rigidity, horizontal nystagmus, and rhabdomyolysis.  The best treatment for these patients is IV fluids and sedation with medications like Geodon, Haldol, Thorazine, and Ativan. You can also add Benadryl to this cocktail.

Review NCCPA Blueprint Topic: Substance-related and addictive disorders

4. A 51-year-old male presents to the ED complaining of substernal burning discomfort. He reports a history of mild hypertension and Raynaud’s phenomenon. On physical exam, you notice he has shortened fingers. When further questioned the patients states that he was once told that he may have a type of autoimmune disease but is unaware of the name. Which of the following medications would be most helpful for the patient’s chronic heartburn??

  1. Omeprazole
  2. Cimetidine
  3. Carafate
  4. Zofran
  5. Amlodipine
Click here to see the answer

Answer: E. Amlodipine

The patient’s symptoms of heartburn, hypertension, Raynaud’s phenomenon, and short fingers are consistent with scleroderma. The best treatment for the reflux esophagitis that can occur with scleroderma is a calcium channel blocker. Calcium channel blockers serve as a smooth muscle relaxant which will relax his tight esophagus resulting in increased mobility and fewer chances for heartburn.

Review NCCPA Blueprint Topic: Systemic sclerosis – Scleroderma

5. You are called to see a patient who is currently on multiple psychiatric medications including Lithium, Lorazepam, Abilify, Trazodone, and Sertraline. He also takes Lasix.  He is in an assisted living facility and is brought to you by his care team for an acute psychotic episode. Which of the following medications is most likely responsible for the patient’s acute psychosis?

  1. Lorazepam
  2. Lasix
  3. Abilify
  4. Trazodone
  5. Sertraline
Click here to see the answer

Answer: B. Lasix

Loop diuretics can be very problematic in patients taking psychiatric medications. Loop diuretic result in a loss of water in the vascular volume, henceforth levels of different psychiatric medications will become higher. For example, if a patient is on lithium and they are currently on Lasix lithium level becomes higher and they can become with lithium toxic.

Review NCCPA Blueprint Topic: Schizophrenia spectrum and other psychotic disorders

6. A young healthy female patient with a recent diagnosis of depression complains of a 4-month history of amenorrhea and decreased sex drive. She has no significant past medical history and usually has normal menstrual cycles. Which of the following medications is most likely the cause of this patient’s symptoms?

  1. Sertraline
  2. Trazodone
  3. Abilify
  4. Venlafaxine
  5. Risperidone
Click here to see the answer

Answer: E. Risperidone

Older (second-generation) atypical antipsychotic agents can cause elevations in plasma prolactin concentrations while the newer atypical antipsychotics have minimal effect. Risperidone (an older second-generation antipsychotic agent) can cause amenorrhea, sexual dysfunction, and infertility.

Review NCCPA Blueprint Topic: Depressive disorders (Pearls)

7. A 64-year-old male presents with a chief complaint of lower extremity pain while walking his dog every afternoon. What is the most important risk factor for developing peripheral vascular disease?

  1. Smoking
  2. Hypertension
  3. Elevated cholesterol
  4. Elevated triglycerides
  5. Diabetes
Click here to see the answer

Answer: A. Smoking

Smoking is the strongest risk factor for PVD. As a class of drugs view nicotine as a very potent vasoconstrictor causing vasoconstriction everywhere. For example vasoconstriction of the vessels in the heart=CAD which can lead to MI. Constricted blood vessels in the brain=CVA and constricted blood vessels in the periphery=PVD.

Review NCCPA Blueprint Topic: Peripheral artery disease (Lesson)

8. A 35-year-old male patient presents concerned about his cardiac risk factors. He has a low HDL level, hypertension, type II DM, and an elevated LDL and triglyceride levels. His brother had CAD at the age of 47. Which of the following risk factor has the highest value when determining this patients risk of developing CAD?

  1. Low HDL levels
  2. Hypertension
  3. Diabetes
  4. High cholesterol
  5. Family history
Click here to see the answer

Answer: C. Diabetes

Diabetes is the strongest risk factor for CAD. All the other choices are risk factors however they do not have the same predictive value as diabetes when it comes to the severity of CAD.

Review NCCPA Blueprint Topic: Coronary Heart Disease (PEARLS)

9. Which of the following is considered to be a 1st line medication used in endocarditis prophylaxis for high-risk patients undergoing invasive procedures such as an esophageal stricture dilation?

  1. Cipro
  2. Clindamycin
  3. Amoxicillin
  4. Clarithromycin
  5. Doxycycline
Click here to see the answer

Answer: C. Amoxicillin

Amoxicillin is the prophylactic agent of choice against endocarditis, as it is able to cover both gram-negative and gram-positive organisms. If the patient is PCN allergic then Clindamycin is the next best choice.

Review NCCPA Blueprint Topic: Acute and subacute bacterial endocarditis

10. In a patient with a VSD murmur which of the following will be seen during the physical exam?

  1. Split S1
  2. Diastolic crescendo murmur
  3. Wide pulse pressures
  4. Holosystolic murmur
  5. Split S2
Click here to see the answer

Answer: D. Holosystolic murmur

In VSD the patient will have a holosystolic murmur moving from left to right during systole causing more blood to move into the right ventricle and pulmonary artery. End game is pulmonary hypertension.

Review NCCPA Blueprint Topic: Ventricular septal defect

[spoiler title=”C

Looking for all the podcast episodes?

This FREE series is limited to every other episode, you can download and enjoy the complete audio series by joining The PANCE and PANRE Exam Academy + SMARTYPANCE

I will be releasing new episodes every few weeks. The Academy is discounted, so sign up now.

Resources and Links From The Show

This Podcast is also available on iTunes and Stitcher Radio for Android

  1. iTunes: The Audio PANCE AND PANRE Podcast iTunes
  2. Stitcher Radio: The Audio PANCE and PANRE Podcast Stitcher
itunes_logo-1

Download The Content Blueprint Checklist

Follow this link to download your FREE copy of the Content Blueprint Checklist

Print it up and start crossing out the topics you understand, marking the ones you don't and making notes of key terms you should remember. The PDF version is interactive and linked directly to the individual lessons on SMARTY PANCE.

Download for PANCE Download for PANRE

The post Podcast Episode 73: Ten PANCE and PANRE Audio Board Review Questions appeared first on The Audio PANCE and PANRE.

]]>
The Audio PANCE and PANRE Physician Assistant Board Review Podcast Welcome to episode 73 of the Audio PANCE and PANRE PA Board Review Podcast. Join me as I cover ten PANCE and PANRE Board review questions from the SMARTYPANCE course content following t... The Audio PANCE and PANRE Physician Assistant Board Review Podcast







Welcome to episode 73 of the Audio PANCE and PANRE PA Board Review Podcast.



Join me as I cover ten PANCE and PANRE Board review questions from the SMARTYPANCE course content following the NCCPA™ content blueprint (download the FREE cheat sheet).



This week we will be covering ten general board review questions based on the NCCPA PANCE and PANRE Content Blueprints. 



Below you will find an interactive exam to complement the podcast.



I hope you enjoy this free audio component to the examination portion of this site. The full board review includes over 2,000 interactive board review questions and is available to all members of the PANCE and PANRE Academy and Smarty PANCE.



* You can download and listen to past FREE episodes here, on iTunes, on Google Play Music or Stitcher Radio.* You can listen to the latest episode, take an interactive quiz and download your results below.



Listen Carefully Then Take The Practice Exam





If you can’t see the audio player click here to listen to the full episode.



Podcast Episode 73: Ten Question PANCE and PANRE Podcast Quiz



The following questions are linked to NCCPA Content Blueprint lessons from the Smarty PANCE and PANRE Board Review Website. If you are a member you will be able to log in and view this interactive video lesson.



1. A 45-year obese female (BMI=36.7 ) presents to your clinic with a random serum glucose level of 242.  You diagnose the patient with type II diabetes and place her on metformin 500 mg twice daily and ask that she check her glucose first thing in the morning and again late at night. The patient states that her glucose first thing in the morning is 424 and by the evening it is 96.  Which of the following tests would you like to order to confirm your diagnosis?



* ACTH level* HgA1C* 24 collection of VMA* Dexamethasone suppression test* Cortisol levels





2. A 49-year-old male is admitted to the hospital with a diagnosis of pneumonia. He has had a cough and fever up to 101.]]>
The Physician Assistant Life | Smarty PANCE full clean 15:57 344
Podcast Episode 71: Ten PANCE and PANRE Audio Board Review Questions http://podcast.thepalife.com/podcast-episode-71/ Thu, 09 May 2019 07:01:50 +0000 http://podcast.thepalife.com/?p=339 http://podcast.thepalife.com/podcast-episode-71/#respond http://podcast.thepalife.com/podcast-episode-71/feed/ 0 <p>The Audio PANCE and PANRE Physician Assistant Board Review Podcast Welcome to episode 71 of the Audio PANCE and PANRE PA Board Review Podcast. Join me as I cover ten PANCE and PANRE Board review questions from the SMARTYPANCE course content following the NCCPA™ content blueprint (download the FREE cheat sheet). This week we will be […]</p> <p>The post <a rel="nofollow" href="http://podcast.thepalife.com/podcast-episode-71/">Podcast Episode 71: Ten PANCE and PANRE Audio Board Review Questions</a> appeared first on <a rel="nofollow" href="http://podcast.thepalife.com">The Audio PANCE and PANRE</a>.</p> The Audio PANCE and PANRE Physician Assistant Board Review Podcast
Episode 71 The Audio PANCE and PANRE Physician Assistant Board Review Podcast

Welcome to episode 71 of the Audio PANCE and PANRE PA Board Review Podcast.

Join me as I cover ten PANCE and PANRE Board review questions from the SMARTYPANCE course content following the NCCPA™ content blueprint (download the FREE cheat sheet).

This week we will be covering ten general board review questions based on the NCCPA PANCE and PANRE Content Blueprints. 

Below you will find an interactive exam to complement the podcast.

I hope you enjoy this free audio component to the examination portion of this site. The full board review includes over 2,000 interactive board review questions and is available to all members of the PANCE and PANRE Academy and Smarty PANCE.

Listen Carefully Then Take The Practice Exam

If you can’t see the audio player click here to listen to the full episode.

Podcast Episode 71: Ten Question PANCE and PANRE Podcast Quiz

The following questions are linked to NCCPA Content Blueprint lessons from the Smarty PANCE and PANRE Board Review Website. If you are a member you will be able to log in and view this interactive video lesson.

1. A 52-year-old male presents complaining of urinary frequency, with hesitancy, and nocturia for the past few months. During his physical examination, you note a nontender, non-enlarged prostate with an isolated right posterior lobe nodule. Which of the following options is most appropriate?

  1. order a serum acid phosphatase level
  2. initiate prazosin and schedule a follow-up appointment in 6 weeks
  3. refer the patient for an ultrasound of the prostate and order a PSA level
  4. reassure the patient and schedule a follow-up appointment in six months
  5. initiate norfloxacin therapy for seven days and schedule follow-up in two weeks
Click here to see the answer

Answer: C. refer the patient for an ultrasound of the prostate and order a PSA level

This patient has an isolated nodule of the prostate gland — cancer until proven otherwise. You should order an ultrasound and a PSA. BPH will present as diffuse enlargement and not a discrete nodule.

Review NCCPA Blueprint Topic: Prostate disorders (PEARLS)

2. Which of the following is a major contraindication to curative surgical resection of a lung tumor?

A. Liver metastases
B. Vagus nerve involvement
C. Non-malignant pleural effusion
D. Chest wall invasion of the tumor

Click here to see the answer

Answer: A. Liver metastases

Distant metastases, except for solitary brain and adrenal metastases are an absolute contraindication for pulmonary resection. Other absolute contraindications include MI within the past 3 months, superior vena cava syndrome due to a metastatic tumor, bilateral endobronchial tumor, contralateral lymph node metastases, and malignant pleural effusion.

Review NCCPA Blueprint Topic: Pulmonary Neoplasms (PEARLS)

3. A 23-year-old female complains of fever and genital pain. Examination reveals the presence of lymphadenopathy in the groin and the presence of vulvar vesicles surrounded by erythematous skin. The diagnosis may be confirmed by

A. A culture of a vesicle using blood agar medium
B. The presence of similar lesions on the fingers and hands
C. A Gram stain of a scraping from the base of the lesion
D. The presence of giant multinucleated cells on Tzanck smear

Click here to see the answer

Answer: D. The presence of giant multinucleated cells on Tzanck smear – A Tzanck smear is the standard laboratory test to confirm the herpes virus as an etiologic agent of a vesicular lesion on an erythematous base.

A. Herpes virus will not grow on blood agar medium.
B. Herpes genitalis occurs in skin that comes into contact with the herpes virus. Usually, the infection is localized.
C. A Gram stain will not identify the herpes virus.

Review NCCPA Blueprint Topic: Herpes simplex (ReelDx)

4. A 28-year-old woman is complaining of heavy uterine bleeding and pelvic pressure that has progressively worsened over the past year. Evaluation reveals multiple moderate-sized uterine fibroids. The patient desires to have more children. The most appropriate definitive treatment is

A. Myomectomy
B. Hysterectomy
C. GnRH agonists
D. Oral progesterone

Click here to see the answer

Answer: A. Myomectomy –  Myomectomy is the definitive treatment of choice for moderate-sized uterine fibroids in young women who desire to maintain reproductive capability.

B. Hysterectomy is indicated as definitive treatment in a patient who does not desire to maintain reproductive capability.
C. GnRH agonists are used as an adjunct to surgery for treatment of uterine fibroids. Used alone, they would not be considered a definitive treatment.
D. Oral progesterone may be used to suppress menorrhagia preoperatively. Used alone, it would not be considered a definitive treatment.;

Review NCCPA Blueprint Topic:

5. A 59-year-old otherwise healthy female develops acute dyspnea and chest pain one week post total abdominal hysterectomy. Echocardiogram demonstrates normal heart size with normal right and left ventricular function. Lung scan demonstrates two segmental perfusion defects. Which of the following is the next step in the management of this patient?

A. Anticoagulation
B. Embolectomy
C. Thrombolysis
D. Inferior vena cava filter

Click here to see the answer

Answer: A. Anticoagulation – Anticoagulation is the treatment of choice in patients with pulmonary embolism with normal ventricular function and no absolute contraindications.

B. Embolectomy is not indicated as the initial treatment of a pulmonary embolism in patients with normal ventricular function.
C. Thrombolysis is contraindicated in patients within 10 days of having major surgery.
D. An inferior vena cava filter is considered in patients with contraindications to anticoagulation therapy or failed anticoagulation therapy.

Review NCCPA Blueprint Topic: Pulmonary embolism (ReelDx)

6. Long term use of which of the following drugs may cause a drug-induced lupus-type eruption?

A. prednisone
B. tetracycline
C. procainamide
D. oral contraceptives

Click here to see the answer

Answer: C. Procainamide – Procainamide, and hydralazine are the most common drugs that may cause a lupus-like eruption.

A. Prednisone is not implicated in drug-induced skin reactions.
B. Tetracycline and sulfonamides are known to cause a photosensitive rash on sun-exposed areas of the skin.
D. Oral contraceptives may induce erythema nodosum.

Review NCCPA Blueprint topic: Drug eruptions (ReelDx)

7. Which of the following clinical manifestations is most commonly seen in viral croup?

A. drooling
B. wheezing
C. sputum production
D. inspiratory stridor

Click here to see the answer

Answer: D. inspiratory stridor – Viral croup typically presents with a barking cough and stridor.

A. Drooling is common in epiglottitis, not viral croup.
B. Wheezing is noted in asthma.
C. Sputum production is noted in bacterial infections.

Review NCCPA Blueprint Topic: Croup (ReelDx)

8. Whispered voice test on a patient reveals decreased hearing in the left ear. Which of the following would be most consistent with conductive hearing loss in the left ear?

A. Sounds best heard in the left ear on Weber test.
B. Air conduction longer than bone conduction in the left ear on Rinne test.
C. Sound best heard in the right ear on Weber test.
D. Bone conduction longer than air conduction in the right ear.

Click here to see the answer

Answer: A. Sounds best heard in the left ear on Weber test – Sound best heard in the ear with a decreased hearing on Weber test (in this case, the left ear) is indicative of conductive hearing loss.

B. With conductive hearing loss, bone conduction should be heard as long as or longer than air conduction of sound in the affected ear. Air conduction lasting longer than bone conduction of sound would indicate a sensorineural hearing loss.
C. Sound best heard in the ear with an unaffected hearing on Weber test (in this case, the right ear) is indicative of sensorineural hearing loss.
D. With conductive hearing loss, bone conduction should be heard as long as or longer than air conduction of sound in the affected ear. The right ear showed normal hearing on physical exam.

Review NCCPA Blueprint Topic: Hearing impairment (Lecture)

9. A 65-year-old male presents with multiple lesions on his back. He denies any pruritis. Physical examination reveals the presence of multiple scattered brown plaques with a raised, warty surface that appears to be stuck onto the skin and feel greasy. Which of the following is the most likely diagnosis?

A. lentigines
B. actinic keratosis
C. keratoacanthomas
D. seborrheic keratosis

Click here to see the answer

Answer: D. seborrheic keratosis – Seborrheic keratosis is a common benign plaque in the elderly that characteristically has a velvety or warty surface associated with a stuck on appearance and greasy feel.

A. Lentigines most commonly are seen on the dorsum of the hand and appear as flat brown spots, often with sharp borders.
B. Actinic keratosis usually presents as small patches of flesh-colored, pink or yellow-brown lesions often with an erythematous component. The lesions are better felt than seen, having a rough, sandpaper feel and are often tender to palpation.
C. Keratoacanthomas usually occur as an isolated lesion on the face appearing as an erythematous, dome-shaped nodule with a central keratinaceous plug.

Review NCCPA Blueprint Topic: Keratotic disorders (PEARLS)

10. You are called to the nursery to see a male infant, born by uncomplicated vaginal delivery. He weighs 2,600 grams and has one deep crease on the anterior third of each foot. Respirations are 88 breaths/minute with expiratory grunting and intercostals retractions. He is cyanotic on room air and becomes pink when placed on 60% oxygen. Chest x-ray shows atelectasis with air bronchograms. Which of the following is the most likely diagnosis?

A. neonatal pneumonia
B. congenital heart disease
C. hyaline membrane disease
D. chronic lung disease of prematurity

Click here to see the answer

Answer: C. hyaline membrane disease – Hyaline membrane disease is the most common cause of respiratory distress in the premature infant. The infant typically presents with tachypnea, cyanosis, and expiratory grunting. A chest x-ray reveals hypoexpansion and air bronchograms.

A. While tachypnea, grunting, retractions, and cyanosis may be signs of neonatal pneumonia, they are primarily late findings of progressive respiratory distress and would not be seen immediately at the time of delivery. A chest x-ray in pneumonia would also most commonly reveal an infiltrate or effusion.
B. While congenital heart disease may present with cyanosis, the chest x-ray will reveal a cardiac abnormality, such as cardiomegaly.
D. Chronic lung disease of prematurity is a complication in about 20% of infants with hyaline membrane disease. It is defined as respiratory symptoms, oxygen requirement and chest x-ray abnormalities at 1 month of age so it cannot be diagnosed at this time in this newborn.;

Review NCCPA Blueprint Topic: Looking for all the podcast episodes?

This FREE series is limited to every other episode, you can download and enjoy the complete audio series by joining The PANCE and PANRE Exam Academy + SMARTYPANCE

I will be releasing new episodes every few weeks. The Academy is discounted, so sign up now.

Resources and Links From The Show

This Podcast is also available on iTunes and Stitcher Radio for Android

  1. iTunes: The Audio PANCE AND PANRE Podcast iTunes
  2. Stitcher Radio: The Audio PANCE and PANRE Podcast Stitcher
itunes_logo-1

Download The Content Blueprint Checklist

Follow this link to download your FREE copy of the Content Blueprint Checklist

Print it up and start crossing out the topics you understand, marking the ones you don’t and making notes of key terms you should remember. The PDF version is interactive and linked directly to the individual lessons on SMARTY PANCE.

Download for PANCE Download for PANRE

The post Podcast Episode 71: Ten PANCE and PANRE Audio Board Review Questions appeared first on The Audio PANCE and PANRE.

]]> The Audio PANCE and PANRE Physician Assistant Board Review Podcast Welcome to episode 71 of the Audio PANCE and PANRE PA Board Review Podcast. Join me as I cover ten PANCE and PANRE Board review questions from the SMARTYPANCE course content following t... The Audio PANCE and PANRE Physician Assistant Board Review Podcast







Welcome to episode 71 of the Audio PANCE and PANRE PA Board Review Podcast.



Join me as I cover ten PANCE and PANRE Board review questions from the SMARTYPANCE course content following the NCCPA™ content blueprint (download the FREE cheat sheet).



This week we will be covering ten general board review questions based on the NCCPA PANCE and PANRE Content Blueprints. 



Below you will find an interactive exam to complement the podcast.



I hope you enjoy this free audio component to the examination portion of this site. The full board review includes over 2,000 interactive board review questions and is available to all members of the PANCE and PANRE Academy and Smarty PANCE.



* You can download and listen to past FREE episodes here, on iTunes, on Google Play Music or Stitcher Radio.* You can listen to the latest episode, take an interactive quiz and download your results below.



Listen Carefully Then Take The Practice Exam





If you can’t see the audio player click here to listen to the full episode.



Podcast Episode 71: Ten Question PANCE and PANRE Podcast Quiz



The following questions are linked to NCCPA Content Blueprint lessons from the Smarty PANCE and PANRE Board Review Website. If you are a member you will be able to log in and view this interactive video lesson.



1. A 52-year-old male presents complaining of urinary frequency, with hesitancy, and nocturia for the past few months. During his physical examination, you note a nontender, non-enlarged prostate with an isolated right posterior lobe nodule. Which of the following options is most appropriate?



* order a serum acid phosphatase level* initiate prazosin and schedule a follow-up appointment in 6 weeks* refer the patient for an ultrasound of the prostate and order a PSA level* reassure the patient and schedule a follow-up appointment in six months* initiate norfloxacin therapy for seven days and schedule follow-up in two weeks





2. Which of the following is a major contraindication to curative surgical resection of a lung tumor?



A. Liver metastasesB. Vagus nerve involvementC. Non-malignant pleural effusionD.]]>
The Physician Assistant Life | Smarty PANCE full clean 22:42 339 Podcast Episode 69: Ten PANCE and PANRE Board Review Audio Questions http://podcast.thepalife.com/podcast-episode-69-ten-pance-and-panre-board-review-audio-questions/ Thu, 28 Mar 2019 00:03:06 +0000 http://podcast.thepalife.com/?p=335 http://podcast.thepalife.com/podcast-episode-69-ten-pance-and-panre-board-review-audio-questions/#respond http://podcast.thepalife.com/podcast-episode-69-ten-pance-and-panre-board-review-audio-questions/feed/ 0 <p>The Audio PANCE/PANRE Board Review Exam Questions Welcome to episode 69 of the Audio PANCE and PANRE PA Board Review Podcast. Join me as I cover ten PANCE and PANRE Board review questions from the SMARTYPANCE course content following the NCCPA™ content blueprint (download the FREE cheat sheet). This week we will be covering ten general board […]</p> <p>The post <a rel="nofollow" href="http://podcast.thepalife.com/podcast-episode-69-ten-pance-and-panre-board-review-audio-questions/">Podcast Episode 69: Ten PANCE and PANRE Board Review Audio Questions</a> appeared first on <a rel="nofollow" href="http://podcast.thepalife.com">The Audio PANCE and PANRE</a>.</p>
The Audio PANCE and PANRE Physician Assistant Board Review Podcast - Episode 69

The Audio PANCE/PANRE Board Review Exam Questions

Welcome to episode 69 of the Audio PANCE and PANRE PA Board Review Podcast.

Join me as I cover ten PANCE and PANRE Board review questions from the SMARTYPANCE course content following the NCCPA™ content blueprint (download the FREE cheat sheet).

This week we will be covering ten general board review questions based on the NCCPA PANCE and PANRE Content Blueprints. 

Below you will find an interactive exam to complement the podcast.

I hope you enjoy this free audio component to the examination portion of this site. The full board review includes over 2,000 interactive board review questions and is available to all members of the PANCE and PANRE Academy and Smarty PANCE.

Listen Carefully Then Take The Practice Exam

If you can’t see the audio player click here to listen to the full episode.

Podcast Episode 69: 10 Question PANCE and PANRE Podcast Quiz

The following questions are linked to NCCPA Content Blueprint lessons from the Smarty PANCE and PANRE Board Review Website. If you are a member you will be able to log in and view this interactive video lesson.

1. Which of the following is the most common cause of acute myocardial infarction?

A. Occlusion caused by coronary microemboli
B. Thrombus development at a site of vascular injury
C. Congenital abnormalities
D. Severe coronary artery spasm

Click here to see the answer

B. Thrombus development at a site of vascular injury – Acute myocardial infarction occurs when a coronary artery thrombus develops rapidly at a site of vascular injury. In most cases, infarction occurs when an atherosclerotic plaque fissures, ruptures, or ulcerates and when conditions favor thrombogenesis so that a mural thrombus forms at the site of rupture and leads to coronary artery occlusion.

A. Coronary microemboli occlusion is a rare cause of acute myocardial infarction.
C. Congenital abnormalities are rare causes of acute MI.
D. Severe coronary artery spasm is more likely to result in Prinzmetal’s angina rather than true infarction.

Review NCCPA Blueprint Topic: Acute myocardial infarction (PEARLS) (Lesson)

2. Endotracheal intubation should be performed with caution in patients with which of the following underlying conditions due to the propensity to cause subluxation of C1 on C2?

A. Rheumatoid arthritis
B. Osteoarthritis
C. Gout
D. Pseudogout

Click here to see the answer

A. Rheumatoid arthritis – Patients with advanced rheumatoid arthritis will have synovitis of the atlantoaxial joint (C1-C2) which may damage the transverse ligament of the atlas, producing a forward displacement of the atlas on the axis (atlantoaxial subluxation).

B. Although patients with osteoarthritis may have neck pain and stiffness, there is no predilection for the atlantoaxial joints.
C. Patients with gout are likely to have involvement of peripheral joints rather than spinal joints.
D. Patients with pseudogout are more likely to have involvement of the knees, wrist, shoulder, ankle, elbow, and hands rather than the cervical spine.

Review NCCPA Blueprint Topic: Rheumatoid arthritis (Lecture) (Lesson)

3. Which of the following interventions is the treatment of choice for an actinic keratosis?

A. Mohs surgery
B. Cryotherapy
C. Acid peels
D. Radiation therapy

Click here to see the answer

B. Cryotherapy – Cryotherapy is the treatment of choice for isolated superficial actinic keratosis.

A. Mohs surgery and radiation therapy are not indicated in the treatment for actinic keratosis.
C. Acid peels can be used to treat actinic keratosis but are not the treatment of choice.

Review NCCPA Blueprint Topic: Actinic keratosis

4. A 55-year-old secretary presents with ongoing pain and numbness in her hand. These symptoms are worse at night and she must shake her hand to regain feeling in it. Which of the following physical examination signs will be
present?

A. Hypothenar atrophy
B. Weakness of finger abduction
C. Inability to maintain wrist extension against resistance
D. Weakness of thumb abduction

Click here to see the answer

D. A weakness of thumb abduction – Median nerve injury causes weakness of thumb abduction (measured by thumb opposition strength) along with thenar atrophy. Tinel’s and Phalen’s signs will also be positive with carpal tunnel syndrome.

A. Hypothenar atrophy may occur with aging and disuse but it is not part of the median nerve involvement that occurs with carpal tunnel syndrome.
B. Finger abduction weakness is associated with ulnar nerve injury, which does not occur with carpal tunnel syndrome.
C. Radial nerve injury causes weakness of wrist extension and this is not part of carpal tunnel syndrome.

Review NCCPA Blueprint Topic: Carpal Tunnel Syndrome (Lesson)

5. A 78-year-old male with history of coronary artery disease status post-CABG and ischemic cardiomyopathy presents with a complaint of progressive dyspnea and orthopnea. He also complains of lower extremity edema. The patient denies fever, chest pain, or cough. On physical examination, vital signs are BP 120/68, HR 75 and regular, RR 22, afebrile. You note the patient to have an S3 heart sound, jugular venous distention, and 2+ lower extremity edema. The patient is admitted and treated. Upon discharge from the hospital, the patient should be educated to monitor which of the following at home?

A. Daily weights
B. Daily spirometry
C. Daily blood glucose
D. Daily fat intake

Click here to see the answer

A. Daily weights – Home monitoring of daily weights can alert the health care provider to the early recognition of worsening heart failure.

B. Spirometry monitoring is important in a patient with asthma, not heart failure.
C. Daily blood glucose monitoring is important in a patient with diabetes, not heart failure.
D. Daily fat intake is important, but will not improve his heart failure management.

Review NCCPA Blueprint Topic: Heart Failure (ReelDx) (Lesson)

6. Which of the following rotator cuff tendons is most likely to sustain injury because of its repeated impingement (carpal tunnel syndrome) between the humeral head and the undersurface of the anterior third of the acromion and coracoacromial ligament?

A. Supraspinatus
B. Infraspinatus
C. Teres minor
D. Subscapularis

Click here to see the answer

A. Supraspinatus – A critical zone exists for the supraspinatus tendon due to its superior insertion site. It is susceptible for injury because it has a reduction in its blood supply that occurs with abduction of the arm. Impingement of the shoulder is most commonly seen with the supraspinatus tendon, the long head of the biceps tendon and/or the subacromial bursa.

Review NCCPA Blueprint topic: Soft tissue injuries of the forearm, wrist, and hand (ReelDx) (Lesson)

7. Which of the following diagnostic findings in the urinary sediment is specific for a diagnosis of chronic renal failure?

A. Hematuria
B. Proteinuria
C. Broad waxy casts
D. Hyaline casts

Click here to see the answer

C. Broad waxy casts – Broad waxy casts in urinary sediment are a specific finding in chronic renal failure.

A. Hematuria and proteinuria are frequent, but nonspecific, findings in chronic renal failure.
D. Hyaline casts may be found in normal urine or in states of dehydration.

Review NCCPA Blueprint Topic: Chronic kidney disease (Lesson)

8. In addition to tobacco products, which of the following is also considered a major risk factor in the development of oral cancer?

A. Sun exposure
B. Alcohol abuse
C. Occupational exposure
D. History of oral candidiasis

Click here to see the answer

B. Alcohol abuse – Major risk factors for the development of oral cancer are the use of tobacco products and alcohol abuse.

A. Sun exposure is a risk factor for cancer of the lip but is not considered a major risk factor for oral cancer.
C. While occupational exposures and the presence of premalignant lesions, such as leukoplakia, are risk factors for the development of oral cancer, they are not considered major risk factors.
D. History of oral candidiasis has no correlation to the development of oral cancer.

Review NCCPA Blueprint Topic: EENT Benign and Malignant Neoplasms (Lesson)

9. A 13-year-old boy with leukemia presents with epistaxis for 2 hours. The bleeding site appears to be from Kiesselbach’s area. The most appropriate intervention is

A. electrocautery of the bleeding site
B. silver nitrate application
C. posterior nasal packing
D. intranasal petrolatum gauze

Click here to see the answer

D. intranasal petrolatum gauze – Petrolatum gauze will provide pressure to the bleeding point while the cause of bleeding is corrected.

A. Cautery is not used because the edges of the cauterized area may begin to bleed.
B. Silver nitrate is not used in children because it increases the risk of nasal septal perforation.
C. Posterior nasal packing is indicated for posterior bleeds in the inferior meatus.

Review NCCPA Blueprint Topic: Epistaxis (ReelDx + Lecture) (Lesson)

10. A 2-year-old female presents with purulent nasal discharge bilaterally with fever and cough for several days. Her mom had taken her out of daycare for a similar occurrence 2 months ago, that was treated with Amoxicillin. Exam further reveals halitosis and periorbital edema. Treatment should be initiated with which of the following?

A. Antihistamines
B. Ribavirin (Rebetol)
C. Intranasal corticosteroids
D. Amoxicillin-clavulanate (Augmentin)

Click here to see the answer

D. Amoxicillin-clavulanate (Augmentin) – High dose amoxicillin-clavulanate is the treatment of choice for resistant bacterial sinusitis, especially in children presenting with risk factors (daycare attendance, previous antibiotic treatment 1-3 months prior, age younger than 2 years).

A. Antihistamines and intranasal corticosteroids have not been adequately studied in children to prove they make a difference in treating recurrent sinusitis.
B. Ribavirin is approved for the treatment of RSV infection.

Review NCCPA Blueprint Topic: Acute and chronic sinusitis (ReelDx + Lecture) (Lesson)

Looking for all the podcast episodes?

This FREE series is limited to every other episode, you can download and enjoy the complete audio series by joining The PANCE and PANRE Exam Academy + SMARTYPANCE

I will be releasing new episodes every few weeks. The Academy is discounted, so sign up now.

Resources and Links From The Show

This Podcast is also available on iTunes and Stitcher Radio for Android

  1. iTunes: The Audio PANCE AND PANRE Podcast iTunes
  2. Stitcher Radio: The Audio PANCE and PANRE Podcast Stitcher
itunes_logo-1

Download The Content Blueprint Checklist

Follow this link to download your FREE copy of the Content Blueprint Checklist

Print it up and start crossing out the topics you understand, marking the ones you don’t and making notes of key terms you should remember. The PDF version is interactive and linked directly to the individual lessons on SMARTY PANCE.

Download for PANCE Download for PANRE

The post Podcast Episode 69: Ten PANCE and PANRE Board Review Audio Questions appeared first on The Audio PANCE and PANRE.

]]> The Audio PANCE/PANRE Board Review Exam Questions Welcome to episode 69 of the Audio PANCE and PANRE PA Board Review Podcast. Join me as I cover ten PANCE and PANRE Board review questions from the SMARTYPANCE course content following the NCCPA™ content...



The Audio PANCE/PANRE Board Review Exam Questions



Welcome to episode 69 of the Audio PANCE and PANRE PA Board Review Podcast.



Join me as I cover ten PANCE and PANRE Board review questions from the SMARTYPANCE course content following the NCCPA™ content blueprint (download the FREE cheat sheet).



This week we will be covering ten general board review questions based on the NCCPA PANCE and PANRE Content Blueprints. 



Below you will find an interactive exam to complement the podcast.



I hope you enjoy this free audio component to the examination portion of this site. The full board review includes over 2,000 interactive board review questions and is available to all members of the PANCE and PANRE Academy and Smarty PANCE.



* You can download and listen to past FREE episodes here, on iTunes, on Google Play Music or Stitcher Radio.* You can listen to the latest episode, take an interactive quiz and download your results below.



Listen Carefully Then Take The Practice Exam





If you can’t see the audio player click here to listen to the full episode.



Podcast Episode 69: 10 Question PANCE and PANRE Podcast Quiz



The following questions are linked to NCCPA Content Blueprint lessons from the Smarty PANCE and PANRE Board Review Website. If you are a member you will be able to log in and view this interactive video lesson.



1. Which of the following is the most common cause of acute myocardial infarction?



A. Occlusion caused by coronary microemboliB. Thrombus development at a site of vascular injuryC. Congenital abnormalitiesD. Severe coronary artery spasm





2. Endotracheal intubation should be performed with caution in patients with which of the following underlying conditions due to the propensity to cause subluxation of C1 on C2?



A. Rheumatoid arthritisB. OsteoarthritisC. GoutD. Pseudogout





3. Which of the following interventions is the treatment of choice for an actinic keratosis?



A. Mohs surgeryB.]]>
The Physician Assistant Life | Smarty PANCE full clean 18:48 335 Podcast Episode 67: Ten PANCE and PANRE Board Review Audio Questions http://podcast.thepalife.com/episode-67/ Fri, 21 Dec 2018 08:01:04 +0000 http://podcast.thepalife.com/?p=326 http://podcast.thepalife.com/episode-67/#respond http://podcast.thepalife.com/episode-67/feed/ 0 <p>The Audio PANCE/PANRE – Ten PA Board Review Exam Questions Welcome to episode 67 of the Audio PANCE and PANRE PA Board Review Podcast. Join me as I cover ten PANCE and PANRE Board review questions from the SMARTYPANCE course content following the NCCPA™ content blueprint (download the FREE cheat sheet). This week we will be covering ten general […]</p> <p>The post <a rel="nofollow" href="http://podcast.thepalife.com/episode-67/">Podcast Episode 67: Ten PANCE and PANRE Board Review Audio Questions</a> appeared first on <a rel="nofollow" href="http://podcast.thepalife.com">The Audio PANCE and PANRE</a>.</p> The Audio PANCE and PANRE Physician Assistant Board Review Podcast Episode 67
Ten Mixed PA Board Review Questions

The Audio PANCE/PANRE – Ten PA Board Review Exam Questions

Welcome to episode 67 of the Audio PANCE and PANRE PA Board Review Podcast.

Join me as I cover ten PANCE and PANRE Board review questions from the SMARTYPANCE course content following the NCCPA™ content blueprint (download the FREE cheat sheet).

This week we will be covering ten general board review questions based on the NCCPA PANCE and PANRE Content Blueprint. 

Below you will find an interactive exam to complement the podcast.

I hope you enjoy this free audio component to the examination portion of this site. The full board review includes over 2,000 interactive board review questions and is available to all members of the PANCE and PANRE Academy and SMARTYPANCE which are now bundled together into one very low price.

Listen Carefully Then Take The Practice Exam

If you can’t see the audio player click here to listen to the full episode.

Podcast Episode 67: 10 Question PANCE and PANRE Podcast Quiz

The following questions are linked to NCCPA Content Blueprint lessons from the SMARTYPANCE & PANRE Board Review Website. If you are a member you will be able to log in and view this interactive video lesson.

A 26-year-old female presents with several pruritic lesions on her dorsal forearms. The lesions are erythematous with vesicles, with a few beginning to weep. She works in a photography laboratory but denies any other possible exposures. Which of the following is the most useful diagnostic test?

A. VDRL serology
B. KOH prep
C. Patch testing
D. Gram’s stain

Click here to see the answer

C. Patch testing with a suspected agent is usually positive in cases of allergic contact dermatitis

A. VDRL serology is useful in the diagnosis of syphilis, not contact dermatitis
B. KOH prep is used for diagnosis of fungal infections, not contact dermatitis
D. Gram’s stain is useful in the diagnosis of bacterial infections, not contact dermatitis

2. An elderly female presents for evaluation of exertional syncope, dyspnea, and angina. She admits that previous to these symptoms she had insidious progression of fatigue that caused her to curtail her activities. Which of the following is the most likely diagnosis?

A. Aortic stenosis
B. Aortic regurgitation
C. Mitral stenosis
D. Mitral valve prolapse

Click here to see the answer

A. The major symptoms of aortic stenosis are exertional syncope, dyspnea, and angina. Symptoms do not become apparent for a number of years and usually are not present until the valve is narrowed to less than 0.5 cm to 2 cm of the valve surface area.

B. Patients with aortic regurgitation are likely to complain of an uncomfortable awareness of their heart, especially when lying down. These patients develop sinus tachycardia with exertion and complain of palpitations and head pounding with activity.
C. The symptoms related to mitral stenosis are related to increased pulmonary pressure after the left atrium can no longer overcome the outflow obstruction.
D. Patients with mitral valve prolapse are typically asymptomatic throughout their lives, although a wide range of
symptoms is possible. When symptoms do occur, palpitations from arrhythmias are most common along with lightheadedness. Syncope is not part of this disease process.

3. A 40-year-old female G5P5 complains of small quantities of urine leaking when she coughs, sneezes, or laughs. Her genitourinary examination is unremarkable and her urinalysis is normal. At this time, which of the following is the most appropriate management plan?

A. Refer for a cystoscopy
B. Recommend Kegel exercises
C. Refer for surgical correction
D. Recommend hormone replacement therapy

Click here to see the answer

B. Strengthening the pelvic muscles by Kegel exercises and emptying the bladder frequently may resolve the problem.

A. Conservative therapy for stress incontinence should be attempted prior to any evaluation, such as cystoscopy, that might indicate the need for surgical correction
D. There is no indication in the history for hormone replacement therapy and no vaginal atrophy was noted on pelvic examination

4. A 62-year-old male presents with complaints of vague epigastric abdominal pain associated with jaundice and generalized pruritus. Physical examination reveals jaundice and a palpable non-tender gallbladder but is otherwise unremarkable. Which of the following is the most likely diagnosis?

A. Viral hepatitis
B. Pancreatic cancer
C. Acute cholecystitis
D. Gilbert’s syndrome

Click here to see the answer

B. Pancreatic cancer is suggested by the vague epigastric pain with jaundice resulting from biliary obstruction due to cancer involving the pancreatic head. The presence of a palpable non-tender gallbladder (Courvoisier’s sign) also indicates obstruction due to the cancer

A. While viral hepatitis may cause jaundice, the liver is enlarged and tender
C. While acute cholecystitis may present with jaundice and an enlarged gallbladder, the pain is classically colicky and located in the right upper quadrant. On physical examination with deep inspiration and palpation of the right subcostal area increased pain and respiratory arrest (Murphy’s sign) is usually seen.
D. Gilbert’s syndrome is the most common of the hereditary hyperbilirubinemias. It is most often diagnosed near puberty or adult life based on results of a comprehensive metabolic panel.

5. Which of the following would you expect on physical examination in a patient with mitral valve stenosis?

A. Systolic blowing murmur
B. Opening snap
C. Mid-systolic click
D. Paradoxically split S2

Click here to see the answer

B. Mitral stenosis is characterized by a mid-diastolic opening snap

A. Mitral stenosis is a diastolic, not a systolic murmur
C. Mid-systolic clicks are noted in mitral valve prolapse, not mitral stenosis
D. Paradoxical splitting of S2 occurs in aortic stenosis, not mitral stenosis

Review PANCE and PANRE Valvular disorders pearls

6. A positive Wood’s light examination (fluorescence) demonstrates

A. viral infection with herpes zoster
B. bacterial infection with Treponema pallidum
C. parasitic infestation with Pediculus humanus
D. mycotic infection with Microsporum canis

Click here to see the answer

D. Microsporum causes tinea capitis and fluoresces blue-green under Wood’s light

A. Lesions of herpes, Treponema, and Pediculus infections do not fluoresce.

Review topic PANCE Dermatologic system dermatophyte infections

7. In which of the following patients would one most likely find acanthosis nigricans?

A. A 55-year-old obese female with hyperinsulinemia
B. A 55-year-old male with an enlarged spleen and pancytopenia
C. A 24-year-old female with increased Lyme titers
D. A 60-year-old male with increased triglycerides

Click here to see the answer

A. Acanthosis nigricans is associated with patients who have hyperinsulinemia

B. Hairy cell leukemia is associated with an enlarged spleen and pancytopenia. Acanthosis nigricans is not a cutaneous manifestation
C. Erythema chronicum migrans is the characteristic lesion associated with Lyme disease
D. Eruptive xanthelasma is associated with increased triglycerides

8. A 65-year-old male presents with back pain two days after he was shoveling snow. The patient complains of pain in his low back that radiates into his buttocks, posterior thigh and calf, and the bottom of his foot. There is associated numbness of the lateral and plantar surface of his foot. Which of the following disc herniations is most likely to be affected?

A. L3-L4
B. L4-L5
C. L5-S1
D. S1-S2

Click here to see the answer

C. The S1 nerve root impingement is most likely to occur from the herniation of the L5-S1 disc space. The S1 disc affects Achilles’ reflex, the gastrocnemius and soleus muscles, and the abductor hallucis and gluteus maximus muscles.

9. A 22-year-old female presents to the emergency department with a rapid heart rate. She appears quite thin and dehydrated. She denies that she is thin, stating “I am so fat that I can hardly stand myself! That is why I exercise every day.” She runs twelve to fifteen miles a day, and on weekends also bicycles forty to fifty miles. Her LMP was six months ago. On exam, she is 5′ 6″ tall and weighs 98 pounds. Temp 98 degrees F, pulse 100, respirations 18, BP 98/60. EKG shows sinus tachycardia. Laboratory findings include Na 138 mEq/L, K 2.8 mEq/L, Cl 91 mEq/L, BUN 35 mg/dL, Creatinine 1.1 mg/dL. Which of the following is the next most appropriate treatment?

A. Propylthiouracil (PTU) and individual psychotherapy
B. Hormone replacement and cognitive therapy
C. Weight restoration and family therapy
D. IV hydration and antidepressant therapy

Click here to see the answer

C. Anorexia nervosa requires a comprehensive, multidisciplinary approach to treatment that integrates medical management, individual psychotherapy, and family therapy. Currently, the best results have been shown with weight restoration accompanied by family therapy for patients with adolescent-onset anorexia nervosa and individual therapy for patients with onset after 18 years of age. Inpatient treatment is often required.

A. Propylthiouracil is used to treat hyperthyroidism, not anorexia nervosahttps://smartypance.com/lessons/ear-disorders/inner-ear-pearls/acoustic-neuroma/
B. Hormone replacement is not indicated for the amenorrhea of anorexia nervosa, but nutritional support may help.
D. Antidepressant therapy may be useful in bulimia nervosa

10. A 52-year-old female presents with complaints of intermittent episodes of dizziness, tinnitus, and hearing loss in the right ear for 6 months. She describes the dizziness as the “room spinning around her,” with the episodes typically lasting for 2 to 4 hours. Physical examination reveals horizontal nystagmus and right ear hearing loss, but the remainder of the examination is unremarkable. Which of the following is the most likely diagnosis?

A. Acute labyrinthitis
B. Positional vertigo
C. Acoustic neuroma
D. Ménière’s syndrome

Click here to see the answer

D. Ménière’s syndrome uhttps://smartypance.com/lessons/ear-disorders/inner-ear-pearls/vertigo-reeldx294/sually presents with episodes of vertigo that last from 1 to 8 hours, sensorineural hearing loss and tinnitus.

A. Acute labyrinthitis typically presents with an acute onset of continuous vertigo that lasts several days to a week and is associated with nausea and vomiting. It does not have any associated auditory or neurologic symptoms.
B. Positional vertigo occurs following changes in head positioning with very brief, less than 1 minute, episodes. Nystagmus occurs following the position change.
C. Acoustic neuroma typically presents with hearing loss and tinnitus. The neuroma grows slowly and central compensatory mechanisms can prevent or minimize the vertigo. Vertigo, when present, is continuous and not episodic.

Looking for all the podcast episodes?

This FREE series is limited to every other episode, you can download and enjoy the complete audio series by joining The PANCE and PANRE Exam Academy + SMARTYPANCE

I will be releasing new episodes every few weeks. The Academy is discounted, so sign up now.

Resources and Links From The Show

This Podcast is also available on iTunes and Stitcher Radio for Android

  1. iTunes: The Audio PANCE AND PANRE Podcast iTunes
  2. Stitcher Radio: The Audio PANCE and PANRE Podcast Stitcher
itunes_logo-1

Download The Content Blueprint Checklist

Follow this link to download your FREE copy of the Content Blueprint Checklist

Print it up and start crossing out the topics you understand, marking the ones you don’t and making notes of key terms you should remember. The PDF version is interactive and linked directly to the individual lessons on SMARTY PANCE.

pance-and-panre-nccpa-content-blueprint

Download

The post Podcast Episode 67: Ten PANCE and PANRE Board Review Audio Questions appeared first on The Audio PANCE and PANRE.

]]>
The Audio PANCE/PANRE – Ten PA Board Review Exam Questions Welcome to episode 67 of the Audio PANCE and PANRE PA Board Review Podcast. Join me as I cover ten PANCE and PANRE Board review questions from the SMARTYPANCE course content following the NCCPA... Ten Mixed PA Board Review Questions



The Audio PANCE/PANRE – Ten PA Board Review Exam Questions



Welcome to episode 67 of the Audio PANCE and PANRE PA Board Review Podcast.



Join me as I cover ten PANCE and PANRE Board review questions from the SMARTYPANCE course content following the NCCPA™ content blueprint (download the FREE cheat sheet).



This week we will be covering ten general board review questions based on the NCCPA PANCE and PANRE Content Blueprint. 



Below you will find an interactive exam to complement the podcast.



I hope you enjoy this free audio component to the examination portion of this site. The full board review includes over 2,000 interactive board review questions and is available to all members of the PANCE and PANRE Academy and SMARTYPANCE which are now bundled together into one very low price.



* You can download and listen to past FREE episodes here, on iTunes, on Google Play Music or Stitcher Radio.* You can listen to the latest episode, take an interactive quiz and download your results below.



Listen Carefully Then Take The Practice Exam





If you can’t see the audio player click here to listen to the full episode.



Podcast Episode 67: 10 Question PANCE and PANRE Podcast Quiz



The following questions are linked to NCCPA Content Blueprint lessons from the SMARTYPANCE & PANRE Board Review Website. If you are a member you will be able to log in and view this interactive video lesson.



A 26-year-old female presents with several pruritic lesions on her dorsal forearms. The lesions are erythematous with vesicles, with a few beginning to weep. She works in a photography laboratory but denies any other possible exposures. Which of the following is the most useful diagnostic test?



A. VDRL serologyB. KOH prepC.]]>
The Physician Assistant Life | Smarty PANCE full clean 18:58 326
Podcast Episode 65: Hepatitis B Breakdown With Joe Gilboy PA-C http://podcast.thepalife.com/podcast-episode-65-hepatitis-b-breakdown-with-joe-gilboy-pa-c/ Mon, 01 Oct 2018 23:38:14 +0000 http://podcast.thepalife.com/?p=314 http://podcast.thepalife.com/podcast-episode-65-hepatitis-b-breakdown-with-joe-gilboy-pa-c/#respond http://podcast.thepalife.com/podcast-episode-65-hepatitis-b-breakdown-with-joe-gilboy-pa-c/feed/ 0 <p>NCCPA™ PANCE Content Blueprint – The ABC’s of Hepatitis +The Hepatitis B Breakdown Welcome to episode 65 of the Audio PANCE and PANRE PA Board Review Podcast. In today’s episode, I welcome my friend, exclusive smartypance lecturer, and PA board review extraordinaire Joe Gilboy PA-C who will be breaking down Hepatitis B as you’ve never seen […]</p> <p>The post <a rel="nofollow" href="http://podcast.thepalife.com/podcast-episode-65-hepatitis-b-breakdown-with-joe-gilboy-pa-c/">Podcast Episode 65: Hepatitis B Breakdown With Joe Gilboy PA-C</a> appeared first on <a rel="nofollow" href="http://podcast.thepalife.com">The Audio PANCE and PANRE</a>.</p> The Audio PANCE and PANRE Episode 65 Hepatitis B Breakdown with Joe Gilboy PA-C

NCCPA™ PANCE Content Blueprint – The ABC’s of Hepatitis +The Hepatitis B Breakdown

Welcome to episode 65 of the Audio PANCE and PANRE PA Board Review Podcast.

In today’s episode, I welcome my friend, exclusive smartypance lecturer, and PA board review extraordinaire Joe Gilboy PA-C who will be breaking down Hepatitis B as you’ve never seen it before.

This is a recorded lecture featured on my smartypance.com board review website covering Hepatitis A, B, and C testing with twelve follow up questions at the end.

Below you will find an interactive exam to complement the podcast.

I hope you enjoy this free audio component to the examination portion of the site. The full board review covers the entire NCCPA Content Blueprint, all indexed, searchable, and now available with Joe’s Gilboy’s priceless lectures.

To top it off you can sign up today for the free 60-day board review email series which includes Joe’s audio explanations.

If you enjoyed this podcast episode:

Listen Carefully Then Take The Practice Exam

If you can’t see the audio player click here to listen to the full episode.

Podcast Episode 65: Hepatitis PANCE/PANRE Podcast Quiz

Episode 65 The Audio PANCE and PANRE Hepatitis B Breakdown

The following questions are linked to NCCPA Content Blueprint lessons from the SMARTYPANCE & PANRE Board Review Website. If you are a member you will be able to log in and view this interactive video lesson.

1. A patient presents wanting to be tested to see if they presently have Hepatitis A which labs would you order?

a. Anti-HAV (IgG)
b. Anti-HAV (IgA)
c. Anti-HAV (IgM)
d. Anti-HAV (IgE)

Click here to see the answer

c. Anti-HAV (IgM)

2. Which lab result would a PA and an IV drug abuser both have that are positive?

a. Anti-HBs
b. Anti-HBe
c. Anti-HBc
d. HBeAg

Click here to see the answer

a. Anti-HBs

3. Which positive lab tells you a person is currently actively infected with hepatitis B?

a. HBsAg
b. HBsAb
c. HBcAg
d. HBeAg

Click here to see the answer

d. HBeAg

4. Which of the following lab result would a chronic hepatitis B patient have that is positive?

a. HBsAg
b. HBsAb
c. HBcAg
d. HBeAg

Click here to see the answer

a. HBsAg

5. Immunoglobulin is given within two weeks for family members and close contacts when they are exposed to hepatitis A. Which immunoglobulin is given?

a. IgM
b. IgG
c. IgA
d. IgE

Click here to see the answer

b. IgG

6. Hepatitis D needs what part of hepatitis B to exist?

a. HBsAg
b. HBsAb
c. HBcAg
d. HBeAg

Click here to see the answer

a. HBsAg

7. Hepatitis E is transmitted by which vector?

a. IV drug abuse
b. Homosexual sex
c. Fecal-oral
d. Blood transfusion

Click here to see the answer

c. Fecal-oral

8. What is the 1st evidence of infection with hepatitis B?

a. HBcAg
b. HBeAg
c. Anti-HBs
d. HBsAg

Click here to see the answer

c. Anti-HBs

9. What is the lab test to indicate a diagnosis of acute hepatitis B?

a. Anti-HBs
b. Anti-HBe
c. Anti-HBc
d. HBeAg

Click here to see the answer

c. Anti-HBc

10. What is the lab test that indicates viral replication and infectious Hepatitis B?

a. HBcAg
b. HBeAg
c. Anti-HBs
d. HBsAg

Click here to see the answer

b. HBeAg

11. Which of the following is the first sign of a patient with an elevated conjugated bilirubin (direct)?

a. Icteric sclera
b. Jaundice
c. Acholic stool
d. Tea-colored urine

Click here to see the answer

d. Tea-colored urine

12. A 52-yo male with upper abdominal pain and an elevated aminotransferase(AST) >10,000 u/l is consistent with which problem?

a. Gallstones
b. Alcoholic liver disease
c. Bile duct stones
d. Viral Hepatitis

Click here to see the answer

d. Viral Hepatitis

Looking for all the podcast episodes?

This FREE series is limited to every other episode, you can download and enjoy the complete audio series by joining The PANCE and PANRE Exam Academy + SMARTYPANCE

I will be releasing new episodes every few weeks. The Academy is discounted, so sign up now.

Resources and Links From The Show

This Podcast is also available on iTunes and Stitcher Radio for Android

  1. iTunes: The Audio PANCE AND PANRE Podcast iTunes
  2. Stitcher Radio: The Audio PANCE and PANRE Podcast Stitcher

itunes_logo-1

Download The Content Blueprint Checklist

Follow this link to download your FREE copy of the Content Blueprint Checklist

Print it up and start crossing out the topics you understand, marking the ones you don’t and making notes of key terms you should remember. The PDF version is interactive and linked directly to the individual lessons on SMARTY PANCE.

pance-and-panre-nccpa-content-blueprint

Download

The post Podcast Episode 65: Hepatitis B Breakdown With Joe Gilboy PA-C appeared first on The Audio PANCE and PANRE.

]]>
NCCPA™ PANCE Content Blueprint – The ABC’s of Hepatitis +The Hepatitis B Breakdown Welcome to episode 65 of the Audio PANCE and PANRE PA Board Review Podcast. In today’s episode, I welcome my friend, exclusive smartypance lecturer,
NCCPA™ PANCE Content Blueprint – The ABC’s of Hepatitis +The Hepatitis B Breakdown
Welcome to episode 65 of the Audio PANCE and PANRE PA Board Review Podcast.
In today’s episode, I welcome my friend, exclusive smartypance lecturer, and PA board review extraordinaire Joe Gilboy PA-C who will be breaking down Hepatitis B as you’ve never seen it before.
This is a recorded lecture featured on my smartypance.com board review website covering Hepatitis A, B, and C testing with twelve follow up questions at the end.
Below you will find an interactive exam to complement the podcast.
I hope you enjoy this free audio component to the examination portion of the site. The full board review covers the entire NCCPA Content Blueprint, all indexed, searchable, and now available with Joe’s Gilboy’s priceless lectures.
To top it off you can sign up today for the free 60-day board review email series which includes Joe’s audio explanations.
If you enjoyed this podcast episode:

* You can download and listen to past FREE episodes here, on iTunesGoogle Play Music, or Stitcher Radio.

Listen Carefully Then Take The Practice Exam
If you can’t see the audio player click here to listen to the full episode.
Podcast Episode 65: Hepatitis PANCE/PANRE Podcast Quiz

The following questions are linked to NCCPA Content Blueprint lessons from the SMARTYPANCE & PANRE Board Review Website. If you are a member you will be able to log in and view this interactive video lesson.
1. A patient presents wanting to be tested to see if they presently have Hepatitis A which labs would you order?
a. Anti-HAV (IgG)
b. Anti-HAV (IgA)
c. Anti-HAV (IgM)
d. Anti-HAV (IgE)
2. Which lab result would a PA and an IV drug abuser both have that are positive?
a. Anti-HBs
b. Anti-HBe
c. Anti-HBc
d. HBeAg
3. Which positive lab tells you a person is currently actively infected with hepatitis B?
a. HBsAg
b. HBsAb
c. HBcAg
d. HBeAg
4. Which of the following lab result would a chronic hepatitis B patient have that is positive?
a. HBsAg
b. HBsAb
c. HBcAg
d. HBeAg
5. Immunoglobulin is given within two weeks for family members and close contacts when they are exposed to hepatitis A. Which immunoglobulin is given?
a. IgM
b. IgG
c. IgA
d. IgE
6. Hepatitis D needs what part of hepatitis B to exist?
a. HBsAg
b. HBsAb
c.]]>
The Physician Assistant Life | Smarty PANCE full clean 22:26 314
Episode 63: The Audio PANCE and PANRE – Ten Mixed NCCPA™ Content Blueprint Multiple Choice Questions http://podcast.thepalife.com/episode-63-the-audio-pance-and-panre-ten-mixed-nccpa-content-blueprint-multiple-choice-questions/ Thu, 09 Aug 2018 16:40:58 +0000 http://podcast.thepalife.com/?p=310 http://podcast.thepalife.com/episode-63-the-audio-pance-and-panre-ten-mixed-nccpa-content-blueprint-multiple-choice-questions/#respond http://podcast.thepalife.com/episode-63-the-audio-pance-and-panre-ten-mixed-nccpa-content-blueprint-multiple-choice-questions/feed/ 0 <p>Ten Mixed NCCPA™ PANCE Content Blueprint Multiple Choice Questions Welcome to episode 63 of the FREE Audio PANCE and PANRE Physician Assistant Board Review Podcast. Join me as I cover ten PANCE and PANRE Board review questions from the SMARTYPANCE course content following the NCCPA™ content blueprint (download the FREE cheat sheet). This week we will be covering ten […]</p> <p>The post <a rel="nofollow" href="http://podcast.thepalife.com/episode-63-the-audio-pance-and-panre-ten-mixed-nccpa-content-blueprint-multiple-choice-questions/">Episode 63: The Audio PANCE and PANRE – Ten Mixed NCCPA™ Content Blueprint Multiple Choice Questions</a> appeared first on <a rel="nofollow" href="http://podcast.thepalife.com">The Audio PANCE and PANRE</a>.</p> The Audio PANCE and PANRE Episode 63 Ten Mixed Multiple Choice Questions

Ten Mixed NCCPA™ PANCE Content Blueprint Multiple Choice Questions

Welcome to episode 63 of the FREE Audio PANCE and PANRE Physician Assistant Board Review Podcast.

Join me as I cover ten PANCE and PANRE Board review questions from the SMARTYPANCE course content following the NCCPA™ content blueprint (download the FREE cheat sheet).

This week we will be covering ten general board review questions based on the NCCPA PANCE and PANRE Content Blueprint. 

Below you will find an interactive exam to complement the podcast.

I hope you enjoy this free audio component to the examination portion of this site. The full board review includes over 2,000 interactive board review questions and is available to all members of the PANCE and PANRE Academy and SMARTYPANCE which are now bundled together into one very low price.

Listen Carefully Then Take The Practice Exam

If you can’t see the audio player click here to listen to the full episode.

Episode 63 – General PANCE/PANRE Podcast Quiz

The following 10 questions are linked to NCCPA Content Blueprint lessons from the SMARTYPANCE and PANRE Board review website. If you are a member you will be able to log in and view this interactive video content.

1. A 24-year-old male presents complaining of a 9-month history of increasing shortness of breath, dyspnea on exertion, and a cough productive of white sputum, mostly in the mornings. He denies orthopnea, PND, peripheral edema, fever, chills, night sweats, recent changes in weight, palpitations, chest pain, food intolerances, or other complaints. Patient has a history of recurrent lung infections. He states that his father had chronic pulmonary problems and died at age 42 from unknown lung disease. The patient denies smoking, alcohol or illicit drug use. On physical examination, the respiratory rate is 22 breaths per minute, a pulse of 98 bpm, a temperature of 98.7 degrees. Pulmonary exam reveals end-expiratory wheezes bilaterally and hyperresonance to percussion. His cardiac exam is normal. Chest X-ray shows decreased lung markings. ECG is normal. Pulmonary function tests show an FEV1 63% of expected and residual capacity is 123% of expected. Which of the following is the most likely diagnosis?

A. Emphysema
B. Pulmonary fibrosis
C. Ventricular septal defect
D. Congestive heart failure

Click here to see the answer

Answer: A. Emphysema

This person has an obstructive lung disease based on PFTs. Emphysema is the most likely diagnosis and may be related to alpha-1 antitrypsin deficiency based on family history and lack of smoking history and young age.

Emphysema is covered as part of the NCCPA Pulmonary Blueprint (12%)

B. The PFTs from a person with pulmonary fibrosis would be consistent with a restrictive pattern. This patient has an obstructive pattern of lung disease.
C. Ventricular septal defect will have a systolic murmur associated with it.
D. Congestive heart failure might explain some of the symptoms of this patient (increasing shortness of breath and DOE), he denies other common symptoms, such as orthopnea and peripheral edema. CHF should not result in changes in the PFTs.

2. A male patient complains of chronic dysuria, frequency, and urgency with associated perineal pain. The most likely diagnosis is

A. cystitis
B. gonococcal urethritis
C. epididymitis
D. prostatitis

Click here to see the answer

Answer: D. prostatitis

Some patients are asymptomatic, but low back or perineal pain, fever, chills, and irritative urinary symptoms are common in prostatitis.

Prostatitis is covered as part of the NCCPA PANCE Genitourinary Blueprint

A. Cystitis is characterized by dysuria without urethral discharge.
B. Initially, there is burning on urination and serous or milky discharge in gonococcal urethritis.
C. Epididymitis is characterized by dysuria, unilateral scrotal pain, and swelling.

3. Which of the following preventive strategies against osteoporosis-associated vertebral fractures has a known side effect of increasing the incidence of hot flashes when used in a perimenopausal female?

A. Calcitonin (Miacalcin) nasal spray
B. Alendronate (Fosamax)
C. Estrogen/progesterone (Prempro)replacement
D. Raloxifene (Evista)

Osteoporosis is covered as part of the PANCE Musculoskeletal Blueprint

Click here to see the answer

Answer: D. Raloxifene (Evista)

Raloxifene has effects on bone turnover and bone mass and has been shown to decrease vertebral fractures. It has anti-estrogen effects on the non-skeletal portions of the body and increases hot flashes in perimenopausal females.

A. Calcitonin does not have any estrogen effects on the body and serves as an analgesic when used in the management of vertebral fractures.
B. Alendronate is a bisphosphonate that does not have any hormonal effects on the body.
C. Estrogen/progesterone replacement has the benefit of maintaining bone and decreasing vertebral fractures but it would improve perimenopausal hot flashes.

4. A 70-year-old presents with a headache and neck stiffness. On physical exam, the patient is febrile, Kernig’s sign is present, and no rash is noted. A spinal tap reveals a white count of 250/cm3 with 100% neutrophils, total protein 250mg/dL, and glucose 35 mg/dL. Which of the following is the most appropriate treatment?

A. Acyclovir (Zovirax)
B. Fluconazole (Diflucan)
C. Ampicillin and ceftriaxone (Rocephin)
D. Penicillin and chloramphenicol (Chloromycetin)

Bacterial meningitis is covered as part of the PANCE Neurology Blueprint

Click here to see the answer

Answer: C. Ampicillin and ceftriaxone (Rocephin)

Ampicillin and ceftriaxone are used to treat bacterial meningitis, secondary to Listeria monocytogenes, which is common in the elderly. Ceftriaxone will cover other common etiologic agents such as Streptococcal pneumonia

A. Acyclovir is used to treat meningitis secondary to herpes. Viral meningitis presents with an increased number of lymphocytes and elevated glucose in the CSF.
B. Fluconazole is used to treat fungal meningitis. Fungal meningitis, typically noted in immunocompromised hosts, presents with an increased number of lymphocytes in the CSF.
D. Penicillin and chloramphenicol is used to treat bacterial meningitis, secondary to Neisseria meningitidis. Bacterial meningitis due to N

5. A 45-year-old patient with type 1 diabetes mellitus is being screened for diabetic nephropathy. Which of the following urinalysis findings is most consistent with early diabetic nephropathy?

A. Microalbuminuria
B. Red cell casts
C. White cell casts
D. Renal epithelial cells

Diabetic neuropathy is covered as part of the PANCE Endocrinology Blueprint

Click here to see the answer

Answer: A. Microalbuminuria

Microalbuminuria is most consistent with early diabetic neuropathy.

B. Red cell casts are more indicative of acute glomerular nephritis.
C. White cell casts are more consistent with acute pyelonephritis.
D. A few renal epithelial cells normally may be found in the urine.

6. A 74-year-old female is being treated for mild hypertension. She is found at home with right hemiparesis and brought to the emergency department. Her daughter states that the patient fell in her kitchen 2 days ago, but had no complaints at that time. She did state that her mother sounded a little confused this morning. The patient’s left pupil is dilated. Which of the following diagnostic studies should be ordered first?

A. MRI of the brain
B. CT scan of the brain
C. Skull x-ray
D. Lumbar puncture

Click here to see the answer

Answer:  B. CT scan of the brain

This patient presents with a history of minor trauma and progressive neurological abnormalities consistent with subdural hematoma. Diagnosis would be confirmed by CT scan, which is less expensive and more sensitive for blood than an MRI.

Intracranial Hemorrhage is covered as part of the PANCE Neurology Blueprint

C. Skull x-rays would not be helpful because they evaluate bony, not soft tissue, injury.
D. A lumbar puncture is contraindicated because of the potential for brain herniation.

7. A post-op patient has signs and symptoms highly suggestive of a pulmonary embolism. The results of the CT scan of the lung is nondiagnostic. What is the most appropriate next step in the evaluation?

A. Ventilation-perfusion (V/Q) scan
B. Ultrasound of the legs
C. Echocardiography
D. D-dimer

Pulmonary embolism and is part of the PANCE Pulmonary Blueprint

Click here to see the answer

Answer: B. Ultrasound of the legs

In a patient with a high likelihood of pulmonary embolism or an inpatient, as in this case, ultrasound of the legs would be the next diagnostic step after a nondiagnostic CT.

A. Ventilation-perfusion scans are performed prior to the CT scan of the chest and would not likely add additional information to this clinical scenario.
C. Although echocardiography may show right ventricular free wall hypokinesis with normal motion of the apex suggestive of pulmonary embolism, more than 50% of patients with a pulmonary embolism will have normal echocardiography. Echocardiography is not used in the diagnosis of inpatients.
D. In a post-op patient, a d-dimer will be positive regardless of the presence or absence of a pulmonary embolism.

8. Seizures that first manifest in early to middle adult life should be considered suspicious of which of the following causes?

A. Cerebrovascular disease
B. Encephalitis
C. Tumor
D. Idiopathic epilepsy

Seizure disorders are covered as part of the PANCE Neurology Blueprint

Click here to see the answer

Answer: C. Tumor

Seizures that develop during adolescence and adult life are predominantly due to tumor, trauma, drug use, or alcohol withdrawal.

9. Which of the following medications used in the treatment of supraventricular tachycardia is able to cause sinus arrest and asystole for a few seconds while it breaks the paroxysmal supraventricular tachycardia?

A. Digoxin (Lanoxin)
B. Adenosine (Adenocard)
C. Verapamil (Calan)
D. Quinidine (Quinaglute)

Paroxysmal supraventricular tachycardia is covered as part of the PANCE Cardiology Blueprint

Click here to see the answer

Answer: B. Adenosine (Adenocard)

Adenosine is an endogenous nucleoside that results in profound (although transient) slowing of the AV conduction and sinus node discharge rate. This agent has a very short half-life of 6 seconds.

A. Digoxin is not used for the acute termination of supraventricular tachycardia.
C. Although verapamil may be used for the termination of acute supraventricular tachycardia, it does not lead to sinus arrest in therapeutic doses.
D. Quinidine is rarely used today and is not indicated for the termination of supraventricular tachycardia.

10. A 32-year-old presents with a 3-day history of diarrhea. The patient denies blood, mucus, or night awakening with diarrhea. He recently returned from a business trip to Canada. On physical examination, the patient is afebrile and vital signs reveal BP 115/80, the pulse is 76, and respirations are 14. The abdominal examination reveals hyperactive bowel sounds but is otherwise unremarkable. Which of the following is the most appropriate initial intervention?

A. Stool for culture, ova, and parasites
B. Proctosigmoidoscopy
C. Metronidazole (Flagyl)
D. Supportive treatment

Infectious and Noninfectious Diarrhea are covered as part of the NCCPA PANCE GI and Nutrition Blueprint

Click here to see the answer

Answer: D. Supportive treatment

Symptomatic treatment, including dietary management and over-the-counter antidiarrheals, is indicated for afebrile patients with watery diarrhea of less than 5 days duration.

A. Stool culture and examination for ova and parasites are indicated when diarrhea has persisted longer than 3 weeks or is associated with abdominal pain, fever, and/or bloody stools.
B. Proctosigmoidoscopy is indicated when inflammatory bowel disease is suspected on the basis of fever, bloody diarrhea, or abdominal pain.
C. Metronidazole is indicated with a confirmed diagnosis of Giardia lamblia or amebic disease.

Looking for all the podcast episodes?

This FREE series is limited to every other episode, you can download and enjoy the complete audio series by joining The PANCE and PANRE Exam Academy + SMARTYPANCE

I will be releasing new episodes every few weeks. The Academy is discounted, so sign up now.

Resources and Links From The Show

This Podcast is also available on iTunes and Stitcher Radio for Android

  1. iTunes: The Audio PANCE AND PANRE Podcast iTunes
  2. Stitcher Radio: The Audio PANCE and PANRE Podcast Stitcher

itunes_logo-1

Download The Content Blueprint Checklist

Follow this link to download your FREE copy of the Content Blueprint Checklist

Print it up and start crossing out the topics you understand, marking the ones you don’t and making notes of key terms you should remember. The PDF version is interactive and linked directly to the individual lessons on SMARTY PANCE.

pance-and-panre-nccpa-content-blueprint

Download

The post Episode 63: The Audio PANCE and PANRE – Ten Mixed NCCPA™ Content Blueprint Multiple Choice Questions appeared first on The Audio PANCE and PANRE.

]]>
Ten Mixed NCCPA™ PANCE Content Blueprint Multiple Choice Questions Welcome to episode 63 of the FREE Audio PANCE and PANRE Physician Assistant Board Review Podcast. Join me as I cover ten PANCE and PANRE Board review questions from the SMARTYPANCE cour...
Ten Mixed NCCPA™ PANCE Content Blueprint Multiple Choice Questions
Welcome to episode 63 of the FREE Audio PANCE and PANRE Physician Assistant Board Review Podcast.
Join me as I cover ten PANCE and PANRE Board review questions from the SMARTYPANCE course content following the NCCPA™ content blueprint (download the FREE cheat sheet).
This week we will be covering ten general board review questions based on the NCCPA PANCE and PANRE Content Blueprint. 
Below you will find an interactive exam to complement the podcast.
I hope you enjoy this free audio component to the examination portion of this site. The full board review includes over 2,000 interactive board review questions and is available to all members of the PANCE and PANRE Academy and SMARTYPANCE which are now bundled together into one very low price.

* You can download and listen to past FREE episodes here, on iTunes, on Google Play Music or Stitcher Radio.
* You can listen to the latest episode, take an interactive quiz and download your results below.

Listen Carefully Then Take The Practice Exam
If you can’t see the audio player click here to listen to the full episode.
Episode 63 – General PANCE/PANRE Podcast Quiz

The following 10 questions are linked to NCCPA Content Blueprint lessons from the SMARTYPANCE and PANRE Board review website. If you are a member you will be able to log in and view this interactive video content.
1. A 24-year-old male presents complaining of a 9-month history of increasing shortness of breath, dyspnea on exertion, and a cough productive of white sputum, mostly in the mornings. He denies orthopnea, PND, peripheral edema, fever, chills, night sweats, recent changes in weight, palpitations, chest pain, food intolerances, or other complaints. Patient has a history of recurrent lung infections. He states that his father had chronic pulmonary problems and died at age 42 from unknown lung disease. The patient denies smoking, alcohol or illicit drug use. On physical examination, the respiratory rate is 22 breaths per minute, a pulse of 98 bpm, a temperature of 98.7 degrees. Pulmonary exam reveals end-expiratory wheezes bilaterally and hyperresonance to percussion.]]>
The Physician Assistant Life | Smarty PANCE full clean 17:38 310
Episode 61: The Audio PANCE and PANRE – Ten Mixed NCCPA™ Content Blueprint Multiple Choice Questions http://podcast.thepalife.com/episode-61-the-audio-pance-and-panre-ten-mixed-nccpa-content-blueprint-multiple-choice-questions/ Mon, 14 May 2018 07:53:07 +0000 http://podcast.thepalife.com/?p=303 http://podcast.thepalife.com/episode-61-the-audio-pance-and-panre-ten-mixed-nccpa-content-blueprint-multiple-choice-questions/#respond http://podcast.thepalife.com/episode-61-the-audio-pance-and-panre-ten-mixed-nccpa-content-blueprint-multiple-choice-questions/feed/ 0 <p>Ten Mixed NCCPA™ Content Blueprint Multiple Choice Questions Welcome to episode 61 of the FREE Audio PANCE and PANRE Physician Assistant Board Review Podcast. Join me as I cover ten PANCE and PANRE Board review questions from the SMARTYPANCE course content following the NCCPA™ content blueprint (download the FREE cheat sheet). This week we will be covering ten general […]</p> <p>The post <a rel="nofollow" href="http://podcast.thepalife.com/episode-61-the-audio-pance-and-panre-ten-mixed-nccpa-content-blueprint-multiple-choice-questions/">Episode 61: The Audio PANCE and PANRE – Ten Mixed NCCPA™ Content Blueprint Multiple Choice Questions</a> appeared first on <a rel="nofollow" href="http://podcast.thepalife.com">The Audio PANCE and PANRE</a>.</p> Ten Mixed NCCPA™ Content Blueprint Multiple Choice Questions

Welcome to episode 61 of the FREE Audio PANCE and PANRE Physician Assistant Board Review Podcast.

Episode 61 The Audio PANCE and PANRE Board Review PodcastJoin me as I cover ten PANCE and PANRE Board review questions from the SMARTYPANCE course content following the NCCPA™ content blueprint (download the FREE cheat sheet).

This week we will be covering ten general board review questions based on the NCCPA PANCE and PANRE Content Blueprint. 

Below you will find an interactive exam to complement the podcast.

I hope you enjoy this free audio component to the examination portion of this site. The full board review includes over 2,000 interactive board review questions and is available to all members of the PANCE and PANRE Academy and SMARTYPANCE which are now bundled together into one very low price.

Listen Carefully Then Take The Practice Exam

If you can’t see the audio player click here to listen to the full episode.

Episode 5 – General PANCE/PANRE Podcast Quiz

The following 10 questions are linked to NCCPA Content Blueprint lessons from the SMARTYPANCE and PANRE Board review website. If you are a member you will be able to log in and view this interactive video content.

1. An 18-year-old male presents with pain in his wrist after he fell off of a moving motorcycle. Physical examination reveals tenderness in the anatomic snuffbox. No fracture is noted on plain radiography of the wrist. Which of the following is the recommended treatment for this patient?

A. An ace wrap of the wrist
B. Closed reduction of the fracture site
C. Thumb spica cast application
D. Open reduction of the fracture site

Click here to see the answer

Answer: C. Thumb spica cast application

Even with normal initial radiographs, patients with a consistent history and tenderness in the anatomical snuffbox are treated as a stable fracture with immobilization in a thumb spica cast. Casting is recommended for all presumed nondisplaced scaphoid fractures.

Scaphoid fractures covered in fractures and dislocation of the forearm, wrist, and hand and are part of the NCCPA Endocrinology Musculoskeletal Blueprint (10%)

2. A solitary pulmonary nodule is found on a pre-employment screening chest x-ray in a 34-year-old non-smoking male. There are no old chest x-rays to compare. Which of the following is the most appropriate next step in the evaluation?

A. CT scan of the chest
B. Needle biopsy of the lesion
C. Positron emission tomography of the chest
D. Fiberoptic bronchoscopy

Solitary pulmonary nodules are covered as part of the NCCPA PANCE Blueprint Pulmonary (12%) Content Blueprint (12%)

Click here to see the answer

Answer: A. CT scan of the chest

In the absence of old x-rays in a nonsmoking individual less than 35 years old, CT scan of the chest is the next step in the evaluation of a solitary pulmonary nodule.

B. A needle biopsy would be indicated for a person greater than 35 years old and/or with a history of smoking to evaluate a solitary pulmonary nodule.
C. Positron emission tomography (PET scan) would be indicated if the CT scan was nonconclusive.
D. Fiberoptic bronchoscopy would be indicated only in the presence of a history of tobacco use or if the lesion was suggestive of malignancy.

3. Early clues to impending delirium tremens include

A. agitation and decreased cognition.
B. visual hallucinations and diaphoresis.
C. autonomic hyperactivity and dehydration.
D. mental confusion and sensory hyperacuity.

Delirium tremens is covered as part of the NCCPA Psychiatry content blueprint (6%) under the topic of withdrawal

Click here to see the answer

Answer: A. agitation and decreased cognition.

Anxiety, decreased cognition, tremulousness, increasing irritability, and hyperactivity are common early clues to impending delirium tremens.

Mental confusion, tremor, sensory hyperacuity, visual hallucinations, autonomic hyperactivity, diaphoresis, dehydration, electrolyte disturbances, seizures, and cardiovascular abnormalities are common signs and/or symptoms of full-blown delirium tremens.

4. Dental caries are caused by which of the following organisms?

A. Streptococcus mutans
B. Streptococcus pyogenes
C. Staphylococcus epidermidis
D. Staphylococcus aureus

Dental caries are covered as part of the NCCPA EENT Content Blueprint (9%) under diseases of the teeth and gums

Click here to see the answer

Answer: A. Streptococcus mutans

Streptococcus mutans is the principal organism that helps to demineralize the enamel.

5. The most definitive treatment for primary enuresis is

A. oxybutynin chloride (Ditropan).
B. imipramine (Tofranil).
C. trimethoprim-sulfamethoxazole (Bactrim).
D. desmopressin (DDAVP)

Primary enuresis is covered under incontinence as part of the NCCPA Genitourinary Content Blueprint (6%)

Click here to see the answer

Answer: D. desmopressin (DDAVP)

Intranasal desmopressin is effective in 50% of patients treated and is the treatment of choice.

A. Oxybutynin chloride is used for bladder spasms. It cannot be used for children under 5 years of age and is not indicated in primary enuresis.
B. Imipramine is an older form of treatment that is moderately effective, but many patients relapse when therapy is stopped. This is no longer considered the treatment of choice.
C. TMP-SMX is indicated for urinary tract infections that may cause secondary enuresis, but it is not used in primary enuresis.

6. A 47-year-old female presents to the clinic with complaints of prolonged, heavy menses that have been getting progressively worse for 3 years. She denies any pain. On physical examination, enlargement of the uterus with multiple smooth, spherical, firm masses is noted. A CBC is consistent with a mild anemia. Which of the following is the most likely diagnosis?

A. Leiomyoma
B. Adenomyosis
C. Endometriosis
D. Endometrial polyps

Click here to see the answer

Leiomyoma is covered as part of the NCCPA Reproductive Content Blueprint (8%)

Answer:  A. Leiomyoma

Abnormal uterine bleeding and irregular enlargement of the uterus are most consistent with leiomyoma. Pain is rarely present unless vascular compromise occurs.

B. While adenomyosis may present with hypermenorrhea, dysmenorrhea is often also present. Physical examination would reveal the presence of diffuse globular uterine enlargement, not the irregular enlargement as noted in the case presented.
C. Endometriosis presents with dyspareunia, dysmenorrhea, and infertility. If the pelvic exam were abnormal, uterine findings would include tender nodules in the cul de sac, not the uterus.
D. While endometrial polyps are compatible with the history of abnormal uterine bleeding, the uterus would be normal size without the irregular enlargement noted in the case presented.

7. Which of the following is the laboratory test that marks recovery from Hepatitis B infection and non-infectivity?

A. Hepatitis B surface antibody(anti-HBs)
B. Hepatitis B surface antigen (HBsAg)
C. Hepatitis B core antigen (HBcAg)
D. Hepatitis A antibody (anti-HAV)

Hepatitis B serology is covered as part of acute and chronic hepatitis and is part of the NCCPA GI and Nutrition Content Blueprint (10%)

Click here to see the answer

Answer: A. Hepatitis B surface antibody(anti-HBs)

Specific antibody to HBsAg appears in most individuals after clearance of HBsAg which indicates recovery from hepatitis B infection, non-infectivity, and immunity.

B. This test establishes infection with HBV and implies infectivity.
C. Presence of the hepatitis B core antigen reflects active infection.
D. Patients who have immunity to hepatitis A do not have immunity to hepatitis B.

8. A 36-year-old woman admits that her husband has abused her for over ten years. You should inform the woman that she is at most risk for injury or death

A. just before a holiday.
B. just after leaving an abusive spouse.
C. when an abusive spouse arrives home after work.
D. when an abusive spouse has been drinking heavily.

Domestic violence is covered as part of the NCCPA Psychiatry Content Blueprint (6%)

Click here to see the answer

Answer: B. just after leaving an abusive spouse.

Women are more likely to be assaulted or murdered when attempting to report the abuse or leave the abusive relationship; up to 75% of domestic assaults occur after separation.

9. A patient presents with an acutely painful and cold left leg. Distal pulses are absent. Leg is cyanotic. There are no signs of gangrene or other open lesions. Symptoms occurred one hour ago. Which of the following treatments is most appropriate?

A. Vena cava filter
B. Embolectomy
C. Amputation
D. Aspirin

Arterial embolism/thrombosis is covered as part of the NCCPA Cardiology Content Blueprint (16%)

Click here to see the answer

Answer: B. Embolectomy

Embolectomy within 4 to 6 hours is the treatment of choice.

A. Vena cava filters are used in the management of venous thromboembolic disease when anticoagulation cannot be done.
C. Amputation is done only when no viable tissue is present. Cutting off a viable limb is never a good idea.
D. Aspirin is used in the prevention and treatment of coronary disease and has no role in the treatment of peripheral arterial embolism.

10. A 53-year-old male is seen in the emergency department following a motor vehicle collision in which his knee impacted against the dashboard. The patient has a posterior knee dislocation that is promptly reduced in the emergency department. The patient currently has a palpable pulse in the dorsalis pedis and posterior tibial areas. Which of the following studies is mandatory?

A. Anterior plain film of knee
B. Sunrise view of the knee
C. Measurement of compartment pressures
D. Angiography

Fractures and dislocations of the knee are covered as part of the NCCPA Musculoskeletal Content Blueprint (10%)

Click here to see the answer

Answer: D. Angiography

The popliteal artery is at risk for injury whenever a patient sustains a posterior dislocation of the knee and should be evaluated with an arteriogram despite the presence of pedal pulses.

C. Compartment pressures are performed in cases of suspected compartment syndrome, not to determine the patency of the popliteal artery.

Looking for all the podcast episodes?

This FREE series is limited to every other episode, you can download and enjoy the complete audio series by joining The PANCE and PANRE Exam Academy + SMARTYPANCE

I will be releasing new episodes every few weeks. The Academy is discounted, so sign up now.

Resources and Links From The Show

This Podcast is also available on iTunes and Stitcher Radio for Android

  1. iTunes: The Audio PANCE AND PANRE Podcast iTunes
  2. Stitcher Radio: The Audio PANCE and PANRE Podcast Stitcher

itunes_logo-1

Download The Content Blueprint Checklist

Follow this link to download your FREE copy of the Content Blueprint Checklist

Print it up and start crossing out the topics you understand, marking the ones you don’t and making notes of key terms you should remember. The PDF version is interactive and linked directly to the individual lessons on SMARTY PANCE.

pance-and-panre-nccpa-content-blueprint

Download

The post Episode 61: The Audio PANCE and PANRE – Ten Mixed NCCPA™ Content Blueprint Multiple Choice Questions appeared first on The Audio PANCE and PANRE.

]]>
Ten Mixed NCCPA™ Content Blueprint Multiple Choice Questions Welcome to episode 61 of the FREE Audio PANCE and PANRE Physician Assistant Board Review Podcast. Join me as I cover ten PANCE and PANRE Board review questions from the SMARTYPANCE course con... Welcome to episode 61 of the FREE Audio PANCE and PANRE Physician Assistant Board Review Podcast.
Join me as I cover ten PANCE and PANRE Board review questions from the SMARTYPANCE course content following the NCCPA™ content blueprint (download the FREE cheat sheet).
This week we will be covering ten general board review questions based on the NCCPA PANCE and PANRE Content Blueprint. 
Below you will find an interactive exam to complement the podcast.
I hope you enjoy this free audio component to the examination portion of this site. The full board review includes over 2,000 interactive board review questions and is available to all members of the PANCE and PANRE Academy and SMARTYPANCE which are now bundled together into one very low price.

* You can download and listen to past FREE episodes here, on iTunes, on Google Play Music or Stitcher Radio.
* You can listen to the latest episode, take an interactive quiz and download your results below.

Listen Carefully Then Take The Practice Exam
If you can’t see the audio player click here to listen to the full episode.
Episode 5 – General PANCE/PANRE Podcast Quiz
The following 10 questions are linked to NCCPA Content Blueprint lessons from the SMARTYPANCE and PANRE Board review website. If you are a member you will be able to log in and view this interactive video content.
1. An 18-year-old male presents with pain in his wrist after he fell off of a moving motorcycle. Physical examination reveals tenderness in the anatomic snuffbox. No fracture is noted on plain radiography of the wrist. Which of the following is the recommended treatment for this patient?

A. An ace wrap of the wrist
B. Closed reduction of the fracture site
C. Thumb spica cast application
D. Open reduction of the fracture site
2. A solitary pulmonary nodule is found on a pre-employment screening chest x-ray in a 34-year-old non-smoking male. There are no old chest x-rays to compare. Which of the following is the most appropriate next step in the evaluation?
A. CT scan of the chest
B. Needle biopsy of the lesion
C. Positron emission tomography of the chest
D. Fiberoptic bronchoscopy
Solita...]]>
The Physician Assistant Life | Smarty PANCE full clean 14:01 303
Episode 59: Emergency Medicine EOR – The Audio PANCE and PANRE Board Review Podcast http://podcast.thepalife.com/episode-59-emergency-medicine-eor-the-audio-pance-and-panre-board-review-podcast/ Mon, 09 Apr 2018 07:01:43 +0000 http://podcast.thepalife.com/?p=298 http://podcast.thepalife.com/episode-59-emergency-medicine-eor-the-audio-pance-and-panre-board-review-podcast/#respond http://podcast.thepalife.com/episode-59-emergency-medicine-eor-the-audio-pance-and-panre-board-review-podcast/feed/ 0 <p>The Audio PANCE/PANRE PA Board Review Podcast Welcome to episode 59 of the FREE Audio PANCE and PANRE Physician Assistant Board Review Podcast. Join me as I cover ten Emergency Medicine End of Rotation Exam (EOR) review questions from the SMARTYPANCE course content following the NCCPA™ and PAEA content blueprint (download the FREE cheat sheet). This week […]</p> <p>The post <a rel="nofollow" href="http://podcast.thepalife.com/episode-59-emergency-medicine-eor-the-audio-pance-and-panre-board-review-podcast/">Episode 59: Emergency Medicine EOR – The Audio PANCE and PANRE Board Review Podcast</a> appeared first on <a rel="nofollow" href="http://podcast.thepalife.com">The Audio PANCE and PANRE</a>.</p> The Audio PANCE and PANRE Emergency Medicine End of Rotation Exam

The Audio PANCE/PANRE PA Board Review Podcast

Welcome to episode 59 of the FREE Audio PANCE and PANRE Physician Assistant Board Review Podcast.

Join me as I cover ten Emergency Medicine End of Rotation Exam (EOR) review questions from the SMARTYPANCE course content following the NCCPA™ and PAEA content blueprint (download the FREE cheat sheet).

This week we will be covering ten emergency medicine end of rotation exam questions based on the NCCPA PANCE and PANRE Content Blueprint. 

Below you will find an interactive exam to complement the podcast.

I hope you enjoy this free audio component to the examination portion of this site. The full board review includes over 2,000 interactive board review questions and is available to all members of the PANCE and PANRE Academy and SMARTYPANCE which are now bundled together into one very low price.

Listen Carefully Then Take The Practice Exam

If you can’t see the audio player click here to listen to the full episode.

Episode 59 – Emergency Medicine EOR Podcast Quiz

The following 10 questions are linked to NCCPA Content Blueprint lessons from the SMARTYPANCE and PANRE Board review website. If you are a member you will be able to log in and view this interactive video content.

1. During a baseball game, a 22-year-old college student is hit in the right eye by a baseball. He complains of blurry vision in that eye. On physical exam, the physician assistant notes proptosis of the right eye and limitation of movement in all directions. On CT scan, which of the following is most likely to be seen?

A. Fracture of the medial orbital wall
B. Prolapse of orbital soft tissue
C. Hematoma of the orbit
D. Orbital emphysema

Click here to see the answer

Answer: C. Hematoma of the orbit

Orbital hemorrhage into the space surrounding the globe following blunt trauma and rupture of the orbital vessels results in increased ocular pressure, proptosis, visual loss, and limitation of movement in all directions. CT reveals a hematoma.

A. Fracture of the medial orbital wall is associated with diplopia from medial rectus impingement, orbital emphysema, and epistaxis.
B. Prolapse of orbital soft tissue, including inferior rectus muscle, inferior oblique muscle, orbital fat, and connective tissue results in enophthalmos, ptosis, diplopia, anesthesia of the ipsilateral cheek and upper lip, and limitation of upward gaze and is seen with fractures of the orbital floor.
D. Orbital emphysema is seen with fractures of the medial orbital wall or floor of the orbit into the maxillary and ethmoid sinuses respectively. It will not lead to proptosis.

Blowout fractures are covered as part of the NCCPA EENT Content Blueprint which accounts for 9% of your exam.

2. A 29-year-old male presents with a complaint of substernal chest pain for 12 hours. The patient states that the pain radiates to his shoulders and is relieved with sitting forward. The patient admits to recent upper respiratory symptoms. On examination vital signs are BP 126/68, HR 86, RR 20, temp 100.3 degrees F. There is no JVD noted. Heart exam reveals regular rate and rhythm with no S3 or S4. There is a friction rub noted. Lungs are clear to auscultation. EKG shows diffuse ST-segment elevation. What is the treatment of choice for this patient?

A. Pericardiocentesis
B. Nitroglycerin
C. Percutaneous coronary intervention
D. Indomethacin (Indocin)

Click here to see the answer

Answer: D. Indomethacin

Indomethacin, a nonsteroidal anti-inflammatory medication, is the treatment of choice in a patient with acute pericarditis.

A. Pericardiocentesis is the treatment of choice in a patient with a pericardial effusion and cardiac tamponade, there is no evidence of either of these in this patient.
B. Nitroglycerin is indicated in the treatment of chest pain related to angina.
C. Percutaneous coronary intervention is the treatment of choice in a patient with an acute myocardial infarction.

Acute pericarditis is covered the NCCPA Cardiology Content Blueprint which accounts for 16% of your exam.

3. A 19-year-old female presents with a sore throat for nearly two weeks. She complains of fatigue and a low-grade fever. On physical examination, there is cervical, axillary, and inguinal lymphadenopathy, and mild splenomegaly. On review of the blood smear, which of the following would be expected?

A. Atypical lymphocytes
B. Hypersegmented neutrophils
C. Hypochromic red blood cells
D. Schistocytes

Click here to see the answer

Answer: A. Atypical lymphocytes

The hallmark of infectious mononucleosis is the presence of lymphocytosis with atypical large lymphocytes seen in the blood smear. These are larger than normal mature lymphocytes, stain more darkly, and frequently show vacuolated, foamy cytoplasm, and dark chromatin in the nucleus.

B. Hypersegmented neutrophils are seen in vitamin B12 deficiency.
C. Anemia, if seen in mononucleosis, is normocytic and normochromic.
D. Schistocytes are noted in hemolytic anemias.

Mononucleosis as part of the NCCPA Infectious Disease Content Blueprint which accounts for 3% of your exam.

4. A 30-year-old female complains of fatigue, weakness, diminished appetite, weight loss, and syncope. She denies fever, chest or abdominal pain, palpitations, changes in bowel patterns or sleep patterns. Physical examination reveals a thin female, BP 90/65 mmHg, and pulse 80 beats per minute. Pulmonary, cardiovascular, abdominal, and neurologic exam are without abnormalities. Areas of brown and bronze hyperpigmentation are noted on her elbows and the creases of her hands. Which of the following tests will be the most useful in making the diagnosis?

A. Drug screen
B. Urine dipstick
C. Complete blood count
D. Serum creatinine kinase

Click here to see the answer

Answer: D. Serum creatinine kinase

Serum creatinine kinase is the most sensitive test to detect rhabdomyolysis, a serious complication of seizures and hyperthermia related to drug abuse.

A. Although a drug screen may identify specific drugs, the results will not alter the care of this patient.
B. Urine dipstick is not sensitive for myoglobinuria.
C. This patient is at risk for myoglobinuria, and a complete blood count will not alter the treatment.

5. A 15-year-old male was seen last week with complaints of a sore throat, headache, and mild cough. A diagnosis of URI was made and supportive treatment was initiated. He returns today with complaints of worsening cough and increasing fatigue. At this time, chest x-ray reveals bilateral hilar infiltrates. A WBC count is normal and a cold hemagglutinin titer is elevated. The most likely diagnosis is

A. tuberculosis.
B. mycoplasma pneumonia.
C. pneumococcal pneumonia.
D. staphylococcal pneumonia.

Click here to see the answer

Answer: B. mycoplasma pneumonia.

The insidious onset of symptoms, the interstitial infiltrates on chest x-ray, and elevated cold hemagglutinin titer makes this diagnosis the most likely.

A. Most children with pulmonary tuberculosis are asymptomatic with few physical examination findings. The results of the diagnostic studies do not support tuberculosis as the most likely diagnosis.
C. The clinical presentation of bacterial pneumonia in children is variable, but usually involves fever of acute onset. The WBC count is also usually elevated, making this a less likely diagnosis.

Mycoplasma pneumonia is covered as part of the NCCPA Pulmonary Content Blueprint and accounts for 12% of the exam

6. Which of the following clinical manifestations is common in candidal vulvovaginitis?

A. Extreme vulvar irritation
B. Firm, painless ulcer
C. Tender lymphadenopathy
D. Purulent discharge

Click here to see the answer

Answer:  A. Extreme vulvar irritation 

Candida infection presents with pruritus, vulvovaginal erythema, and white, cheese-like (curd) discharge that may be malodorous.

B. A firm painless ulcer is seen in syphilis.
C. Tender lymphadenopathy is associated with bacterial infections and is not a feature of candidal vulvovaginitis.
D. Purulent discharge is noted in gonorrhea.

Vaginitis is covered as part of the NCCPA Reproductive System Content Blueprint and accounts for 8% of the exam

7. A 63-year-old female presents with a complaint of chest pressure for one hour noticed upon awakening. She admits to associated nausea, vomiting, and shortness of breath. 12 lead EKG reveals ST-segment elevation in leads II, III, and AVF. Which of the following is the most likely diagnosis?

A. Aortic dissection
B. Inferior wall myocardial infarction
C. Acute pericarditis
D. Pulmonary embolus

Click here to see the answer

Answer: B. Inferior wall myocardial infarction 

Myocardial infarction often presents with chest pressure and associated nausea and vomiting. ST-segment elevation in leads II, III, and AVF are classic findings seen in acute inferior wall myocardial infarction.

A. A patient with aortic dissection will complain of tearing, ripping pain. EKG is often normal but may reveal left ventricular strain pattern.
C. Acute pericarditis presents with atypical chest pain and diffuse ST-segment elevation.
D. Pulmonary embolism often presents with either no EKG changes or sinus tachycardia. Classically described, rarely seen findings include a large S wave in lead I, a Q wave with T wave inversion in lead III, ST-segment depression in lead II, T wave inversion in leads V1-V4 and a transient right bundle branch block.

Acute myocardial infarction is covered as part of the NCCPA Cardiology Content Blueprint and accounts for 16% of the exam

8. Small grayish vesicles and punched-out ulcers in the posterior pharynx in a child with pharyngitis is representative of which organism?

A. Epstein-Barr virus
B. Group C Streptococcus
C. Coxsackievirus
D. Gonorrhea

Click here to see the answer

Answer: C. Coxsackievirus

Coxsackievirus presents with small grayish vesicles and punched-out ulcers in the posterior pharynx.

A. Epstein-Barr virus presents with enlarged tonsils with exudates and petechiae of the palate.
B. Group C Streptococcus presents with a red pharynx and enlarged tonsils with a yellow, blood tinged exudates
D. Neisseria gonorrhea of the pharynx may be asymptomatic

Coxsackievirus is covered as part of the NCCPA Dermatology Content Blueprint which accounts for 5% of your exam.

9. Which of the following is the most appropriate management of acute psychosis in a patient with schizophrenia?

A. Amitriptyline (Elavil)
B. Risperidone (Risperdal)
C. Lithium (Eskalith)
D. Sertraline (Zoloft)

Click here to see the answer

Answer: B. Risperidone (Risperdal)

There are numerous options used in the management of acute psychosis including benzodiazepines and antipsychotics such as haloperidol (first generation – typical -antipsychotic), risperidone or aripiprazole (second Generation – atypical – antipsychotics). Antidepressants and lithium have no benefit in the treatment of acute psychosis.

Schizophrenia is covered as part of the NCCPA psychiatry Content Blueprint which accounts for 6% of your exam.

10. A 25-year-old presents with pain in the proximal ulna after falling directly on the forearm. X-ray shows a fracture of the proximal 1/3rd of the ulna. There is an associated anterior radial head dislocation. What is the proper name for this condition?

A. Galeazzi fracture
B. Monteggia fracture
C. Colles’ fracture
D. Smith fracture

Click here to see the answer

Answer: B. Monteggia fracture

A Monteggia fracture is a fracture of the proximal ulna with anterior dislocation of the radial head.

A. A Galeazzi fracture is a fracture along the length of the radius with an injury to the distal radioulnar joint.
C. A Colles’ fracture is a fracture of the distal radius with dorsal displacement of the radial head.
D. A Smith fracture is a fracture of the distal radius with a ventral displacement of the radial head.

Monteggia fracture is covered as part of the NCCPA Musculoskeletal Content Blueprint which accounts for 10% of your exam.

Looking for all the podcast episodes?

This FREE series is limited to every other episode, you can download and enjoy the complete audio series by joining The PANCE and PANRE Exam Academy + SMARTYPANCE

I will be releasing new episodes every few weeks. The Academy is discounted, so sign up now.

Resources and Links From The Show

This Podcast is also available on iTunes and Stitcher Radio for Android

  1. iTunes: The Audio PANCE AND PANRE Podcast iTunes
  2. Stitcher Radio: The Audio PANCE and PANRE Podcast Stitcher

itunes_logo-1

Download The Content Blueprint Checklist

Follow this link to download your FREE copy of the Content Blueprint Checklist

Print it up and start crossing out the topics you understand, marking the ones you don’t and making notes of key terms you should remember. The PDF version is interactive and linked directly to the individual lessons on SMARTY PANCE.

pance-and-panre-nccpa-content-blueprint

Download

The post Episode 59: Emergency Medicine EOR – The Audio PANCE and PANRE Board Review Podcast appeared first on The Audio PANCE and PANRE.

]]> The Audio PANCE/PANRE PA Board Review Podcast Welcome to episode 59 of the FREE Audio PANCE and PANRE Physician Assistant Board Review Podcast. Join me as I cover ten Emergency Medicine End of Rotation Exam (EOR) review questions from the SMARTYPANCE c...
The Audio PANCE/PANRE PA Board Review Podcast
Welcome to episode 59 of the FREE Audio PANCE and PANRE Physician Assistant Board Review Podcast.
Join me as I cover ten Emergency Medicine End of Rotation Exam (EOR) review questions from the SMARTYPANCE course content following the NCCPA™ and PAEA content blueprint (download the FREE cheat sheet).
This week we will be covering ten emergency medicine end of rotation exam questions based on the NCCPA PANCE and PANRE Content Blueprint. 
Below you will find an interactive exam to complement the podcast.
I hope you enjoy this free audio component to the examination portion of this site. The full board review includes over 2,000 interactive board review questions and is available to all members of the PANCE and PANRE Academy and SMARTYPANCE which are now bundled together into one very low price.

* You can download and listen to past FREE episodes here, on iTunes, on Google Play Music or Stitcher Radio.
* You can listen to the latest episode, take an interactive quiz and download your results below.
* Members can take Emergency medicine EOR 1 (200 questions) and Emergency Medicine EOR 2 (184 questions)

Listen Carefully Then Take The Practice Exam
If you can’t see the audio player click here to listen to the full episode.
Episode 59 – Emergency Medicine EOR Podcast Quiz
The following 10 questions are linked to NCCPA Content Blueprint lessons from the SMARTYPANCE and PANRE Board review website. If you are a member you will be able to log in and view this interactive video content.
1. During a baseball game, a 22-year-old college student is hit in the right eye by a baseball. He complains of blurry vision in that eye. On physical exam, the physician assistant notes proptosis of the right eye and limitation of movement in all directions. On CT scan, which of the following is most likely to be seen?
A. Fracture of the medial orbital wall
B. Prolapse of orbital soft tissue
C. Hematoma of the orbit
D. Orbital emphysema
2. A 29-year-old male presents with a complaint of substernal chest pain for 12 hours. The patient states that the pain radiates to his shoulders and is relieved with sitting forward.]]>
The Physician Assistant Life | Smarty PANCE full clean 18:14 298 Episode 57: Endocrinology – The Audio PANCE/PANRE Board Review Podcast – Content Blueprint Review Endocrinology http://podcast.thepalife.com/episode-57-endocrinology/ Tue, 20 Feb 2018 05:55:20 +0000 http://podcast.thepalife.com/?p=290 http://podcast.thepalife.com/episode-57-endocrinology/#respond http://podcast.thepalife.com/episode-57-endocrinology/feed/ 0 <p>The Audio PANCE/PANRE PA Board Review Podcast Welcome to episode 57 of the FREE Audio PANCE and PANRE Physician Assistant Board Review Podcast. Join me as I cover ten PANCE and PANRE Board review questions from the SMARTYPANCE course content following the NCCPA™ content blueprint (download the FREE cheat sheet). This week we will be covering ten endocrinology […]</p> <p>The post <a rel="nofollow" href="http://podcast.thepalife.com/episode-57-endocrinology/">Episode 57: Endocrinology – The Audio PANCE/PANRE Board Review Podcast – Content Blueprint Review Endocrinology</a> appeared first on <a rel="nofollow" href="http://podcast.thepalife.com">The Audio PANCE and PANRE</a>.</p> Episode 57 - Endocrinology NCCPA Content Blueprint Board Review

The Audio PANCE/PANRE PA Board Review Podcast

Welcome to episode 57 of the FREE Audio PANCE and PANRE Physician Assistant Board Review Podcast.

Join me as I cover ten PANCE and PANRE Board review questions from the SMARTYPANCE course content following the NCCPA™ content blueprint (download the FREE cheat sheet).

This week we will be covering ten endocrinology board review questions based on the NCCPA PANCE and PANRE Content Blueprint. 

Below you will find an interactive exam to complement the podcast.

I hope you enjoy this free audio component to the examination portion of this site. The full board review includes over 2,000 interactive board review questions and is available to all members of the PANCE and PANRE Academy and SMARTYPANCE which are now bundled together into one very low price.

Listen Carefully Then Take The Practice Exam

If you can’t see the audio player click here to listen to the full episode.

Episode 57 – Endocrinology PANCE/PANRE Podcast Quiz

The following 10 questions are linked to NCCPA Content Blueprint lessons from the SMARTYPANCE and PANRE Board review website. If you are a member you will be able to log in and view this interactive video content.

1. A 53-year-old Hispanic woman comes to your clinic for her annual physical exam. She is obese, does not exercise, and regularly eats fried foods. A random blood glucose is 249 mg/dL. Her hemoglobin A1C is 9.5. Which of the following treatments would be weight neutral or cause weight loss in this patient?

A. Glargine
B. Glyburide
C. Actos
D. Metformin
E. Glipizide

Diabetes Mellitus Type 2 is covered as part of the NCCPA Endocrinology Content Blueprint which accounts for 6% of your exam.

Click here to see the answer

Answer: D. Metformin

The patient in this vignette most likely has type II diabetes. Of the given treatments, only metformin is weight neutral in the majority of cases (i.e. it does not cause significant weight gain/loss). Metformin is a first-line treatment for type II DM in most patients. Although the exact mechanism is unknown, it appears to decrease gluconeogenesis and increase insulin sensitivity. There is no risk of hypoglycemia or weight gain (though some patients even lose weight). The most high yield side effect involves lactic acidosis, particularly in patients with renal insufficiency. 

A. Glargine, a long-acting insulin, can cause weight gain.
B. Glyburide, a second-generation sulfonylurea, can cause weight gain.
C. Actos, a thiazolidinedione, can cause weight gain.
E. Glipizide, a second-generation sulfonylurea, can cause weight gain.

2. A solitary thyroid nodule is noted on physical examination. The TSH level is normal. The next step in the evaluation is:

A. measurement of T4 and free T3 levels.
B. a radionuclide thyroid scan.
C. a fine needle biopsy.
D. a surgical excision.

Solitary thyroid nodule is covered as part of thyroid neoplastic disease in the NCCPA Endocrinology Content Blueprint which accounts for 6% of your exam.

Click here to see the answer

Answer: C. a fine needle biopsy

Fine needle aspiration (FNA) is the first step in the evaluation of a solitary nodule with a normal TSH level. FNA has a high level of accuracy in diagnosing benign versus malignant nodules in this setting.

A. Measurement of T4 and T3 levels would not be of benefit in the evaluation of a solitary thyroid nodule with a normal TSH level.
B. A thyroid scan would be the next step if there were a low TSH level.
D. Surgical excision would be the final step after determination of malignancy or suspicion of malignancy by FNA.

3. An 18-year-old male with a past medical history of type I diabetes presents to the emergency room with polyuria, polydipsia, and dehydration. Vital signs reveal tachycardia and hypotension. The physical exam is significant for dry mucous membranes and decreased skin turgor. In the waiting room, he begins vomiting and complains of intense abdominal pain. You observe him taking rapid, deep breaths, and over the course of his brief stay, getting more somnolent. Which of the following abnormalities would be expected in this patient?

A. Hypernatremia
B. Decreased total body potassium
C. Hypoglycemia
D. Absence of urinary beta-OH-butyrate
E. Non-anion-gap metabolic acidosis

Diabetic ketoacidosis is covered under Diabetes Mellitus Type 1 as part of the NCCPA Endocrinology Content Blueprint which accounts for 6% of your exam.

Click here to see the answer

Answer: B. Decreased total body potassium

This type I diabetic is presenting with signs and symptoms of diabetic ketoacidosis (DKA). In DKA, total body potassium stores are generally decreased due to osmotic diuresis. DKA is a life-threatening emergency that may occur in either type I or type II diabetics but is significantly more common in patients with type I. The pathogenesis is related to insulin deficiency resulting in hyperglycemia that leads to osmotic diuresis and hypovolemia. The inability of the body to use the available glucose for ATP production results in ketone formation and eventually an anion gap metabolic acidosis. Serum potassium levels may be low, normal, or elevated, but total body stores are generally low and require repletion. Common precipitating factors include infection, trauma, myocardial infarction, sepsis and, of course, inadequate insulin administration. Patients may present with nausea, vomiting, abdominal pain, Kussmaul respirations (rapid, deep breaths), dehydration, polydipsia, polyuria and may eventually progress to altered mental status.

A. Patients with DKA more often present with hyponatremia. Remember that serum sodium decreases 1.6 mEq/L for every 100 mg/dL increase in glucose.
C. DKA requires hyperglycemia by definition. Hypoglycemia can be a complication of treatment if glucose is not monitored closely.
D. Ketones, such as Beta-OH-Butyrate, are commonly found in patients with DKA since ketogenesis is a normal response to starvation caused by the inadequate transit of serum glucose into cells.
E. Patients with DKA present with an anion-gap metabolic acidosis secondary to ketoacids.

4. A 30-year-old female complains of fatigue, weakness, diminished appetite, weight loss, and syncope. She denies fever, chest or abdominal pain, palpitations, changes in bowel patterns or sleep patterns. Physical examination reveals a thin female, BP 90/65 mmHg, and pulse 80 beats per minute. Pulmonary, cardiovascular, abdominal, and neurologic exam are without abnormalities. Areas of brown and bronze hyperpigmentation are noted on her elbows and the creases of her hands. Which of the following is the most likely diagnosis?

A. Addison’s disease
B. Cushing’s disease
C. Anorexia nervosa
D. Porphyria

This condition is covered as part of the NCCPA Endocrinology Content Blueprint and accounts for 6% of the exam

Click here to see the answer

Answer: A. Addison’s disease

Addison’s disease (adrenal insufficiency) would account for all her symptoms, the hypotension, and the hyperpigmentation of the skin.

B. Cushing’s disease, the presence of an ACTH-producing adenoma, is characterized by central obesity, hypertension, moon facies, purple striae, and glucose intolerance.
C. Anorexia nervosa may explain the weakness, weight loss, hypotension, and syncope, however, a normal pulse rate would be an unexpected finding along with the hyperpigmentation.
D. Porphyria presents acutely with anxiety, depression, disorientation, and insomnia.

5. A 39-year-old male presents to your clinic complaining of increasing constant headaches and progressive loss of peripheral vision. His medical and family history is unremarkable. Physical examination reveals bitemporal hemianopsia but is otherwise without any abnormalities. Which of the following is the most likely diagnosis?
Answers

A. An aneurysm involving the circle of Willis
B. A migraine headache
C. Multiple sclerosis
D. Pituitary tumor

This condition is covered as part of the NCCPA Endocrinology Content Blueprint and accounts for 6% of the exam

Click here to see the answer

Answer: D. A pituitary tumor 

A pituitary tumor would account for the headaches and the loss of the peripheral vision in both visual fields. As the tumor grows, the optic chiasm will be compressed by the tumor.

A. An aneurysm involving the circle of Willis would result in CN III palsy. This would be a rare finding.
B. Although a migraine headache may produce visual field defects, these defects would remit upon resolution of the migraine. It would also be unusual to have the scotomas occur bilaterally.
C. Optic neuritis associated with multiple sclerosis presents with decreased visual acuity, dimness, or color desaturation in the central visual field. It would not affect the periphery.

6. Radioactive iodine is most successful in treating hyperthyroidism that results from

A. Grave’s disease.
B. subacute thyroiditis.
C. Hashimoto’s thyroiditis.
D. papillary thyroid carcinoma.

Watch this ReelDx Video of a 16-year-old with ADHD presents with chest pain and exophthalmos

Diseases of the thyroid gland are covered as part of the NCCPA Endocrinology Content Blueprint and accounts for 6% of the exam

Click here to see the answer

Answer:  A. Grave’s disease.

Radioactive iodine is an excellent method to destroy overactive thyroid tissue of Grave’s disease.

B. Radioactive iodine is ineffective in subacute thyroiditis due to the thyroid’s low uptake of iodine.
C. Radioiodine uptake is low in Hashimoto’s thyroiditis and is often transient.
D. Papillary thyroid carcinoma is a common thyroid malignancy and must be treated by a thyroidectomy.

7. A newborn infant exhibits prolonged jaundice, feeding problems, hypotonia, and an enlarged tongue. Proper treatment in this infant would consist of which of the following?

A. IV antibiotics
B. Thyroid hormone replacement
C. Hepatitis B immunoglobulin
D. Vitamin B6 supplement

This condition is covered as part of the NCCPA Endocrinology Content Blueprint and accounts for 6% of the exam

Click here to see the answer

Answer: B. Thyroid hormone replacement 

This scenario is consistent with congenital hypothyroidism. Measurement of TSH or T4 would confirm this and T4 should be given.

C. Hepatitis and sepsis may account for the presence of jaundice, feeding problems, and hypotonia, but would not result in an enlarged tongue.
D. A deficiency in vitamin B6 may lead to glossitis but would not account for or any of the other signs.

8. Which of the following glucose-lowering agents act by delaying glucose absorption?

A. Metformin (Glucophage)
B. Acarbose (Precose)
C. Glipizide (Glucotrol)
D. Pioglitazone (Actos)

Diabetes Mellitus Type 2 and associated medications are covered as part of the NCCPA Endocrinology Content Blueprint which accounts for 6% of your exam.

Click here to see the answer

Answer: B. Acarbose (Precose)

Alpha-glucosidase inhibitors, such as acarbose, reduce glucose by delaying glucose absorption.

A. Metformin, a biguanide, lowers glucose by decreasing hepatic glucose production and increased glucose utilization.
C. Glipizide and other sulfonylureas work by increasing insulin secretion.
D. Pioglitazone is a thiazolidinedione and decreases insulin resistance and increases glucose utilization.c

9. Which of the following conditions may result in hypokalemia?

A. Adrenal adenoma
B. Hypoparathyroidism
C. Hyperthyroidism
D. Adrenal insufficiency

Diseases of the Adrenal Glands are covered as part of the NCCPA Endocrinology Content Blueprint which accounts for 6% of your exam.

Click here to see the answer

Answer: A. Adrenal adenoma

Excessive secretion of aldosterone from an adrenal adenoma will lead to sodium retention and the secretion of potassium in the distal tubule of the kidney, eventually leading to hypokalemia.

B. Hypoparathyroidism and hyperthyroidism should not have any effect on potassium levels.
D. Adrenal insufficiency would lead to hyperkalemia.

10. A 7-year-old child with a history of type 1 diabetes mellitus for 3 years presents for routine follow-up. The mother states that the child has been having nightmares and night sweats. Additionally, his average morning glucose readings have risen from an average of 100 mg/dL to 145 mg/dL over the past week. This child is most likely experiencing

A. a growth spurt.
B. emotional problems.
C. the Somogyi effect.
D. the dawn phenomenon.

The Somogyi effect and the dawn phenomenon are covered under Diabetes Mellitus Type 1 as part of the NCCPA Endocrinology Content Blueprint which accounts for 6% of your exam.

Click here to see the answer

Answer: C. the Somogyi effect.

This refers to nocturnal hypoglycemia, which stimulates counter-regulatory hormone release resulting in rebound hyperglycemia.

A. Nightmares and night sweats are not associated with growth spurts.
B. With this limited history, it is impossible to label the child as emotionally unstable.
D. This refers to an early morning rise in plasma glucose due to reduced tissue sensitivity to insulin between 5 AM and 8 AM. It is not associated with nightmares and night sweats.

Looking for all the podcast episodes?

This FREE series is limited to every other episode, you can download and enjoy the complete audio series by joining The PANCE and PANRE Exam Academy + SMARTYPANCE

I will be releasing new episodes every few weeks. The Academy is discounted, so sign up now.

Resources and Links From The Show

This Podcast is also available on iTunes and Stitcher Radio for Android

  1. iTunes: The Audio PANCE AND PANRE Podcast iTunes
  2. Stitcher Radio: The Audio PANCE and PANRE Podcast Stitcher

itunes_logo-1

Download The Content Blueprint Checklist

Follow this link to download your FREE copy of the Content Blueprint Checklist

Print it up and start crossing out the topics you understand, marking the ones you don’t and making notes of key terms you should remember. The PDF version is interactive and linked directly to the individual lessons on SMARTY PANCE.

pance-and-panre-nccpa-content-blueprint

Download

The post Episode 57: Endocrinology – The Audio PANCE/PANRE Board Review Podcast – Content Blueprint Review Endocrinology appeared first on The Audio PANCE and PANRE.

]]>
The Audio PANCE/PANRE PA Board Review Podcast Welcome to episode 57 of the FREE Audio PANCE and PANRE Physician Assistant Board Review Podcast. Join me as I cover ten PANCE and PANRE Board review questions from the SMARTYPANCE course content following ...
The Audio PANCE/PANRE PA Board Review Podcast
Welcome to episode 57 of the FREE Audio PANCE and PANRE Physician Assistant Board Review Podcast.
Join me as I cover ten PANCE and PANRE Board review questions from the SMARTYPANCE course content following the NCCPA™ content blueprint (download the FREE cheat sheet).
This week we will be covering ten endocrinology board review questions based on the NCCPA PANCE and PANRE Content Blueprint. 
Below you will find an interactive exam to complement the podcast.
I hope you enjoy this free audio component to the examination portion of this site. The full board review includes over 2,000 interactive board review questions and is available to all members of the PANCE and PANRE Academy and SMARTYPANCE which are now bundled together into one very low price.

* You can download and listen to past FREE episodes here, on iTunes, on Google Play Music or Stitcher Radio.
* You can listen to the latest episode, take an interactive quiz and download your results below.

Listen Carefully Then Take The Practice Exam
If you can’t see the audio player click here to listen to the full episode.
Episode 57 – Endocrinology PANCE/PANRE Podcast Quiz
The following 10 questions are linked to NCCPA Content Blueprint lessons from the SMARTYPANCE and PANRE Board review website. If you are a member you will be able to log in and view this interactive video content.
1. A 53-year-old Hispanic woman comes to your clinic for her annual physical exam. She is obese, does not exercise, and regularly eats fried foods. A random blood glucose is 249 mg/dL. Her hemoglobin A1C is 9.5. Which of the following treatments would be weight neutral or cause weight loss in this patient?
A. Glargine
B. Glyburide
C. Actos
D. Metformin
E. Glipizide
Diabetes Mellitus Type 2 is covered as part of the NCCPA Endocrinology Content Blueprint which accounts for 6% of your exam.
2. A solitary thyroid nodule is noted on physical examination. The TSH level is normal. The next step in the evaluation is:
A. measurement of T4 and free T3 levels.
B. a radionuclide thyroid scan.
C.]]>
The Physician Assistant Life | Smarty PANCE full clean 17:07 290
Episode 55: The Audio PANCE and PANRE Board Review Podcast – Mixed Content Blueprint Review http://podcast.thepalife.com/episode-55-audio-pance-panre-board-review-podcast-mixed-content-blueprint-review/ Thu, 30 Nov 2017 05:41:24 +0000 http://podcast.thepalife.com/?p=285 http://podcast.thepalife.com/episode-55-audio-pance-panre-board-review-podcast-mixed-content-blueprint-review/#respond http://podcast.thepalife.com/episode-55-audio-pance-panre-board-review-podcast-mixed-content-blueprint-review/feed/ 0 <p>Welcome to episode 55 of the FREE Audio PANCE and PANRE Physician Assistant Board Review Podcast. Join me as I cover ten PANCE and PANRE Board review questions from the SMARTYPANCE course content following the NCCPA™ content blueprint (download the FREE cheat sheet). This week we will be covering ten general board review questions based on the NCCPA PANCE […]</p> <p>The post <a rel="nofollow" href="http://podcast.thepalife.com/episode-55-audio-pance-panre-board-review-podcast-mixed-content-blueprint-review/">Episode 55: The Audio PANCE and PANRE Board Review Podcast – Mixed Content Blueprint Review</a> appeared first on <a rel="nofollow" href="http://podcast.thepalife.com">The Audio PANCE and PANRE</a>.</p> Episode 55 The Audio PANCE and PANRE Physician Assistant Board ReviewWelcome to episode 55 of the FREE Audio PANCE and PANRE Physician Assistant Board Review Podcast.

Join me as I cover ten PANCE and PANRE Board review questions from the SMARTYPANCE course content following the NCCPA™ content blueprint (download the FREE cheat sheet).

This week we will be covering ten general board review questions based on the NCCPA PANCE and PANRE Content Blueprint. 

Below you will find an interactive exam to complement the podcast.

I hope you enjoy this free audio component to the examination portion of this site. The full board review includes over 2,000 interactive board review questions and is available to all members of the PANCE and PANRE Academy and SMARTYPANCE which are now bundled together into one very low price.

Listen Carefully Then Take The Quiz

If you can’t see the audio player click here to listen to the full episode.

Episode 55 PANCE and PANRE Podcast Quiz

The following 10 questions are linked to NCCPA Content Blueprint lessons from the SMARTYPANCE and PANRE Board review website. If you are a member you will be able to log in and view this interactive video content.

1. Which of the following physical examination findings would be consistent with a pleural effusion?

A. Hyperresonance to percussion
B. Increased tactile fremitus
C. Unilateral lag on chest expansion
D. Egophony

Pleural effusions are covered as part of the NCCPA Content Blueprint Pulmonology section which accounts for 10% of the exam

View this ReelDx patient video case of a 68-year-old female complaining of bilateral chest pain and difficulty breathing. Included in this lesson is the Picmonic explaning Light’s Criteria.

Click here to see the answer

Answer: C. Unilateral lag on chest expansion

A lag on chest expansion may be seen in the presence of a pleural effusion.

A. Hyperresonance to percussion would be suggestive of emphysema or pneumothorax.
B. Increased tactile fremitus would be consistent with a consolidation.
D. The presence of egophony would be consistent with a consolidation.

2. A 55 year-old female presents with complaints of stiffness, aching, and pain in the muscles of her neck, shoulders, lower back, hips, and thighs. There is no associated weakness associated with the stiffness and achiness. Laboratory evaluation shows an elevated C reactive protein and erythrocyte sedimentation rate. Which of the following medications is used to treat this condition immediately and will also serve to prevent a known complication from this disorder?

A. Glucocorticoids
B. Cyclophosphamide (Cytoxan)
C. Methotrexate (Rheumatrex)
D. Azathioprine (Imuran)

This condition is covered as part of the NCCPA Musculoskeletal Content Blueprint and accounts for 10% of the exam

Click here to see the answer

Answer: A. Glucocorticoids

This patient has polymyalgia rheumatica and treatment with glucocorticoids can relieve discomfort and prevent the associated ischemic temporal arteritis, which threatens vision.

B. Cyclophosphamide is an immunosuppressant used in the treatment of acute leukemia.
C. Methotrexate is a folate inhibitor used to treat rheumatoid arthritis, not polymyalgia rheumatica.
D. Azathioprine is an immunosuppressant that is used to treat rheumatic disease and inflammatory bowel disease, not polymyalgia rheumatica.

3. A 35 year-old pregnant patient presents with fever, chills, and left-sided flank pain. On physical examination left-sided CVA tenderness is noted. Urinalysis reveals numerous white blood cells and white blood cell casts. Which of the following is the most appropriate treatment?

A. Oral ciprofloxacin (Cipro)
B. Oral trimethoprim-sulfamethoxazole (Bactrim)
C. IV gentamicin (Garamycin)
D. IV ceftriaxone (Rocephin)

This condition is covered as part of the NCCPA Genitourinary Content Blueprint and accounts for 6% of the exam

Click here to see the answer

Answer: D. IV ceftriaxone (Rocephin)

IV cephalosporins are first line treatment of pyelonephritis in a pregnant patient, followed by oral step-down therapy.

A. See B for explanation.
B. The fluoroquinolones and trimethoprim-sulfamethoxazole are contraindicated in pregnancy.
C. Gentamicin is not indicated as first line therapy in the treatment of pyelonephritis in a pregnant patient.

4. A 26 year-old female arrives in the emergency department with friends who say she was standing in front of her church, dressed in a white bathrobe, claiming to be the Virgin Mary and handing out $100 bills to all passers-by. Her friends noted that she had been depressed lately, but now seems completely euphoric. She had a similar episode two years ago. Which of the following is the most appropriate treatment?

A. Inpatient olanzapine (Zyprexa) therapy
B. Inpatient electroconvulsive therapy
C. Outpatient paroxetine (Paxil) therapy
D. Outpatient psychotherapy

This condition is covered as part of the NCCPA Psychiatry Content Blueprint and accounts for 6% of the exam

Click here to see the answer

Answer: A. Inpatient olanzapine (Zyprexa) therapy

Treatment of the manic phase is usually done in the hospital to protect patients from behaviors associated with grandiosity (spending inordinate amounts of money, making embarrassing speeches, etc.). Lithium, valproate, and olanzapine are considered effective in the manic stage; the depressive stage is treated with
antidepressants.

5. A 19 year-old patient was involved in a motor vehicle crash and brought to the emergency department fully immobilized. The patient sustained multiple blunt injuries to the chest and abdomen. During the trauma assessment, there was no blood at the urethral meatus and a Foley catheter was placed. The urine was positive for blood on the dipstick. Which of the following is the most appropriate diagnostic test

A. Retrograde urethrography
B. CT scan of abdomen and pelvis
C. Serum haptoglobin
D. Urine myoglobin

Click here to see the answer

Answer: B. CT scan of abdomen and pelvis

CT scan of the abdomen and pelvis is indicated in blunt trauma including those resulting in hematuria or when renal injury is suspected.

A. A retrograde urethrogram should be performed when blood is found at the external urinary meatus prior to insertion of a catheter.
C. A decreased serum haptoglobin is seen in hemolysis and does not provide information on renal status.
D. A positive test for blood in the absence of red blood cells on urine examination suggests myoglobinuria, and should be confirmed by electrophoresis.

6. A 48 year-old male presents with complaints of heartburn that occurs approximately 45 minutes after eating about three times a week that is relieved by antacids. He claims to have followed advice about elevating the head of the bed, avoiding spicy foods, and losing weight, but continues to have heartburn. Which of the following is the most appropriate next step?

A. Ranitidine (Zantac)
B. Sucralfate (Carafate)
C. Metoclopramide (Reglan)
D. Misoprostol (Cytotec)

GERD (Gastroesophageal reflux disease) is covered as part of the NCCPA GI and Nutrition Content Blueprint and accounts for 10% of the exam

Click here to see the answer

Answer: A. Ranitidine (Zantac)

Ranitidine, an H2 receptor blocker, is indicated for the treatment of mild, intermittent symptoms of gastroesophageal reflux disease.

B. Sucralfate is used in the treatment of duodenal ulcers.
C. Metoclopramide is indicated for the treatment of gastroparesis as a first-line agent and as a second-line agent in the treatment of refractory gastroesophageal reflux.
D. Misoprostol is indicated for the prevention of NSAID-induced gastritis.

7. Radioactive iodine (I131) is most successful in treating hyperthyroidism that results from

A. Grave’s disease.
B. subacute thyroiditis.
C. Hashimoto’s thyroiditis.
D. papillary thyroid carcinoma.

Watch this ReelDx Video of a 16-year-old with ADHD presents with chest pain and exophthalmos

Diseases of the thyroid gland are covered as part of the NCCPA Endocrinology Content Blueprint and accounts for 6% of the exam

Click here to see the answer

Answer: A. Grave’s disease.

Radioactive iodine (I131) is an excellent method to destroy overactive thyroid tissue of Grave’s disease.

B. Radioactive iodine is ineffective in subacute thyroiditis due to the thyroid’s low uptake of iodine.
C. Radioiodine uptake is low in Hashimoto’s thyroiditis making radioactive therapy ineffective.
D. Papillary thyroid carcinoma is a common thyroid malignancy and should be treated by a thyroidectomy.

8. An 8 year-old boy is brought to a health care provider complaining of dyspnea and fatigue. On physical examination, a continuous machinery murmur is heard best in the second left intercostal space and is widely transmitted over the precordium. The most likely diagnosis is

A. ventricular septal defect
B. atrial septal defect
C. congenital aortic stenosis
D. patent ductus arteriosus

This disorder is covered as part of the NCCPA Cardiology Content Blueprint and accounts for 9% of the exam

Click here to see the answer

Answer: D. patent ductus arteriosus.

Patent ductus arteriosus is classically described in children as a continuous machinery-type murmur that is widely transmitted across the precordium.

A. Ventricular septal defect causes a holosystolic murmur rather than a continuous machinery-like murmur.
B. Atrial septal defect causes a fixed split S2 rather than a continuous systolic heart murmur.
C. Congenital aortic stenosis causes a crescendo-decrescendo systolic murmur heard best in the second intercostal space.

9. A 63 year-old male with history of hypertension and tobacco abuse presents complaining of dyspnea on exertion for two weeks. The patient admits to one episode of chest discomfort while shoveling snow which was relieved after five minutes of rest. Vital signs are BP 130/70, HR 68, RR 14. Heart exam reveals regular rate and rhythm, normal S1 and S2, no murmur, gallop, or rub. Lungs are clear to auscultation bilaterally. There is no edema noted. Which of the following is the most appropriate initial diagnostic study for this patient?

A. Helical CT scan
B. Chest x-ray
C. Nuclear stress test
D. Cardiac catheterization

Angina is covered as part of the NCCPA Cardiology Content Blueprint which accounts for 10% of the exam

Click here to see the answer

Answer: C. Nuclear stress test

In patients with classic symptoms of angina, nuclear stress testing is the most widely used test for diagnosis of ischemic heart disease.

A. Helical CT scan aids in the diagnosis of pulmonary embolism, not in the evaluation of angina.
B. Chest x-ray is not used as a diagnostic study to evaluate symptoms of angina or coronary heart disease.
D. Coronary angiography is indicated in patients with classic stable angina who are severely symptomatic despite medical therapy and are being considered for percutaneous intervention (PCI), patients with troublesome symptoms that are difficult to diagnose, angina symptoms in a patient who has survived sudden cardiac death event, patients with ischemia on noninvasive testings.

10. A 52 year-old male with history of hypertension and hyperlipidemia presents with an acute myocardial infarction. Urgent cardiac catheterization is performed and shows a 90% occlusion of the left anterior descending artery. The other arteries have minimal disease. Ejection fraction is 45%. Which of the following is the treatment of choice in this patient?

A. Coronary artery bypass grafting (CABG)
B. Streptokinase
C. Percutaneous coronary intervention (PCI)
D. Warfarin (Coumadin)

Acute myocardial infarction [Non-ST-segment elevation (NSTEMI)] and [ST-segment elevation myocardial infarction (STEMI)] are covered as part of the NCCPA Cardiology Content Blueprint which represents 16% of your exam

Click here to see the answer

Answer: C. Percutaneous coronary intervention (PCI)

Immediate coronary angiography and primary percutaneous coronary intervention have been shown to be superior to thrombolysis.

A. Percutaneous coronary intervention is a better, less invasive alternative to CABG for single vessel coronary artery disease.
B. Streptokinase is not commonly used for the treatment of acute myocardial infarction because it is ineffective at opening the occluded artery and reducing mortality. Streptokinase would be harmful because it would increase the risk of bleeding.
D. Warfarin is used to prevent thrombosis and not for acute treatment.

Looking for all the podcast episodes?

This FREE series is limited to every other episode, you can download and enjoy the complete audio series by joining The PANCE and PANRE Exam Academy + SMARTYPANCE

I will be releasing new episodes every few weeks. The Academy is discounted, so sign up now.

Resources and Links From The Show

This Podcast is also available on iTunes and Stitcher Radio for Android

  1. iTunes: The Audio PANCE AND PANRE Podcast iTunes
  2. Stitcher Radio: The Audio PANCE and PANRE Podcast Stitcher

itunes_logo-1

Download The Content Blueprint Checklist

Follow this link to download your FREE copy of the Content Blueprint Checklist

Print it up and start crossing out the topics you understand, marking the ones you don’t and making notes of key terms you should remember. The PDF version is interactive and linked directly to the individual lessons on SMARTY PANCE.

pance-and-panre-nccpa-content-blueprint

Download

The post Episode 55: The Audio PANCE and PANRE Board Review Podcast – Mixed Content Blueprint Review appeared first on The Audio PANCE and PANRE.

]]>
Welcome to episode 55 of the FREE Audio PANCE and PANRE Physician Assistant Board Review Podcast. Join me as I cover ten PANCE and PANRE Board review questions from the SMARTYPANCE course content following the NCCPA™ content blueprint (download the FRE... Welcome to episode 55 of the FREE Audio PANCE and PANRE Physician Assistant Board Review Podcast.
Join me as I cover ten PANCE and PANRE Board review questions from the SMARTYPANCE course content following the NCCPA™ content blueprint (download the FREE cheat sheet).
This week we will be covering ten general board review questions based on the NCCPA PANCE and PANRE Content Blueprint. 
Below you will find an interactive exam to complement the podcast.
I hope you enjoy this free audio component to the examination portion of this site. The full board review includes over 2,000 interactive board review questions and is available to all members of the PANCE and PANRE Academy and SMARTYPANCE which are now bundled together into one very low price.

* You can download and listen to past FREE episodes here, on iTunes, on Google Play Music or Stitcher Radio.
* You can listen to the latest episode, take an interactive quiz and download your results below.

Listen Carefully Then Take The Quiz
If you can’t see the audio player click here to listen to the full episode.
Episode 55 PANCE and PANRE Podcast Quiz
The following 10 questions are linked to NCCPA Content Blueprint lessons from the SMARTYPANCE and PANRE Board review website. If you are a member you will be able to log in and view this interactive video content.
1. Which of the following physical examination findings would be consistent with a pleural effusion?
A. Hyperresonance to percussion
B. Increased tactile fremitus
C. Unilateral lag on chest expansion
D. Egophony
Pleural effusions are covered as part of the NCCPA Content Blueprint Pulmonology section which accounts for 10% of the exam

View this ReelDx patient video case of a 68-year-old female complaining of bilateral chest pain and difficulty breathing. Included in this lesson is the Picmonic explaning Light’s Criteria.
2.]]>
The Physician Assistant Life | Smarty PANCE full clean 19:30 285
Episode 53: General Surgery End of Rotation Exam – The Audio PANCE and PANRE Podcast http://podcast.thepalife.com/episode-53-general-surgery-end-rotation-exam-audio-pance-panre-podcast/ Thu, 05 Oct 2017 06:03:07 +0000 http://podcast.thepalife.com/?p=280 http://podcast.thepalife.com/episode-53-general-surgery-end-rotation-exam-audio-pance-panre-podcast/#respond http://podcast.thepalife.com/episode-53-general-surgery-end-rotation-exam-audio-pance-panre-podcast/feed/ 0 <p>Welcome to episode 53 of the FREE Audio PANCE and PANRE Physician Assistant Board Review Podcast. Join me as I cover ten PANCE and PANRE Board review questions from the Academy course content following the NCCPA™ content blueprint (download the FREE cheat sheet) and the PAEA End of Rotation™ Exams Blueprints & Topic Lists. This week we will be […]</p> <p>The post <a rel="nofollow" href="http://podcast.thepalife.com/episode-53-general-surgery-end-rotation-exam-audio-pance-panre-podcast/">Episode 53: General Surgery End of Rotation Exam – The Audio PANCE and PANRE Podcast</a> appeared first on <a rel="nofollow" href="http://podcast.thepalife.com">The Audio PANCE and PANRE</a>.</p> General Surgery End of Rotation Exam - The Audio PANCE and PANRE Board Review Podacst - The PA Life and SMARTY PANCEWelcome to episode 53 of the FREE Audio PANCE and PANRE Physician Assistant Board Review Podcast.

Join me as I cover ten PANCE and PANRE Board review questions from the Academy course content following the NCCPA™ content blueprint (download the FREE cheat sheet) and the PAEA End of Rotation™ Exams Blueprints & Topic Lists.

This week we will be covering ten General Surgery End of Rotation Exam Questions that are now part of the SMARTYPANCE website. 

Below you will find an interactive exam to complement the podcast.

I hope you enjoy this free audio component to the examination portion of this site. The full board review includes over 2,000 interactive board review questions and is available to all members of the PANCE and PANRE Academy and SMARTYPANCE which are now bundled together into one very low price.

Listen Carefully Then Take The Quiz

If you can’t see the audio player click here to listen to the full episode.

Episode 51: General Surgery Endo of Rotation Exam Podcast Quiz

The following 10 questions are linked to NCCPA™ Content Blueprint lessons from the SMARTYPANCE and PANRE Board review website. If you are a member you will be able to log in and view this interactive video content.

1. An 18-year-old patient has a tibia/fibula fracture following a motorcycle crash. Twelve hours later the patient presents with increased pain despite adequate doses of analgesics and immobilization. Which of the following is the most likely diagnosis?

A. avascular necrosis
B. myositis ossificans
C. compartment syndrome
D. reflex sympathetic dystrophy

Click here to see the answer

Answer: C. Compartment syndrome

Compartment syndrome is characterized by a pathological increase of pressure within a closed space and results from edema or bleeding within the compartment. It may occur as an early local complication of fracture.

A. Avascular necrosis occurs primarily in muscles post-traumatically and may not arise for several months after an injury.

B. Myositis ossificans is a late complication of fracture resulting from disruption of the blood supply to the bone.

D. Reflex sympathetic dystrophy is characterized by painful wasting of the hand muscles that may be secondary to injury and could occur as a late complication.

2. A 25-year-old male presents to the ED with left calf pain and cramping, as well as nausea and vomiting. He admits to “partying with cocaine all night”. He describes his urine as a dark brown color. Serum creatinine kinase (CK) is 1325 IU/L (Normal Range 32-267 IU/L). Which of the following is the initial mainstay of therapy for this condition?

A. IV rehydration
B. Fasciotomy
C. Toradol (Ketorolac)
D. Hydrotherapy

Click here to see the answer

Answer: A. IV rehydration

IV rehydration with crystalloids for 24 to 72 hours is the mainstay of therapy for rhabdomyolysis.

B. Fasciotomy is indicated for compartment syndrome.

C. NSAIDs, such as Toradol (Ketorolac), should not be used due to the vasoconstrictive effects on the kidneys.

D. Hydrotherapy is not useful or indicated for Rhabdomyolysis

3. A 42-year-old male presents complaining of a sudden onset of a severe intermittent pain originating in the flank and radiating into the right testicle. He also complains of nausea and vomiting. On examination, the patient is afebrile but restless. Examination of the abdomen reveals tenderness to palpation along the right flank with no rebound or direct testicular tenderness. Urinalysis reveals a pH of 5.4 and microscopic hematuria but is otherwise unremarkable. Which of the following is the most likely diagnosis?

A. Bladder cancer
B. Nephrolithiasis
C. Acute appendicitis
D. Acute epididymitis

Click here to see the answer

Answer: B. Nephrolithiasis

A sudden onset of severe colicky flank pain associated with nausea and vomiting as well as the absence of rebound or direct testicular tenderness makes nephrolithiasis the most likely diagnosis. This is further supported by the presence of hematuria on the urinalysis.

A. While bladder cancer may have associated microscopic hematuria, it presents with painless hematuria or irritative voiding symptoms.

C. While an acute abdomen, such as acute appendicitis, is in the differential diagnosis, the absence of fever and peritoneal signs makes this diagnosis less likely

D. The absence of fever, as well as non-tenderness to palpation of the testes, suggests a renal rather than gonadal cause of the patient’s symptoms

4. What is the term for blue discoloration about the umbilicus?

A. Cullen’s sign
B. Murphy’s sign
C. Rovsing’s sign
D. Turner sign

Click here to see the answer

Answer: A. Cullen’s sign

Cullen’s sign is a blue discoloration about the umbilicus and can occur in hemorrhagic pancreatitis and results from hemoperitoneum.

B. Murphy’s sign is seen in liver and gallbladder disease by which the patient abruptly halts deep inspiration due to pain in the right upper quadrant while the examiner’s hand is keeping stable pressure to the area.

C. Rovsing’s sign is positive when the patient experiences right-sided abdominal pain with palpation to the left side. This is generally seen in those with appendicitis.

D. Turner sign is also seen in severe, acute pancreatitis but is represented by a green-brown discoloration of bilateral flanks

5. A 25-year-old female presents with right lower quadrant pain, right flank pain, nausea, and vomiting. Her temperature is 39.6 degrees C. There is right CVA tenderness and RLQ tenderness. The pelvic exam is unremarkable. Urinalysis reveals pH 7.0, trace protein, negative glucose, negative ketones, positive blood, and positive nitrates. Specific gravity is 1.022. Microscopy shows 102 RBCs/HPF, 50-75 WBCs/HPF, rare epithelial cells, and WBC casts. The most likely diagnosis is

A. acute salpingitis
B. nephrolithiasis
C. acute pyelonephritis
D. appendicitis

Click here to see the answer

Answer: C. acute pyelonephritis

Acute pyelonephritis presents with flank pain, fever, and generalized muscle tenderness. Urinalysis shows pyuria with leukocyte casts.

A. Acute salpingitis would be suggested if pelvic exam abnormalities were present.

B. Nephrolithiasis does not usually present with fever or casts. Urinalysis will have RBCs present.

D. This scenario is consistent with acute pyelonephritis, not acute appendicitis.

6. A 26-year-old gravida 0 sexually active female presents to the emergency room complaining of colicky pain in her lower abdomen for the past 12 hours. She passed out earlier in the day while trying to have a bowel movement. Her last menstrual period was 6 weeks ago. She has noted vaginal spotting over the last 24 hours. Vital signs show Temp 37 degrees C, BP 96/60mmHg, P 110, R 16, Oxygen Sat. 98%. Abdominal exam is positive for distension and tenderness. Bowel sounds are decreased. Pelvic exam shows cervical motion and adnexal tenderness. Which of the following is the most likely diagnosis?

A. Ectopic pregnancy 
B. Appendicitis 
C. Crohn’s disease 
D. Pelvic inflammatory disease

Click here to see the answer

Answer: A. Ectopic pregnancy 

High suspicion for ectopic pregnancy should be maintained when any possible pregnant woman presents with vaginal bleeding or abdominal pain.

B. Appendicitis presents with nausea, vomiting and periumbilical pain that moves to the right lower quadrant of the abdomen.

C. Crohn’s disease is more common in women and may present with an acute abdomen. However, the pelvic examination would be normal.

D. In pelvic inflammatory disease, the temperature is usually above 38 degrees C and pelvic pain usually follows onset of cessation of menses

7. A 12-year-old boy presents to the office with pain in his legs with activity gradually becoming worse over the past month. He is unable to ride a bicycle with his friends due to the pain in his legs. Examination of the heart reveals an ejection click and accentuation of the second heart sound. Femoral pulses are weak and delayed compared to the brachial pulses. Blood pressure obtained in both arms is elevated. Chest x-ray reveals rib notching. Which of the following is the most likely diagnosis?

A. abdominal aortic aneurysm
B. pheochromocytoma
C. coarctation of the aorta
D. thoracic outlet syndrome

Click here to see the answer

Answer: C. coarctation of the aorta

Coarctation is a discrete or long segment of narrowing adjacent to the left subclavian artery. As a result of the coarctation, systemic collaterals develop. X-ray findings occur from the dilated and pulsatile intercostal arteries and the “3” is due to the coarctation site with proximal and distal dilations.

A. Abdominal aortic aneurysm is usually asymptomatic until the patient has dissection or rupture. It is uncommon in a child.

B. Pheochromocytoma classically causes paroxysms of hypertension due to catecholamine release from the adrenal medulla but does not cause variations in blood pressure in the upper and lower extremities.

D. Thoracic outlet syndrome occurs when the brachial plexus, subclavian artery, or subclavian vein becomes compressed in the region of the thoracic outlet. It is the most common cause of acute arterial occlusion in the upper extremity of adults under 40 years old

8. A 28-year-old male presents with burns sustained from hot grease splashed on his left hand earlier this afternoon. The burn extends from his palm to the volar aspect of his wrist and has an erythematous base, covered by an intact blister. There are a few small scattered blisters over the dorsum of the left hand. Which of the following is the initial intervention of choice?

A. Tetanus prophylaxis 
B. Admission to a burn unit 
C. Intravenous fluid administration 
D. Debridement of blisters

Click here to see the answer

Answer: A. Tetanus prophylaxis

Tetanus prophylaxis should be initially considered in all burn patients

B. Admission to a burn unit is not indicated for adult patients with uncomplicated partial thickness burns covering less than 15 to 20% of total body surface area (TBSA).

C. IV fluids are indicated for severe partial thickness burns covering more than 10% TBSA or in burns with complications.

D. Debridement of blisters is controversial, however blisters on the palms and soles should remain intact.

9. Which of the following is the selected method for the prevention of venous thromboembolism in a 38-year-old male undergoing an inguinal hernia repair?

A. early ambulation
B. elastic stockings
C. intermittent pneumatic compression
D. low-molecular weight heparin

Click here to see the answer

Answer: A. early ambulation 

Early ambulation is recommended for prophylaxis of venous thromboembolism in low-risk, minor procedures when the patient is under 40 years of age and there are no clinical risk factors.

B. Elastic stockings are indicated for patients at moderate risk of venous thromboembolism in ages 40-60 with minor procedures with additional thrombosis risk factor, or major operations for patients under age 40 without additional clinical risk factors.

C. Intermittent pneumatic compression is indicated in patients undergoing a major operation plus an increased risk of bleeding.

D. Low molecular weight heparin is indicated in patients undergoing orthopedic surgery, neurosurgery, or trauma with an identifiable risk factor for thromboembolism.

10. A patient presents with abdominal pain in the right lower quadrant, examination reveals increased pain in the right lower quadrant on deep palpation of the left lower quadrant. This is commonly known as which of the following?

A. Psoas sign
B. Murphy’s sign
C. Rovsing’s sign
D. Obturator sign

Click here to see the answer

Answer: C. Rovsing’s sign 

A positive Rovsing’s sign can be elicited in a patient with appendicitis when increased pain occurs in the right lower quadrant upon palpation of the left lower quadrant. 

A. Psoas sign is right lower quadrant pain with right leg extension seen in acute appendicitis

B. Murphy’s sign is seen in liver and gallbladder disease in which the patient abruptly halts deep inspiration due to discomfort as the examiner’s hand applies pressure to the right upper quadrant.

D. Obturator sign is right lower quadrant pain with internal rotation of the hip seen in acute appendicitis

Appendicitis: RLQ pain, + Obturator and Psoas sign

Looking for all the podcast episodes?

This FREE series is limited to every other episode, you can download and enjoy the complete audio series by joining The PANCE and PANRE Exam Academy + SMARTYPANCE

I will be releasing new episodes every few weeks. The Academy is discounted, so sign up now.

Resources and Links From The Show

This Podcast is also available on iTunes and Stitcher Radio for Android

  1. iTunes: The Audio PANCE AND PANRE Podcast iTunes
  2. Stitcher Radio: The Audio PANCE and PANRE Podcast Stitcher

itunes_logo-1

Download The Content Blueprint Checklist

Follow this link to download your FREE copy of the Content Blueprint Checklist

Print it up and start crossing out the topics you understand, marking the ones you don’t and making notes of key terms you should remember. The PDF version is interactive and linked directly to the individual lessons on SMARTY PANCE.

pance-and-panre-nccpa-content-blueprint

FREE Download

The post Episode 53: General Surgery End of Rotation Exam – The Audio PANCE and PANRE Podcast appeared first on The Audio PANCE and PANRE.

]]>
Welcome to episode 53 of the FREE Audio PANCE and PANRE Physician Assistant Board Review Podcast. Join me as I cover ten PANCE and PANRE Board review questions from the Academy course content following the NCCPA™ content blueprint (download the FREE ch... Welcome to episode 53 of the FREE Audio PANCE and PANRE Physician Assistant Board Review Podcast.
Join me as I cover ten PANCE and PANRE Board review questions from the Academy course content following the NCCPA™ content blueprint (download the FREE cheat sheet) and the PAEA End of Rotation™ Exams Blueprints & Topic Lists.
This week we will be covering ten General Surgery End of Rotation Exam Questions that are now part of the SMARTYPANCE website. 
Below you will find an interactive exam to complement the podcast.
I hope you enjoy this free audio component to the examination portion of this site. The full board review includes over 2,000 interactive board review questions and is available to all members of the PANCE and PANRE Academy and SMARTYPANCE which are now bundled together into one very low price.

* You can download and listen to past FREE episodes here, on iTunes, on Google Play Music or Stitcher Radio.
* You can listen to the latest episode, take an interactive quiz and download your results below.

Listen Carefully Then Take The Quiz
If you can’t see the audio player click here to listen to the full episode.
Episode 51: General Surgery Endo of Rotation Exam Podcast Quiz
The following 10 questions are linked to NCCPA™ Content Blueprint lessons from the SMARTYPANCE and PANRE Board review website. If you are a member you will be able to log in and view this interactive video content.
1. An 18-year-old patient has a tibia/fibula fracture following a motorcycle crash. Twelve hours later the patient presents with increased pain despite adequate doses of analgesics and immobilization. Which of the following is the most likely diagnosis?
A. avascular necrosis
B. myositis ossificans
C. compartment syndrome
D. reflex sympathetic dystrophy
2. A 25-year-old male presents to the ED with left calf pain and cramping, as well as nausea and vomiting.]]>
The Physician Assistant Life | Smarty PANCE full clean 19:22 280
Episode 51: The Audio PANCE and PANRE Board Review Podcast – Comprehensive Audio Quiz http://podcast.thepalife.com/episode-51-audio-pance-panre-board-review-podcast-comprehensive-audio-quiz/ Tue, 11 Jul 2017 16:40:07 +0000 http://podcast.thepalife.com/?p=271 http://podcast.thepalife.com/episode-51-audio-pance-panre-board-review-podcast-comprehensive-audio-quiz/#respond http://podcast.thepalife.com/episode-51-audio-pance-panre-board-review-podcast-comprehensive-audio-quiz/feed/ 0 <p>Welcome to episode 51 of the FREE Audio PANCE and PANRE Physician Assistant Board Review Podcast. Join me as I cover ten PANCE and PANRE Board review questions from the Academy course content following the NCCPA™ content blueprint (download the FREE cheat sheet). This week we will be covering ten general board review questions based on the NCCPA PANCE […]</p> <p>The post <a rel="nofollow" href="http://podcast.thepalife.com/episode-51-audio-pance-panre-board-review-podcast-comprehensive-audio-quiz/">Episode 51: The Audio PANCE and PANRE Board Review Podcast – Comprehensive Audio Quiz</a> appeared first on <a rel="nofollow" href="http://podcast.thepalife.com">The Audio PANCE and PANRE</a>.</p> The Audio PANCE and PANRE Physician Assistant Board Review Episode 51Welcome to episode 51 of the FREE Audio PANCE and PANRE Physician Assistant Board Review Podcast.

Join me as I cover ten PANCE and PANRE Board review questions from the Academy course content following the NCCPA™ content blueprint (download the FREE cheat sheet).

This week we will be covering ten general board review questions based on the NCCPA PANCE and PANRE Content Blueprint. 

Below you will find an interactive exam to complement the podcast.

I hope you enjoy this free audio component to the examination portion of this site. The full board review includes over 2,000 interactive board review questions and is available to all members of the PANCE and PANRE Academy and SmartyPANCE which are now bundled together into one very low price.

Listen Carefully Then Take The Quiz

If you can’t see the audio player click here to listen to the full episode.

Episode 51 PANCE and PANRE Podcast Quiz

The following 10 questions are linked to NCCPA Content Blueprint lessons from the SMARTYPANCE and PANRE Board review website. If you are a member you will be able to log in and view this interactive video content.

1. A 42-year-old male with a history of constipation presents with complaints of severe pain with defecation described as feeling like he is “tearing apart.” He has also noted occasional small amounts of blood on toilet paper. External examination of the rectum is unremarkable and an internal rectal exam cannot be performed due to severe pain when attempted. Which of the following is the most likely diagnosis?

A. Proctitis
B. Anal fissure
C. Rectal prolapse
D. Internal hemorrhoids

Anal fissures are covered as part of the NCCPA Content Blueprint GI and Nutrition which accounts for 10% of the exam

View this ReelDx patient video case of a 51-year-old with rectal bleeding and abdominal pain (proctitis)

Click here to see the answer

Answer: B. Anal fissures

Anal fissures are easily diagnosed from history alone with the classic finding of severe pain upon defecation. Constipation is also a common cause of the trauma that leads to the development of a fissure.

A. Proctitis is usually caused by anorectal infections that produce symptoms of anorectal discomfort, tenesmus, constipation, and discharge from the rectum.
C. Rectal prolapse is commonly seen in elderly females with complaints that include an anal mass, rectal bleeding and a change in bowel habits.
D. While internal hemorrhoids may cause rectal bleeding, tearing pain is an uncommon complaint unless there is evidence of thrombosis of irreducible tissue.

2. What type of chest pain is most commonly associated with a dissecting aortic aneurysm?

A. Squeezing
B. Dull, aching
C. Ripping, tearing
D. Burning

Dissecting aortic aneurysm is covered as part of the NCCPA Cardiology Content Blueprint and accounts for 16% of the exam

Click here to see the answer

Answer: C. A dissecting aortic aneurysm often presents with a very severe ripping, tearing like pain.

A dissecting aortic aneurysm often presents with a very severe ripping, tearing like pain.

A. Squeezing pain is more characteristic of angina or esophageal pain.
B. Dull, aching pain is more characteristic of chest wall pain, possibly angina, or anxiety.
D. Burning pain is more characteristic of esophageal reflux, esophagitis, or tracheobronchitis.

3. A 15-year-old softball player presents after jamming the distal tip of her finger into severe flexion. She is unable to extend the distal phalanx and she has pain on palpation of the distal interphalangeal joint. X-ray of the hand fails to reveal any associated avulsion fracture. Which of the following is the treatment of choice?

A. Open reduction and internal fixation
B. Continuous extension of the DIP with splinting
C. Continuous flexion of the PIP with splinting
D. Application of short arm cast

Soft tissue injuries of the forearm, wrist, and hand are covered as part of the NCCPA Musculoskeletal Content Blueprint and accounts for 10% of the exam

Click here to see the answer

Answer: B. Continuous extension of the DIP with splinting

The treatment of choice for a tear in the extensor tendon of the finger is continuous extension of the DIP via splinting for 6 to 8 weeks.

D. Short arm casting is indicated in wrist and metacarpal injuries but not in DIP extensor injuries.

4. A patient presents with loss of pigmentation on the back of hands, face, and body folds due to the absence of epidermal melanocytes. There has been improvement with PUVA treatment. Which of the following is the most likely diagnosis?

A. Pityriasis alba
B. Tinea versicolor
C. Vitiligo
D. Melasma

View this ReelDx patient video case of a 7-year-old female with localized, depigmented skin patches; rash

Click here to see the answer

Answer: C. Vitiligo

 Vitiligo is the acquired loss of pigmentation due to the absence of epidermal melanocytes presenting on the back of hands, face, or body folds.

A. Pityriasis alba is caused by dermal inflammation that becomes scaly and hypopigmented.
B. Tinea versicolor is a fungal infection of the skin presenting as oval or circular lesions
D. Melasma is an acquired brown hyperpigmentation involving the face and neck in women during their second or third trimester of pregnancy.

5. Which of the following medications used in the management of anxiety has a delayed onset of action?

A. Buspirone (BuSpar)
B. Diphenhydramine (Benadryl)
C. Lorazepam (Ativan)
D. Butalbital (Fiorinal)

Anxiety disorders are covered as part of the NCCPA Content Psychiatry Blueprint which accounts for 6% of your exam

Click here to see the answer

Answer: A. buspirone (BuSpar)

Buspirone takes several days to weeks for it to have clinical activity.

B. Diphenhydramine works as a histamine blocker and will cause sedation immediately because of its anticholinergic effects.
C. Lorazepam is an anxiolytic medication that has an immediate onset of activity.
D. Butalbital is a short to intermediate-acting barbiturate that has immediate activity.

6. A 57-year-old male was working on his farm when some manure was slung hitting his left eye. He presents several days after with a red, tearing, painful eye. Fluorescein stain reveals uptake over the cornea looking like a shallow crater. Which of the following interventions would be harmful?

A. Ophthalmic antibiotics
B. Pressure patch
C. Examination for visual acuity
D. Copious irrigation

Corneal abrasion is covered as part of the NCCPA EENT Content Blueprint and accounts for 9% of the exam

Click here to see the answer

Answer: B. Pressure patch

Patching of the eye after a corneal abrasion associated with organic material contamination is contraindicated due to increased risk of fungal infection.

A. Ophthalmic antibiotics and copious irrigation are indicated when treating a patient with a suspected corneal ulcer due to an infectious cause.
C. Examination for assessment of visual acuity should be performed.

7. A 75-year-old female falls on her outstretched arm. She sustains a humeral mid-shaft fracture. Nerve impingement occurs due to the fracture. What is the most likely physical examination abnormality that will be encountered?

A. Inability to extend the wrist against resistance
B. Numbness over the deltoid muscle in the shoulder
C. Winging of the scapula
D. Weakness of the rotator cuff

Fractures and dislocations of the forearm, wrist, and hand are covered as part of the NCCPA Musculoskeletal Content Blueprint and accounts for 10% of the exam

Click here to see the answer

Answer: D. Inability to extend the wrist against resistance

The radial nerve is most likely entrapped by this fracture. Radial nerve damage will cause an inability to extend the wrist against resistance.

B. Axillary nerve injury results in numbness over the deltoid muscle; this nerve is more commonly injured in proximal humeral fractures and anterior shoulder dislocations.
C. Injury to the long thoracic nerve causes winging of the scapula due to its innervation of the serratus anterior muscle.
D. Injury to the subscapular nerve results in weakness and pain of the infraspinatus muscle; this injury is commonly seen in volleyball players from repetitive stress.

8. A 16-year-old male involved in a fight sustained a laceration to his right upper eyelid. He is unable to open his eye, and a possible laceration of the globe is suspected. Which of the following is the next step?

A. Use a slit lamp to determine the extent of the injury.
B. Use fluorescein strips to determine the extent of the injury.
C. Apply a metal eye shield and refer to an ophthalmologist.
D. Apply antibiotic ointment to the lid and recheck in 24 hours.

Disorders of they eye are covered as part of the NCCPA EENT Content Blueprint and accounts for 9% of the exam

Blowout Fracture

66-year-old female with acute onset of periorbital deformitySmartyPANCE ReelDx + members can view this real-world case!

Click here to see the answer

Answer: C. Apply a metal eye shield and refer to an ophthalmologist.

Protect the eye from any pressure with a rigid metal eye shield and refer for immediate ophthalmologic consultation. Avoid unnecessary actions that would delay treatment or cause further injury.

9. Which of the following typical findings would be revealed during a sigmoidoscopy on a patient with Crohn’s disease of the intestine?

A. Rectal pseudopolyps
B. Diffuse ulceration and bleeding
C. Sheets of WBCs with inflamed mucosa
D. Intermittent longitudinal mucosal ulcers and fissures

Inflammatory bowel disease is covered as part of the NCCPA Content Blueprint GI and Nutrition which accounts for 10% of the exam

Click here to see the answer

Answer: D. Intermittent longitudinal mucosal ulcers and fissures

Ulcerations tend to be linear with transverse fissures in Crohn’s disease. These skip lesions are common with Crohn’s disease.

A. Rectal pseudopolyps are associated with ulcerative colitis rather than Crohn’s.
B. Diffuse ulcerations and bleeding are more characteristic of ulcerative colitis than Crohn’s disease.
C. Sheets of WBCs or “pseudomembranes” can be detected in patients with pseudomembranous colitis.

10. A 72-year-old patient with a history of hypertension and atrial fibrillation presents with episodes of weakness, numbness, and paresthesia’s in the right arm. At the same time, she notes speech difficulty and loss of vision in her left eye. These symptoms come on abruptly and clear within minutes. Physical examination is normal except for the previously known arrhythmia. Which of the following is the most likely diagnosis?

A. Focal seizure
B. Migraine headache
C. Hypoglycemic episodes
D. Transient ischemic attack

Atrial fibrillation is covered as part of the NCCPA Cardiology Content Blueprint which is 16% of your exam

SmartyPANCE + ReelDx members can view this ReelDx patient case video of an 83-year-old with fast heart rate, palpitations

Click here to see the answer

Answer: D. Transient ischemic attack

This patient’s symptoms are consistent with transient ischemia in the carotid territory. Atrial fibrillation is a risk factor for cerebral emboli.

A. Focal seizures usually cause abnormal motor movement rather than weakness or loss of feeling.
B. Patients with migraines commonly have a history of episodes since adolescence.
C. Hypoglycemic episodes do not present with focal neurological findings.

Looking for all the podcast episodes?

This FREE series is limited to every other episode, you can download and enjoy the complete audio series by joining The PANCE and PANRE Exam Academy + SMARTYPANCE

I will be releasing new episodes every few weeks. The Academy is discounted, so sign up now.

Resources and Links From The Show

This Podcast is also available on iTunes and Stitcher Radio for Android

  1. iTunes: The Audio PANCE AND PANRE Podcast iTunes
  2. Stitcher Radio: The Audio PANCE and PANRE Podcast Stitcher

itunes_logo-1

Download The Content Blueprint Checklist

Follow this link to download your FREE copy of the Content Blueprint Checklist

Print it up and start crossing out the topics you understand, marking the ones you don’t and making notes of key terms you should remember. The PDF version is interactive and linked directly to the individual lessons on SMARTY PANCE.

pance-and-panre-nccpa-content-blueprint

FREE Download

The post Episode 51: The Audio PANCE and PANRE Board Review Podcast – Comprehensive Audio Quiz appeared first on The Audio PANCE and PANRE.

]]>
Welcome to episode 51 of the FREE Audio PANCE and PANRE Physician Assistant Board Review Podcast. Join me as I cover ten PANCE and PANRE Board review questions from the Academy course content following the NCCPA™ content blueprint (download the FREE ch... Welcome to episode 51 of the FREE Audio PANCE and PANRE Physician Assistant Board Review Podcast.
Join me as I cover ten PANCE and PANRE Board review questions from the Academy course content following the NCCPA™ content blueprint (download the FREE cheat sheet).
This week we will be covering ten general board review questions based on the NCCPA PANCE and PANRE Content Blueprint. 
Below you will find an interactive exam to complement the podcast.
I hope you enjoy this free audio component to the examination portion of this site. The full board review includes over 2,000 interactive board review questions and is available to all members of the PANCE and PANRE Academy and SmartyPANCE which are now bundled together into one very low price.

* You can download and listen to past FREE episodes here, on iTunes, on Google Play Music or Stitcher Radio.
* You can listen to the latest episode, take an interactive quiz and download your results below.

Listen Carefully Then Take The Quiz
If you can’t see the audio player click here to listen to the full episode.
Episode 51 PANCE and PANRE Podcast Quiz
The following 10 questions are linked to NCCPA Content Blueprint lessons from the SMARTYPANCE and PANRE Board review website. If you are a member you will be able to log in and view this interactive video content.
1. A 42-year-old male with a history of constipation presents with complaints of severe pain with defecation described as feeling like he is “tearing apart.” He has also noted occasional small amounts of blood on toilet paper. External examination of the rectum is unremarkable and an internal rectal exam cannot be performed due to severe pain when attempted. Which of the following is the most likely diagnosis?
A. Proctitis
B. Anal fissure
C. Rectal prolapse
D. Internal hemorrhoids
Anal fissures are covered as part of the NCCPA Content Blueprint GI and Nutrition which accounts for 10% of the exam
]]>
The Physician Assistant Life | Smarty PANCE full clean 17:39 271
Episode 49: The Audio PANCE and PANRE Board Review Podcast – Comprehensive Audio Quiz http://podcast.thepalife.com/episode-49/ Tue, 16 May 2017 18:40:58 +0000 http://podcast.thepalife.com/?p=262 http://podcast.thepalife.com/episode-49/#respond http://podcast.thepalife.com/episode-49/feed/ 0 <p>Welcome to episode 49 of the FREE Audio PANCE and PANRE Physician Assistant Board Review Podcast. Join me as I cover ten PANCE and PANRE board review questions from the Academy course content following the NCCPA™ content blueprint (download the FREE cheat sheet). This week we will be covering 10 general board review questions based on the NCCPA PANCE […]</p> <p>The post <a rel="nofollow" href="http://podcast.thepalife.com/episode-49/">Episode 49: The Audio PANCE and PANRE Board Review Podcast – Comprehensive Audio Quiz</a> appeared first on <a rel="nofollow" href="http://podcast.thepalife.com">The Audio PANCE and PANRE</a>.</p> The Audio PANCE and PANRE Episode 49Welcome to episode 49 of the FREE Audio PANCE and PANRE Physician Assistant Board Review Podcast.

Join me as I cover ten PANCE and PANRE board review questions from the Academy course content following the NCCPA™ content blueprint (download the FREE cheat sheet).

This week we will be covering 10 general board review questions based on the NCCPA PANCE and PANRE Content Blueprint. 

Below you will find an interactive exam to complement the podcast.

I hope you enjoy this free audio component to the examination portion of this site. The full board review includes over 2,000 interactive board review questions and is available to all members of the PANCE and PANRE Academy and SmartyPANCE.

  • You can download and listen to past FREE episodes here, on iTunes or Stitcher Radio.
  • You can listen to the latest episode, take an interactive quiz and download your results below.

Listen Carefully Then Take The Quiz

If you can’t see the audio player click here to listen to the full episode.

Episode 49 PANCE and PANRE Podcast Quiz

The following 10 questions are linked to NCCPA Content Blueprint lessons from the SmartyPANCE PANCE and PANRE board review website. If you are a member and login you will be able to view this content.

1. Which of the following clinical findings would be seen in a patient with food poisoning caused by Staphylococcus aureus?

A. Ingestion of mayonnaise-based salads 48 hours earlier
B. Bloody diarrhea with mucus for one week
C. Abdominal cramps and vomiting for 48 hours
D. High fever for 1 week

Staphylococcus aureus food poisoning is part of the NCCPA Content Blueprint GI and Nutrition which accounts for 10% of the exam

Click here to see the answer

Answer: C. Abdominal cramps and vomiting for 48 hours

Abdominal cramps, nausea, vomiting, and watery diarrhea typically last 1-2 days with staphylococcal food poisoning.

A. A preformed toxin causes staphylococcal food poisoning; it has a short incubation period of 1-8 hours.
B. Because Staphylococcus aureus does not invade the mucus, blood and mucus are not seen with this noninflammatory cause of food poisoning.
D. Staphylococcal food poisoning may be associated with low-grade fever or subnormal temperature.

2. Which of the following diagnostic tests should be ordered initially to evaluate for suspected deep venous thrombosis

A. Venogram
B. Arteriogram
C. Duplex ultrasound
D. Impedance plethysmography

Venous thrombosis is part of the NCCPA Cardiology Content Blueprint and represents 16% of the exam

Click here to see the answer

Answer: C. Duplex ultrasound

Ultrasound is the technique of choice to detect deep venous thrombosis in the leg.

A. Venogram has been replaced by noninvasive tests due to discomfort, cost, technical difficulties, and complications, such as phlebitis.
B. Thrombophlebitis is a venous problem, not an arterial one. Any unnecessary invasive procedure is potentially harmful.
D. Impedance plethysmography is equivalent to ultrasound in detecting thrombi of the femoral and popliteal veins, but it may miss early, nonocclusive thrombi.

3. A patient presents with signs and symptoms of Cushing’s syndrome. Extensive diagnostic evaluation reveals an ACTH-secreting pituitary adenoma. First-line therapy should consist of

A. pituitary radiation
B. medical adrenalectomy
C. transsphenoidal resection of the tumor
D. amiloride (Midamor)

Cushings disease is part of the NCCPA Content Blueprint Endocrinology which represents 6% of your exam

Click here to see the answer

Answer: C. transsphenoidal resection of the tumor

Transsphenoidal resection of the tumor cures about 80% of patients. The remainder can be given a combination of pituitary radiation and medical adrenalectomy with one or more drugs. If these procedures fail, the last option is bilateral adrenalectomy.

4. A 26-year-old man is stung by a bee, and shortly thereafter, a wheal develops at the site of the sting. He soon feels flushed and develops hives, rhinorrhea, and tightness in the chest. He is seen in the urgent care center. Immediate therapy should be to

A. transfer him to a local hospital emergency department
B. apply a cold compress to site of the sting
C. administer subcutaneous epinephrine
D. administer oral albuterol

Click here to see the answer

Answer: C. administer subcutaneous epinephrine

Epinephrine hydrochloride 1:1000, 0.2 to 0.5 mL subcutaneously is indicated for the initial treatment of this systemic reaction. Additional injections may be given every 20 to 30 minutes if needed.

A. Systemic (anaphylactic) reactions can rapidly become life-threatening. Delay in treatment may cause death.
B. This is only supportive local therapy and does not address the need to treat the systemic reaction present.
D. Albuterol is indicated in the presence of bronchospasm (suggested by the presence of chest tightness) but would be delivered by an aerosol, not an oral, route.

5. An 18-year-old sexually active female was seen in the student health clinic 1 week ago for a sore throat. A streptococcal antigen test was positive, and she was given a prescription for oral penicillin. After 3 days, she stopped her medication because she felt better. She now presents with a severe sore throat. On physical examination, she has a temperature of 102.6° F (39.2° C), marked pharyngeal erythema, medial deviation of the soft palate on the left, tender left anterior cervical adenopathy, and a “hot potato” voice. The rest of her history and physical examination are unremarkable. Which of the following is the most likely diagnosis?

A. Recurrent streptococcal pharyngitis
B. Infectious mononucleosis
C. Gonococcal pharyngitis
D. Peritonsillar abscess

Peritonsillar abscess is covered as part of the NCCPA Content Blueprint EENT which accounts for 6% of your exam

Peritonsillar abcess

View this ReelDx patient video case ~4d pain back of throat; swelling; difficulty swallowing

SmartyPANCE ReelDx + members can view this real-world case!

Click here to see the answer

Answer: D. Peritonsillar abscess

The soft palate deviation and a muffled voice are classic signs of peritonsillar abscess.

A. This presentation suggests a complication of an incompletely treated streptococcal pharyngitis rather than recurrent disease
B. Infectious mononucleosis may present with a severe sore throat and cervical adenopathy in this age group, but would not cause deviation of the soft palate or the muffled voice
C. Gonococcal pharyngitis usually follows a more indolent course than this patient’s presentation

6. Which of the following would provide the most specific information regarding the functional cardiac status in a patient with chronic heart failure?

A. Electrocardiogram
B. Chest x-ray
C. Serum electrolytes
D. Echocardiogram

Heart Failure is covered as part of the NCCPA Content Blueprint Cardiology and accounts for 16% of the exam

Heart Failure 2

View this SMARTYPANCE ReelDx patient video case: 45-year-old female with difficulty breathing; edema; 8/10 chest pain and whole-body pain

SmartyPANCE ReelDx + members can view this real-world case!

Click here to see the answer

Answer: D. Echocardiogram

An echocardiogram will estimate ejection fraction, which is an indicator of left ventricular function.

A. Electrocardiogram offers no specific information of functional status but may provide clues about the cause.
B. A chest x-ray may show findings of chronic heart failure, such as cardiomegaly or pulmonary congestion, but does not reflect cardiac functional status.
C. Serum electrolytes may be abnormal, either as a result of heart failure, or as a contributing factor, but they do not indicate functional status.

7.  A 33-year-old female presents for follow-up of her Pap smear that showed a low-grade squamous intraepithelial lesion. Reflex HPV testing is positive. Which of the following is the most appropriate diagnostic procedure?

A. Cone biopsy
B. Aspiration needle biopsy
C. Dilation and curettage
D. Colposcopy-directed biopsy

Cervical dysplasia and cervical cancer are covered as part of the NCCPA Content Blueprint Reproductive system and accounts for 8% of your exam

Click here to see the answer

Answer: D. Colposcopy-directed biopsy

A colposcopy-directed biopsy is the first diagnostic evaluation indicated for cervical dysplasia.

A. A cervical cone biopsy may be indicated in further evaluation of this patient, but it is dependent on the results of the colposcopy.
B. An aspiration needle biopsy has no role in the evaluation of cervical dysplasia.
C. Dilatation and curettage has no role in either the diagnosis or treatment of isolated cervical dysplasia.

8. A decrease in the fetal heart rate (FHR) occurring late during contractions is noted. The FHR returns to the baseline slowly after the uterine contraction. The physician assistant should be alerted to the possibility of

A. pelvic dystocia
B. precipitous labor
C. fetal head compression
D. placental insufficiency

Fetal distress is covered as part of the NCCPA Content Blueprint Reproductive system and accounts for 8% of your exam

SmartyPANCE ReelDx + members can view this real-world case!

Click here to see the answer

Answer: D. placental insufficiency

Placental insufficiency is the probable cause of fetal distress resulting in late decelerations.

A. Pelvic dystocia, particularly that due to small bony architecture, is the most common cause of passage abnormalities and is not directly associated with FHR decelerations.
B. This refers to the length of labor, not decelerations in FHR.
C. The drop in FHR is caused by an interference with uterine blood flow to the intervillous space causing an early, not late, deceleration.

9. A 36-year-old patient with cardiomyopathy secondary to viral myocarditis develops fatigue, increasing dyspnea, and lower extremity edema over the past 3 days. He denies fever. A chest x-ray shows no significant increase in heart size but reveals prominence of the superior pulmonary vessels. Based on these clinical findings, which of the following is the most likely diagnosis?

A. Heart failure
B. Subacute bacterial endocarditis
C. Pulmonary embolus
D. Pneumonia

Heart Failure is covered as part of the NCCPA Content Blueprint Cardiology and accounts for 16% of the exam

Click here to see the answer

Answer: A. Heart failure

Given the presence of cardiomyopathy, the patient’s heart has decreased functional reserve. The symptoms and chest x-ray findings are typical of congestive heart failure.

B. Endocarditis occurs as a result of an infection that primarily occurs in the blood stream. Endocarditis would present with signs of infection or seeding rather than signs of heart failure.
C. Pulmonary embolus usually presents with an acute onset of chest pain, severe dyspnea, and anxiety.
D. Pneumonia is less likely since there is no fever and edema is not usually associated with pneumonia.

10. Which of the following is the first-line treatment for symptomatic bradyarrhythmias due to sick sinus syndrome (SSS)?

A. Permanent pacemaker
B. Radiofrequency ablation
C. Antiarrhythmics
D. Anticoagulation therapy

Sick sinus syndrome is covered as part of the NCCPA Cardiology Content Blueprint which is 16% of your exam

Click here to see the answer

Answer: A. Permanent pacemaker

Permanent pacemakers are the therapy of choice in patients with symptomatic bradyarrhythmias in sick sinus syndrome.

B. Radiofrequency ablation is used for the treatment of accessory pathways in the heart.

Looking for all the podcast episodes?

This FREE series is limited to every other episode, you can download and enjoy the complete audio series by joining The PANCE and PANRE Exam Academy + SMARTYPANCE

I will be releasing new episodes every few weeks. The Academy is discounted, so sign up now.

Resources and Links From The Show

This Podcast is also available on iTunes and Stitcher Radio for Android

  1. iTunes: The Audio PANCE AND PANRE Podcast iTunes
  2. Stitcher Radio: The Audio PANCE and PANRE Podcast Stitcher

itunes_logo-1

Download The Content Blueprint Checklist

Follow this link to download your FREE copy of the Content Blueprint Checklist

Print it up and start crossing out the topics you understand, marking the ones you don’t and making notes of key terms you should remember. The PDF version is interactive and linked directly to the individual lessons on SMARTY PANCE.

pance-and-panre-nccpa-content-blueprint

FREE Download

The post Episode 49: The Audio PANCE and PANRE Board Review Podcast – Comprehensive Audio Quiz appeared first on The Audio PANCE and PANRE.

]]>
Welcome to episode 49 of the FREE Audio PANCE and PANRE Physician Assistant Board Review Podcast. Join me as I cover ten PANCE and PANRE board review questions from the Academy course content following the NCCPA™ content blueprint (download the FREE ch... Welcome to episode 49 of the FREE Audio PANCE and PANRE Physician Assistant Board Review Podcast.
Join me as I cover ten PANCE and PANRE board review questions from the Academy course content following the NCCPA™ content blueprint (download the FREE cheat sheet).
This week we will be covering 10 general board review questions based on the NCCPA PANCE and PANRE Content Blueprint. 
Below you will find an interactive exam to complement the podcast.
I hope you enjoy this free audio component to the examination portion of this site. The full board review includes over 2,000 interactive board review questions and is available to all members of the PANCE and PANRE Academy and SmartyPANCE.

* You can download and listen to past FREE episodes here, on iTunes or Stitcher Radio.
* You can listen to the latest episode, take an interactive quiz and download your results below.

Listen Carefully Then Take The Quiz
If you can’t see the audio player click here to listen to the full episode.
Episode 49 PANCE and PANRE Podcast Quiz
The following 10 questions are linked to NCCPA Content Blueprint lessons from the SmartyPANCE PANCE and PANRE board review website. If you are a member and login you will be able to view this content.
1. Which of the following clinical findings would be seen in a patient with food poisoning caused by Staphylococcus aureus?
A. Ingestion of mayonnaise-based salads 48 hours earlier
B. Bloody diarrhea with mucus for one week
C. Abdominal cramps and vomiting for 48 hours
D. High fever for 1 week
Staphylococcus aureus food poisoning is part of the NCCPA Content Blueprint GI and Nutrition which accounts for 10% of the exam
2. Which of the following diagnostic tests should be ordered initially to evaluate for suspected deep venous thrombosis
A. Venogram
B. Arteriogram
C. Duplex ultrasound
D. Impedance plethysmography
Venous thrombosis is part of the NCCPA Cardiology Content Blueprint and represents 16% of the exam
3. A patient presents with signs and symptoms of Cushing’s syndrome. Extensive diagnostic evaluation reveals an ACTH-secreting pituitary adenoma. First-line therapy should consist of
A. pituitary radiation
B. medical adrenalectomy
C. transsphenoidal resection of the tumor
D. amiloride (Midamor)
full clean 18:13 262
Episode 47: The Audio PANCE and PANRE Podcast – Mixed Content Blueprint Board Review http://podcast.thepalife.com/episode-47-audio-pance-panre-podcast-mixed-content-blueprint-board-review/ Wed, 01 Mar 2017 06:23:50 +0000 http://podcast.thepalife.com/?p=253 http://podcast.thepalife.com/episode-47-audio-pance-panre-podcast-mixed-content-blueprint-board-review/#respond http://podcast.thepalife.com/episode-47-audio-pance-panre-podcast-mixed-content-blueprint-board-review/feed/ 0 <p>Welcome to episode 47 of the FREE Audio PANCE and PANRE Physician Assistant Board Review Podcast. Join me as I cover 10 PANCE and PANRE board review questions from the Academy course content following the NCCPA™ content blueprint (download the FREE cheat sheet). This week we will be covering 10 general board review questions based on the NCCPA PANCE […]</p> <p>The post <a rel="nofollow" href="http://podcast.thepalife.com/episode-47-audio-pance-panre-podcast-mixed-content-blueprint-board-review/">Episode 47: The Audio PANCE and PANRE Podcast – Mixed Content Blueprint Board Review</a> appeared first on <a rel="nofollow" href="http://podcast.thepalife.com">The Audio PANCE and PANRE</a>.</p> Episode 47 Welcome to episode 47 of the FREE Audio PANCE and PANRE Physician Assistant Board Review Podcast.

Join me as I cover 10 PANCE and PANRE board review questions from the Academy course content following the NCCPA™ content blueprint (download the FREE cheat sheet).

This week we will be covering 10 general board review questions based on the NCCPA PANCE and PANRE Content Blueprint. 

Below you will find an interactive exam to complement the podcast.

I hope you enjoy this free audio component to the examination portion of this site. The full genitourinary board review includes over 72 GU specific questions and is available to all members of the PANCE and PANRE Academy and SmartyPANCE

  • You can download and listen to past FREE episodes here, on iTunes or Stitcher Radio.
  • You can listen to the latest episode, take an interactive quiz and download your results below.

Listen Carefully Then Take The Quiz

If you can’t see the audio player click here to listen to the full episode.

Episode 47 PANCE and PANRE Podcast Quiz

The following 10 questions are linked to NCCPA Content Blueprint lessons from the SmartyPANCE PANCE and PANRE board review website. If you are a member and login you will be able to view this content.

1. To relieve dependent edema in a pregnant patient, which of the following should be instituted as treatment?

  1. Limit fluid intake
  2. Elevate the legs
  3. Prescribe thiazide diuretics
  4. Strict avoidance of sodium

Uncomplicated pregnancy is part of the NCCPA Content Blueprint Reproductive System which is 8% of the exam

Click here to see the answer

Answer: B. Elevate the legs.

Dependent edema is a common and rarely serious complication of pregnancy due to impedance of venous return. Leg elevation improves circulation

A. Limitation of fluid is not indicated and may be harmful.
C. Thiazide diuretics are contraindicated and could be harmful.
D. Edema due to impedance of venous return will not respond to sodium restriction.

2. To further assess ascites in a patient, the physician assistant instructs the patient to turn onto one side while performing percussion. Which of the following is the reason for this maneuver?

  1. Testing for shifting of dullness on percussion
  2. Shifting of internal organs making percussion easier
  3. Trying to elicit any pain while moving
  4. Trying to produce a caput medusae

Ascites is a common complication of liver disease. Disorders of the liver is part of the NCCPA GI and Nutrition Content Blueprint and represents 10% of the exam

Click here to see the answer

Answer:  A. Testing for shifting of dullness on percussion.

In ascites, dullness shifts to the more dependent side as the fluid relocates into dependent space, while tympany shifts to the top as the gas-filled organs float to the top of the ascitic fluid

C. Pain with movement is associated with peritonitis and not ascites.
D. Caput medusae is the dilation of the superficial abdominal veins due to increased intra abdominal fluid accumulation. It is visible with the patient standing and does not need a special maneuver for identification.

3.  A 7-year-old child with a history of type 1 diabetes mellitus for 3 years presents for routine follow-up. The mother states that the child has been having nightmares and night sweats. Additionally, his average morning glucose readings have risen from an average of 100 mg/dL to 145 mg/dL over the past week. This child is most likely experiencing

  1. a growth spurt
  2. emotional problems
  3. the Somogyi effect
  4. the dawn phenomenon

Diabetes mellitus type 1 is part of the NCCPA Content Blueprint Endocrinology which represents 6% of your exam

Click here to see the answer

Answer: C. the Somogyi effect.

This refers to nocturnal hypoglycemia, which stimulates counterregulatory hormone release resulting in rebound hyperglycemia

A. Nightmares and night sweats are not associated with growth spurts.
B. With this limited history, it is impossible to label the child as emotionally unstable.
D. This refers to an early morning rise in plasma glucose due to reduced tissue sensitivity to insulin between 5AM and 8AM. It is not associated with nightmares and night sweats.

4. A patient complains of loss of sensation at the level of the umbilicus. Which of the following dermatomes is affected?

  1. T6
  2. T8
  3. T10
  4. T12

Radicular pain is a common symptom of a herniated nucleus pulposus covered as part of the NCCPA Content Blueprint Musculoskeletal System which is 10% of your exam

Click here to see the answer

Answer: C. T10.

The dermatome T10 is at the level of the umbilicus.

5. A 25-year-old female graduate student presents to the student health center for the eighth time in three weeks to be sure she does not have meningitis. She read that there was a student on campus who had meningitis last month, and now she has headaches and is requesting to be tested to make sure she does not have meningitis. She has been evaluated at each visit, and physical examination has been completely normal each time. Which of the following is the most likely diagnosis?

  1. Conversion disorder
  2. Hypochondriasis
  3. Malingering
  4. Somatization disorder

Somatoform disorders are covered as part of the NCCPA Content Blueprint Psychiatry which is 6% of your exam

Click here to see the answer

Answer: B. Hypochondriasis.

Hypochondriasis is the chronic preoccupation with the idea of having a serious disease, which is usually not amenable to reassurance

A. Conversion disorder is characterized by onset of symptoms or deficits mimicking neurologic or medical illness, but the etiology is psychological.
C. Malingering is the intentional production or feigning of physical or psychological signs and symptoms for some gain.
D. Somatization disorder is characterized by complaints of pain, often related to gastrointestinal and sexual dysfunction, and pseudoneurological symptoms

6. What is the mechanism for the relapsing fevers associated with malaria?

  1. Release of malarial merozoites into the bloodstream
  2. Release of toxins from Plasmodium species
  3. Attachment of Plasmodium species to receptor sites on the red blood cells
  4. Invasion of hepatocytes by Plasmodium species

Malaria is covered as part of the NCCPA Content Blueprint Infectious Disease and is 3% of the exam

Click here to see the answer

Answer: A. Release of malarial merozoites into the bloodstream.

Fever, chills, and sweats coincide with the release of merozoites from red blood cells that have been infected with Plasmodium species. Each merozoite may infect a new red blood cell, leading to a cycle of invasion growth, and release.

7. A 29-year-old patient with idiopathic thrombocytopenia purpura (ITP) is treated with prednisone therapy. Despite therapy, platelet counts remain consistently below 20,000/microliter over the course of 6 weeks. Which of the following is the most appropriate intervention for this patient?

  1. Aspirin
  2. Intravenous immunoglobulin
  3. Danazol (Danocrine)
  4. Splenectomy

Idiopathic thrombocytopenic purpura is covered as part of the NCCPA Content Blueprint Hematology and is 3% of your exam

Click here to see the answer

Answer: D. Splenectomy

Persistently low platelet counts (< 20,000) require effective long-term treatment, and splenectomy is the treatment of choice.

A. Aspirin inhibits platelet function and could lead to significant bleeding and death of this patient.
B. Intravenous immunoglobulin can be utilized for short-term treatment, but the platelet count is likely to return to baseline within a month.
C. Danazol is typically reserved for ITP that fails to respond to splenectomy.

8. A 27 year-old female presents to your office for evaluation of weakness, visual loss, and sensory loss over the right great toe. These symptoms have occurred during three episodes approximately three months apart with each episode lasting about three days. Which of the following tests would be most useful in further evaluating this patient?

  1. MRI of the brain
  2. Electromyography
  3. Glucose tolerance test
  4. Electroencephalograph

This condition is covered as part of the NCCPA Content Blueprint Neurology which is 6% of your exam

SmartyPANCE ReelDx + members can view this real-world case!

MS

Click here to see the answer

Answer: A. MRI of the brain.

Multiple sclerosis typically presents with relapsing weakness of the limbs, sensory loss, paresthesias, and visual changes. Diagnosis is based on history and either abnormal brain or spinal cord MRI, or visual, auditory, or somatosensory evoked electrical response.

C. Glucose tolerance test is used in the evaluation of diabetes mellitus.
D. Electroencephalograph is used to evaluate patients for possible seizure disorder.

9. A patient taking bleomycin (Blenoxane) should be monitored for which of the following side effects?

  1. Optic neuritis
  2. Hyperuricemia
  3. Encephalopathy
  4. Pulmonary fibrosis

This condition is is covered as part of the NCCPA Pulmonary Content Blueprint which represents 12% of your exam

Click here to see the answer

Answer: D. Pulmonary fibrosis.

Pulmonary fibrosis and pulmonary infiltrates are known side effects of bleomycin

A. Optic neuritis is a potential side effect of ethambutol, used in the treatment of tuberculosis.
B. Hyperuricemia or encephalopathy are not known side effects of bleomycin.

10. A patient presents for a follow-up visit for chronic hypertension. Which of the following findings may be noted on the fundoscopic examination of this patient?

  1. cherry-red fovea
  2. boxcar segmentation of retinal veins
  3. papilledema
  4. arteriovenous nicking

This condition is a common complications of chronic hypertension covered as part of the NCCPA Cardiology Content Blueprint which is 16% of your exam

Click here to see the answer

Answer: D. arteriovenous nicking

Arteriovenous nicking is common in chronic hypertension

A. Cherry-red fovea and boxcar segmentation of the retinal veins are findings seen in central retinal artery occlusion.
C. Papilledema is noted in conditions causing increased intracranial pressure.

Looking for all the podcast episodes?

This FREE series is limited to every other episode, you can download and enjoy the complete audio series by joining The PANCE and PANRE Exam Academy + SmaryPANCE

I will be releasing new episodes every few weeks. The Academy is discounted, so sign up now.

Resources and Links From The Show

This Podcast is also available on iTunes and Stitcher Radio for Android

  1. iTunes: The Audio PANCE AND PANRE Podcast iTunes
  2. Stitcher Radio: The Audio PANCE and PANRE Podcast Stitcher

itunes_logo-1

Download The Content Blueprint Checklist

Follow this link to download your FREE copy of the Content Blueprint Checklist

Print it up and start crossing out the topics you understand, marking the ones you don’t and making notes of key terms you should remember. The PDF version is interactive and linked directly to the individual lessons on SMARTY PANCE.

pance-and-panre-nccpa-content-blueprint

FREE Download

The post Episode 47: The Audio PANCE and PANRE Podcast – Mixed Content Blueprint Board Review appeared first on The Audio PANCE and PANRE.

]]> Welcome to episode 47 of the FREE Audio PANCE and PANRE Physician Assistant Board Review Podcast. Join me as I cover 10 PANCE and PANRE board review questions from the Academy course content following the NCCPA™ content blueprint (download the FREE che... Welcome to episode 47 of the FREE Audio PANCE and PANRE Physician Assistant Board Review Podcast.
Join me as I cover 10 PANCE and PANRE board review questions from the Academy course content following the NCCPA™ content blueprint (download the FREE cheat sheet).
This week we will be covering 10 general board review questions based on the NCCPA PANCE and PANRE Content Blueprint. 
Below you will find an interactive exam to complement the podcast.
I hope you enjoy this free audio component to the examination portion of this site. The full genitourinary board review includes over 72 GU specific questions and is available to all members of the PANCE and PANRE Academy and SmartyPANCE

* You can download and listen to past FREE episodes here, on iTunes or Stitcher Radio.
* You can listen to the latest episode, take an interactive quiz and download your results below.

Listen Carefully Then Take The Quiz
If you can’t see the audio player click here to listen to the full episode.
Episode 47 PANCE and PANRE Podcast Quiz
The following 10 questions are linked to NCCPA Content Blueprint lessons from the SmartyPANCE PANCE and PANRE board review website. If you are a member and login you will be able to view this content.
1. To relieve dependent edema in a pregnant patient, which of the following should be instituted as treatment?

* Limit fluid intake
* Elevate the legs
* Prescribe thiazide diuretics
* Strict avoidance of sodium

Uncomplicated pregnancy is part of the NCCPA Content Blueprint Reproductive System which is 8% of the exam
2. To further assess ascites in a patient, the physician assistant instructs the patient to turn onto one side while performing percussion. Which of the following is the reason for this maneuver?

* Testing for shifting of dullness on percussion
* Shifting of internal organs making percussion easier
* Trying to elicit any pain while moving
* Trying to produce a caput medusae

Ascites is a common complication of liver disease. Disorders of the liver is part of the NCCPA GI and Nutrition Content Blueprint and represents 10% of the exam
3.  A 7-year-old child with a history of type 1 diabetes mellitus for 3 years presents for routine follow-up. The mother states that the child has been having nightmares and night sweats. Additionally, his average morning glucose readings have risen from an average of 100 mg/dL to 145 mg/dL over the past week. This child is most likely experiencing

]]>
The Physician Assistant Life | Smarty PANCE full clean 17:34 253 Murmur Madness: The Audio PANCE and PANRE Episode 45 http://podcast.thepalife.com/murmur-madness-the-audio-pance-and-panre-episode-45/ Thu, 05 Jan 2017 00:56:12 +0000 http://podcast.thepalife.com/?p=248 http://podcast.thepalife.com/murmur-madness-the-audio-pance-and-panre-episode-45/#respond http://podcast.thepalife.com/murmur-madness-the-audio-pance-and-panre-episode-45/feed/ 0 <p>Welcome to episode 45 of the FREE Audio PANCE and PANRE Physician Assistant Board Review Podcast. Today we are going to take a brief detour away from 10 daily questions and instead cover the murmurs you need to know for your PANCE and PANRE exam. I have still included an 11 question exam at the end […]</p> <p>The post <a rel="nofollow" href="http://podcast.thepalife.com/murmur-madness-the-audio-pance-and-panre-episode-45/">Murmur Madness: The Audio PANCE and PANRE Episode 45</a> appeared first on <a rel="nofollow" href="http://podcast.thepalife.com">The Audio PANCE and PANRE</a>.</p> Murmur Madness The Audio PANCE and PANRE Board Review PodcastWelcome to episode 45 of the FREE Audio PANCE and PANRE Physician Assistant Board Review Podcast.

Today we are going to take a brief detour away from 10 daily questions and instead cover the murmurs you need to know for your PANCE and PANRE exam.

I have still included an 11 question exam at the end of this post so make sure you scroll down after you listen to the podcast.

There are a total of 17 cardiac conditions that may present on exam day in the form of a descriptive sound/murmur.

You will often be given supporting evidence such as the location of auscultation or radiation which you can use to narrow down your options.

There are total of 51 cardiac topics in the  NCCPA™ Cardiology PANCE and PANRE Content Blueprint which represents 16% of the PANCE and PANRE exam.

(click here to download the FREE NCCPA Content Blueprint cheat sheet)

These cardiac conditions and their associated murmurs cover a whopping 33% of the cardiology section of the PANCE and PANRE exam!

Take a listen to this week’s podcast episode

If you can’t see the audio player click here to listen to the full episode.

Let’s break this all down:

Each of the links below opens the corresponding lesson on SmartyPANCE and is available to members (you must be logged in or join now)

There are 9 valvular disorders associated with murmurs:

Valvular Disorders (PEARLS and Flashcards)

  1. Aortic stenosis (ReelDx)
  2. Aortic regurgitation (Diastolic Murmur)
  3. Mitral stenosis (Diastolic)
  4. Mitral regurgitation
  5. Mitral valve prolapse
  6. Tricuspid stenosis
  7. Tricuspid regurgitation
  8. Pulmonary stenosis
  9. Pulmonary regurgitation

Five congenital heart diseases that have corresponding murmurs

Congenital Heart Diseases (PEARLS)

  1. Atrial septal defect
  2. Coarctation of the aorta
  3. Patent ductus arteriosus
  4. Tetralogy of Fallot
  5. Ventricular septal defect

One murmur associated with cardiomyopathy

  1. Hypertrophic cardiomyopathy

And a pair of conditions nested under the label of “other forms of heart disease” that have associated murmurs/heart sounds worth mentioning

Other Forms of Heart Disease (PEARLS)

  1. Acute and subacute bacterial endocarditis
  2. Acute pericarditis

Making Sense of Murmurs

Let’s start with the valvular disorders:

You have 4 heart valves with two main conditions – half are diastolic murmurs and half are systolic murmurs. Here is a helpful mnemonic:

Recalling Common Systolic Heart Murmurs: MR PASS

  • itral
  • egurgitation
  • hysiologic (also known as functional, systolic flow murmur, a heart murmur heard in the absence of cardiac abnormality)
  • ortic
  • tenosis
  • ystolic – All the above murmurs are heard during systole.

MR PASS wins the Most Valuable Player award.

  • itral
  • alve
  • rolapse – Add MVP as another systolic murmur.

MR PASS often hangs around with MS ARD.

  • itral
  • tenosis
  • ortic
  • egurgitation
  • iastolic – All the above murmurs are heard during diastole.

Here are the nine valvular murmurs and their associated descriptions

Remember which are systolic and diastolic this can be very helpful at ruling out or ruling in a condition come exam day.

  1. Aortic Stenosis – Systolic harsh ejection crescendo decrescendo murmur at RUSB (aortic area) with radiation to the neck and apex
  2. Aortic Regurgitation – diastolic – soft high pitched blowing at LSB with patient sitting leaning forward and exhaling
  3. Mitral stenosis – diastolic – low pitched decrescendo rumbling with opening snap at the APEX (the mitral area) enhanced by expiration
  4. Mitral regurgitation – blowing holosystolic murmur loudest at the APEX (the mitral area) with a split S2 that radiates to the axilla and is increased by squatting, handgrip and expiration
  5. Tricuspid Stenosis – diastolic – mid diastolic rumbling at LLSB (tricuspid area) with opening snap
  6. Tricuspid regurgitation – High pitched holosystolic blowing murmur that radiates to the LSB (tricuspid area)
  7. Pulmonic stenosis – harsh midsystolic ejection crescendo-decrescendo murmur with widely split s2 at LSB that radiates to the left shoulder and neck
  8. Pulmonic regurgitation – diastolic –  high pitched, decrescendo murmur at LUSB increase with inspiration
  9. Mitral valve prolapse – Midsystolic ejection click head best at the APEX (the mitral area)

With these valvular murmurs you have 4 auscultation points which can be easily remembered using the mnemonic APT Ment watch this amazing video so you never forget!

Another helpful mnemonic: Aortic Pulmonic Tricuspid Mitral – ll P hysicians T ake M oney!

cardiac-auscultation-locations

  1. Aortic area: right 2nd interspace (Right upper sternal border RUSB)
  2. Pulmonic area: 2nd left interspace (Left upper sternal border LUSB)
  3. Tricuspid area: Left lower sternal border LLSB
  4. Mitral area: APEX

You can use the auscultation point to successfully narrow down your murmur and use the designation of systolic or diastolic to narrow down even further.

Five congenital heart diseases that have corresponding murmurs

Congenital Heart Diseases (PEARLS)

  1. Atrial septal defect – Systolic ejection murmur at 2nd left intercostal space with an early to mid-systolic rumble and fixed splitting of the 2nd heart sound (s2) and CXR will show pulmonary vascular markings.
  2. Coarctation of the aorta – Late systolic ejection murmur-posterior or continuous murmur if collateral flow. Will have absent or weak femoral pulses with a delay of palpable femoral pulse and HTN in arms but low or normal blood pressure in the legs
  3. Patent ductus arteriosus – Continuous, rough, machinery-like murmur, heard best in the first interspaces of the LSB
  4. Tetralogy of Fallot – Harsh systolic ejection murmur heard best at the left sternal border. Associated with bluish skin, trouble gaining weight, and sudden loss of consciousness during crying or feeding
  5. Ventricular septal defect – Harsh high pitched holosystolic murmur heard best at the LSB with ride radiation and a fixed split S2

Several of these conditions have a “tell” that make it easy to identify the condition and they are usually always part of the root of the question.

  • PDA = machinery like murmur (remember that the physician assistant got a patent on his machine).
  • Tetralogy of Fallot usually presents with a young child having TET spells which are described as bluish skin, trouble gaining weight, and sudden loss of consciousness during crying or feeding. 
  • Coarctation will almost always present with absent or weak femoral pulses with a delay of palpable femoral pulse and HTN in arms but low or normal blood pressure in the legs. 
  • ASD is a bit tougher but they will usually give you a wide and fixed splitting of the S2 and patients will be easily fatigued. CXR will show pulmonary vascular markings.
  • VSD will have a high pitched holosystolic murmur and newborns may show dyspnea.

One murmur associated with cardiomyopathy

  1. Hypertrophic cardiomyopathy – Medium-pitched, mid-systolic murmur that decreases with squatting and increases with straining. S4 gallop and apical lift with thick, stiff left ventricle. HCM is the leading cause of sudden death in athletes and may cause angina.

And a pair of conditions nested under the label of “other forms of heart disease” that have associated murmurs/heart sounds worth mentioning

Other Forms of Heart Disease (PEARLS)

  1. Acute and subacute bacterial endocarditis  – A new mitral regurgitant murmur in a patient with a history of IVDA, fever (39.0º C),* and a blood culture that reveals 2 out of 2 positive growth
  2. Acute pericarditis – Although this is not a murmur, it is important to identify a pericardial friction rub heard best with patient upright and leaning forward. Chest pain is also relieved by sitting and/or leaning forward

You can listen to all these murmurs and see their associated waveforms at www.smartypance.com/courses/cardiology

Here is a wonderful video from the Khan Academy of how to approach murmurs video of murmurs

Murmur Flash Cards

Episode 45 PANCE and PANRE Murmur Quiz

You will see from these questions on the PANCE and PANRE things aren’t always so straightforward.

You can take the exam here: thepalife.com/murmur-madness

Looking for all the podcast episodes?

This FREE podcast series is limited to every other episode, you can download and enjoy the complete audio series by joining The PANCE and PANRE Exam Academy + SmartyPANCE

I will be releasing new episodes every few weeks. The Academy is discounted and now includes complimentary access to SmartyPANCE so sign up now.

Included is a full cardiology content blueprint board review exam with over 147 cardiology specific questions and 51 cardiology blueprint topics covered in detail. This is in addition to 1,000’s of additional board review questions and NCCPA content blueprint courses covering all 13 organ areas.

I am also happy to introduce ReelDX™ patient case based integration into many of the SmartyPANCE blueprint lessons. It’s like a virtual rotation from the comfort of your couch!

This podcast is available on iTunes and Stitcher Radio (among others)

  1. iTunes: The Audio PANCE AND PANRE Podcast iTunes
  2. Stitcher Radio: The Audio PANCE and PANRE Podcast Stitcher

The post Murmur Madness: The Audio PANCE and PANRE Episode 45 appeared first on The Audio PANCE and PANRE.

]]>
Welcome to episode 45 of the FREE Audio PANCE and PANRE Physician Assistant Board Review Podcast. Today we are going to take a brief detour away from 10 daily questions and instead cover the murmurs you need to know for your PANCE and PANRE exam. Welcome to episode 45 of the FREE Audio PANCE and PANRE Physician Assistant Board Review Podcast.
Today we are going to take a brief detour away from 10 daily questions and instead cover the murmurs you need to know for your PANCE and PANRE exam.
I have still included an 11 question exam at the end of this post so make sure you scroll down after you listen to the podcast.
There are a total of 17 cardiac conditions that may present on exam day in the form of a descriptive sound/murmur.
You will often be given supporting evidence such as the location of auscultation or radiation which you can use to narrow down your options.
There are total of 51 cardiac topics in the  NCCPA™ Cardiology PANCE and PANRE Content Blueprint which represents 16% of the PANCE and PANRE exam.
(click here to download the FREE NCCPA Content Blueprint cheat sheet)
These cardiac conditions and their associated murmurs cover a whopping 33% of the cardiology section of the PANCE and PANRE exam!
Take a listen to this week’s podcast episode
If you can’t see the audio player click here to listen to the full episode.
Let’s break this all down:
Each of the links below opens the corresponding lesson on SmartyPANCE and is available to members (you must be logged in or join now)
There are 9 valvular disorders associated with murmurs:
Valvular Disorders (PEARLS and Flashcards)

Aortic stenosis (ReelDx)
Aortic regurgitation (Diastolic Murmur)
Mitral stenosis (Diastolic)
Mitral regurgitation
Mitral valve prolapse
Tricuspid stenosis
Tricuspid regurgitation
Pulmonary stenosis
Pulmonary regurgitation

Five congenital heart diseases that have corresponding murmurs
Congenital Heart Diseases (PEARLS)

]]>
The Physician Assistant Life | Smarty PANCE full clean 14:41 248
Episode 43: The Audio PANCE and PANRE Board Review Podcast http://podcast.thepalife.com/episode-43-the-audio-pance-and-panre-board-review-podcast/ Thu, 27 Oct 2016 05:43:30 +0000 http://podcast.thepalife.com/?p=240 http://podcast.thepalife.com/episode-43-the-audio-pance-and-panre-board-review-podcast/#respond http://podcast.thepalife.com/episode-43-the-audio-pance-and-panre-board-review-podcast/feed/ 0 <p>Welcome to episode 43 of the FREE Audio PANCE and PANRE Physician Assistant Board Review Podcast. Join me as I cover 10 PANCE and PANRE board review questions from the Academy course content following the NCCPA™ content blueprint (download the FREE cheat sheet). This week we will continue to take a break from topic specific board review and […]</p> <p>The post <a rel="nofollow" href="http://podcast.thepalife.com/episode-43-the-audio-pance-and-panre-board-review-podcast/">Episode 43: The Audio PANCE and PANRE Board Review Podcast</a> appeared first on <a rel="nofollow" href="http://podcast.thepalife.com">The Audio PANCE and PANRE</a>.</p> episode-43-the-audio-pance-and-panre-board-review-podcastWelcome to episode 43 of the FREE Audio PANCE and PANRE Physician Assistant Board Review Podcast.

Join me as I cover 10 PANCE and PANRE board review questions from the Academy course content following the NCCPA™ content blueprint (download the FREE cheat sheet).

This week we will continue to take a break from topic specific board review and covering 10 general board review questions.

Below you will find an interactive exam to complement the podcast.

I hope you enjoy this free audio component to the examination portion of this site. The full genitourinary board review includes over 72 GU specific questions and is available to all members of the PANCE and PANRE Academy.

  • You can download and listen to past FREE episodes here, on iTunes or Stitcher Radio.
  • You can listen to the latest episode, take an interactive quiz and download your results below.

Listen Carefully Then Take The Quiz

If you can’t see the audio player click here to listen to the full episode.

Episode 43 PANCE and PANRE Podcast Quiz

1. A mother brings in her five-year-old boy for his school physical. She voices some concerns about his readiness for school, saying he seems to be socially immature. She has noticed he does not interact with other children well, and that when he plays with them, he has a tendency to “place them” and then run around them as if they were statues. He rarely cries when he is hurt, and he shrugs off any attempt to hug him. He has good attention to details, and will sit and draw the same geometric shapes over and over again, but does not seem interested in learning the alphabet. He avoids eye contact with anyone. Which of the following is the most likely diagnosis?

  1. Normal 5 year-old
  2. Social phobia
  3. Autism
  4. Avoidant personality

Click here to see the answer

Answer: C. Autism

Children with autism do not tend to make eye contact, and even avoid it. They do not accept comfort when hurt and stiffen up when hugged. They do not tend to play with others, and do not tend to imitate grown-ups in play.
They approach play in a more mechanical way, using others as props rather than interacting with them.

Explanations

  • A. This behavior is not normal for a child this age.
  • B. Social phobia is an excessive and persistent fear of social situations in which the person may be scrutinized by others.
  • D. Avoidant personality disorder is characterized by timidity, social awkwardness, and a pervasive sense of inadequacy and fear of criticism.

2. A 3-week-old male infant presents with recurrent regurgitation after feeding that has progressed to projectile vomiting in the last few days. The mother states that the child appears hungry all of the time. She denies any diarrhea in the child. Which of the following clinical findings is most likely?

  1. Bile-stained vomitus
  2. Hemoccult positive stools
  3. Olive-sized mass in the right upper abdomen
  4. Sausage-shaped mass in the upper-mid abdomen

Click here to see the answer

Answer: C. Olive-sized mass in the right upper abdomen

An olive-sized mass may be palpated in the right upper abdomen in pyloric stenosis and if found, is pathognomonic for pyloric stenosis.

Explanations

  • D. A sausage-shaped mass may be noted in intussusception, not pyloric stenosis.
  • A. Gastric obstruction, such as that seen with pyloric stenosis, causes vomiting that is not bilious.
  • B. Blood-streaked vomitus, but not hemoccult positive stools, may be seen in pyloric stenosis.

3. A 65 year-old with COPD receiving their first pneumococcal polysaccharide PPSV23 vaccination should be revaccinated in

  1. 1 year
  2. 3 years
  3. 5 years
  4. Never

Click here to see the answer

Answer: D. Never

People who are first vaccinated with PPSV23 at age 65 years or older should receive only 1 dose, regardless of their underlying medical condition.

ACIP recommendations for revaccination remain unchanged from the 1997 recommendations. For most persons for whom PPSV23 is indicated, ACIP does not recommend routine revaccination. A second dose of PPSV23 is recommended 5 years after the first dose for persons aged 19–64 years with functional or anatomic asplenia and for persons with immunocompromising conditions . ACIP does not recommend multiple revaccinations because of insufficient data regarding clinical benefit, particularly the degree and duration of protection, and safety.

All adults age 65 years and older should also receive one dose of PCV13.

Click here to review the CDC guidelines

4. Which of the following therapies is recommended for a 13 month-old child with sickle cell disease?

  1. Folic acid and penicillin V
  2. Ferrous sulfate and penicillin V
  3. Folic acid and ferrous sulfate
  4. Folic acid, ferrous sulfate and penicillin V

Click here to see the answer

Answer: A. Folic acid and penicillin V

Patients with sickle cell disease should receive prophylactic penicillin V starting at 2 months of age and folic acid starting at 1 year of age. Ferrous sulfate is not globally recommended for patients with sickle cell disease.

5. A patient with severe COPD presents to the Emergency Department with a 3 day history of increasing shortness of breath with exertion and cough productive of purulent sputum. An arterial blood gas reveals a pH of 7.25, PaCO2 of 70 mmHg and PaO2 of 50 mmHg. He is started on albuterol nebulizer, nasal oxygen at 2 liters per minute, and an IV is started. After one hour of treatment, his arterial blood gas now reveals a pH of 7.15, PaCO2 100 mmHg and PaO2 of 70 mmHg. Which of the following is the most appropriate next step in his treatment?

A. Decrease the oxygen flow rate.
B. Administer oral corticosteroids.
C. Intubate the patient.
D. Administer salmeterol (Serevent)

Click here to see the answer

Answer: C. Intubate the patient.

This person has increasing respiratory failure as indicated by the raising PaCO2 levels. Intubation is required at this time.

Explanations

  • A. Decreasing the oxygen flow rate would be harmful as it would decrease the amount of oxygen delivered to the patient.
  • B. Administration of steroids is an important treatment modality but this patient is in respiratory failure and needs more immediate therapy.
  • D. Long-acting beta agonist therapy such as salmeterol is not utilized for rescue therapy.

6. A patient should be tested for tuberculosis prior to being treated with

A. etanercept (Enbrel).
B. cyclosporine (Neoral).
C. methotrexate (Rheumatrex).
D. prednisone (Deltasone).

Click here to see the answer

Answer: A. etanercept (Enbrel)

Etanercept is an anti-cytokine agent used in the treatment of rheumatoid arthritis and has as a side effect the potential for serious infections. One of these side effects includes reactivation of dormant tuberculosis.

Explanations

B. Cyclosporine, methotrexate, and prednisone do not have the requirement to check for tuberculosis prior to initiating treatment.

7. Which of the following side effects is associated with long-term administration of phenytoin (Dilantin)?

A. Ataxia
B. Hypotension
C. Osteomalacia
D. Cardiac dysrhythmia

Click here to see the answer

Answer: C. Osteomalacia

Osteomalacia, or demineralization of bone, is a side effect of phenytoin that may occur after chronic administration.

Explanations

  • A. Ataxia is associated most often with acute oral overdosage of phenytoin.
  •  B. Cardiac dysrhythmia, with or without hypotension, is an expected side effect of rapid IV phenytoin administration.

8. Hairy leukoplakia has the greatest prevalence of distribution on the

A. palate.
B. floor of the mouth.
C. lateral tongue.
D. gingiva.

Click here to see the answer

Answer: C. lateral tongue.

The lateral border of the tongue is where hairy leukoplakia is commonly seen.

9. 75 year-old male presents for a routine physical. Vitals are normal with no orthostatic changes. On physical examination, a fine cortical movement with repetitive rubbing of the tip of the thumb along the tips of the fingers is noted at rest. Which of the following is the most likely diagnosis?

A. Seizure disorder
B. Peripheral neuropathy
C. Shy-Drager syndrome
D. Parkinson's disease

Click here to see the answer

Answer: D. Parkinson’s disease

Parkinson’s disease presents with tremor at rest (pill-rolling), bradykinesia, rigidity, and postural instability.

Explanations

  • A. While a seizure may present with fine or gross uncontrolled motor movements, the tremor described is classic pill-rolling tremor noted in Parkinson’s disease.
  • B. Peripheral neuropathy presents with loss of sensation not tremor.
  • C. Shy-Drager syndrome is due to autonomic degeneration and typically presents with orthostatic hypotension.

10. A 45 year-old female presents to the emergency department with generalized, hot, erythema of the skin. Physical exam reveals an oral temperature of 102 degrees Fahrenheit, purulent conjunctivitis, and mucosal erosions. Her skin is painful and separates from the dermis with touch. Which of the following is the most likely cause for this condition?

A. Penicillin
B. Prednisolone
C. Aspirin
D. Hydrochlorothiazide (HCTZ)

Click here to see the answer

Answer: A. Penicillin

Medications are most frequently implicated in toxic epidermal necrolysis. These usually include, analgesics (NSAIDs), antibiotics – sulfonamides (sulfamethoxazole, sulfadiazine, sulfapyridine,
beta-lactams (cephalosporins, penicillins, carbapenems) and anticonvulsants (Carbamazepine).

Explanations

  • B. Systemic glucocorticoids may be used early in the treatment of this condition and are not a cause.
  • C. Aspirin is not linked to toxic epidermal necrolysis.
  • D. Hydrochlorothiazide diuretics are not associated with the production of toxic epidermal necrolysis.

Looking for all the podcast episodes?

This FREE series is limited to every other episode, you can download and enjoy the complete audio series by joining The PANCE and PANRE Exam Academy.

I will be releasing new episodes every few weeks. The Academy is discounted, so sign up now.

Resources From The Show

This Podcast is also available on iTunes and Stitcher Radio for Android

  1. iTunes: The Audio PANCE AND PANRE Podcast iTunes
  2. Stitcher Radio: The Audio PANCE and PANRE Podcast Stitcher

itunes_logo-1

The post Episode 43: The Audio PANCE and PANRE Board Review Podcast appeared first on The Audio PANCE and PANRE.

]]>
Welcome to episode 43 of the FREE Audio PANCE and PANRE Physician Assistant Board Review Podcast. Join me as I cover 10 PANCE and PANRE board review questions from the Academy course content following the NCCPA™ content blueprint (download the FREE che... Join me as I cover 10 PANCE and PANRE board review questions from the Academy course content following the NCCPA™ content blueprint (download the FREE cheat sheet).
This week we will continue to take a break from topic specific board review and covering 10 general board review questions.
Below you will find an interactive exam to complement the podcast.
I hope you enjoy this free audio component to the examination portion of this site. The full genitourinary board review includes over 72 GU specific questions and is available to all members of the PANCE and PANRE Academy.

* You can download and listen to past FREE episodes here, on iTunes or Stitcher Radio.
* You can listen to the latest episode, take an interactive quiz and download your results below.

Listen Carefully Then Take The Quiz
If you can’t see the audio player click here to listen to the full episode.
Episode 43 PANCE and PANRE Podcast Quiz
1. A mother brings in her five-year-old boy for his school physical. She voices some concerns about his readiness for school, saying he seems to be socially immature. She has noticed he does not interact with other children well, and that when he plays with them, he has a tendency to “place them” and then run around them as if they were statues. He rarely cries when he is hurt, and he shrugs off any attempt to hug him. He has good attention to details, and will sit and draw the same geometric shapes over and over again, but does not seem interested in learning the alphabet. He avoids eye contact with anyone. Which of the following is the most likely diagnosis?

* Normal 5 year-old
* Social phobia
* Autism
* Avoidant personality

2. A 3-week-old male infant presents with recurrent regurgitation after feeding that has progressed to projectile vomiting in the last few days. The mother states that the child appears hungry all of the time. She denies any diarrhea in the child. Which of the following clinical findings is most likely?

* Bile-stained vomitus
* Hemoccult positive stools
* Olive-sized mass in the right upper abdomen
* Sausage-shaped mass in the upper-mid abdomen


3. A 65 year-old with COPD receiving their first pneumococcal polysaccharide PPSV23 vaccination should be revaccinated in

* 1 year
* 3 years
* 5 years
* Never

4. Which of the following therapies is recommended for a 13 month-old child with sickle cell disease?


* Folic acid and penicillin V
* Ferrous sulfate and penicillin V
* Folic acid and ferrous sulfate
* Folic acid, ferrous sulfate and penicillin V

5. A patient with severe COPD presents to the Emergency Department with a 3 day history of increasing shortness of breath with exertion and cough productive of purulent sputum. An arterial blood gas reveals a pH of 7.25, PaCO2 of 70 mmHg and PaO2 of 50 mmHg. He is started on albuterol nebulizer, nasal oxygen at 2 liters per minute,]]>
The Physician Assistant Life | Smarty PANCE full clean 16:12 240
Episode 41: The Audio PANCE and PANRE Board Review Podcast http://podcast.thepalife.com/episode-41-the-audio-pance-and-panre-board-review-podcast/ Mon, 29 Aug 2016 21:02:32 +0000 http://podcast.thepalife.com/?p=227 http://podcast.thepalife.com/episode-41-the-audio-pance-and-panre-board-review-podcast/#respond http://podcast.thepalife.com/episode-41-the-audio-pance-and-panre-board-review-podcast/feed/ 0 <p>Welcome to episode 41 of the FREE Audio PANCE and PANRE Physician Assistant Board Review Podcast. Join me as I cover 10 PANCE and PANRE board review questions from the Academy course content following the NCCPA™ content blueprint. This week we will be taking a break from topic specific board review and covering 10 general board review questions. […]</p> <p>The post <a rel="nofollow" href="http://podcast.thepalife.com/episode-41-the-audio-pance-and-panre-board-review-podcast/">Episode 41: The Audio PANCE and PANRE Board Review Podcast</a> appeared first on <a rel="nofollow" href="http://podcast.thepalife.com">The Audio PANCE and PANRE</a>.</p> The Audio PANCE and PANRE Episode 41Welcome to episode 41 of the FREE Audio PANCE and PANRE Physician Assistant Board Review Podcast.

Join me as I cover 10 PANCE and PANRE board review questions from the Academy course content following the NCCPA™ content blueprint.

This week we will be taking a break from topic specific board review and covering 10 general board review questions.

Below you will find an interactive exam to complement the podcast.

I hope you enjoy this free audio component to the examination portion of this site. The full genitourinary board review includes over 72 GU specific questions and is available to all members of the PANCE and PANRE Academy.

  • You can download and listen to past FREE episodes here, on iTunes or Stitcher Radio.
  • You can listen to the latest episode, take an interactive quiz and download your results below.

Listen Carefully Then Take The Quiz

If you can’t see the audio player click here to listen to the full episode.

Episode 41 PANCE and PANRE Podcast Quiz

1. A mother brings her 6-year-old boy for evaluation of school behavior problems. She says the teacher told her that the boy does not pay attention in class, that he gets up and runs around the room when the rest of the children are listening to a story, and that he seems to be easily distracted by events outside or in the hall. He refuses to remain in his seat during class, and occasionally sits under his desk or crawls around under a table. The teacher told the mother this behavior is interfering with the child’s ability to function in the classroom and to learn. The mother states that she has noticed some of these behaviors at home, including his inability to watch his favorite cartoon program all the way through. Which of the following is the most likely diagnosis?

  1. Antisocial disorder
  2. Dysthymic mood disorder
  3. Obsessive-compulsive disorder
  4. Attention deficit hyperactivity disorder

Click here to see the answer

Answer: D. Attention deficit hyperactivity disorder

Attention deficit hyperactivity disorder is characterized by inattention, including increased distractibility and difficulty sustaining attention; poor impulse control and decreased self-inhibitory capacity; and motor over activity and motor restlessness, which are pervasive and interfere with the individual’s ability to function under normal circumstances.

Explanations

  • Antisocial behavior disorder is characterized by disregard for rights of others; a defect in the experience of compunction or remorse for harming others.
  • Dysthymic mood disorder is characterized by chronic, sad mood occurring for at least 2 years in an adult (one year in a child). Behavioral problems are not part of this disorder.
  • Obsessive-compulsive disorder is characterized by recurrent obsessions and compulsions that result in anxiety and disruptive behaviors related to those compulsions.

2. Which of the following is the treatment of choice for a torus (buckle) fracture involving the distal radius?

A. Open reduction and internal fixation
B. Ace wrap or anterior splinting
C. Closed reduction and casting
D. Corticosteroid injection followed by splinting

Click here to see the answer

Answer: B. Ace wrap or anterior splinting

A torus or buckle fracture occurs after a minor fall on the hand. These fractures are very stable and are not as painful as unstable fractures. They heal uneventfully in 3-4 weeks.

3. Which of the following can be used to treat chronic bacterial prostatitis?

A. Penicillin
B. Cephalexin (Keflex)
C. Nitrofurantoin (Macrobid)
D. Levofloxacin (Levaquin)

Click here to see the answer

Answer: D. Levofloxacin (Levaquin)

Chronic bacterial prostatitis (Type II prostatitis) can be difficult to treat and requires the use of fluoroquinolones or trimethoprim-sulfamethoxazole, both of which penetrate the prostate.

4. A 25 year-old male with history of syncope presents for evaluation. The patient admits to intermittent episodes of rapid heart beating that resolve spontaneously. 12 Lead EKG shows delta waves and a short PR interval. Which of the following is the treatment of choice in this patient?

A. Radiofrequency catheter ablation
B. Verapamil (Calan)
C. Percutaneous coronary intervention
D. Digoxin (Lanoxin)

Click here to see the answer

Answer: A. Radiofrequency catheter ablation

Radiofrequency catheter ablation is the treatment of choice on patients with accessory pathways, such as Wolff-Parkinson-White Syndrome.

Explanations

  • Calcium channel blockers such as verapamil decrease refractoriness of the accessory pathway or increase that of the AV node leading to faster ventricular rates, therefore calcium channel blockers should be avoided in patients with WPW.
  • Percutaneous coronary intervention is indicated in the treatment of coronary artery disease, not preexcitation syndromes.
  • (Digoxin decreases refractoriness of the accessory pathway and increases that of the AV node leading to faster ventricular rates. It should therefore be avoided in patients with WPW.

5. Which of the following pathophysiological processes is associated with chronic bronchitis?

A. Destruction of the lung parenchyma
B. Mucous gland enlargement and goblet cell hyperplasia
C. Smooth muscle hypertrophy in the large airways
D. Increased mucus adhesion secondary to reduction in the salt and water content of the mucus

Click here to see the answer

Answer: B. Mucous gland enlargement and goblet cell hyperplasia

Chronic bronchitis results from the enlargement of mucous glands and goblet cell hypertrophy in the large airways.

Explanations

  • Destruction of the gas-exchanging structures in the lung is characteristic of emphysema.
  • There may be smooth muscle hypertrophy in chronic bronchitis but it is not to the extent as found in asthma and is not an underlying factor in the pathology of chronic bronchitis.
  • Abnormal absorption of sodium and a reduced rate of chloride secretion in cystic fibrosis leads to thickening of the mucus and increase in adhesion of the mucus.

6. Which of the following dietary substances interact with monoamine oxidase-inhibitor antidepressant drugs?

A. Lysine
B. Glycine
C. Tyramine
D. Phenylalanine

Click here to see the answer

Answer: C. Tyramine

Monoamine oxidase inhibitors are associated with serious food/drug and drug/drug interactions. Patient must restrict intake of foods having a high tyramine content to avoid serious reactions. Tyramine is a precursor to norepinephrine.

Explanations

Lysine, glycine, and phenylalanine are not known to interact with MAO inhibitors.

7. Gallstones usually result in biliary symptoms by causing inflammation or obstruction following migration into the common bile duct or

A. cystic duct
B. pancreatic duct
C. duodenal ampulla
D. common hepatic duct

Click here to see the answer

Answer: A. cystic duct

Obstruction of the cystic duct by gallstones causes the typical symptom of biliary colic. Once obstructed the gallbladder distends and becomes edematous and inflamed. Gallstones can also migrate into the common bile duct through the cystic duct leading to a condition known as choledocholithiasis.

Explanations

  • Obstruction of the pancreatic duct leads to development of acute pancreatitis.
  • The duodenal ampulla is the area where the pancreatic duct and the common bile duct empty into the duodenum. Gallstones do not cause obstruction at this distal site.
  • The common hepatic duct from the liver joins the cystic duct from the gallbladder to form the common bile duct. Stone migration occurs along the pathway of the cystic duct to the common bile duct, not along the common hepatic duct.

8. An elderly patient with poorly-controlled Type 2 diabetes and renal disease develops a fever of 102°F orally, productive cough, and dyspnea. Physical examination demonstrates a respiratory rate of 32/min, labored breathing, and rales at the left base. Pulse oximetry is 90%. Which of the following is the next appropriate step in the management of this patient?

A. Administer nebulized corticosteroids
B. Admit to the hospital
C. Oral antimicrobial therapy
D. Endotracheal intubation

Click here to see the answer

Answer: B. Admit to the hospital

Community acquired pneumonia is the most deadly infectious disease in the U.S. Important risk factors for increased morbidity and mortality include advanced age, alcoholism, comorbid medical conditions, altered mental status, respiratory rate greater than 30 breaths/min, hypotension, and a BUN greater than 30.

Explanations

  • Inhaled corticosteroids are not utilized in the management of community-acquired pneumonia.
  • Due to the age of the patient, comorbid diseases, and current signs of respiratory distress, intravenous not oral antimicrobial therapy is indicated.
  • Endotracheal intubation is indicated for respiratory failure unresponsive to conservative management.

9. A 53 year-old female who is well known to the practice presents to the office complaining of increasing fatigue, constipation, and a weight gain of 10 lb (4.5 kg) over the past year. She also states others have noticed a recent hoarseness to her voice, and she is bothered by “charley horses” in her legs that wake her up at night. Her past medical history is unremarkable except for a history of hyperthyroidism treated by radioactive iodine 5 years ago. She is currently taking no medications and has no known drug allergies. Which of the following is the most likely cause of the patient’s symptoms?

A. Hypothyroidism
B. Hypoparathyroidism
C. Vocal cord paralysis
D. Radiation thyroiditis

Click here to see the answer

Answer: A. Hypothyroidism

The current symptoms, along with the past treatment of hyperthyroidism with radioactive iodine, would indicate hypothyroidism.

Explanations

  • This is a possible later complication of subtotal thyroidectomy, not radioactive iodine therapy.
  • This is an immediate complication of subtotal thyroidectomy or injury and does not occur with radioactive iodine therapy.
  • Radiation thyroiditis may occur following radiation therapy but there is no history of the patient having previous external beam radiation therapy.

10. Which of the following is most frequently associated with bladder cancer?

A. Hematuria
B. Dysuria
C. Urgency
D. Frequency

Click here to see the answer

Answer: A. Hematuria

Significant persistent hematuria >3 RBC/HPF on three urinalyses, a single urinalysis with >100 RBC, or gross hematuria, identifies significant renal or urologic lesions. Bladder cancer usually presents with painless hematuria.

Explanations

Dysuria, urgency, and frequency are associated with irritative voiding symptoms associated with cystitis.

Looking for all the podcast episodes?

This FREE series is limited to every other episode, you can download and enjoy the complete audio series by joining The PANCE and PANRE Exam Academy.

I will be releasing new episodes every few weeks. The Academy is discounted, so sign up now.

reeldx_signup1-ssmallGet ReelDx at 40% off!

I am so excited to be introducing ReelDx cases into my new SmartyPANCE board review website. This is a $99 value now included to all registered SmartyPANCE users. If you are just interested in access to the ReelDx cases through their website as a listener of this podcast you can get 40% off a one year subscription by entering the code “ThePALife” at checkout:

Resources From The Show

This Podcast is also available on iTunes and Stitcher Radio for Android

  1. iTunes: The Audio PANCE AND PANRE Podcast iTunes
  2. Stitcher Radio: The Audio PANCE and PANRE Podcast Stitcher

itunes_logo-1

The post Episode 41: The Audio PANCE and PANRE Board Review Podcast appeared first on The Audio PANCE and PANRE.

]]>
Welcome to episode 41 of the FREE Audio PANCE and PANRE Physician Assistant Board Review Podcast. Join me as I cover 10 PANCE and PANRE board review questions from the Academy course content following the NCCPA™ content blueprint. Welcome to episode 41 of the FREE Audio PANCE and PANRE Physician Assistant Board Review Podcast.
Join me as I cover 10 PANCE and PANRE board review questions from the Academy course content following the NCCPA™ content blueprint.
This week we will be taking a break from topic specific board review and covering 10 general board review questions.
Below you will find an interactive exam to complement the podcast.
I hope you enjoy this free audio component to the examination portion of this site. The full genitourinary board review includes over 72 GU specific questions and is available to all members of the PANCE and PANRE Academy.

* You can download and listen to past FREE episodes here, on iTunes or Stitcher Radio.
* You can listen to the latest episode, take an interactive quiz and download your results below.

Listen Carefully Then Take The Quiz
If you can’t see the audio player click here to listen to the full episode.
Episode 41 PANCE and PANRE Podcast Quiz
1. A mother brings her 6-year-old boy for evaluation of school behavior problems. She says the teacher told her that the boy does not pay attention in class, that he gets up and runs around the room when the rest of the children are listening to a story, and that he seems to be easily distracted by events outside or in the hall. He refuses to remain in his seat during class, and occasionally sits under his desk or crawls around under a table. The teacher told the mother this behavior is interfering with the child’s ability to function in the classroom and to learn. The mother states that she has noticed some of these behaviors at home, including his inability to watch his favorite cartoon program all the way through. Which of the following is the most likely diagnosis?

* Antisocial disorder
* Dysthymic mood disorder
* Obsessive-compulsive disorder
* Attention deficit hyperactivity disorder

2. Which of the following is the treatment of choice for a torus (buckle) fracture involving the distal radius?
A. Open reduction and internal fixation
B. Ace wrap or anterior splinting
C. Closed reduction and casting
D. Corticosteroid injection followed by splinting
3. Which of the following can be used to treat chronic bacterial prostatitis?
A. Penicillin
B. Cephalexin (Keflex)
C. Nitrofurantoin (Macrobid)
D. Levofloxacin (Levaquin)
4. A 25 year-old male with history of syncope presents for evaluation. The patient admits to intermittent episodes of rapid heart beating that resolve spontaneously. 12 Lead EKG shows delta waves and a short PR interval. Which of the following is the treatment of choice in this patient?
A. Radiofrequency catheter ablation
B. Verapamil (Calan)
C. Percutaneous coronary intervention
D. Digoxin (Lanoxin)
5. Which of the following pathophysiological processes is associated with chronic bronchitis?
A. Destruction of the lung parenchyma
B. Mucous gland enlargement and goblet cell hyperplasia
C. Smooth muscle hypertrophy in the large airways
D. Increased mucus adhesion secondary to reduction in the salt and water content of the mucus
The Physician Assistant Life | Smarty PANCE full clean 15:52 227
REPRODUCTIVE SYSTEM : THE AUDIO PANCE AND PANRE PODCAST TOPIC SPECIFIC REVIEW EPISODE 39 http://podcast.thepalife.com/reproductive-system-the-audio-pance-and-panre-podcast-topic-specific-review-episode-39/ Mon, 20 Jun 2016 20:52:43 +0000 http://podcast.thepalife.com/?p=220 http://podcast.thepalife.com/reproductive-system-the-audio-pance-and-panre-podcast-topic-specific-review-episode-39/#respond http://podcast.thepalife.com/reproductive-system-the-audio-pance-and-panre-podcast-topic-specific-review-episode-39/feed/ 0 <p>Welcome to episode 39 of the FREE Audio PANCE and PANRE Physician Assistant Board Review Podcast. Join me as I continue to cover topic specific PANCE and PANRE review from the Academy course content following the NCCPA™ content blueprint. Click here to download my interactive content blueprint checklist This week we will be covering 10 topic specific OBGYN/reproductive board […]</p> <p>The post <a rel="nofollow" href="http://podcast.thepalife.com/reproductive-system-the-audio-pance-and-panre-podcast-topic-specific-review-episode-39/">REPRODUCTIVE SYSTEM : THE AUDIO PANCE AND PANRE PODCAST TOPIC SPECIFIC REVIEW EPISODE 39</a> appeared first on <a rel="nofollow" href="http://podcast.thepalife.com">The Audio PANCE and PANRE</a>.</p> Reproductive System Board Review PodcastWelcome to episode 39 of the FREE Audio PANCE and PANRE Physician Assistant Board Review Podcast.

Join me as I continue to cover topic specific PANCE and PANRE review from the Academy course content following the NCCPA™ content blueprint.

Click here to download my interactive content blueprint checklist

This week we will be covering 10 topic specific OBGYN/reproductive board review questions.

The reproductive system accounts for 8% of your PANCE/PANRE board exam.  

Below you will find an interactive exam to complement the podcast.

I hope you enjoy this free audio component to the examination portion of this site. The full reproductive review includes over 107 reproductive system content blueprint specific questions and is available to all members of the PANCE and PANRE Academy or my new site SMARTY PANCE!!

Listen Carefully Then Take The Quiz

If you can’t see the audio player click here to listen to the full episode.

Reproductive System PANCE and PANRE Podcast Quiz

You can take the interactive quiz by clicking here

Looking for all the podcast episodes?

This FREE series is limited to every other episode, you can download and enjoy the complete audio series by joining The PANCE and PANRE Exam Academy.

I will be releasing new episodes every few weeks. The Academy is discounted, so sign up now.

Resources and Show Notes:

This Podcast is also available on iTunes and Stitcher Radio for Android

  1. iTunes: The Audio PANCE AND PANRE Podcast iTunes
  2. Google Play: The Audio PANCE and PANRE Podcast Google Play
  3. Stitcher Radio: The Audio PANCE and PANRE Podcast Stitcher

itunes_logo-1

The post REPRODUCTIVE SYSTEM : THE AUDIO PANCE AND PANRE PODCAST TOPIC SPECIFIC REVIEW EPISODE 39 appeared first on The Audio PANCE and PANRE.

]]>
Welcome to episode 39 of the FREE Audio PANCE and PANRE Physician Assistant Board Review Podcast. Join me as I continue to cover topic specific PANCE and PANRE review from the Academy course content following the NCCPA™ content blueprint. Welcome to episode 39 of the FREE Audio PANCE and PANRE Physician Assistant Board Review Podcast.
Join me as I continue to cover topic specific PANCE and PANRE review from the Academy course content following the NCCPA™ content blueprint.
Click here to download my interactive content blueprint checklist
This week we will be covering 10 topic specific OBGYN/reproductive board review questions.
The reproductive system accounts for 8% of your PANCE/PANRE board exam.  
Below you will find an interactive exam to complement the podcast.
I hope you enjoy this free audio component to the examination portion of this site. The full reproductive review includes over 107 reproductive system content blueprint specific questions and is available to all members of the PANCE and PANRE Academy or my new site SMARTY PANCE!!

* You can download and listen to past FREE episodes here, on iTunes, Google Play and Stitcher Radio.
* You can listen to the latest episode, take an interactive quiz and download your results below.

Listen Carefully Then Take The Quiz
If you can’t see the audio player click here to listen to the full episode.
Reproductive System PANCE and PANRE Podcast Quiz
You can take the interactive quiz by clicking here
Looking for all the podcast episodes?
This FREE series is limited to every other episode, you can download and enjoy the complete audio series by joining The PANCE and PANRE Exam Academy.
I will be releasing new episodes every few weeks. The Academy is discounted, so sign up now.
Resources and Show Notes:

* Reproductive system PANCE and PANRE content blueprint course on Smarty PANCE
* Reproductive System Contentent Blueprint 107 question exam on Smarty PANCE (members only)
* Download my free interactive content blueprint checklist
* My list of recommended PANCE and PANRE review books
* USE CODE “PALIFE” TO GET 10% OFF THE RUTGERS PANCE AND PANRE REVIEW COURSE

This Podcast is also available on iTunes and Stitcher Radio for Android

* iTunes: The Audio PANCE A...]]>
The Physician Assistant Life | Smarty PANCE full clean 13:46 220
Musculoskeletal 1: The Audio PANCE and PANRE Podcast Topic Specific Review Episode 37 http://podcast.thepalife.com/musculoskeletal-1-the-audio-pance-and-panre-podcast-topic-specific-review-episode-37/ Mon, 25 Apr 2016 04:06:08 +0000 http://podcast.thepalife.com/?p=214 http://podcast.thepalife.com/musculoskeletal-1-the-audio-pance-and-panre-podcast-topic-specific-review-episode-37/#respond http://podcast.thepalife.com/musculoskeletal-1-the-audio-pance-and-panre-podcast-topic-specific-review-episode-37/feed/ 0 <p>Welcome to episode 37 of the FREE Audio PANCE and PANRE Physician Assistant Board Review Podcast. Join me as I continue to cover topic specific PANCE and PANRE review from the Academy course content following the NCCPA™ content blueprint. Click here to download my interactive content blueprint checklist This week we will be covering 10 topic specific […]</p> <p>The post <a rel="nofollow" href="http://podcast.thepalife.com/musculoskeletal-1-the-audio-pance-and-panre-podcast-topic-specific-review-episode-37/">Musculoskeletal 1: The Audio PANCE and PANRE Podcast Topic Specific Review Episode 37</a> appeared first on <a rel="nofollow" href="http://podcast.thepalife.com">The Audio PANCE and PANRE</a>.</p> Musculoskeletal PART 1 - The Audio PANCE and PANRE Board Review PodcastWelcome to episode 37 of the FREE Audio PANCE and PANRE Physician Assistant Board Review Podcast.

Join me as I continue to cover topic specific PANCE and PANRE review from the Academy course content following the NCCPA™ content blueprint.

Click here to download my interactive content blueprint checklist

This week we will be covering 10 topic specific Musculoskeletal board review questions.

The Musculoskeletal System accounts for 10% of your PANCE/PANRE board exam.  

Below you will find an interactive exam to complement the podcast.

I hope you enjoy this free audio component to the examination portion of this site. The full genitourinary board review includes over 72 GU specific questions and is available to all members of the PANCE and PANRE Academy.

  • You can download and listen to past FREE episodes here, on iTunes or Stitcher Radio.
  • You can listen to the latest episode, take an interactive quiz and download your results below.

Listen Carefully Then Take The Quiz

If you can’t see the audio player click here to listen to the full episode.

Musculoskeletal PANCE and PANRE Podcast Quiz

You can take the interactive exam by clicking here.

Looking for all the podcast episodes?

This FREE series is limited to every other episode, you can download and enjoy the complete audio series by joining The PANCE and PANRE Exam Academy.

I will be releasing new episodes every few weeks. The Academy is discounted, so sign up now.

Resources and Show Notes:

This Podcast is also available on iTunes and Stitcher Radio for Android

  1. iTunes: The Audio PANCE AND PANRE Podcast iTunes
  2. Stitcher Radio: The Audio PANCE and PANRE Podcast Stitcher

itunes_logo-1

The post Musculoskeletal 1: The Audio PANCE and PANRE Podcast Topic Specific Review Episode 37 appeared first on The Audio PANCE and PANRE.

]]> Welcome to episode 37 of the FREE Audio PANCE and PANRE Physician Assistant Board Review Podcast. Join me as I continue to cover topic specific PANCE and PANRE review from the Academy course content following the NCCPA™ content blueprint. Welcome to episode 37 of the FREE Audio PANCE and PANRE Physician Assistant Board Review Podcast.
Join me as I continue to cover topic specific PANCE and PANRE review from the Academy course content following the NCCPA™ content blueprint.
Click here to download my interactive content blueprint checklist
This week we will be covering 10 topic specific Musculoskeletal board review questions.
The Musculoskeletal System accounts for 10% of your PANCE/PANRE board exam.  
Below you will find an interactive exam to complement the podcast.
I hope you enjoy this free audio component to the examination portion of this site. The full genitourinary board review includes over 72 GU specific questions and is available to all members of the PANCE and PANRE Academy.

* You can download and listen to past FREE episodes here, on iTunes or Stitcher Radio.
* You can listen to the latest episode, take an interactive quiz and download your results below.

Listen Carefully Then Take The Quiz
If you can’t see the audio player click here to listen to the full episode.
Musculoskeletal PANCE and PANRE Podcast Quiz
You can take the interactive exam by clicking here.
Looking for all the podcast episodes?
This FREE series is limited to every other episode, you can download and enjoy the complete audio series by joining The PANCE and PANRE Exam Academy.
I will be releasing new episodes every few weeks. The Academy is discounted, so sign up now.
Resources and Show Notes:

* My list of recommended PANCE and PANRE review books
* My secret upcoming website which will cover the entire NCCPA Content Blueprint (coming soon)
* USE CODE “PALIFE” TO GET 10% OFF THE RUTGERS PANCE AND PANRE REVIEW COURSE

This Podcast is also available on iTunes and Stitcher Radio for Android

* iTunes: The Audio PANCE AND PANRE Podcast iTunes
* Stitcher Radio: The Audio PANCE and PANRE Podcast Stitcher


]]>
The Physician Assistant Life | Smarty PANCE full clean 13:50 214 Genitourinary 1: The Audio PANCE and PANRE Podcast Topic Specific Review Episode 35 http://podcast.thepalife.com/genitourinary-1-the-audio-pance-and-panre-podcast-topic-specific-review-episode-35/ Mon, 14 Mar 2016 03:41:59 +0000 http://podcast.thepalife.com/?p=203 http://podcast.thepalife.com/genitourinary-1-the-audio-pance-and-panre-podcast-topic-specific-review-episode-35/#respond http://podcast.thepalife.com/genitourinary-1-the-audio-pance-and-panre-podcast-topic-specific-review-episode-35/feed/ 0 <p>Welcome to episode 35 of the FREE Audio PANCE and PANRE Physician Assistant Board Review Podcast. Join me as I continue to cover topic specific PANCE and PANRE review from the Academy course content following the NCCPA™ content blueprint. This week we will be covering 10 topic specific Genitourinary board review questions. GU accounts for 6% of […]</p> <p>The post <a rel="nofollow" href="http://podcast.thepalife.com/genitourinary-1-the-audio-pance-and-panre-podcast-topic-specific-review-episode-35/">Genitourinary 1: The Audio PANCE and PANRE Podcast Topic Specific Review Episode 35</a> appeared first on <a rel="nofollow" href="http://podcast.thepalife.com">The Audio PANCE and PANRE</a>.</p> The Audio PANCE and PANRE Genitourinary Review 1Welcome to episode 35 of the FREE Audio PANCE and PANRE Physician Assistant Board Review Podcast.

Join me as I continue to cover topic specific PANCE and PANRE review from the Academy course content following the NCCPA™ content blueprint.

This week we will be covering 10 topic specific Genitourinary board review questions.

GU accounts for 6% of your PANCE/PANRE board exam.  

Below you will find an interactive exam to complement the podcast.

I hope you enjoy this free audio component to the examination portion of this site. The full genitourniary board review includes over 72 GU specific questions and is available to all members of the PANCE and PANRE Academy.

  • You can download and listen to past FREE episodes here, on iTunes or Stitcher Radio.
  • You can listen to the latest episode, take an interactive quiz and download your results below.

Listen Carefully Then Take The Quiz

If you can’t see the audio player click here to listen to the full episode.

Genitourinary PANCE and PANRE Podcast Quiz

You can take the interactive exam by clicking here.

Looking for all the podcast episodes?

This FREE series is limited to every other episode, you can download and enjoy the complete audio series by joining The PANCE and PANRE Exam Academy.

I will be releasing new episodes every few weeks. The Academy is discounted, so sign up now.

Resources and Show Notes:

This Podcast is also available on iTunes and Stitcher Radio for Android

  1. iTunes: The Audio PANCE AND PANRE Podcast iTunes
  2. Stitcher Radio: The Audio PANCE and PANRE Podcast Stitcher

itunes_logo-1

The post Genitourinary 1: The Audio PANCE and PANRE Podcast Topic Specific Review Episode 35 appeared first on The Audio PANCE and PANRE.

]]>
Welcome to episode 35 of the FREE Audio PANCE and PANRE Physician Assistant Board Review Podcast. Join me as I continue to cover topic specific PANCE and PANRE review from the Academy course content following the NCCPA™ content blueprint. Welcome to episode 35 of the FREE Audio PANCE and PANRE Physician Assistant Board Review Podcast.
Join me as I continue to cover topic specific PANCE and PANRE review from the Academy course content following the NCCPA™ content blueprint.
This week we will be covering 10 topic specific Genitourinary board review questions.
GU accounts for 6% of your PANCE/PANRE board exam.  
Below you will find an interactive exam to complement the podcast.
I hope you enjoy this free audio component to the examination portion of this site. The full genitourniary board review includes over 72 GU specific questions and is available to all members of the PANCE and PANRE Academy.

* You can download and listen to past FREE episodes here, on iTunes or Stitcher Radio.
* You can listen to the latest episode, take an interactive quiz and download your results below.

Listen Carefully Then Take The Quiz
If you can’t see the audio player click here to listen to the full episode.
Genitourinary PANCE and PANRE Podcast Quiz
You can take the interactive exam by clicking here.
Looking for all the podcast episodes?
This FREE series is limited to every other episode, you can download and enjoy the complete audio series by joining The PANCE and PANRE Exam Academy.
I will be releasing new episodes every few weeks. The Academy is discounted, so sign up now.
Resources and Show Notes:

* My list of recommended PANCE and PANRE review books
* My secret upcoming website which will cover the entire NCCPA Content Blueprint (coming soon)
* USE CODE “PALIFE” TO GET 10% OFF THE RUTGERS PANCE AND PANRE REVIEW COURSE

This Podcast is also available on iTunes and Stitcher Radio for Android

* iTunes: The Audio PANCE AND PANRE Podcast iTunes
* Stitcher Radio: The Audio PANCE and PANRE Podcast Stitcher


]]>
The Physician Assistant Life | Smarty PANCE full clean 17:54 203
EENT 1: The Audio PANCE and PANRE Podcast Topic Specific Review Episode 33 http://podcast.thepalife.com/eent-1-the-audio-pance-and-panre-podcast-topic-specific-review-episode-33/ Fri, 05 Feb 2016 06:23:01 +0000 http://podcast.thepalife.com/?p=197 http://podcast.thepalife.com/eent-1-the-audio-pance-and-panre-podcast-topic-specific-review-episode-33/#respond http://podcast.thepalife.com/eent-1-the-audio-pance-and-panre-podcast-topic-specific-review-episode-33/feed/ 0 <p>Welcome to episode 33 of the FREE Audio PANCE and PANRE Physician Assistant Board Review Podcast. Join me as I continue to cover topic specific PANCE and PANRE review from the Academy course content following the NCCPA™ content blueprint. This week we will be covering 10 topic specific HEENT board review questions. Eyes, Ears Nose and Throat accounts […]</p> <p>The post <a rel="nofollow" href="http://podcast.thepalife.com/eent-1-the-audio-pance-and-panre-podcast-topic-specific-review-episode-33/">EENT 1: The Audio PANCE and PANRE Podcast Topic Specific Review Episode 33</a> appeared first on <a rel="nofollow" href="http://podcast.thepalife.com">The Audio PANCE and PANRE</a>.</p> PANCE and PANRE HEENT Audio Review Part OneWelcome to episode 33 of the FREE Audio PANCE and PANRE Physician Assistant Board Review Podcast.

Join me as I continue to cover topic specific PANCE and PANRE review from the Academy course content following the NCCPA™ content blueprint.

This week we will be covering 10 topic specific HEENT board review questions.

Eyes, Ears Nose and Throat accounts for 9% of your PANCE/PANRE board exam.  

Below you will find an interactive exam to complement the podcast.

I hope you enjoy this free audio component to the examination portion of this site. The full Gastroenterology/Nutrition review includes over 111 EENT specific questions and is available to all members of the PANCE and PANRE Academy.

  • You can download and listen to past FREE episodes here, on iTunes or Stitcher Radio.
  • You can listen to the latest episode, take an interactive quiz and download your results below.

Listen Carefully Then Take The Quiz

If you can’t see the audio player click here to listen to the full episode.

EENT PANCE and PANRE Podcast Quiz

You can take the interactive exam by clicking here.

Looking for all the podcast episodes?

This FREE series is limited to every other episode, you can download and enjoy the complete audio series by joining The PANCE and PANRE Exam Academy.

I will be releasing new episodes every few weeks. The Academy is discounted, so sign up now.

Resources and Show Notes:

This Podcast is also available on iTunes and Stitcher Radio for Android

  1. iTunes: The Audio PANCE AND PANRE Podcast iTunes
  2. Stitcher Radio: The Audio PANCE and PANRE Podcast Stitcher

itunes_logo-1

The post EENT 1: The Audio PANCE and PANRE Podcast Topic Specific Review Episode 33 appeared first on The Audio PANCE and PANRE.

]]>
Welcome to episode 33 of the FREE Audio PANCE and PANRE Physician Assistant Board Review Podcast. Join me as I continue to cover topic specific PANCE and PANRE review from the Academy course content following the NCCPA™ content blueprint. Welcome to episode 33 of the FREE Audio PANCE and PANRE Physician Assistant Board Review Podcast.
Join me as I continue to cover topic specific PANCE and PANRE review from the Academy course content following the NCCPA™ content blueprint.
This week we will be covering 10 topic specific HEENT board review questions.
Eyes, Ears Nose and Throat accounts for 9% of your PANCE/PANRE board exam.  
Below you will find an interactive exam to complement the podcast.
I hope you enjoy this free audio component to the examination portion of this site. The full Gastroenterology/Nutrition review includes over 111 EENT specific questions and is available to all members of the PANCE and PANRE Academy.

* You can download and listen to past FREE episodes here, on iTunes or Stitcher Radio.
* You can listen to the latest episode, take an interactive quiz and download your results below.

Listen Carefully Then Take The Quiz
If you can’t see the audio player click here to listen to the full episode.
EENT PANCE and PANRE Podcast Quiz
You can take the interactive exam by clicking here.
Looking for all the podcast episodes?
This FREE series is limited to every other episode, you can download and enjoy the complete audio series by joining The PANCE and PANRE Exam Academy.
I will be releasing new episodes every few weeks. The Academy is discounted, so sign up now.
Resources and Show Notes:

* My list of recommended PANCE and PANRE review books
* Physician Assistant Exam Review Podcast covering Diseases of the Gallbladder and Liver

This Podcast is also available on iTunes and Stitcher Radio for Android

* iTunes: The Audio PANCE AND PANRE Podcast iTunes
* Stitcher Radio: The Audio PANCE and PANRE Podcast Stitcher


]]>
The Physician Assistant Life | Smarty PANCE full clean 18:12 197
Gastroenterology 1: The Audio PANCE and PANRE Podcast Topic Specific Review Episode 31 http://podcast.thepalife.com/gastroenterology-1-the-audio-pance-and-panre-podcast-topic-specific-review-episode-31/ Mon, 21 Dec 2015 17:50:45 +0000 http://podcast.thepalife.com/?p=191 http://podcast.thepalife.com/gastroenterology-1-the-audio-pance-and-panre-podcast-topic-specific-review-episode-31/#respond http://podcast.thepalife.com/gastroenterology-1-the-audio-pance-and-panre-podcast-topic-specific-review-episode-31/feed/ 0 <p>Welcome to episode 31 of the FREE Audio PANCE and PANRE Physician Assistant Board Review Podcast. Join me as I continue to cover topic specific PANCE and PANRE review from the Academy course content following the NCCPA™ content blueprint. This week we will be covering 10 topic specific Gastroenterology board review questions. Gastroenterology and Nutrition accounts for 10% […]</p> <p>The post <a rel="nofollow" href="http://podcast.thepalife.com/gastroenterology-1-the-audio-pance-and-panre-podcast-topic-specific-review-episode-31/">Gastroenterology 1: The Audio PANCE and PANRE Podcast Topic Specific Review Episode 31</a> appeared first on <a rel="nofollow" href="http://podcast.thepalife.com">The Audio PANCE and PANRE</a>.</p> Gastroenterology 1 The Audio PANCE and PANRE Podcast Topic Specific Review Episode 31Welcome to episode 31 of the FREE Audio PANCE and PANRE Physician Assistant Board Review Podcast.

Join me as I continue to cover topic specific PANCE and PANRE review from the Academy course content following the NCCPA™ content blueprint.

This week we will be covering 10 topic specific Gastroenterology board review questions.

Gastroenterology and Nutrition accounts for 10% of your PANCE/PANRE board exam.  

Below you will find an interactive exam to complement the podcast.

I hope you enjoy this free audio component to the examination portion of this site. The full Gastroenterology/Nutrition review includes over 149 GI/Nutrition specific questions and is available to all members of the PANCE and PANRE Academy.

  • You can download and listen to past FREE episodes here, on iTunes or Stitcher Radio.
  • You can listen to the latest episode, take an interactive quiz and download your results below.

Listen Carefully Then Take The Quiz

If you can’t see the audio player click here to listen to the full episode.

Pulmonology PANCE and PANRE Podcast Quiz

You can take the interactive exam by clicking here.

Looking for all the podcast episodes?

This FREE series is limited to every other episode, you can download and enjoy the complete audio series by joining The PANCE and PANRE Exam Academy.

I will be releasing new episodes every few weeks. The Academy is currently discounted, so sign up now.

Resources and Show Notes:

This Podcast is also available on iTunes and Stitcher Radio for Android

  1. iTunes: The Audio PANCE AND PANRE Podcast iTunes
  2. Stitcher Radio: The Audio PANCE and PANRE Podcast Stitcher

itunes_logo-1

 

The post Gastroenterology 1: The Audio PANCE and PANRE Podcast Topic Specific Review Episode 31 appeared first on The Audio PANCE and PANRE.

]]>
Welcome to episode 31 of the FREE Audio PANCE and PANRE Physician Assistant Board Review Podcast. Join me as I continue to cover topic specific PANCE and PANRE review from the Academy course content following the NCCPA™ content blueprint. Welcome to episode 31 of the FREE Audio PANCE and PANRE Physician Assistant Board Review Podcast.
Join me as I continue to cover topic specific PANCE and PANRE review from the Academy course content following the NCCPA™ content blueprint.
This week we will be covering 10 topic specific Gastroenterology board review questions.
Gastroenterology and Nutrition accounts for 10% of your PANCE/PANRE board exam.  
Below you will find an interactive exam to complement the podcast.
I hope you enjoy this free audio component to the examination portion of this site. The full Gastroenterology/Nutrition review includes over 149 GI/Nutrition specific questions and is available to all members of the PANCE and PANRE Academy.

* You can download and listen to past FREE episodes here, on iTunes or Stitcher Radio.
* You can listen to the latest episode, take an interactive quiz and download your results below.

Listen Carefully Then Take The Quiz
If you can’t see the audio player click here to listen to the full episode.
Pulmonology PANCE and PANRE Podcast Quiz
You can take the interactive exam by clicking here.
Looking for all the podcast episodes?
This FREE series is limited to every other episode, you can download and enjoy the complete audio series by joining The PANCE and PANRE Exam Academy.
I will be releasing new episodes every few weeks. The Academy is currently discounted, so sign up now.
Resources and Show Notes:

* My list of recommended PANCE and PANRE review books
* Physician Assistant Exam Review Podcast covering Diseases of the Gallbladder and Liver

This Podcast is also available on iTunes and Stitcher Radio for Android

* iTunes: The Audio PANCE AND PANRE Podcast iTunes
* Stitcher Radio: The Audio PANCE and PANRE Podcast Stitcher


 
]]>
The Physician Assistant Life | Smarty PANCE full clean 13:53 191
Pulmonology 1: The Audio PANCE and PANRE Podcast Topic Specific Review Episode 29 http://podcast.thepalife.com/episode-29/ Mon, 09 Nov 2015 19:40:36 +0000 http://podcast.thepalife.com/?p=183 http://podcast.thepalife.com/episode-29/#respond http://podcast.thepalife.com/episode-29/feed/ 0 <p>Welcome to episode 29 of the FREE Audio PANCE and PANRE Physician Assistant Board Review Podcast. Over the next few episodes I will be covering topic specific PANCE and PANRE review from the Academy course content following the NCCPA content blueprint. This week we will be covering 10 topic specific Pulmonology board review questions. Below you […]</p> <p>The post <a rel="nofollow" href="http://podcast.thepalife.com/episode-29/">Pulmonology 1: The Audio PANCE and PANRE Podcast Topic Specific Review Episode 29</a> appeared first on <a rel="nofollow" href="http://podcast.thepalife.com">The Audio PANCE and PANRE</a>.</p> Pulmonology 1 The Audio PANCE and PANRE Episode 29Welcome to episode 29 of the FREE Audio PANCE and PANRE Physician Assistant Board Review Podcast.

Over the next few episodes I will be covering topic specific PANCE and PANRE review from the Academy course content following the NCCPA content blueprint.

This week we will be covering 10 topic specific Pulmonology board review questions.

Below you will find an interactive exam to complement the podcast.

I hope you enjoy this free audio component to the examination portion of this site. The full pulmonology review includes over 142 pulmonology specific questions and is available to all members of the PANCE and PANRE Academy.

  • You can download and listen to past FREE episodes here, on iTunes or Stitcher Radio.
  • You can listen to the latest episode, take an interactive quiz and download your results below.

Listen Carefully Then Take The Quiz

If you can’t see the audio player click here to listen to the full episode.

Pulmonology PANCE and PANRE Podcast Quiz

You can take the interactive exam by clicking here.

Looking for all the podcast episodes?

This FREE series is limited to every other episode, you can download and enjoy the complete audio series by joining The PANCE and PANRE Exam Academy.

I will be releasing new episodes every few weeks. The Academy is currently discounted, so sign up now.

Resources and Show Notes:

This Podcast is also available on iTunes and Stitcher Radio for Android

  1. iTunes: The Audio PANCE AND PANRE Podcast iTunes
  2. Stitcher Radio: The Audio PANCE and PANRE Podcast Stitcher

itunes_logo-1

The post Pulmonology 1: The Audio PANCE and PANRE Podcast Topic Specific Review Episode 29 appeared first on The Audio PANCE and PANRE.

]]>
Welcome to episode 29 of the FREE Audio PANCE and PANRE Physician Assistant Board Review Podcast. Over the next few episodes I will be covering topic specific PANCE and PANRE review from the Academy course content following the NCCPA content blueprint.... Welcome to episode 29 of the FREE Audio PANCE and PANRE Physician Assistant Board Review Podcast.
Over the next few episodes I will be covering topic specific PANCE and PANRE review from the Academy course content following the NCCPA content blueprint.
This week we will be covering 10 topic specific Pulmonology board review questions.
Below you will find an interactive exam to complement the podcast.
I hope you enjoy this free audio component to the examination portion of this site. The full pulmonology review includes over 142 pulmonology specific questions and is available to all members of the PANCE and PANRE Academy.

* You can download and listen to past FREE episodes here, on iTunes or Stitcher Radio.
* You can listen to the latest episode, take an interactive quiz and download your results below.

Listen Carefully Then Take The Quiz
If you can’t see the audio player click here to listen to the full episode.
Pulmonology PANCE and PANRE Podcast Quiz
You can take the interactive exam by clicking here.
Looking for all the podcast episodes?
This FREE series is limited to every other episode, you can download and enjoy the complete audio series by joining The PANCE and PANRE Exam Academy.
I will be releasing new episodes every few weeks. The Academy is currently discounted, so sign up now.
Resources and Show Notes:

* My list of recommended PANCE and PANRE review books

This Podcast is also available on iTunes and Stitcher Radio for Android

* iTunes: The Audio PANCE AND PANRE Podcast iTunes
* Stitcher Radio: The Audio PANCE and PANRE Podcast Stitcher


]]>
The Physician Assistant Life | Smarty PANCE full clean 15:43 183
Cardiology 1: The Audio PANCE and PANRE Board Review Podcast Episode 27 http://podcast.thepalife.com/cardiology-1-the-audio-pance-and-panre-board-review-podcast-episode-27/ Wed, 30 Sep 2015 21:14:41 +0000 http://podcast.thepalife.com/?p=176 http://podcast.thepalife.com/cardiology-1-the-audio-pance-and-panre-board-review-podcast-episode-27/#respond http://podcast.thepalife.com/cardiology-1-the-audio-pance-and-panre-board-review-podcast-episode-27/feed/ 0 <p>Welcome to episode 27 of the FREE Audio PANCE and PANRE Physician Assistant Board Review Podcast. Over the next few episodes I will be covering topic specific PANCE and PANRE review from the Academy course content following the NCCPA content blueprint. This week we will be covering 10 topic specific Cardiology board review questions. Below you […]</p> <p>The post <a rel="nofollow" href="http://podcast.thepalife.com/cardiology-1-the-audio-pance-and-panre-board-review-podcast-episode-27/">Cardiology 1: The Audio PANCE and PANRE Board Review Podcast Episode 27</a> appeared first on <a rel="nofollow" href="http://podcast.thepalife.com">The Audio PANCE and PANRE</a>.</p> The Audio PANCE and PANRE Board Review Podcast Episode 27Welcome to episode 27 of the FREE Audio PANCE and PANRE Physician Assistant Board Review Podcast.

Over the next few episodes I will be covering topic specific PANCE and PANRE review from the Academy course content following the NCCPA content blueprint.

This week we will be covering 10 topic specific Cardiology board review questions.

Below you will find an interactive exam to complement the podcast.

I hope you enjoy this free audio component to the examination portion of this site. The full cardiology review includes over 147 cardiology specific questions and is available to all members of the PANCE and PANRE Academy.

  • You can download and listen to past FREE episodes here, on iTunes or Stitcher Radio.
  • You can listen to the latest episode, take an interactive quiz and download your results below.

Listen Carefully Then Take The Quiz

If you can’t see the audio player click here to listen to the full episode.

Cardiology Questions 1-10

You can take the interactive exam by clicking here.

Looking for all the podcast episodes?

This FREE series is limited to every other episode, you can download and enjoy the complete audio series by joining The PANCE and PANRE Exam Academy.

I will be be releasing new episodes every two weeks. The Academy is currently discounted, so sign up now.

Resources and Show Notes:

This Podcast is also available on iTunes and Stitcher Radio for Android

  1. iTunes: The Audio PANCE AND PANRE Podcast iTunes
  2. Stitcher Radio: The Audio PANCE and PANRE Podcast Stitcher

itunes_logo-1

The post Cardiology 1: The Audio PANCE and PANRE Board Review Podcast Episode 27 appeared first on The Audio PANCE and PANRE.

]]>
Welcome to episode 27 of the FREE Audio PANCE and PANRE Physician Assistant Board Review Podcast. Over the next few episodes I will be covering topic specific PANCE and PANRE review from the Academy course content following the NCCPA content blueprint.... Over the next few episodes I will be covering topic specific PANCE and PANRE review from the Academy course content following the NCCPA content blueprint.
This week we will be covering 10 topic specific Cardiology board review questions.
Below you will find an interactive exam to complement the podcast.
I hope you enjoy this free audio component to the examination portion of this site. The full cardiology review includes over 147 cardiology specific questions and is available to all members of the PANCE and PANRE Academy.

* You can download and listen to past FREE episodes here, on iTunes or Stitcher Radio.
* You can listen to the latest episode, take an interactive quiz and download your results below.

Listen Carefully Then Take The Quiz
If you can’t see the audio player click here to listen to the full episode.
Cardiology Questions 1-10
You can take the interactive exam by clicking here.
Looking for all the podcast episodes?
This FREE series is limited to every other episode, you can download and enjoy the complete audio series by joining The PANCE and PANRE Exam Academy.
I will be be releasing new episodes every two weeks. The Academy is currently discounted, so sign up now.
Resources and Show Notes:

* My list of recommended PANCE and PANRE review books

This Podcast is also available on iTunes and Stitcher Radio for Android

* iTunes: The Audio PANCE AND PANRE Podcast iTunes
* Stitcher Radio: The Audio PANCE and PANRE Podcast Stitcher


]]>
The Physician Assistant Life | Smarty PANCE full clean 13:31 176
The Audio PANCE and PANRE Board Review Podcast Episode 25 http://podcast.thepalife.com/the-audio-pance-and-panre-board-review-podcast-episode-25/ Mon, 31 Aug 2015 19:44:06 +0000 http://podcast.thepalife.com/?p=165 http://podcast.thepalife.com/the-audio-pance-and-panre-board-review-podcast-episode-25/#respond http://podcast.thepalife.com/the-audio-pance-and-panre-board-review-podcast-episode-25/feed/ 0 <p>Welcome to episode 25 of the FREE Audio PANCE and PANRE Physician Assistant Board Review Podcast. The Audio PANCE and PANRE is an audio board review series that includes 10 Multiple Choice PANCE and PANRE Board Review Questions in each episode. I hope you enjoy this free audio component to the examination portion of this site. The […]</p> <p>The post <a rel="nofollow" href="http://podcast.thepalife.com/the-audio-pance-and-panre-board-review-podcast-episode-25/">The Audio PANCE and PANRE Board Review Podcast Episode 25</a> appeared first on <a rel="nofollow" href="http://podcast.thepalife.com">The Audio PANCE and PANRE</a>.</p> Episode 25 The Audio PANCE and PANRE Board Review Podcast They Physician Assistant LifeWelcome to episode 25 of the FREE Audio PANCE and PANRE Physician Assistant Board Review Podcast.

The Audio PANCE and PANRE is an audio board review series that includes 10 Multiple Choice PANCE and PANRE Board Review Questions in each episode.

I hope you enjoy this free audio component to the examination portion of this site. The full series is available to all members of the PANCE and PANRE Academy.

  • You can download and listen to past FREE episodes here, on iTunes or Stitcher Radio.
  • You can listen to the latest episode, download the transcript and take an interactive quiz of the questions below.

adobe pdfDownload the FREE PDF transcript for FREE here or on Scribd.

Listen Carefully Then Take The Quiz

If you can’t see the audio player click here to listen to the full episode.

Questions 1-10

The Audio PANCE and PANRE Episode 25

1. Which of the following conditions will produce a transudative pleural effusion?

A. Kaposi’s sarcoma
B. Pneumonia
C. Cirrhosis
D. Mesothelioma

Click here to see the answer

Answer: C. Cirrhosis

Transudative pleural effusions result from alteration in the formation of pleural fluid, the absorption of pleural fluid, or both, by systemic factors. Local factors affecting pleural fluid absorption and/or formation produce exudative pleural effusions.

2. Which of the following pathophysiological processes is believed to initiate acute appendicitis?

A. Obstruction
B. Perforation
C. Hemorrhage
D. Vascular compromise

Click here to see the answer

Answer: A. Obstruction

Obstruction of the appendiceal lumen by lymphoid hyperplasia, a fecalith or foreign body initiates most cases of appendicitis.

3. A 23 year-old female with a history of palpitations presents for evaluation. She admits to acute onset of rapid heart beating lasting seconds to minutes with associated shortness of breath and chest pain. The patient states she can relieve her symptoms with Valsalva. Which of the following is the most appropriate diagnostic study to establish a definitive diagnosis in this patient?

A. Cardiac catheterization
B. Cardiac MRI
C. Chest CT scan
D. Electrophysiology study

Click here to see the answer

Answer: D. Electrophysiology study

Electrophysiology study is useful in establishing the diagnosis and pathway of complex arrhythmias such as supraventricular tachycardia.

4. A known alcoholic presents to the emergency department with altered level of consciousness and a blood glucose level of 35 mg/dL. Which of the following best explains this glucose result?

A. Excess pancreatic insulin release
B. Rapid carbohydrate discharge into the small bowel
C. Agonist insulin receptor antibody formation
D. Hepatic glycogen depletion and impaired gluconeogenesis

Click here to see the answer

Answer: D. Hepatic glycogen depletion and impaired gluconeogenesis

Alcohol-related hypoglycemia results from hepatic glycogen depletion and impaired gluconeogenesis and not due to antibody formation, excessive insulin release from the pancreas, or rapid release of carbohydrate into the small bowel.

5. Which of the following is the chief adverse effect of thiazide diuretics?

A. Hypokalemia
B. Hypernatremia
C. Hypocalcemia
D. Hypermagnesemia

Click here to see the answer

Answer: A. Hypokalemia

Thiazide diuretics can induce electrolyte changes. Principle among those is hypokalemia.

6. The most distinctive sign of pertussis is

A. stridor without cough.
B. productive cough with basilar rales.
C. loose cough with coarse rhonchi.
D. paroxysmal cough with crowing inspiration.

Click here to see the answer

Answer: D. paroxysmal cough with crowing inspiration.

A paroxysmal cough with a loud inspiration (the whoop) is noted in pertussis.

7. The finding of egophony is most consistent with

A. emphysema.
B. atelectasis.
C. pneumothorax.
D. lobar pneumonia

Click here to see the answer

Answer D. Lobar pneumonia

Egophony occurs with consolidation caused by lobar pneumonia.

8. A patient presents with edema, which is most noticeable in the hands and face. Laboratory findings include proteinuria, hypoalbuminemia, and hyperlipidemia. The most likely diagnosis is

A. congestive heart failure.
B. end-stage liver disease.
C. nephrotic syndrome.
D. malnutrition.

Click here to see the answer

Answer: C. nephrotic syndrome.

Proteinuria, hyperlipidemia, and hypoalbuminemia are consistent with nephrotic syndrome.

9. The best course of action for a patient with a bothersome inflamed pinguecula (pingueculitis) is

A. antibiotic drops.
B. excision.
C. Visine drops.
D. no treatment.

Click here to see the answer

Answer: D. no treatment.

With pingueculitis, no treatment is necessary; a short course of NSAID drops or steroids may help.

pinguekula

10. An immunocompromised patient presents with signs and symptoms consistent with Legionella pneumophila who has not responded to initial antibiotic therapy with a macrolide. Which of the following should be added?

A. Clarithromycin (Biaxin)
B. Rifampin (Rifadin)
C. Levofloxacin (Levaquin)
D. Amoxicillin-clavulanate (Augmentin)

Click here to see the answer

Answer B. Rifampin

Rifampin should be used as an adjunct in patients with either a macrolide or quinolone antibiotic, who have failed therapy, are immunocompromised or have severe illness.

Looking for all the episodes?

This FREE series is limited to every other episode, you can download and enjoy the complete audio series by joining The PANCE and PANRE Exam Academy.

I will be be releasing new episodes every two weeks. The Academy is currently discounted, so sign up now.

This Podcast is also available on iTunes and Stitcher Radio for Android

  1. iTunes: The Audio PANCE AND PANRE Podcast iTunes
  2. Stitcher Radio: The Audio PANCE and PANRE Podcast Stitcher

itunes_logo-1


While you are over there, download and subscribe to Brian Wallaces’ excellent Physician Assistant Exam Review Podcast. Follow along with Brian who covers new topics twice monthly and really does an amazing job!

Cheers,

Stephen Pasquini PA-C

The post The Audio PANCE and PANRE Board Review Podcast Episode 25 appeared first on The Audio PANCE and PANRE.

]]>
Welcome to episode 25 of the FREE Audio PANCE and PANRE Physician Assistant Board Review Podcast. The Audio PANCE and PANRE is an audio board review series that includes 10 Multiple Choice PANCE and PANRE Board Review Questions in each episode. Welcome to episode 25 of the FREE Audio PANCE and PANRE Physician Assistant Board Review Podcast.
The Audio PANCE and PANRE is an audio board review series that includes 10 Multiple Choice PANCE and PANRE Board Review Questions in each episode.
I hope you enjoy this free audio component to the examination portion of this site. The full series is available to all members of the PANCE and PANRE Academy.

* You can download and listen to past FREE episodes here, on iTunes or Stitcher Radio.
* You can listen to the latest episode, download the transcript and take an interactive quiz of the questions below.

Download the FREE PDF transcript for FREE here or on Scribd.
Listen Carefully Then Take The Quiz
If you can’t see the audio player click here to listen to the full episode.
Questions 1-10
The Audio PANCE and PANRE Episode 25
1. Which of the following conditions will produce a transudative pleural effusion?
A. Kaposi’s sarcoma
B. Pneumonia
C. Cirrhosis
D. Mesothelioma
2. Which of the following pathophysiological processes is believed to initiate acute appendicitis?
A. Obstruction
B. Perforation
C. Hemorrhage
D. Vascular compromise
3. A 23 year-old female with a history of palpitations presents for evaluation. She admits to acute onset of rapid heart beating lasting seconds to minutes with associated shortness of breath and chest pain. The patient states she can relieve her symptoms with Valsalva. Which of the following is the most appropriate diagnostic study to establish a definitive diagnosis in this patient?
A. Cardiac catheterization
B. Cardiac MRI
C. Chest CT scan
D. Electrophysiology study
4. A known alcoholic presents to the emergency department with altered level of consciousness and a blood glucose level of 35 mg/dL. Which of the following best explains this glucose result?
A. Excess pancreatic insulin release
B. Rapid carbohydrate discharge into the small bowel
C. Agonist insulin receptor antibody formation
D. Hepatic glycogen depletion and impaired gluconeogenesis
5. Which of the following is the chief adverse effect of thiazide diuretics?
A. Hypokalemia
B. Hypernatremia
C. Hypocalcemia
D. Hypermagnesemia
6. The most distinctive sign of pertussis is
A. stridor without cough.
B. productive cough with basilar rales.
C. loose cough with coarse rhonchi.
D. paroxysmal cough with crowing inspiration.
7. The finding of egophony is most consistent with
A. emphysema.
B. atelectasis.
C. pneumothorax.
D.]]>
The Physician Assistant Life | Smarty PANCE full clean 8:29 165
The Audio PANCE and PANRE Board Review Podcast Episode 23 http://podcast.thepalife.com/the-audio-pance-and-panre-board-review-podcast-episode-23/ Tue, 04 Aug 2015 08:25:31 +0000 http://podcast.thepalife.com/?p=110 http://podcast.thepalife.com/the-audio-pance-and-panre-board-review-podcast-episode-23/#respond http://podcast.thepalife.com/the-audio-pance-and-panre-board-review-podcast-episode-23/feed/ 0 <p>Welcome to episode 23 of the FREE Audio PANCE and PANRE Physician Assistant Board Review Podcast. The Audio PANCE and PANRE is an audio board review series that includes 10 Multiple Choice PANCE and PANRE Board Review Questions in each episode. I hope you enjoy this free audio component to the examination portion of this site. The […]</p> <p>The post <a rel="nofollow" href="http://podcast.thepalife.com/the-audio-pance-and-panre-board-review-podcast-episode-23/">The Audio PANCE and PANRE Board Review Podcast Episode 23</a> appeared first on <a rel="nofollow" href="http://podcast.thepalife.com">The Audio PANCE and PANRE</a>.</p> Welcome to episode 23 of the FREE Audio PANCE and PANRE Physician Assistant Board Review Podcast.

The Audio PANCE and PANRE is an audio board review series that includes 10 Multiple Choice PANCE and PANRE Board Review Questions in each episode.

I hope you enjoy this free audio component to the examination portion of this site. The full series is available to all members of the PANCE and PANRE Academy.

  • You can download and listen to past FREE episodes here, on iTunes or Stitcher Radio.
  • You can listen to the latest episode, download the transcript and take an interactive quiz of the questions below.

adobe pdfDownload the FREE PDF transcript for FREE here or on  Scribd.

Listen Carefully Then Take The Quiz

If you can’t see the audio player click here to listen to the full episode.

Questions 1-10

The Audio PANCE and PANRE Episode 23

1. A 72 year-old female presents with vulvular pruritus for the last nine months, which has progressively worsened over the last two months. She states that she went through menopause at age 54 and has been on estrogen and progesterone therapy since that time. Physical examination reveals red lesions with white plaques on the vulva. What should the next course of management include?

A. Refer to a gynecologist for biopsy.

B. Refer to a dermatologist for antifungal therapy.

C. Treat with a topical steroid.

D. Treat with estrogen cream.

Click here to see the answer

Answer: A. Refer to a gynecologist for biopsy.

Vulvular squamous cell hyperplasia causes thickening and hyperkeratosis of the vulva. The lesions are red and moist and cause intense pruritus over time the area becomes thickened and a white plaque may develop. Biopsy must be done to evaluate for intraepithelial neoplasm or invasive tumor.

2. A 30 year-old female presents to the emergency department with a syncopal episode. She has a history of irregular menstrual cycles and infertility. She has scanty, persistent vaginal bleeding and sharp pelvic pain. A left adnexal mass is palpated. The most likely diagnosis is

A. placenta abruptio.

B. ectopic pregnancy.

C. pelvic inflammatory disease.

D. ruptured ovarian cyst.

Click here to see the answer

Answer B. ectopic pregnancy.

Infertility increases the risk of developing ectopic pregnancy. The onset of vaginal bleeding, pelvic pain, and formation of an adnexal mass makes this the most likely diagnosis.

3. A patient presents complaining of periumbilical pain. Which of the following anatomical sites is this finding associated with?

A. Bladder

B. Stomach

C. Pancreas

D. Small bowel

Click here to see the answer

Answer: D. Small bowel

Pain from the small intestine, appendix, or proximal colon causes periumbilical pain.

4. A 53 year-old patient presents with severe pain at the base of the thumb and no other finger involvement. The pain is worse with activity and lasts a short period of time following rest. There is no specific history of trauma to the thumb but the patient admits working with her hands as a typist. Which of the following is the most likely diagnosis?

A. Rheumatoid arthritis

B. Osteoarthritis

C. Hemochromatosis

D. Pseudogout

Click here to see the answer

Answer: B. Osteoarthritis

The base of the thumb is typically involved with osteoarthritis as are the DIP joints of the other fingers.

5. A 38 year-old female with history of coarctation of the aorta repair at the age of two presents with fevers for four weeks. The patient states that she has felt fatigued and achy during this time. Maximum temperature has been 102.1 degrees F. She denies cough, congestion, or other associated symptoms. Physical examination reveals a pale tired appearing female in no acute distress. Heart reveals a new grade III-IV/VI systolic ejection border at the apex, and a II/VI diastolic murmur at the right sternal border. What is the most likely diagnosis?

A. Acute myocardial infarction

B. Bacterial endocarditis

C. Acute pericarditis

D. Restrictive cardiomyopathy

Click here to see the answer

Answer B: Bacterial endocarditis

Bacterial endocarditis presents as febrile illness lasting several days to weeks, commonly with nonspecific symptoms, echocardiogram often reveals vegetations on affected valves.

6. A 45 year-old male presents with abdominal pain and one episode of mild hematemesis, which happened days ago. On physical examination, vital signs are stable and he is in no acute distress. Hemoglobin and hematocrit are unremarkable; endoscopy reveals non-bleeding small superficial ulceration of the duodenal bulb. Rapid urease test is positive. Which of the following is the most appropriate treatment at this time?

A. Schedule for a selective vagotomy and antrectomy

B. Start an antacid along with omeprazole (Prilosec)

C. Schedule elective ulcer excision and start sucralfate (Carafate)

D. Start omeprazole (Prilosec) and antibiotic therapy against H. pylori

Click here to see the answer

Answer: D. Start omeprazole (Prilosec) and antibiotic therapy against H. pylori

Treatment goals of H. pylori associated ulcers include eradicating the infection with appropriate antibiotics as well as use of a proton pump inhibitor, such as omeprazole,

7. Which of the following findings is usually associated with Addison’s disease?

A. Weight gain

B. Hypertension

C. Increased pigmentation

D. High plasma cortisol levels

Click here to see the answer

Answer: C. Increased pigmentation

Patients with Addison’s disease have diffuse tanning over nonexposed and exposed skin due to increased melanocytic factor that is released with adrenocorticotropic hormone.

8. A 60 year-old patient with COPD characteristic of emphysema presents with a cough and increased sputum production. The following information is noted: Temperature 100°F (37.8°C); Respiratory rate 20/min; Heart rate 88 beats/min; pH 7.44; PaO2 75 mmHg; PaCO2 40 mmHg; O2 saturation 92%. Physical examination is remarkable for increased AP diameter, diminished breath sounds without wheezes, rhonchi, or other signs of respiratory distress. Which of the following would be an appropriate treatment for this patient?

A. Broad-spectrum antibiotic

B. Admission to the hospital

C. Oxygen at 6 L/min by nasal cannula

D. Brief course of oral theophylline

Click here to see the answer

Answer: A. Broad-spectrum antibiotic

A. Sputum production is extremely variable from patient to patient, but any increase in sputum with a history of COPD reported by a patient must be regarded as potentially infectious and treated promptly.

9. Which of the following physical findings suggest pernicious anemia?

A. Splenomegaly and hepatomegaly

B. Petechiae and ecchymosis

C. Loss of position and vibratory sensation

D. Cheilosis and koilonychia

Click here to see the answer

Answer: C. Loss of position and vibratory sensation

Loss of position and vibratory sensation are common neurologic findings in pernicious anemia.

10. A 60 year-old male presents with a normochromic, normocytic anemia and splenomegaly. His past history reveals several episodes of bacterial pneumonia in the past year. The WBC count is 43,000 mm3 with 25% segmented neutrophils, 3% blasts, 70% mature lymphocytes, 1% basophils, and 1% eosinophils. This most likely represents

A. myelodysplastic syndrome.

B. acute lymphocytic leukemia.

C. chronic lymphocytic leukemia.

D. chronic myelogenous leukemia.

Click here to see the answer

Answer: C. chronic lymphocytic leukemia.

Chronic lymphocytic leukemia usually occurs after the age of 50 presenting with lymphocytosis > 20,000 mm3 and lymphocytes that appear mature.

Watch the video

Looking for all the episodes?

This FREE series is limited to every other episode, you can download and enjoy the complete audio series by joining The PANCE and PANRE Exam Academy.

I will be be releasing new episodes every two weeks. The Academy is currently discounted, so sign up now.

This Podcast is also available on iTunes and Stitcher Radio for Android

  1. iTunes: The Audio PANCE AND PANRE Podcast iTunes
  2. Stitcher Radio: The Audio PANCE and PANRE Podcast Stitcher

itunes_logo-1

Cheers,

Stephen Pasquini PA-C

The post The Audio PANCE and PANRE Board Review Podcast Episode 23 appeared first on The Audio PANCE and PANRE.

]]>
Welcome to episode 23 of the FREE Audio PANCE and PANRE Physician Assistant Board Review Podcast. The Audio PANCE and PANRE is an audio board review series that includes 10 Multiple Choice PANCE and PANRE Board Review Questions in each episode. Welcome to episode 23 of the FREE Audio PANCE and PANRE Physician Assistant Board Review Podcast.
The Audio PANCE and PANRE is an audio board review series that includes 10 Multiple Choice PANCE and PANRE Board Review Questions in each episode.
I hope you enjoy this free audio component to the examination portion of this site. The full series is available to all members of the PANCE and PANRE Academy.

* You can download and listen to past FREE episodes here, on iTunes or Stitcher Radio.
* You can listen to the latest episode, download the transcript and take an interactive quiz of the questions below.

Download the FREE PDF transcript for FREE here or on  Scribd.
Listen Carefully Then Take The Quiz
If you can’t see the audio player click here to listen to the full episode.
Questions 1-10
The Audio PANCE and PANRE Episode 23
1. A 72 year-old female presents with vulvular pruritus for the last nine months, which has progressively worsened over the last two months. She states that she went through menopause at age 54 and has been on estrogen and progesterone therapy since that time. Physical examination reveals red lesions with white plaques on the vulva. What should the next course of management include?
A. Refer to a gynecologist for biopsy.
B. Refer to a dermatologist for antifungal therapy.
C. Treat with a topical steroid.
D. Treat with estrogen cream.
2. A 30 year-old female presents to the emergency department with a syncopal episode. She has a history of irregular menstrual cycles and infertility. She has scanty, persistent vaginal bleeding and sharp pelvic pain. A left adnexal mass is palpated. The most likely diagnosis is
A. placenta abruptio.
B. ectopic pregnancy.
C. pelvic inflammatory disease.
D. ruptured ovarian cyst.
3. A patient presents complaining of periumbilical pain. Which of the following anatomical sites is this finding associated with?
A. Bladder
B. Stomach
C. Pancreas
D. Small bowel
4. A 53 year-old patient presents with severe pain at the base of the thumb and no other finger involvement. The pain is worse with activity and lasts a short period of time following rest. There is no specific history of trauma to the thumb but the patient admits working with her hands as a typist. Which of the following is the most likely diagnosis?
A. Rheumatoid arthritis
B. Osteoarthritis
C. Hemochromatosis
D. Pseudogout
5. A 38 year-old female with history of coarctation of the aorta repair at the age of two presents with fevers for four weeks. The patient states that she has felt fatigued and achy during this time. Maximum temperature has been 102.]]>
The Physician Assistant Life | Smarty PANCE full clean 11:25 110
The Audio PANCE and PANRE Board Review Podcast Episode 21 http://podcast.thepalife.com/the-audio-pance-and-panre-board-review-podcast-episode-21/ Sun, 07 Jun 2015 09:04:27 +0000 http://podcast.thepalife.com/?p=106 http://podcast.thepalife.com/the-audio-pance-and-panre-board-review-podcast-episode-21/#respond http://podcast.thepalife.com/the-audio-pance-and-panre-board-review-podcast-episode-21/feed/ 0 <p>Welcome to episode 21 of the FREE Audio PANCE and PANRE Physician Assistant Board Review Podcast. The Audio PANCE and PANRE is an audio board review series that includes 10 Multiple Choice PANCE and PANRE Board Review Questions in each episode. I hope you enjoy this free audio component to the examination portion of this site. The […]</p> <p>The post <a rel="nofollow" href="http://podcast.thepalife.com/the-audio-pance-and-panre-board-review-podcast-episode-21/">The Audio PANCE and PANRE Board Review Podcast Episode 21</a> appeared first on <a rel="nofollow" href="http://podcast.thepalife.com">The Audio PANCE and PANRE</a>.</p> Welcome to episode 21 of the FREE Audio PANCE and PANRE Physician Assistant Board Review Podcast.

The Audio PANCE and PANRE is an audio board review series that includes 10 Multiple Choice PANCE and PANRE Board Review Questions in each episode.

I hope you enjoy this free audio component to the examination portion of this site. The full series is available to all members of the PANCE and PANRE Academy.

  • You can download and listen to past FREE episodes here, on iTunes or Stitcher Radio.
  • You can listen to the latest episode, download the transcript and take an interactive quiz of the questions below.

adobe pdfDownload the FREE PDF transcript for FREE here or on  Scribd.

Listen Carefully Then Take The Quiz

If you can’t see the audio player click here to listen to the full episode.

Questions 1-10

The Audio PANCE and PANRE Episode 21

1. A 28-year-old female, who has experienced occasional painful migratory arthralgias, complains now of a tender, swollen, and hot left ankle. The joint was aspirated and the synovial fluid showed 55,000 WBCs, 75% polymorphonuclear lymphocytes, low glucose level, and no crystals. Which of the following would be the most likely diagnosis?

A. Rheumatoid arthritis
B. Septic arthritis
C. Gouty arthritis
D. Osteoarthritis

Click here to see the answer

Answer: B. Cyanosis

Cyanosis is very common in tetralogy of Fallot.

2. When the diagnosis of gonococcal urethritis is confirmed, which of the following is the treatment of choice?

A. Ceftriaxone (Rocephin)
B. Azithromycin 1g orally in a single dose
C. Ceftriaxone 250 mg IM in a single dose PLUS Azithromycin 1g orally in a single dose
D. Doxycycline (Vibramycin)

Click here to see the answer

Answer: C. Ceftriaxone 250 mg IM in a single dose PLUS Azithromycin 1g orally in a single dose.

Because of resistance, the CDC now recommends dual therapy with Ceftriaxone 250 mg IM in a single dose PLUS Azithromycin 1g orally in a single dose Alertnative regimens if ceftriaxone is not available include Cefixime 400 mg orally in a single dose PLUS Azithromycin 1 g orally in a single dose

3. Erythema nodosum is characterized by

A. subcutaneous red tender nodules.
B. brown pigmentation on the lower extremities.
C. tender lymph nodes in the groin.
D. scaling red macules.

Click here to see the answer

Answer: A, subcutaneous red tender nodules.

Erythema nodosum produces erythematous red tender nodules, especially on the shins.

4. Use of systemic corticosteroids can cause which of the following adverse effects in the eye?

A. Cortical blindness
B. Optic atrophy
C. Glaucoma
D. Papilledema

Click here to see the answer

Answer: C. Glaucoma

Glaucoma can be caused by the long-term use of steroids.

5. It is determined that a woman has a nonexistent rubella titer level during her first trimester of pregnancy. When should she receive the rubella vaccine?

A. During the first trimester of pregnancy
B. During the second trimester of pregnancy
C. During the third trimester of pregnancy
D. After delivery of the infant

Click here to see the answer

Answer: D. After delivery of the infant

The patient should not receive the rubella vaccine during the course of her pregnancy as the possibility of transmission of the rubella virus does exist. During the time that the patient is without protective titer she should avoid anyone with active rubella infection. The proper time to receive the vaccine is after delivery of the infant.

6. A patient with which of the following is at highest risk for coronary artery disease?

A. Congenital heart disease
B. Polycystic ovary syndrome
C. Acute renal failure
D. Diabetes mellitus

Click here to see the answer

Answer: D. Diabetes mellitus

Patients with diabetes mellitus are in the same risk category for coronary artery disease as those patients with established atherosclerotic disease.

7. A 44-year-old female presents with ongoing arthralgias and myalgias with intermittent flares of arthritis. She is found to have a malar rash that worsens with sun exposure. She is known to have progressive renal damage and has recurrent infections that are slow to respond to therapy. She takes ibuprofen (Motrin) as needed for her joint pain and takes no other medication. Which of the following tests would be the initial test recommended to screen for this diagnosis?

A. Rheumatoid factor
B. Antihistone antibodies
C. Anti-Smith (Anti-Sm) antibodies
D. Anti-nuclear antibodies (ANA)

Click here to see the answer

Answer: A. Glipizide

Sulfonylureas increase insulin levels and predispose patients to hypoglycemia.

8. Upon stroking of the lateral aspect of the sole from the heel to the ball of the foot, the great toe dorsiflexes and the other toes fan. This is a positive

A. Kernig’s sign.
B. Brudzinski’s sign.
C. Babinski’s sign.
D. Gower’s sign.

Click here to see the answer

Answer: C. A Babinski test is performed by stroking the lateral aspect of the sole from the heel to the ball of the foot, the great toe dorsiflexes and the other toes fan in a positive test.

For your knowledge:

Kernig’s sign is positive when pain is noted on straightening the knee after flexing both the hip and knee.

Brudzinski’s sign occurs with neck flexion resulting in resultant flexion of the hips. It is a sign of meningeal irritation.

A positive Gower’s sign is noted in certain types of muscular dystrophy and is described as children rising to stand by rolling over prone and pushing off the floor with arms while the legs remain extended.

9. Which of the following strategies promotes improved carbohydrate metabolism and is recommended for all Type 2 diabetic patients?

A. Low-carbohydrate, high-protein diet
B. Routine aerobic exercise
C. Metformin (Glucophage)
D. Acupuncture

Click here to see the answer

Answer: B. Routine aerobic exercise

Routine exercise improves carbohydrate metabolism and insulin sensitivity.

10. Acute rebound hypertensive episodes have been reported to occur with the sudden withdrawal of

A. verapamil (Calan).
B. lisinopril (Prinivil).
C. clonidine (Catapres).
D. hydrochlorothiazide (HCTZ)

Click here to see the answer

Answer: C, Clonodine

Clonidine (Catapres) is a central alpha agonist and abrupt withdrawal may produce a rebound hypertensive crisis.

Looking for all the episodes?

This FREE series is limited to every other episode, you can download and enjoy the complete audio series by joining The PANCE and PANRE Exam Academy.

I will be be releasing new episodes every two weeks. The Academy is currently discounted, so sign up now.

This Podcast is also available on iTunes and Stitcher Radio for Android

  1. iTunes: The Audio PANCE AND PANRE Podcast iTunes
  2. Stitcher Radio: The Audio PANCE and PANRE Podcast Stitcher

itunes_logo-1

 

Cheers,

Stephen Pasquini PA-C

The post The Audio PANCE and PANRE Board Review Podcast Episode 21 appeared first on The Audio PANCE and PANRE.

]]>
Welcome to episode 21 of the FREE Audio PANCE and PANRE Physician Assistant Board Review Podcast. The Audio PANCE and PANRE is an audio board review series that includes 10 Multiple Choice PANCE and PANRE Board Review Questions in each episode. Welcome to episode 21 of the FREE Audio PANCE and PANRE Physician Assistant Board Review Podcast.
The Audio PANCE and PANRE is an audio board review series that includes 10 Multiple Choice PANCE and PANRE Board Review Questions in each episode.
I hope you enjoy this free audio component to the examination portion of this site. The full series is available to all members of the PANCE and PANRE Academy.

* You can download and listen to past FREE episodes here, on iTunes or Stitcher Radio.
* You can listen to the latest episode, download the transcript and take an interactive quiz of the questions below.

Download the FREE PDF transcript for FREE here or on  Scribd.

Listen Carefully Then Take The Quiz
If you can’t see the audio player click here to listen to the full episode.
Questions 1-10
The Audio PANCE and PANRE Episode 21
1. A 28-year-old female, who has experienced occasional painful migratory arthralgias, complains now of a tender, swollen, and hot left ankle. The joint was aspirated and the synovial fluid showed 55,000 WBCs, 75% polymorphonuclear lymphocytes, low glucose level, and no crystals. Which of the following would be the most likely diagnosis?
A. Rheumatoid arthritis
B. Septic arthritis
C. Gouty arthritis
D. Osteoarthritis
2. When the diagnosis of gonococcal urethritis is confirmed, which of the following is the treatment of choice?
A. Ceftriaxone (Rocephin)
B. Azithromycin 1g orally in a single dose
C. Ceftriaxone 250 mg IM in a single dose PLUS Azithromycin 1g orally in a single dose
D. Doxycycline (Vibramycin)
3. Erythema nodosum is characterized by
A. subcutaneous red tender nodules.
B. brown pigmentation on the lower extremities.
C. tender lymph nodes in the groin.
D. scaling red macules.
4. Use of systemic corticosteroids can cause which of the following adverse effects in the eye?
A. Cortical blindness
B. Optic atrophy
C. Glaucoma
D. Papilledema
5. It is determined that a woman has a nonexistent rubella titer level during her first trimester of pregnancy. When should she receive the rubella vaccine?
A. During the first trimester of pregnancy
B. During the second trimester of pregnancy
C. During the third trimester of pregnancy
D. After delivery of the infant
6. A patient with which of the following is at highest risk for coronary artery disease?
A. Congenital heart disease
B. Polycystic ovary syndrome
C. Acute renal failure
D. Diabetes mellitus
7. A 44-year-old female presents with ongoing arthralgias and myalgias with intermittent flares of arthritis. She is found to have a malar rash that worsens with sun exposure. She is known to have progressive renal damage and has recurrent infections that are slow to respond to therapy.]]>
The Physician Assistant Life | Smarty PANCE full clean 9:43 106
The Audio PANCE and PANRE Board Review Podcast Episode 19 http://podcast.thepalife.com/the-audio-pance-and-panre-board-review-podcast-episode-19/ Sat, 02 May 2015 14:34:53 +0000 http://podcast.thepalife.com/?p=99 http://podcast.thepalife.com/the-audio-pance-and-panre-board-review-podcast-episode-19/#respond http://podcast.thepalife.com/the-audio-pance-and-panre-board-review-podcast-episode-19/feed/ 0 <p>Welcome to episode 19 of the FREE Audio PANCE and PANRE Physician Assistant Board Review Podcast. The Audio PANCE and PANRE is an audio board review series that includes 10 Multiple Choice PANCE and PANRE Board Review Questions in each episode. I hope you enjoy this free audio component to the examination portion of this site. The […]</p> <p>The post <a rel="nofollow" href="http://podcast.thepalife.com/the-audio-pance-and-panre-board-review-podcast-episode-19/">The Audio PANCE and PANRE Board Review Podcast Episode 19</a> appeared first on <a rel="nofollow" href="http://podcast.thepalife.com">The Audio PANCE and PANRE</a>.</p> Welcome to episode 19 of the FREE Audio PANCE and PANRE Physician Assistant Board Review Podcast.

The Audio PANCE and PANRE is an audio board review series that includes 10 Multiple Choice PANCE and PANRE Board Review Questions in each episode.

I hope you enjoy this free audio component to the examination portion of this site. The full series is available to all members of the PANCE and PANRE Academy.

  • You can download and listen to past FREE episodes here, on iTunes or Stitcher Radio.
  • You can listen to the latest episode, download the transcript and take an interactive quiz of the questions below.

adobe pdfUse one of the social links below to unlock and download the FREE PDF Transcript of this podcast:

[sociallocker id=”10126″]Download the PDF for FREE here or on  Scribd [/sociallocker]

Listen Carefully Then Take The Quiz

Questions 1-10

The Audio PANCE and PANRE Episode 17

1. A 12 month-old child with tetralogy of Fallot is most likely to have which of the following clinical features?

A. Chest pain
B. Cyanosis
C. Convulsions
D. Palpitations

Click here to see the answer

Answer: B. Cyanosis

Cyanosis is very common in tetralogy of Fallot.

2. Intraarticular injection of hyaluronic acid has been approved for treatment of patients with which of the following conditions?

A. Rheumatoid arthritis of the knee
B. Osteoarthritis of the knee
C. Olecranon bursitis
D. Gouty arthritis

Click here to see the answer

Answer: B. Osteoarthritis of the knee

Intraarticular injection of hyaluronic acid has been approved recently for treatment of patients with osteoarthritis of the knee that have failed other therapies. Although the onset of action of this medication is slower than injected glucocorticoids, it has a sustained length of activity outlasting the injected glucocorticoids.

3. A 28 year-old female, who has experienced occasional painful migratory arthralgias, complains now of a tender, swollen, and hot left ankle. The joint was aspirated and the synovial fluid showed 55,000 WBCs, 75% polymorphonuclear lymphocytes, low glucose level, and no crystals. Which of the following would be the most likely diagnosis?

A. Rheumatoid arthritis
B. Septic arthritis
C. Gouty arthritis
D. Osteoarthritis

Click here to see the answer

Answer:  B. Septic arthritis

Septic arthritis presents with a large number of WBCs, predominantly polymorphonuclear, and with glucose levels much lower than serum levels.

4. A 25 year-old presents with pain in the proximal ulna after falling directly on the forearm. X-ray shows fracture of the proximal 1/3rd of the ulna. There is an associated anterior radial head dislocation. What is the proper name for this condition?

A. Galeazzi fracture
B. Monteggia fracture
C. Colles’ fracture
D. Smith fracture

Click here to see the answer

Answer: B. Monteggia fracture

A Monteggia fracture is a fracture of the proximal ulna with anterior dislocation of the radial head.

5. A 20 year-old male presents with a mass in the groin. On examination with the patient standing, a mass is noted that extends into the scrotum. The patient denies any trauma. The most likely diagnosis is

A. an indirect inguinal hernia.
B. a direct inguinal hernia.
C. an obturator hernia.
D. a femoral hernia.

Click here to see the answer

Answer: A. an indirect inguinal hernia.

An indirect inguinal hernia is caused by a patent processus vaginalis and the hernial contents may be felt in the ipsilateral scrotum.

6. A patient with type 2 diabetes mellitus presents for a yearly eye exam. Ophthalmoscopic exam reveals neovascularization. Which of the following is the most likely complication related to this finding?

A. Glaucoma
B. Cataracts
C. Vitreous hemorrhage
D. Optic neuritis

Click here to see the answer

Answer: C. Vitreous hemorrhage

Proliferative retinopathy, as evidenced by neovascularization, is associated with an increased risk of vitreous hemorrhage.

7. Which of the following oral hypoglycemic agents when used as monotherapy is most likely to cause hypoglycemia?

A. Glipizide (Glucotrol)
B. Metformin (Glucophage)
C. Pioglitazone (Actos)
D. Acarbose (Precose)

Click here to see the answer

Answer: A. Glipizide

Sulfonylureas increase insulin levels and predispose patients to hypoglycemia.

8. A 75 year-old female presents with medial knee pain that worsens with stair climbing. Physical examination reveals swelling and point tenderness inferior and medial to the patella and tenderness overlying the medial tibial plateau. Which of the following is the most likely diagnosis?

A. Pes anserine bursitis
B. Prepatellar bursitis
C. Infrapatellar bursitis
D. Trochanteric bursitis

Click here to see the answer

Answer: A. Pes anserine bursitis

The pes anserine bursa underlies the semimembranosus tendon and may become inflamed or painful owing to trauma, overuse, or inflammation. It is a common cause of knee pain and it is often misdiagnosed in adults.

9. A 23 year-old male presents with syncope. On physical examination you note a medium-pitched, mid-systolic murmur that decreases with squatting and increases with straining. Which of the following is the most likely diagnosis?

A. Hypertrophic cardiomyopathy
B. Aortic stenosis
C. Mitral regurgitation
D. Pulmonic stenosis

Click here to see the answer

Answer: A. Hypertrophic cardiomyopathy

Hypertrophic cardiomyopathy is characterized by a medium- pitched, mid-systolic murmur that decreases with squatting and increases with straining.

10. Which of the following can be a very serious consequence of using antidiarrheals in a patient with inflammatory bowel disease?

A. Lymphoma
B. Toxic megacolon
C. Bone marrow suppression
D. Delayed serum sickness-like reaction

Click here to see the answer

Answer: B. Toxic megagolon.

Antidiarrheals may cause the development of toxic megacolon when used by patients with active severe inflammatory bowel disease.

Looking for all the episodes?

This FREE series is limited to every other episode, you can download and enjoy the complete audio series by joining The PANCE and PANRE Exam Academy.

I will be be releasing new episodes every two weeks. The Academy is currently discounted, so sign up now.

This Podcast is also available on iTunes and Stitcher Radio for Android

  1. iTunes: The Audio PANCE AND PANRE Podcast iTunes
  2. Stitcher Radio: The Audio PANCE and PANRE Podcast Stitcher

itunes_logo-1

The post The Audio PANCE and PANRE Board Review Podcast Episode 19 appeared first on The Audio PANCE and PANRE.

]]>
Welcome to episode 19 of the FREE Audio PANCE and PANRE Physician Assistant Board Review Podcast. The Audio PANCE and PANRE is an audio board review series that includes 10 Multiple Choice PANCE and PANRE Board Review Questions in each episode. Welcome to episode 19 of the FREE Audio PANCE and PANRE Physician Assistant Board Review Podcast.
The Audio PANCE and PANRE is an audio board review series that includes 10 Multiple Choice PANCE and PANRE Board Review Questions in each episode.
I hope you enjoy this free audio component to the examination portion of this site. The full series is available to all members of the PANCE and PANRE Academy.

* You can download and listen to past FREE episodes here, on iTunes or Stitcher Radio.
* You can listen to the latest episode, download the transcript and take an interactive quiz of the questions below.

Use one of the social links below to unlock and download the FREE PDF Transcript of this podcast:
[sociallocker id=”10126″]Download the PDF for FREE here or on  Scribd [/sociallocker]
Listen Carefully Then Take The Quiz
Questions 1-10
The Audio PANCE and PANRE Episode 17
1. A 12 month-old child with tetralogy of Fallot is most likely to have which of the following clinical features?
A. Chest pain
B. Cyanosis
C. Convulsions
D. Palpitations
2. Intraarticular injection of hyaluronic acid has been approved for treatment of patients with which of the following conditions?
A. Rheumatoid arthritis of the knee
B. Osteoarthritis of the knee
C. Olecranon bursitis
D. Gouty arthritis
3. A 28 year-old female, who has experienced occasional painful migratory arthralgias, complains now of a tender, swollen, and hot left ankle. The joint was aspirated and the synovial fluid showed 55,000 WBCs, 75% polymorphonuclear lymphocytes, low glucose level, and no crystals. Which of the following would be the most likely diagnosis?
A. Rheumatoid arthritis
B. Septic arthritis
C. Gouty arthritis
D. Osteoarthritis
4. A 25 year-old presents with pain in the proximal ulna after falling directly on the forearm. X-ray shows fracture of the proximal 1/3rd of the ulna. There is an associated anterior radial head dislocation. What is the proper name for this condition?
A. Galeazzi fracture
B. Monteggia fracture
C. Colles’ fracture
D. Smith fracture
5. A 20 year-old male presents with a mass in the groin. On examination with the patient standing, a mass is noted that extends into the scrotum. The patient denies any trauma. The most likely diagnosis is
A. an indirect inguinal hernia.
B. a direct inguinal hernia.
C. an obturator hernia.
D. a femoral hernia.
6. A patient with type 2 diabetes mellitus presents for a yearly eye exam. Ophthalmoscopic exam reveals neovascularization. Which of the following is the most likely complication related to this finding?
A. Glaucoma
B. Cataracts
C. Vitreous hemorrhage
D. Optic neuritis
7. Which of the following oral hypoglycemic agents when used as monotherapy is most likely to cause hypoglycemia?
A. Glipizide (Glucotrol)
B. Metformin (Glucophage)
C. Pioglitazone (Actos)
D. Acarbose (Precose)
8.]]>
The Physician Assistant Life | Smarty PANCE full clean 8:09 99
The Audio PANCE and PANRE Board Review Podcast Episode 17 http://podcast.thepalife.com/episode-17/ Wed, 25 Mar 2015 12:00:38 +0000 http://podcast.thepalife.com/?p=93 http://podcast.thepalife.com/episode-17/#respond http://podcast.thepalife.com/episode-17/feed/ 0 <p>Welcome to episode 17 of the FREE Audio PANCE and PANRE Physician Assistant Board Review Podcast. The Audio PANCE and PANRE is an audio board review series that includes 10 Multiple Choice PANCE and PANRE Board Review Questions in each episode. I hope you enjoy this free audio component to the examination portion of this site. The […]</p> <p>The post <a rel="nofollow" href="http://podcast.thepalife.com/episode-17/">The Audio PANCE and PANRE Board Review Podcast Episode 17</a> appeared first on <a rel="nofollow" href="http://podcast.thepalife.com">The Audio PANCE and PANRE</a>.</p> Welcome to episode 17 of the FREE Audio PANCE and PANRE Physician Assistant Board Review Podcast.

The Audio PANCE and PANRE is an audio board review series that includes 10 Multiple Choice PANCE and PANRE Board Review Questions in each episode.

I hope you enjoy this free audio component to the examination portion of this site. The full series is available to all members of the PANCE and PANRE Academy.

  • You can download and listen to past FREE episodes here, on iTunes or Stitcher Radio.
  • You can listen to the latest episode, download the transcript and take an interactive quiz of the questions below.

You can download a Free PDF Transcript of the Questions and Answers

Listen Carefully Then Take The Quiz

Questions 1-10

The Audio PANCE and PANRE Episode 17

1. A 64 year-old male presents complaining of new onset of fatigue, weight gain, constipation, erectile dysfunction, and loss of body hair. Laboratory investigation demonstrates: TSH less than 0.5 microunits/mL (normal range 0.5-5.0 microU/mL); Thyroxine (T4) 2 mcg/dL (normal range 5-12 mcg/dL); Prolactin 10 nanograms/ml (normal 2 – 18 ng/mL.) What is the most likely diagnosis?

A. Primary hypothyroidism
B. Excessive dosing of levothyroxine (Synthroid)
C. Hypopituitarism
D. Subacute thyroiditis

Click here to see the answer
C. The low trophic and target hormone levels combined with symptoms of hypogonadism indicate this patient has hypopituitarism.

2. A 15 year-old male was seen last week with complaints of sore throat, headache, and mild cough. A diagnosis of URI was made and supportive treatment was initiated. He returns today with complaints of worsening cough and increasing fatigue. At this time, chest x-ray reveals bilateral hilar infiltrates. A WBC count is normal and a cold hemagglutinin titer is elevated. The most likely diagnosis is

A. tuberculosis.
B. mycoplasma pneumonia.
C. pneumococcal pneumonia.
D. staphylococcal pneumonia.

Click here to see the answer
B. The insidious onset of symptoms, the interstitial infiltrate on chest x-ray, and elevated cold hemagglutinin titer make this diagnosis the most likely.

3. Which of the following clinical manifestations is common in candidal vulvovaginitis?

A. Extreme vulvar irritation
B. Firm, painless ulcer
C. Tender lymphadenopathy
D. Purulent discharge

Click here to see the answer
A. Candida infection presents with pruritus, vulvovaginal erythema, and white, cheese-like (curd) discharge that may be malodorous.

4. A 63 year-old female presents with a complaint of chest pressure for one hour, noticed upon awakening. She admits to associated nausea, vomiting, and shortness of breath. 12 lead EKG reveals ST segment elevation in leads II, III, and AVF. Which of the following is the most likely diagnosis?

A. Aortic dissection
B. Inferior wall myocardial infarction
C. Acute pericarditis
D. Pulmonary embolus

Click here to see the answer
B. Myocardial infarction often presents with chest pressure and associated nausea and vomiting. ST segment elevation in leads II, III, and AVF are classic findings seen in acute inferior wall myocardial infarction.

5. An 18 year-old woman presents to the clinic complaining of fatigue. She reports a past history of lifelong frequent nosebleeds and bleeding gums. She also has menorrhagia. Her mother and maternal grandfather have a similar bleeding history. Initial lab results are as follows: WBC 9,500/mm3, Hgb 10.9 g/dL, HCT 33%, MCV 69 fL, MCHC 26 pg and platelets 284,000/mm3. Which of the following tests should be ordered to evaluate this patient’s diagnosis?

A. Hemoglobin electrophoresis
B. Bleeding time and platelet aggregometry
C. Bone marrow aspiration
D. PT and aPTT

Click here to see the answer
B. The patient’s presentation is consistent with a congenital qualitative platelet disorder, most likely von Willebrand’s Disease, necessitating a bleeding time and evaluation of platelet function.

6. A 35 year-old patient has recurrent seasonal rhinitis and a history of mild asthma. Which of the following should be included for first-line management?

A. Immunotherapy
B. Decongestants
C. Corticosteroid inhalers
D. Cromolyn sodium (Intal)

Click here to see the answer
C. Regular use of corticosteroid nasal spray and oral inhalers prior to the allergy season is among the best means of preventing allergies.

7. A 35-year-old female presents with multiple ulcerative lesions on her labia and perineum. A Tzanck preparation of one of the lesions reveals multinucleated giant cells. Which of the following is the most likely diagnosis?

A. Herpes Simplex Virus (HSV)
B. Molluscum Contagiosum Virus (MCV)
C. Human Papilloma Virus (HPV)
D. Syphilis

Click here to see the answer
A. The presentation seen on the Tzanck preparation is characteristic of HSV.

8. Small grayish vesicles and punched-out ulcers in the posterior pharynx in a child with pharyngitis is representative of which organism?

A. Epstein-Barr virus
B. Group C Streptococcus
C. Coxsackievirus
D. Gonorrhea

Click here to see the answer
C. Coxsackievirus presents with small grayish vesicles and punched-out ulcers in the posterior pharynx.

9. A 53 year-old female has a diagnosis of migraine headaches. She had been using sumatriptan (Imitrex) to abort her headaches, but she is now having one or two headaches per week. The most appropriate preventive therapy is

A. zolmitriptan (Zomig).
B. promethazine (Phenergan).
C. propranolol (Inderal).
D. fluoxetine (Prozac).

Click here to see the answer
C. Propanolol is useful in preventing migraine headaches and may be maintained indefinitely.

10. Which of the following primitive reflexes should begin to disappear at about 2 months of age in a normal infant?

A. Moro
B. Grasp
C. Tonic neck
D. Parachute

Click here to see the answer
B. The grasp reflex starts to disappear at about 2-3 months of age.

Looking for all the episodes?

This FREE series is limited to every other episode, you can download and enjoy the complete audio series by joining The PANCE and PANRE Exam Academy.

I will be be releasing new episodes every two weeks. The Academy is currently discounted, so sign up now.

This Podcast is also available on iTunes and Stitcher Radio for Android

  1. iTunes: The Audio PANCE AND PANRE Podcast iTunes
  2. Stitcher Radio: The Audio PANCE and PANRE Podcast Stitcher

itunes_logo-1

The post The Audio PANCE and PANRE Board Review Podcast Episode 17 appeared first on The Audio PANCE and PANRE.

]]>
Welcome to episode 17 of the FREE Audio PANCE and PANRE Physician Assistant Board Review Podcast. The Audio PANCE and PANRE is an audio board review series that includes 10 Multiple Choice PANCE and PANRE Board Review Questions in each episode. Welcome to episode 17 of the FREE Audio PANCE and PANRE Physician Assistant Board Review Podcast.
The Audio PANCE and PANRE is an audio board review series that includes 10 Multiple Choice PANCE and PANRE Board Review Questions in each episode.
I hope you enjoy this free audio component to the examination portion of this site. The full series is available to all members of the PANCE and PANRE Academy.

* You can download and listen to past FREE episodes here, on iTunes or Stitcher Radio.
* You can listen to the latest episode, download the transcript and take an interactive quiz of the questions below.

Listen Carefully Then Take The Quiz
Questions 1-10
The Audio PANCE and PANRE Episode 17
1. A 64 year-old male presents complaining of new onset of fatigue, weight gain, constipation, erectile dysfunction, and loss of body hair. Laboratory investigation demonstrates: TSH less than 0.5 microunits/mL (normal range 0.5-5.0 microU/mL); Thyroxine (T4) 2 mcg/dL (normal range 5-12 mcg/dL); Prolactin 10 nanograms/ml (normal 2 – 18 ng/mL.) What is the most likely diagnosis?
A. Primary hypothyroidism
B. Excessive dosing of levothyroxine (Synthroid)
C. Hypopituitarism
D. Subacute thyroiditis
2. A 15 year-old male was seen last week with complaints of sore throat, headache, and mild cough. A diagnosis of URI was made and supportive treatment was initiated. He returns today with complaints of worsening cough and increasing fatigue. At this time, chest x-ray reveals bilateral hilar infiltrates. A WBC count is normal and a cold hemagglutinin titer is elevated. The most likely diagnosis is
A. tuberculosis.
B. mycoplasma pneumonia.
C. pneumococcal pneumonia.
D. staphylococcal pneumonia.
3. Which of the following clinical manifestations is common in candidal vulvovaginitis?
A. Extreme vulvar irritation
B. Firm, painless ulcer
C. Tender lymphadenopathy
D. Purulent discharge
4. A 63 year-old female presents with a complaint of chest pressure for one hour, noticed upon awakening. She admits to associated nausea, vomiting, and shortness of breath. 12 lead EKG reveals ST segment elevation in leads II, III, and AVF. Which of the following is the most likely diagnosis?
A. Aortic dissection
B. Inferior wall myocardial infarction
C. Acute pericarditis
D. Pulmonary embolus
5. An 18 year-old woman presents to the clinic complaining of fatigue. She reports a past history of lifelong frequent nosebleeds and bleeding gums. She also has menorrhagia. Her mother and maternal grandfather have a similar bleeding history. Initial lab results are as follows: WBC 9,500/mm3, Hgb 10.9 g/dL, HCT 33%, MCV 69 fL, MCHC 26 pg and platelets 284,000/mm3. Which of the following tests should be ordered to evaluate this patient’s diagnosis?
A. Hemoglobin electrophoresis
B. Bleeding time and platelet aggregometry
C. Bone marrow aspiration
D. PT and aPTT
6. A 35 year-old patient has recurrent seasonal rhinitis and a history of mild asthma. Which of the following should be included for first-line management?
A. Immunotherapy
B. Decongestants
C. Corticosteroid inhalers
D. Cromolyn sodium (Intal)
7. A 35-year-old female presents with multiple ulcerative lesions on her labia and perineum. A Tzanck preparation of one of the lesions reveal...]]>
The Physician Assistant Life | Smarty PANCE full clean 8:05 93
The Audio PANCE and PANRE Board Review Podcast Episode 15 http://podcast.thepalife.com/the-audio-pance-and-panre-board-review-podcast-episode-15/ Sat, 14 Feb 2015 03:26:01 +0000 http://podcast.thepalife.com/?p=83 http://podcast.thepalife.com/the-audio-pance-and-panre-board-review-podcast-episode-15/#respond http://podcast.thepalife.com/the-audio-pance-and-panre-board-review-podcast-episode-15/feed/ 0 <p>Welcome to episode 15 of the FREE Audio PANCE and PANRE Physician Assistant Board Review Podcast. The Audio PANCE and PANRE is an audio board review series that includes 10 Multiple Choice PANCE and PANRE Board Review Questions in each episode. I hope you enjoy this free audio component to the examination portion of this site. The […]</p> <p>The post <a rel="nofollow" href="http://podcast.thepalife.com/the-audio-pance-and-panre-board-review-podcast-episode-15/">The Audio PANCE and PANRE Board Review Podcast Episode 15</a> appeared first on <a rel="nofollow" href="http://podcast.thepalife.com">The Audio PANCE and PANRE</a>.</p> The Audio PANCE and PANRE Episode 15 - The Physician Assistant Life Board Review PodcastWelcome to episode 15 of the FREE Audio PANCE and PANRE Physician Assistant Board Review Podcast.

The Audio PANCE and PANRE is an audio board review series that includes 10 Multiple Choice PANCE and PANRE Board Review Questions in each episode.

I hope you enjoy this free audio component to the examination portion of this site. The full series is available to all members of the PANCE and PANRE Academy.

  • You can download and listen to past FREE episodes here, on iTunes or Stitcher Radio.
  • You can listen to the latest episode, download the transcript and take an interactive quiz of the questions below.

Download a Free PDF Transcript of the Questions and Answers

Listen Carefully Then Take The Quiz

Questions 1-10

The Audio PANCE and PANRE Episode 15

1. A patient with prostate cancer has a nonpalpable, focal lesion, and the patient is reluctant to have surgery at this time. Which of the following would best monitor disease progression?

A. Periodic rectal exams
B. Transrectal ultrasonography
C. Measurements of serum acid phosphatase
D. Measurements of prostate-specific antigen

Click here to see the answer
D. Measurements of prostate-specific antigen – PSA measurement correlates well with volume and stage of disease and is the recommended examination for monitoring disease progression

2. If a woman has a normal 28-day menstrual cycle what tissue and hormonal phase occurs during the last 14 days

A. Proliferative follicular phase under the influence of estrogen.
B. Secretory luteal phase under the influence of estrogen and progesterone.
C. Proliferative follicular phase under the influence of estrogen and progesterone.
D. Secretory luteal phase under the influence of estrogen

Click here to see the answer
B. Secretory luteal phase under the influence of estrogen and progesterone. – The endometrial changes seen in the latter half of the cycle are under the influence of both estrogen and progesterone from the corpus luteum. During this phase, the endometrium becomes more vascularized and slightly edematous.

3. A 24 year-old male presents for routine physical examination. On physical examination, you find that the patient’s upper extremity blood pressure is higher than the blood pressure in the lower extremity. Heart exam reveals a late systolic murmur heard best posteriorly. What is the most likely diagnosis in this patient?

A. Hypertrophic obstructive cardiomyopathy
B. Patent foramen ovale
C. Coarctation of the aorta
D. Patent ductus arteriosus

Click here to see the answer
C. Coarctation of the aorta -Coarctation of the aorta commonly presents with higher systolic pressures in the upper extremities than the lower extremities and absent or weak femoral pulses

4. A mother brings a 3 month-old infant to the office because she is concerned about a red, vascular, nodular growth on the child’s back. It appears to be enlarging slightly and the vessels are slightly dilated. It seems to cause the child no discomfort. The most likely diagnosis is

A. a hemangioma.
B. a pigmented nevus.
C. a salmon patch (stork bite).
D. a malignant melanoma.

Click here to see the answer
A. A hemangioma – A hemangioma is a bright red to deep purple vascular nodule or plaque that often develops at birth, may enlarge, and may regress and disappear with aging.

5. A 45 year-old male with Type 1 diabetes presents with the following lipid panel: Total cholesterol 321 mg/dL; Triglycerides 225 mg/dL; HDL 30 mg/dL; LDL 155 mg/dL. The treatment of choice for this patient is

A. Nicotinic acid (Niacin).
B. Cholestyramine (Questran).
C. Gemfibrozil (Lopid).
D. Simvastatin (Zocor).

Click here to see the answer
D. Simvastatin – As of now, Simvastatin is still considered the drug of choice – as it will decreases triglyceride level, decrease LDL, and increase HDL. (I do foresee changes to this recommendation in the future)

6. A 45 year-old male presents with abdominal pain and one episode of mild hematemesis, which happened days ago. On physical examination, vital signs are stable and he is in no acute distress. Hemoglobin and hematocrit are unremarkable; endoscopy reveals non-bleeding small superficial ulceration of the duodenal bulb. Rapid urease test is positive. Which of the following is the most appropriate treatment at this time?

A. Schedule for a selective vagotomy and antrectomy
B. Start an antacid along with omeprazole (Prilosec)
C. Schedule elective ulcer excision and start sucralfate (Carafate)
D. Start omeprazole (Prilosec) and antibiotic therapy against H. pylori

Click here to see the answer
D. Start omeprazole (Prilosec) and antibiotic therapy against H. pylori – Treatment goals of H. pylori associated ulcers include eradicating the infection with appropriate antibiotics as well as use of a proton pump inhibitor, such as omeprazole, to promote ulcer healing.

7. A 19 year-old female presents with a sore throat for nearly two weeks. She complains of fatigue and a low-grade fever. On physical examination, there is cervical, axillary, and inguinal lymphadenopathy, and mild splenomegaly. On review of the blood smear, which of the following would be expected?

A. Atypical lymphocytes
B. Hypersegmented neutrophils
C. Hypochromic red blood cells
D. Schistocytes

Click here to see the answer
A. Atypical lymphocytes – The hallmark of infectious mononucleosis is the presence of lymphocytosis with atypical large lymphocytes seen in the blood smear. These are larger than normal mature lymphocytes, stain more darkly, and frequently show vacuolated, foamy cytoplasm, and dark chromatin in the nucleus.

8. Which of the following increases the risk of developing testicular cancer?

A. Low socioeconomic status
B. History of cryptorchidism
C. Multiple episodes of epididymitis
D. Being of African-American ethnicity

Click here to see the answer
History of cryptorchidism – The major predisposing risk factor is cryptorchidism unrepaired until after age two.

9. A 17 year-old patient presents to the emergency department with agitation and hallucinations, and has one seizure. He admits to using “some drugs” but does not know what they were. On physical examination, temperature is 103 degrees F, BP 140/90, pulse 120, respirations 20. Remainder of the examination is unremarkable. Which of the following diagnostic studies will be of most help in managing this patient?

A. Drug screen
B. Urine dipstick
C. Complete blood count
D. Serum creatinine kinase

Click here to see the answer
D. Serum creatinine kinase – Serum creatinine kinase is the most sensitive test to detect rhabdomyolysis, a serious complication of seizures and hyperthermia related to drug abuse.
(note) * A. Although a drug screen may identify specific drugs, the results will not alter the care of this patient.
.

10. Which of the following medications is the treatment of choice for patients with chronic gout to prevent recurrence of symptoms during its quiescent phase?

A. Probenecid (Benemid)
B. Allopurinol (Zyloprim)
C. Colchicine
D. Indomethacin (Indocin)

Click here to see the answer
B. Allopurinol – Allopurinol is the best drug to lower serum urate in overproducers, stone formers, and patients with advanced renal failure. It is a xanthine oxidase inhibitor that is used to prevent the formation of uric acid.

Looking for all the episodes?

This FREE series is limited to every other episode, you can download and enjoy the complete audio series by joining The PANCE and PANRE Exam Academy.

I will be be releasing new episodes every two weeks. The Academy is currently discounted, so sign up now.

This Podcast is also available on iTunes and Stitcher Radio for Android

  1. iTunes: The Audio PANCE AND PANRE Podcast iTunes
  2. Stitcher Radio: The Audio PANCE and PANRE Podcast Stitcher

itunes_logo-1

The post The Audio PANCE and PANRE Board Review Podcast Episode 15 appeared first on The Audio PANCE and PANRE.

]]>
Welcome to episode 15 of the FREE Audio PANCE and PANRE Physician Assistant Board Review Podcast. The Audio PANCE and PANRE is an audio board review series that includes 10 Multiple Choice PANCE and PANRE Board Review Questions in each episode. Welcome to episode 15 of the FREE Audio PANCE and PANRE Physician Assistant Board Review Podcast.
The Audio PANCE and PANRE is an audio board review series that includes 10 Multiple Choice PANCE and PANRE Board Review Questions in each episode.
I hope you enjoy this free audio component to the examination portion of this site. The full series is available to all members of the PANCE and PANRE Academy.

* You can download and listen to past FREE episodes here, on iTunes or Stitcher Radio.
* You can listen to the latest episode, download the transcript and take an interactive quiz of the questions below.

Listen Carefully Then Take The Quiz
Questions 1-10
The Audio PANCE and PANRE Episode 15
1. A patient with prostate cancer has a nonpalpable, focal lesion, and the patient is reluctant to have surgery at this time. Which of the following would best monitor disease progression?
A. Periodic rectal exams
B. Transrectal ultrasonography
C. Measurements of serum acid phosphatase
D. Measurements of prostate-specific antigen
2. If a woman has a normal 28-day menstrual cycle what tissue and hormonal phase occurs during the last 14 days
A. Proliferative follicular phase under the influence of estrogen.
B. Secretory luteal phase under the influence of estrogen and progesterone.
C. Proliferative follicular phase under the influence of estrogen and progesterone.
D. Secretory luteal phase under the influence of estrogen
3. A 24 year-old male presents for routine physical examination. On physical examination, you find that the patient’s upper extremity blood pressure is higher than the blood pressure in the lower extremity. Heart exam reveals a late systolic murmur heard best posteriorly. What is the most likely diagnosis in this patient?
A. Hypertrophic obstructive cardiomyopathy
B. Patent foramen ovale
C. Coarctation of the aorta
D. Patent ductus arteriosus
4. A mother brings a 3 month-old infant to the office because she is concerned about a red, vascular, nodular growth on the child’s back. It appears to be enlarging slightly and the vessels are slightly dilated. It seems to cause the child no discomfort. The most likely diagnosis is
A. a hemangioma.
B. a pigmented nevus.
C. a salmon patch (stork bite).
D. a malignant melanoma.
5. A 45 year-old male with Type 1 diabetes presents with the following lipid panel: Total cholesterol 321 mg/dL; Triglycerides 225 mg/dL; HDL 30 mg/dL; LDL 155 mg/dL. The treatment of choice for this patient is
A. Nicotinic acid (Niacin).
B. Cholestyramine (Questran).
C. Gemfibrozil (Lopid).
D. Simvastatin (Zocor).
6. A 45 year-old male presents with abdominal pain and one episode of mild hematemesis, which happened days ago. On physical examination, vital signs are stable and he is in no acute distress. Hemoglobin and hematocrit are unremarkable; endoscopy reveals non-bleeding small superficial ulceration of the duodenal bulb. Rapid urease test is positive. Which of the following is the most appropriate treatment at this time?
A. Schedule for a selective vagotomy and antrectomy
B. Start an antacid along with omeprazole (Prilosec)
C. Schedule elective ulcer excision and start sucralfate (Carafate)
D.]]>
The Physician Assistant Life | Smarty PANCE full clean 9:27 83
The Audio PANCE and PANRE Board Review Podcast Episode 13 http://podcast.thepalife.com/the-audio-pance-and-panre-episode-13/ Fri, 16 Jan 2015 01:37:30 +0000 http://podcast.thepalife.com/?p=67 http://podcast.thepalife.com/the-audio-pance-and-panre-episode-13/#respond http://podcast.thepalife.com/the-audio-pance-and-panre-episode-13/feed/ 0 <p>The Audio PANCE and PANRE is an audio board review series that includes 10 Multiple Choice PANCE and PANRE Board Review Questions in each episode. I hope you enjoy this free audio component to the examination portion of this site. The full series is available to all members of the PANCE and PANRE Academy. You can […]</p> <p>The post <a rel="nofollow" href="http://podcast.thepalife.com/the-audio-pance-and-panre-episode-13/">The Audio PANCE and PANRE Board Review Podcast Episode 13</a> appeared first on <a rel="nofollow" href="http://podcast.thepalife.com">The Audio PANCE and PANRE</a>.</p> The Audio PANCE and PANRE Episode 13 - The Physician Assistant LifeThe Audio PANCE and PANRE is an audio board review series that includes 10 Multiple Choice PANCE and PANRE Board Review Questions in each episode.

I hope you enjoy this free audio component to the examination portion of this site. The full series is available to all members of the PANCE and PANRE Academy.

  • You can download and listen to past FREE episodes here, on iTunes or Stitcher Radio.
  • You can listen to the latest episode, download the transcript and take an interactive quiz of the questions below.

Download a Free PDF Transcript of the Questions and Answers

Listen Carefully Then Take The Quiz

Questions 1-10

The Audio PANCE and PANRE Episode 13

1. During a baseball game, a 22 year-old college student is hit in the right eye by a baseball. He complains of blurry vision in that eye. On physical exam, the physician assistant notes proptosis of the right eye, and limitation of movement in all directions. On CT scan, which of the following is most likely to be seen?

A. Fracture of the medial orbital wall
B. Prolapse of orbital soft tissue
C. Hematoma of the orbit
D. Orbital emphysema

Click here to see the answer
C. Hematoma of the orbit – Orbital hemorrhage into the space surrounding the globe following blunt trauma and rupture of the orbital vessels results in increased ocular pressure, proptosis, visual loss, and limitation of movement in all directions. CT reveals a hematoma.

2. Which of the following physical findings is suggestive of atrial septal defect?

A. Fixed split S2
B. Increased pulse pressure
C. Continuous mechanical murmur
D. Difference in blood pressure between the left and right arm

Click here to see the answer
A. Fixed split S2 – An atrial septal defect will cause a shunt of blood from the left to the right atrium. This will result in an equalization in the amount of blood entering both the left and right ventricles which effectively eliminates the normally wide splitting that inspiration typically causes in hearts without an atrial septal defect.

3. Which of the following is essential to make a diagnosis of cystic fibrosis?

A. Positive family history
B. Elevated sweat chloride
C. Recurrent respiratory infections
D. Elevated trypsinogen levels

Click here to see the answer
B. Elevated sweat chloride – The diagnosis of cystic fibrosis is made only after an elevated sweat chloride test or demonstration of a genotype consistent with cystic fibrosis.

4. In infants, the eyes should move in parallel without deviation by the age of

A. 2 weeks.
B. 3 months.
C. 6 months.
D. 1 year.

Click here to see the answer
C. 6 months – Intermittent alternating convergent strabismus is frequently noted for the first 6 months of life, but referral is indicated if it persists beyond 6 months.

5. Which of the following physical exam findings is consistent with moderate emphysema?

A. Increased tactile fremitus
B. Dullness to percussion
C. Distant heart sounds
D. Deviated trachea

Click here to see the answer
C. Distant heart sounds – Distant heart sounds are common in emphysema patients due to hyperinflation of the lungs.

6. Which of the following is the most common indication for operative intervention in patients with chronic pancreatitis?

A. Weight loss
B. Intractable pain
C. Exocrine deficiency
D. To decrease risk of cancer

Click here to see the answer
B. Intractable pain – Indications for surgical treatment of chronic pancreatitis include severe pain that limits the patient’s functioning or intractable pain despite the use of non-narcotic analgesics and absence of alcohol intake.

7. A 22 year old male presents to the ED with pain that radiates to his shoulders and is relieved with sitting forward. The patient admits to recent upper respiratory symptoms. On examination vital signs are BP 126/68, HR 86, RR 20, temp 100.3 degrees F. There is no JVD noted. Heart exam reveals regular rate and rhythm with no S3 or S4. There is a friction rub noted. Lungs are clear to auscultation. EKG shows diffuse ST segment elevation. What is the treatment of choice in this patient?

A. Pericardiocentesis
B. Nitroglycerin
C. Percutaneous coronary intervention
D. Indomethacin (Indocin)

Click here to see the answer
D. Indomethacin (Indocin) – Indomethacin, a nonsteroidal anti-inflammatory medication, is the treatment of choice in a patient with acute pericarditis.

8. As a general rule, sutures in the face should be removed in

A. 3 days.
B. 5 days.
C. 7 days.
D. 10 days.

Click here to see the answer
B. 5 days – Sutures of the face should be removed in 5 days in order to allow for adequate healing and to limit the amount of scarring.

9. Patient education for a 23 year-old using oral contraceptives should include which of the following?

A. Rifampin may decrease the effectiveness of the oral contraceptives.
B. Acetaminophen may decrease the effectiveness of the oral contraceptives.
C. Oral contraceptives may provide some protection from coronary artery disease.
D. Changing to the “minipill” (progestin only) will inhibit ovulation more consistently than combination oral contraceptives.

Click here to see the answer
A. Rifampin may interfere with the efficacy of the oral contraceptives.

10. When performing a rectal examination, prostatic massage is contraindicated in

A. acute bacterial prostatitis.
B. chronic bacterial prostatitis.
C. nonbacterial prostatitis.
D. prostatodynia.

Click here to see the answer
A. Acute bacterial prostatitis – Vigorous manipulation of the prostate during rectal examination may result in septicemia. This is contraindicated in the presence of fever, irritative voiding symptoms, and perineal/sacral pain.

This Podcast is also available on iTunes and Stitcher Radio for Android

  1. iTunes: The Audio PANCE AND PANRE Podcast iTunes
  2. Stitcher Radio: The Audio PANCE and PANRE Podcast Stitcher

itunes_logo-1

The post The Audio PANCE and PANRE Board Review Podcast Episode 13 appeared first on The Audio PANCE and PANRE.

]]>
The Audio PANCE and PANRE is an audio board review series that includes 10 Multiple Choice PANCE and PANRE Board Review Questions in each episode. I hope you enjoy this free audio component to the examination portion of this site. The Audio PANCE and PANRE is an audio board review series that includes 10 Multiple Choice PANCE and PANRE Board Review Questions in each episode.
I hope you enjoy this free audio component to the examination portion of this site. The full series is available to all members of the PANCE and PANRE Academy.

* You can download and listen to past FREE episodes here, on iTunes or Stitcher Radio.
* You can listen to the latest episode, download the transcript and take an interactive quiz of the questions below.

Listen Carefully Then Take The Quiz
Questions 1-10
The Audio PANCE and PANRE Episode 13
1. During a baseball game, a 22 year-old college student is hit in the right eye by a baseball. He complains of blurry vision in that eye. On physical exam, the physician assistant notes proptosis of the right eye, and limitation of movement in all directions. On CT scan, which of the following is most likely to be seen?
A. Fracture of the medial orbital wall
B. Prolapse of orbital soft tissue
C. Hematoma of the orbit
D. Orbital emphysema
2. Which of the following physical findings is suggestive of atrial septal defect?
A. Fixed split S2
B. Increased pulse pressure
C. Continuous mechanical murmur
D. Difference in blood pressure between the left and right arm
3. Which of the following is essential to make a diagnosis of cystic fibrosis?
A. Positive family history
B. Elevated sweat chloride
C. Recurrent respiratory infections
D. Elevated trypsinogen levels
4. In infants, the eyes should move in parallel without deviation by the age of
A. 2 weeks.
B. 3 months.
C. 6 months.
D. 1 year.
5. Which of the following physical exam findings is consistent with moderate emphysema?
A. Increased tactile fremitus
B. Dullness to percussion
C. Distant heart sounds
D. Deviated trachea
6. Which of the following is the most common indication for operative intervention in patients with chronic pancreatitis?
A. Weight loss
B. Intractable pain
C. Exocrine deficiency
D. To decrease risk of cancer
7. A 22 year old male presents to the ED with pain that radiates to his shoulders and is relieved with sitting forward. The patient admits to recent upper respiratory symptoms. On examination vital signs are BP 126/68, HR 86, RR 20, temp 100.3 degrees F. There is no JVD noted. Heart exam reveals regular rate and rhythm with no S3 or S4. There is a friction rub noted. Lungs are clear to auscultation. EKG shows diffuse ST segment elevation. What is the treatment of choice in this patient?
A. Pericardiocentesis
B. Nitroglycerin
C. Percutaneous coronary intervention
D. Indomethacin (Indocin)
8. As a general rule, sutures in the face should be removed in
A. 3 days.
B. 5 days.
C. 7 days.
D. 10 days.
9. Patient education for a 23 year-old using oral contraceptives should include which of the following?
A. Rifampin may decrease the effectiveness of the oral contraceptives.
B. Acetaminophen may decrease the effectiveness of the oral contraceptives.
C. Oral contraceptives may provide some protection from coronary artery disease.
D. Changing to the “minipill” (progestin only) will inhibit ovulation more consistently than combination oral co...]]>
The Physician Assistant Life | Smarty PANCE full clean 7:42 67
The Audio PANCE and PANRE Board Review Podcast Episode 11 http://podcast.thepalife.com/the-audio-pance-and-panre-episode-11/ Thu, 01 Jan 2015 01:24:08 +0000 http://podcast.thepalife.com/?p=64 http://podcast.thepalife.com/the-audio-pance-and-panre-episode-11/#respond http://podcast.thepalife.com/the-audio-pance-and-panre-episode-11/feed/ 0 <p>The Audio PANCE and PANRE is an audio board review series that includes 10 Multiple Choice PANCE and PANRE Board Review Questions in each episode. I hope you enjoy this free audio component to the examination portion of this site. The full series is available to all members of the PANCE and PANRE Academy. You can […]</p> <p>The post <a rel="nofollow" href="http://podcast.thepalife.com/the-audio-pance-and-panre-episode-11/">The Audio PANCE and PANRE Board Review Podcast Episode 11</a> appeared first on <a rel="nofollow" href="http://podcast.thepalife.com">The Audio PANCE and PANRE</a>.</p> The Audio PANCE and PANRE is an audio board review series that includes 10 Multiple Choice PANCE and PANRE Board Review Questions in each episode.

I hope you enjoy this free audio component to the examination portion of this site. The full series is available to all members of the PANCE and PANRE Academy.

  • You can download and listen to past FREE episodes here, on iTunes or Stitcher. (sometimes it takes a few hours to show up on iTunes after publication)
  • You can listen to the latest episode, download the transcript and take an interactive quiz of the questions below.

Download a Free PDF Transcript of the Questions and Answers

Listen Carefully Then Take The Quiz

Questions 1-10

The Audio PANCE and PANRE Episode 11

1. What is the initial treatment of choice for hyperthyroidism in a 10-week pregnant patient?

A. No treatment is necessary.
B. Propylthiouracil (PTU)
C. Radioiodine treatment
D. Subtotal thyroidectomy

Click here to see the answer
B. Propylthiouracil (PTU) – This is the initial treatment of choice.

2. A patient presents with eye pain and blurred vision. Snellen testing reveals vision of 20/200 in the affected eye and 20/20 in the unaffected eye. Fluorescein staining reveals the presence of a dendritic ulcer. Which of the following is the most likely diagnosis?

A. Viral keratitis
B. Fungal corneal ulcer
C. Acanthamoeba keratitis
D. Bacterial corneal ulcer

Click here to see the answer
A. Viral keratitis – Herpes Simplex virus is a common cause of dendritic ulceration noted on fluorescein staining.

And in case you were wondering:

B. Fungal corneal ulcers have an indolent course with intraocular infection being common but fluorescein staining is negative for a dendritic pattern.

C. Acanthamoeba keratitis has a waxing and waning course over several months and has no fluorescein staining in a dendritic pattern.

D. Bacterial corneal ulcers can progress aggressively resulting in corneal perforation. Fluorescein staining does not occur in a dendritic pattern.

3. A 63 year-old female with history of diabetes mellitus presents for blood pressure follow-up. At her last two visits her blood pressure was 150/92 and 152/96. Today in the office her blood pressure is 146/92. Recent blood work shows a Sodium 140 mEq/L, Potassium 4.2 mEq/L, BUN of 23 mg/dL, and Creatinine of 1.1 mg/dL. Which of the following is the most appropriate initial medication in this patient?

A. Terazosin (Hytrin)
B. Atenolol (Tenormin)
C. Lisinopril (Zestril)
D. Hydrochlorothiazide (HCTZ)

Click here to see the answer
C. Lisinopril (Zestril) – For the boards ACE inhibitors should be part of the initial treatment of hypertension in diabetics because of beneficial effects in diabetic nephropathy and is the most appropriate initial medication. In real practice, despite what we have been led to believe, it actually makes little difference which antihypertensive is used in diabetic patients just as long as we reach our blood pressure goals.

4. What is the EKG manifestation of cardiac end-organ damage due to hypertension?

A. Right bundle branch block
B. Left ventricular hypertrophy
C. Right ventricular hypertrophy
D. ST segment elevation in lateral precordial leads

Click here to see the answer
B. Left ventricular hypertrophy – Long-standing hypertension can lead to left ventricular hypertrophy with characteristic changes noted on EKG.

5. Annual blood pressure determinations should be obtained beginning at the age of

A. 3 years.
B. 5 years.
C. 12 years.
D. 18 years.

Click here to see the answer
A. 3 years – Periodic measurements of blood pressure should be part of routine preventive health assessments beginning at the age of 3 years.

6. In adults and intravenous drug abusers, which of the following bones is most commonly affected with acute osteomyelitis?

A. Femur
B. Humerus
C. Vertebral spine
D. Tibia

Click here to see the answer
C. Vertebral spine – The bones of the vertebral spine are most commonly affected in a patient with osteomyelitis. Organisms reach the well-perfused vertebral body of adults via spinal arteries and quickly spread from the end plate into the disk space and then to the adjacent vertebral body. The infection may originate in the urinary tract and intravenous drug use carries an increased risk of spinal infection.

7. Treatment of the patient with Pediculosis pubis consists of which of the following?

A. Permethrin (Nix) cream
B. Clotrimazole (Gyne-Lotrimin)
C. Podofilox (Condylox) solution
D. Selenium sulfide (Selsun) suspension

Click here to see the answer
A. Permethrin 1% cream/shampoo is used to kill the louse and remove the eggs from the hair shafts.

8. A 43 year-old asymptomatic diabetic female is found to have an elevated total calcium level of 12.4 mg/dL. Which of the following tests must be assessed in order to evaluate this laboratory abnormality?

A. Intact parathyroid hormone
B. Serum albumin
C. 24 hour urine calcium level
D. Complete blood count

Click here to see the answer
B. Serum albumin – Since approximately 50% of calcium is protein bound, total calcium levels should be interpreted relative to albumin levels.

9. Which of the following conditions would cause a positive Kussmaul’s sign on physical examination?

A. Left ventricular failure
B. Pulmonary edema
C. Coarctation of the aorta
D. Constrictive pericarditis

Click here to see the answer
D. Constrictive pericarditis – Kussmaul’s sign is an increase rather than the normal decrease in the CVP during inspiration. It is most often caused by severe right-sided heart failure; it is a frequent finding in patients with constrictive pericarditis or right ventricular infarction.

10. Treatment of Bell’s palsy includes which of the following?

A. Acyclovir
B. Reassurance of the patient’s recovery
C. Referral to a neurosurgeon
D. Electromyography

Click here to see the answer
B. Reassurance of the patient’s recovery – Although I usually always treat with Acyclovir, Bell’s palsy is a peripheral neuropathy of cranial nerve VII. Although it has been suggested it may be related to an activation of herpes simplex virus, there is little empiric evidence for this. Approximately 60% of cases of Bell’s palsy recover without treatment and patient reassurance of this is advised. Electromyography may provide aid in the prognosis, but not as a treatment option. A neurosurgeon has no role in the management of Bell’s palsy.

Looking for all the episodes?

This FREE series is limited to every 3rd episode, you can download and enjoy the complete audio series by joining The PANCE and PANRE Exam Academy.

I will be be releasing new episodes every two weeks. The Academy is currently discounted, so sign up now.

This Podcast is also available on iTunes and Stitcher Radio for Android

  1. iTunes: The Audio PANCE AND PANRE Podcast iTunes
  2. Stitcher Radio: The Audio PANCE and PANRE Podcast Stitcher

itunes_logo-1

The post The Audio PANCE and PANRE Board Review Podcast Episode 11 appeared first on The Audio PANCE and PANRE.

]]>
The Audio PANCE and PANRE is an audio board review series that includes 10 Multiple Choice PANCE and PANRE Board Review Questions in each episode. I hope you enjoy this free audio component to the examination portion of this site. The Audio PANCE and PANRE is an audio board review series that includes 10 Multiple Choice PANCE and PANRE Board Review Questions in each episode.
I hope you enjoy this free audio component to the examination portion of this site. The full series is available to all members of the PANCE and PANRE Academy.

* You can download and listen to past FREE episodes here, on iTunes or Stitcher. (sometimes it takes a few hours to show up on iTunes after publication)
* You can listen to the latest episode, download the transcript and take an interactive quiz of the questions below.

Listen Carefully Then Take The Quiz
Questions 1-10
The Audio PANCE and PANRE Episode 11
1. What is the initial treatment of choice for hyperthyroidism in a 10-week pregnant patient?
A. No treatment is necessary.
B. Propylthiouracil (PTU)
C. Radioiodine treatment
D. Subtotal thyroidectomy
2. A patient presents with eye pain and blurred vision. Snellen testing reveals vision of 20/200 in the affected eye and 20/20 in the unaffected eye. Fluorescein staining reveals the presence of a dendritic ulcer. Which of the following is the most likely diagnosis?
A. Viral keratitis
B. Fungal corneal ulcer
C. Acanthamoeba keratitis
D. Bacterial corneal ulcer
And in case you were wondering:
B. Fungal corneal ulcers have an indolent course with intraocular infection being common but fluorescein staining is negative for a dendritic pattern.
C. Acanthamoeba keratitis has a waxing and waning course over several months and has no fluorescein staining in a dendritic pattern.
D. Bacterial corneal ulcers can progress aggressively resulting in corneal perforation. Fluorescein staining does not occur in a dendritic pattern.
3. A 63 year-old female with history of diabetes mellitus presents for blood pressure follow-up. At her last two visits her blood pressure was 150/92 and 152/96. Today in the office her blood pressure is 146/92. Recent blood work shows a Sodium 140 mEq/L, Potassium 4.2 mEq/L, BUN of 23 mg/dL, and Creatinine of 1.1 mg/dL. Which of the following is the most appropriate initial medication in this patient?
A. Terazosin (Hytrin)
B. Atenolol (Tenormin)
C. Lisinopril (Zestril)
D. Hydrochlorothiazide (HCTZ)
4. What is the EKG manifestation of cardiac end-organ damage due to hypertension?
A. Right bundle branch block
B. Left ventricular hypertrophy
C. Right ventricular hypertrophy
D. ST segment elevation in lateral precordial leads
5. Annual blood pressure determinations should be obtained beginning at the age of
A. 3 years.
B. 5 years.
C. 12 years.
D. 18 years.
6. In adults and intravenous drug abusers, which of the following bones is most commonly affected with acute osteomyelitis?
A. Femur
B. Humerus
C. Vertebral spine
D. Tibia
7. Treatment of the patient with Pediculosis pubis consists of which of the following?
A. Permethrin (Nix) cream
B. Clotrimazole (Gyne-Lotrimin)
C. Podofilox (Condylox) solution
D. Selenium sulfide (Selsun) suspension
8. A 43 year-old asymptomatic diabetic female is found to have an elevated total calcium level of 12.4 mg/dL. Which of the following tests must be assessed in order to evaluate this laboratory abnormality?
A. Intact parathyroid hormone
B.]]>
The Physician Assistant Life | Smarty PANCE full clean 8:25 64
The Audio PANCE and PANRE Board Review Podcast Episode 9 http://podcast.thepalife.com/the-audio-pance-and-panre-episode-9/ Thu, 18 Dec 2014 23:29:20 +0000 http://podcast.thepalife.com/?p=60 http://podcast.thepalife.com/the-audio-pance-and-panre-episode-9/#respond http://podcast.thepalife.com/the-audio-pance-and-panre-episode-9/feed/ 0 <p>The Audio PANCE and PANRE is an audio board review series that includes 10 Multiple Choice PANCE and PANRE Board Review Questions in each episode. I created this audio series to help you fill some downtime when you are unable to read (like in the car) with some useful board review. I hope you enjoy this free audio component […]</p> <p>The post <a rel="nofollow" href="http://podcast.thepalife.com/the-audio-pance-and-panre-episode-9/">The Audio PANCE and PANRE Board Review Podcast Episode 9</a> appeared first on <a rel="nofollow" href="http://podcast.thepalife.com">The Audio PANCE and PANRE</a>.</p> The Audio PANCE and PANRE is an audio board review series that includes 10 Multiple Choice PANCE and PANRE Board Review Questions in each episode.

I created this audio series to help you fill some downtime when you are unable to read (like in the car) with some useful board review.

I hope you enjoy this free audio component to the examination portion of this site. The full series is available to all members of the PANCE and PANRE Academy.

  • You can download and listen to past FREE episodes here or on iTunes.
  • You can listen to the latest episode and take an interactive quiz of the questions below.

Download a Free PDF Transcript of the Questions and Answers

Listen Carefully Then Take The Quiz

Questions 1-10

The Audio PANCE and PANRE Episode 9

1. A 3 year-old girl is diagnosed with atopic dermatitis. Which of the following disorders is this child at risk for in the future?

A. Asthma
B. Tinea pedis
C. Squamous carcinoma
D. Systemic lupus erythematosus (SLE)

Click here to see the answer
A. Up to 50% of patients with atopic dermatitis develop asthma and/or allergic rhinitis in the future.

2. A 3 year-old boy is seen in the office with a 5-day history of fever, erythema, edema of the hands and feet, a generalized rash over the body, bilateral conjunctival injections, fissuring and erythema of the lips, and cervical adenopathy. Antistreptolysin A (ASO) titer and throat culture are negative. The most serious systemic complication associated with this disorder is

A. renal.
B. cardiac.
C. pulmonary.
D. hepatic.

Click here to see the answer
B. The patient most likely has Kawasaki syndrome. The major complication with this disorder is coronary artery aneurysms, which are reported in up to 20% of affected children. The etiology of this disorder is uncertain, although a bacterial toxin with super antigen properties may be involved.

3. Which of the following medication classes is the treatment of choice in a patient with variant or Prinzmetal’s angina?

A. Calcium channel blockers
B. ACE inhibitors
C. Beta blockers
D. Angiotensin II receptor blockers

Click here to see the answer
A. Calcium channel blockers are effective prophylactically to treat coronary vasospasm associated with variant or Prinzmetal’s angina.

4. Pharmacologic treatment of a patient with gestational diabetes should consist of which of the following?

A. Oral hypoglycemic agents
B. Regular insulin
C. Oral corticosteroids
D. Glucagon

Click here to see the answer
B. Regular insulin is the drug of choice as this will maintain the mother’s blood sugar but not cross the placenta.

5. Which of the following is an indication for vaccination against hepatitis A?

A. Illicit drug users
B. Health care workers
C. Renal dialysis patients
D. Routine vaccination starting at birth

Click here to see the answer
A. Hepatitis A vaccine is recommended for illicit drug users, anyone living or traveling to endemic areas, sewage workers, food handlers, homosexual and bisexual men, animal handlers, patients with a history of chronic liver disease or a clotting factor disease as well as children and workers in day care settings and institutions.

6. A 2 month-old infant presents for a routine health maintenance visit. The mother has been concerned about the infant’s hearing since birth. Physical examination reveals no apparent response to a sudden loud sound. Which of the following is the most appropriate diagnostic evaluation?

A. audiometry
B. tympanometry
C. acoustic reflectometry
D. auditory-evoked potentials

Click here to see the answer
D. Brainstem auditory-evoked potentials evaluate the sensory pathway and identify the site of any anatomical disruption. The test does not require any active response from the patient and is useful in the evaluation of suspected hearing loss in an infant.

7. A 53 year-old female status post abdominal hysterectomy 3 days ago suddenly develops pleuritic chest pain and dyspnea. On exam, she is tachycardic and tachypneic with rales in the left lower lobe. A chest x-ray is unremarkable and an EKG reveals sinus tachycardia. Which of the following is the most likely diagnosis?

A. Atelectasis
B. Pneumothorax
C. Pulmonary embolism
D. Myocardial infarction

Click here to see the answer
C. This patient’s risk factors for pulmonary embolism include advanced age, surgery, and prolonged bedrest. While the diagnosis of pulmonary embolism is difficult to make due to nonspecific clinical findings, the most common symptoms include pleuritic chest pain and dyspnea associated with tachypnea. Chest x-ray and EKG are usually normal.

8. Which of the following is an independent risk factor for development of a mesothelioma?

A. Cigarette smoking
B. Asbestos exposure
C. Radon gas exposure
D. Chronic obstructive lung disease

Click here to see the answer
B. Studies confirm the association of asbestos exposure to the development of mesothelioma.

9. A 34 year-old female with a history of asthma presents with complaints of increasing asthma attacks. The patient states she has been well-controlled on albuterol inhaler until one month ago. Since that time she notices that she has had to use her inhaler 3-4 times a week and also has had increasing nighttime use averaging about three episodes in the past month. Spirometry reveals greater than 85% predicted value. Which of the following is the most appropriate intervention at this time?

A. Oral prednisone
B. Oral theophylline (Theo-Dur)
C. Salmeterol (Serevent) inhaler
D. Beclomethasone (Qvar)inhaler

Click here to see the answer
D. This patient has progressed to mild persistent asthma. In addition to her inhaled beta2-agonist (albuterol), she should be started on an anti-inflammatory agent. Inhaled corticosteroids, such as beclomethasone, are preferred for long-term control.

10. Which of the following mechanisms leads to a primary pneumothorax?

A. Penetrating or blunt trauma forces
B. Underlying lung cancer
C. Pressure of air in the pleural space exceeds room air pressure
D. Rupture of subpleural apical blebs due to high negative intrapleural pressures

Click here to see the answer
D. A primary spontaneous pneumothorax is thought to result from a rupture of subpleural apical blebs secondary to high negative intrapleural pressures.

Looking for all the episodes?

This FREE series is limited to every 3rd episode, you can download and enjoy the complete audio series by joining The PANCE and PANRE Exam Academy.

I will be be releasing new episodes every two weeks. The Academy is currently discounted, so sign up now.

This Podcast is also available on iTunes and Stitcher Radio for Android

  1. iTunes: The Audio PANCE AND PANRE Podcast iTunes
  2. Stitcher Radio: The Audio PANCE and PANRE Podcast Stitcher

itunes_logo-1

The post The Audio PANCE and PANRE Board Review Podcast Episode 9 appeared first on The Audio PANCE and PANRE.

]]>
The Audio PANCE and PANRE is an audio board review series that includes 10 Multiple Choice PANCE and PANRE Board Review Questions in each episode. I created this audio series to help you fill some downtime when you are unable to read (like in the car) ... The Audio PANCE and PANRE is an audio board review series that includes 10 Multiple Choice PANCE and PANRE Board Review Questions in each episode.
I created this audio series to help you fill some downtime when you are unable to read (like in the car) with some useful board review.
I hope you enjoy this free audio component to the examination portion of this site. The full series is available to all members of the PANCE and PANRE Academy.

* You can download and listen to past FREE episodes here or on iTunes.
* You can listen to the latest episode and take an interactive quiz of the questions below.

Listen Carefully Then Take The Quiz
Questions 1-10
The Audio PANCE and PANRE Episode 9
1. A 3 year-old girl is diagnosed with atopic dermatitis. Which of the following disorders is this child at risk for in the future?
A. Asthma
B. Tinea pedis
C. Squamous carcinoma
D. Systemic lupus erythematosus (SLE)
2. A 3 year-old boy is seen in the office with a 5-day history of fever, erythema, edema of the hands and feet, a generalized rash over the body, bilateral conjunctival injections, fissuring and erythema of the lips, and cervical adenopathy. Antistreptolysin A (ASO) titer and throat culture are negative. The most serious systemic complication associated with this disorder is
A. renal.
B. cardiac.
C. pulmonary.
D. hepatic.
3. Which of the following medication classes is the treatment of choice in a patient with variant or Prinzmetal’s angina?
A. Calcium channel blockers
B. ACE inhibitors
C. Beta blockers
D. Angiotensin II receptor blockers
4. Pharmacologic treatment of a patient with gestational diabetes should consist of which of the following?
A. Oral hypoglycemic agents
B. Regular insulin
C. Oral corticosteroids
D. Glucagon
5. Which of the following is an indication for vaccination against hepatitis A?
A. Illicit drug users
B. Health care workers
C. Renal dialysis patients
D. Routine vaccination starting at birth
6. A 2 month-old infant presents for a routine health maintenance visit. The mother has been concerned about the infant’s hearing since birth. Physical examination reveals no apparent response to a sudden loud sound. Which of the following is the most appropriate diagnostic evaluation?
A. audiometry
B. tympanometry
C. acoustic reflectometry
D. auditory-evoked potentials
7. A 53 year-old female status post abdominal hysterectomy 3 days ago suddenly develops pleuritic chest pain and dyspnea. On exam, she is tachycardic and tachypneic with rales in the left lower lobe. A chest x-ray is unremarkable and an EKG reveals sinus tachycardia. Which of the following is the most likely diagnosis?
A. Atelectasis
B. Pneumothorax
C. Pulmonary embolism
D. Myocardial infarction
8. Which of the following is an independent risk factor for development of a mesothelioma?
A. Cigarette smoking
B. Asbestos exposure
C. Radon gas exposure
D. Chronic obstructive lung disease
9. A 34 year-old female with a history of asthma presents with complaints of increasing asthma attacks. The patient states she has been well-controlled on albuterol inhaler until one month ago. Since that time she notices that she has had to use her inhaler 3-4 times a week and also has had increasing nighttime use averaging about three episodes in the past month.]]>
The Physician Assistant Life | Smarty PANCE full clean 7:30 60
The Audio PANCE and PANRE Board Review Podcast Episode 7 http://podcast.thepalife.com/the-audio-pance-and-panre-episode-7/ Sat, 08 Nov 2014 20:32:15 +0000 http://podcast.thepalife.com/?p=54 http://podcast.thepalife.com/the-audio-pance-and-panre-episode-7/#respond http://podcast.thepalife.com/the-audio-pance-and-panre-episode-7/feed/ 0 <p>The Audio PANCE and PANRE is an audio board review series that includes 10 Multiple Choice PANCE and PANRE Board Review Questions in each episode. I created this audio series to help you fill some downtime when you are unable to read (like in the car) with some useful board review. I hope you enjoy this free audio component […]</p> <p>The post <a rel="nofollow" href="http://podcast.thepalife.com/the-audio-pance-and-panre-episode-7/">The Audio PANCE and PANRE Board Review Podcast Episode 7</a> appeared first on <a rel="nofollow" href="http://podcast.thepalife.com">The Audio PANCE and PANRE</a>.</p> THE-AUDIO-PANCE-AND-PANRE 300x300The Audio PANCE and PANRE is an audio board review series that includes 10 Multiple Choice PANCE and PANRE Board Review Questions in each episode.

I created this audio series to help you fill some downtime when you are unable to read (like in the car) with some useful board review.

I hope you enjoy this free audio component to the examination portion of this site. The full series is available to all members of the PANCE and PANRE Academy.

Questions 1-10

1. Which of the following is a staphylococcal infection characterized by a localized red swollen and acutely tender abscess of the upper or lower eyelid?

A. Hordeolum
B. Uveitis
C. Chalazion
D. Dacryocystitis

Click here to see the answer
A. Hordeolum (stye) is a staphylococcal infection characterized by a localized red swollen and acutely tender abscess of the upper or lower eyelid..

2. Progesterone influence on the breast tissue prior to menstruation causes

A. proliferation of the mammary ducts.
B. growth of the lobules and alveoli.
C. proliferation of Cooper’s ligaments.
D. increase in the number of glands of Montgomery.

Click here to see the answer
B. Growth of the lobules and alveoli is under the influence of progesterone. Prior to menses, the breast swelling that women notice is a result of the progesterone which is secreted from the corpus luteum. During menses, the swelling subsides.

3. A 3 year-old girl is diagnosed with atopic dermatitis. Which of the following disorders is this child at risk for in the future?

A. Asthma
B. Tinea pedis
C. Squamous carcinoma
D. Systemic lupus erythematosus (SLE)

Click here to see the answer
A. Up to 50% of patients with atopic dermatitis develop asthma and/or allergic rhinitis in the future.

4. A patient presents with chronic back pain. On physical examination testing, the patient is found to have abnormalities of proprioception and vibration discrimination. Which of the following portions of the spinal column are most likely affected?

A. Lateral spinothalamic tract
B. Ventral spinothalamic tract
C. Posterior column
D. Transection of the cord

Click here to see the answer
C. The posterior column affects proprioception (position sense) and vibration sense.

5. A 44 year-old female presents for follow-up results of her lipid profile. She is asymptomatic and has a past medicalhistory of hypothyroidism treated with levothyroxine and hypertension controlled with atenolol (Tenormin). She drinks an average of 6 alcoholic beverages a day and smokes 1 pack per day for the last 32 years. Her family history is unremarkable for premature coronary artery disease. Her fasting blood glucose is 98 mg/dL, total cholesterol is 198 mg/dL, LDL cholesterol is 132 mg/dL, HDL cholesterol is 56 mg/dL and triglycerides of 90 mg/dL. Excluding LDL cholesterol levels, how many major risk factors for coronary artery disease does this female possess?

A. 2
B. 3
C. 4
D. 5

Click here to see the answer
A. This patient’s major cardiac risk factors are smoking and hypertension in addition to the elevated LDL
cholesterol.

6. A 65 year-old patient with steroid-dependent chronic obstructive lung disease presents with a headache that has been increasing in severity over the past week, accompanied by nausea and vomiting. He denies fever, but has had photophobia and a stiff neck. Which of the following is the most likely diagnosis?

A. Transient ischemic attack
B. Bacterial meningitis
C. Migraine headache
D. Cryptococcosis

Click here to see the answer
D. Cryptococcus is an opportunistic fungal infection that affects immunocompromised patients, including those with HIV, chronic steroid use, organ transplants, diabetes mellitus, and chronic renal or liver disease. The most common clinical presentation is that of meningitis; fever is present in only about half of patients.

7. A 40 year-old female presents with a 1.5 cm well-circumscribed mass noted on mammography in the right upper, outer quadrant. Which of the following procedures is most appropriate and should be done next?

A. Fine needle aspiration
B. BRCA 1 and BRCA 2 genetic testing
C. Serum CA-125
D. Radiation therapy

Click here to see the answer
A. Fine needle aspiration is fairly accurate, easily performed, and has minimal morbidity.

8. On examination of a pregnant patient the physician assistant notes the fundal height is at the level of the umbilicus. This corresponds to what gestational age?

A. 16 weeks
B. 20 weeks
C. 24 weeks
D. 28 weeks

Click here to see the answer
B. At 20-22 weeks, the fundal height is typically at the level of the umbilicus.

9. Which of the following is a common symptom associated with laryngotracheobronchitis (viral croup)?

A. Drooling
B. High fever
C. “Hot potato” voice
D. Barking cough

Click here to see the answer
D. Viral croup is characterized by a history of upper respiratory tract symptoms followed by onset of a barking cough and stridor..

10. A 23 year-old female with history of asthma for the past 5 years presents with complaints of increasing shortness ofbreath for 2 days. Her asthma has been well-controlled until 2 days ago. Since yesterday, she has been using heralbuterol inhaler every 4 to 6 hours. She is normally very active, however yesterday she did not complete her 30minute exercise routine due to increasing dyspnea. She denies any cough, fever, recent surgeries, or use of oralcontraceptives. On examination, you note the presence of prolonged expiration and diffuse wheezing. The remainder of the exam is unremarkable. Which of the following is the most appropriate initial diagnostic evaluation prior to initiation of treatment?

A. Chest x-ray
B. Sputum gram stain
C. Peak flow
D. Ventilation-perfusion scan

Click here to see the answer
C. A peak flow reading will help you to gauge her current extent of airflow obstruction and is helpful in monitoring the effectiveness of any treatment interventions.

Looking for all the episodes?

This FREE series is limited to every 3rd episode, you can download and enjoy the complete audio series by joining The PANCE and PANRE Exam Academy.

I will be be releasing new episodes every two weeks. The Academy is currently discounted at $35 for the entire year, so sign up now.

This Podcast is also available on iTunes and Stitcher Radio for Android

  1. iTunes: The Audio PANCE AND PANRE Podcast iTunes
  2. Stitcher Radio: The Audio PANCE and PANRE Podcast Stitcher

itunes_logo-1


And, as always, if you haven’t already, make sure to download Brian Wallaces’ excellent Physician Assistant Exam Review Podcast. Follow along with Brian who covers new topics twice monthly and really does an amazing job! He is running a contest on his site to win some free goodies! You can sign up to win free stuff here!

The post The Audio PANCE and PANRE Board Review Podcast Episode 7 appeared first on The Audio PANCE and PANRE.

]]>
The Audio PANCE and PANRE is an audio board review series that includes 10 Multiple Choice PANCE and PANRE Board Review Questions in each episode. I created this audio series to help you fill some downtime when you are unable to read (like in the car) ... I created this audio series to help you fill some downtime when you are unable to read (like in the car) with some useful board review.
I hope you enjoy this free audio component to the examination portion of this site. The full series is available to all members of the PANCE and PANRE Academy.
Questions 1-10
1. Which of the following is a staphylococcal infection characterized by a localized red swollen and acutely tender abscess of the upper or lower eyelid?
A. Hordeolum
B. Uveitis
C. Chalazion
D. Dacryocystitis
2. Progesterone influence on the breast tissue prior to menstruation causes
A. proliferation of the mammary ducts.
B. growth of the lobules and alveoli.
C. proliferation of Cooper’s ligaments.
D. increase in the number of glands of Montgomery.
3. A 3 year-old girl is diagnosed with atopic dermatitis. Which of the following disorders is this child at risk for in the future?
A. Asthma
B. Tinea pedis
C. Squamous carcinoma
D. Systemic lupus erythematosus (SLE)
4. A patient presents with chronic back pain. On physical examination testing, the patient is found to have abnormalities of proprioception and vibration discrimination. Which of the following portions of the spinal column are most likely affected?
A. Lateral spinothalamic tract
B. Ventral spinothalamic tract
C. Posterior column
D. Transection of the cord
5. A 44 year-old female presents for follow-up results of her lipid profile. She is asymptomatic and has a past medicalhistory of hypothyroidism treated with levothyroxine and hypertension controlled with atenolol (Tenormin). She drinks an average of 6 alcoholic beverages a day and smokes 1 pack per day for the last 32 years. Her family history is unremarkable for premature coronary artery disease. Her fasting blood glucose is 98 mg/dL, total cholesterol is 198 mg/dL, LDL cholesterol is 132 mg/dL, HDL cholesterol is 56 mg/dL and triglycerides of 90 mg/dL. Excluding LDL cholesterol levels, how many major risk factors for coronary artery disease does this female possess?
A. 2
B. 3
C. 4
D. 5
6. A 65 year-old patient with steroid-dependent chronic obstructive lung disease presents with a headache that has been increasing in severity over the past week, accompanied by nausea and vomiting. He denies fever, but has had photophobia and a stiff neck. Which of the following is the most likely diagnosis?
A. Transient ischemic attack
B. Bacterial meningitis
C. Migraine headache
D. Cryptococcosis
7. A 40 year-old female presents with a 1.5 cm well-circumscribed mass noted on mammography in the right upper, outer quadrant. Which of the following procedures is most appropriate and should be done next?
A. Fine needle aspiration
B. BRCA 1 and BRCA 2 genetic testing
C. Serum CA-125
D. Radiation therapy
8. On examination of a pregnant patient the physician assistant notes the fundal height is at the level of the umbilicus. This corresponds to what gestational age?
A. 16 weeks
B. 20 weeks
C. 24 weeks
D. 28 weeks
9. Which of the following is a common symptom associated with laryngotracheobronchitis (viral croup)?
A. Drooling
B. High fever
C. “Hot potato” voice
D. Barking cough
10. A 23 year-old female with history of asthma for the past 5 years presents with complaints of increasing shortness ofbreath for 2 days. Her asthma has been well-controlled until 2 days ago. Since yesterday, she has been using heralbuterol inhaler every 4 to 6 hours. She is normally very active, however yesterday she did not complete her 30minu...]]>
The Physician Assistant Life | Smarty PANCE full clean 7:19 54
The Audio PANCE and PANRE Board Review Podcast Episode 5 http://podcast.thepalife.com/audiopance-5/ Thu, 04 Sep 2014 13:55:09 +0000 http://podcast.thepalife.com/?p=48 http://podcast.thepalife.com/audiopance-5/#respond http://podcast.thepalife.com/audiopance-5/feed/ 0 <p>The Audio PANCE and PANRE is an audio board review series that includes 10 Multiple Choice PANCE and PANRE Board Review Questions in each episode. I created this audio series to help you fill some downtime when you are unable to read (like in the car) with some useful board review. I hope you enjoy this free audio component […]</p> <p>The post <a rel="nofollow" href="http://podcast.thepalife.com/audiopance-5/">The Audio PANCE and PANRE Board Review Podcast Episode 5</a> appeared first on <a rel="nofollow" href="http://podcast.thepalife.com">The Audio PANCE and PANRE</a>.</p> THE-AUDIO-PANCE-AND-PANRE 300x300The Audio PANCE and PANRE is an audio board review series that includes 10 Multiple Choice PANCE and PANRE Board Review Questions in each episode.

I created this audio series to help you fill some downtime when you are unable to read (like in the car) with some useful board review.

I hope you enjoy this free audio component to the examination portion of this site. The full series is available to all members of the PANCE and PANRE Academy.

Questions

6 y/o male presents w/ hemarthrosis of the L knee. Coag studies: PT 12.5s (normal12-14 sec), INR 1.0, aPTT 58s (normal 18-28 sec), platelet 430,000/microliter (normal 150,000-450,000/microliter), & bleeding time 4m (normal 2-12m). Which is the best tx option for this pt?

A. Desmopressin acetate
B. Corticosteroids
C. Vitamin K
D. Cryoprecipitate

Click here to see the answer
D. Hemophilia A presents with a prolonged aPTT and normal platelet count and function. Hemophilia A is treated with factor VIII concentrate or cryoprecipitate.

HIV+ pt presents w/ worsening dementia, fever, HA, & R hemiparesis. Brain MRI reveals 6 lesions throughout the brain that show ring enhancement & surrounding edema. Which is the tx of choice?

A. Sulfadiazine and pyrimethamine
B. Trimethoprim-sulfamethoxazole
C. Radiation therapy
D. Ventricular shunt placement

Click here to see the answer
A. Toxoplasmosis is commonly noted in HIV positive patients and presents with multiple ring-enhancing lesions. Treatment of choice for possible toxoplasmosis is sulfadiazine and pyrimethamine.

On examination of a pregnant patient the physician assistant notes a bluish or purplish discoloration of the vagina and cervix. This is called

A. Hegar’s sign.
B. McDonald’s sign.
C. Cullen’s sign
D. Chadwick’s sign

Click here to see the answer
D. Chadwick’s sign is a bluish or purplish discoloration of the vagina and cervix.

A 52 year-old female comes to the office because of black stools for the past 3 days. She is afebrile and she has no pertinent physical examination abnormalities. Which of the following is the most appropriate initial diagnostic study?

A. Stool for occult blood
B. Stool cultures
C. Sigmoidoscopy
D. Abdominal CT scan

Click here to see the answer
A. Occult bleeding, as evidenced by the patient’s history of black stools, is initially verified by a positive fecal occult blood test.

Which of the following is considered a risk factor for the development of malignant melanoma?

A. male gender
B. inability to tan
C. Japanese ethnicity
D. brown-haired individuals

Click here to see the answer
B. Inability to tan and propensity to burn are risk factors for developing malignant melanoma.

A 43 year-old data entry clerk presents with a one-month history of pain and tingling in the right thumb, index finger, and middle finger. Tinel’s sign and Phalen’s maneuver are positive. The most appropriate intervention at this time is

A. methylprednisolone (Medrol) dose pack.
B. splint in neutral position.
C. observation.
D. surgery.

Click here to see the answer
B. Splinting in neutral position relieves impingement of the median nerve, thus improving symptoms of carpal tunnel.

Which of the following is a cause of prerenal azotemia?

A. Infection
B. Renal toxins
C. Poor renal perfusion
D. Urinary tract obstruction

Click here to see the answer
C. Renal hypoperfusion is the cause of prerenal azotemia, which may be rapidly reversible when renal blood flow and glomerular ultrafiltration pressure are restored.

Which of the following is the most common cause of secondary hypertension?

A. Renal parenchymal disease
B. Primary aldosteronism
C. Oral contraceptive use
D. Cushing’s syndrome

Click here to see the answer
A. Renal parenchymal disease is the most common cause of secondary hypertension.

A newborn weighs 8 pounds at birth. On average, what should the infant weigh at 1 year of age?

A. 16 pounds
B. 20 pounds
C. 24 pounds
D. 28 pounds

Click here to see the answer
C. An infant will triple birth weight within the first year of life. A newborn that weighs 8 pounds at birth will weigh approximately 24 pounds at 1 year of age.

The most reliable sign of acute otitis media (AOM) is

A. bulging of the tympanic membrane.
B. loss of tympanic membrane mobility.
C. reddening of the tympanic membrane.
D. air bubbles behind the tympanic membrane.

Click here to see the answer
B. Loss of tympanic membrane mobility during pneumoinsufflation is the most reliable sign for diagnosing acute
otitis media.

Looking for all the episodes?

This FREE series is limited to every 3rd episode, you can download and enjoy the complete audio series by joining The PANCE and PANRE Exam Academy.

I will be be releasing new episodes every two weeks. The Academy is currently discounted at $35 for the entire year, so sign up now.

This Podcast is also available on iTunes and Stitcher Radio for Android

  1. iTunes: The Audio PANCE AND PANRE Podcast iTunes
  2. Stitcher Radio: The Audio PANCE and PANRE Podcast Stitcher

itunes_logo-1


And, if you haven’t already, make sure to download Brian Wallaces’ excellent Physician Assistant Exam Review Podcast. Follow along with Brian who covers new topics twice monthly and really does an amazing job!

The post The Audio PANCE and PANRE Board Review Podcast Episode 5 appeared first on The Audio PANCE and PANRE.

]]>
The Audio PANCE and PANRE is an audio board review series that includes 10 Multiple Choice PANCE and PANRE Board Review Questions in each episode. I created this audio series to help you fill some downtime when you are unable to read (like in the car) ... I created this audio series to help you fill some downtime when you are unable to read (like in the car) with some useful board review.
I hope you enjoy this free audio component to the examination portion of this site. The full series is available to all members of the PANCE and PANRE Academy.
Questions
6 y/o male presents w/ hemarthrosis of the L knee. Coag studies: PT 12.5s (normal12-14 sec), INR 1.0, aPTT 58s (normal 18-28 sec), platelet 430,000/microliter (normal 150,000-450,000/microliter), & bleeding time 4m (normal 2-12m). Which is the best tx option for this pt?
A. Desmopressin acetate
B. Corticosteroids
C. Vitamin K
D. Cryoprecipitate
HIV+ pt presents w/ worsening dementia, fever, HA, & R hemiparesis. Brain MRI reveals 6 lesions throughout the brain that show ring enhancement & surrounding edema. Which is the tx of choice?
A. Sulfadiazine and pyrimethamine
B. Trimethoprim-sulfamethoxazole
C. Radiation therapy
D. Ventricular shunt placement
On examination of a pregnant patient the physician assistant notes a bluish or purplish discoloration of the vagina and cervix. This is called
A. Hegar’s sign.
B. McDonald’s sign.
C. Cullen’s sign
D. Chadwick’s sign
A 52 year-old female comes to the office because of black stools for the past 3 days. She is afebrile and she has no pertinent physical examination abnormalities. Which of the following is the most appropriate initial diagnostic study?
A. Stool for occult blood
B. Stool cultures
C. Sigmoidoscopy
D. Abdominal CT scan
Which of the following is considered a risk factor for the development of malignant melanoma?
A. male gender
B. inability to tan
C. Japanese ethnicity
D. brown-haired individuals
A 43 year-old data entry clerk presents with a one-month history of pain and tingling in the right thumb, index finger, and middle finger. Tinel’s sign and Phalen’s maneuver are positive. The most appropriate intervention at this time is
A. methylprednisolone (Medrol) dose pack.
B. splint in neutral position.
C. observation.
D. surgery.
Which of the following is a cause of prerenal azotemia?
A. Infection
B. Renal toxins
C. Poor renal perfusion
D. Urinary tract obstruction
Which of the following is the most common cause of secondary hypertension?
A. Renal parenchymal disease
B. Primary aldosteronism
C. Oral contraceptive use
D. Cushing’s syndrome
A newborn weighs 8 pounds at birth. On average, what should the infant weigh at 1 year of age?
A. 16 pounds
B. 20 pounds
C. 24 pounds
D. 28 pounds
The most reliable sign of acute otitis media (AOM) is
A. bulging of the tympanic membrane.
B. loss of tympanic membrane mobility.
C. reddening of the tympanic membrane.
D. air bubbles behind the tympanic membrane.
Looking for all the episodes?
This FREE series is limited to every 3rd episode, you can download and enjoy the complete audio series by joining The PANCE and PANRE Exam Academy.
I will be be releasing new episodes every two weeks. The Academy is currently discounted at $35 for the entire year, so sign up now.
This Podcast is also available on iTunes and Stitcher Radio for Android

* iTunes: The Audio PANCE AND PANRE Podcast iTunes
* Stitcher Radio:  full clean 7:30 48
The Audio PANCE and PANRE Board Review Podcast Episode 3 http://podcast.thepalife.com/audiopance-3/ Mon, 21 Jul 2014 15:00:15 +0000 http://podcast.thepalife.com/?p=21 http://podcast.thepalife.com/audiopance-3/#respond http://podcast.thepalife.com/audiopance-3/feed/ 0 <p>This FREE series is limited to every 3rd episode,  you can download and enjoy the complete audio series by joining the PANCE and PANRE Exam Academy. Questions and Answers A 62 year-old homeless patient presents complaining of fever, weight loss, anorexia, night sweats and a chronic cough that recently became productive of purulent sputum that is […]</p> <p>The post <a rel="nofollow" href="http://podcast.thepalife.com/audiopance-3/">The Audio PANCE and PANRE Board Review Podcast Episode 3</a> appeared first on <a rel="nofollow" href="http://podcast.thepalife.com">The Audio PANCE and PANRE</a>.</p> THE-AUDIO-PANCE-AND-PANRE-300x3001

This FREE series is limited to every 3rd episode,  you can download and enjoy the complete audio series by joining the PANCE and PANRE Exam Academy.

Questions and Answers

A 62 year-old homeless patient presents complaining of fever, weight loss, anorexia, night sweats and a chronic cough that recently became productive of purulent sputum that is blood streaked. On physical examination, the patient appears chronically ill and malnourished. Which of the following chest x-ray findings supports your suspected diagnosis?

A. Hyperinflation and flat diaphragms
B. Interstitial fibrosis and pleural thickening
C. Cavitary lesions involving the upper lobes
D. “Eggshell” calcification of hilar lymph nodes C. This patient most likely has tuberculosis.

A chest x-ray finding of cavitary lesions involving the upper lobes would support this suspected diagnosis.

Abduction of the shoulder against resistance helps localize pain in which of the following muscles of the shoulder girdle?

A. Supraspinatus
B. Infraspinatus
C. Teres minor
D. Subscapularis

A. Abduction against resistance tests the supraspinatus.

What is the recommended method for screening pregnant women for gestational diabetes?

A. Fasting blood sugar and 2 hour post prandial
B. 50 gram glucose load followed by a blood sugar in 1 hour
C. 75 gram glucose load followed by a blood sugar in 2 hours
D. 100 gram glucose load followed by a blood sugar at 1 hour, 2 hours, and 3 hours

B. One hour Glucola is the screening test for gestational diabetes. It is a 50 gram glucose load, with a serum glucose obtained 1 hour after the dose. Normal value is less than 140 mg/dL.

A 45 year-old smoker presents with a sore mouth and increasing difficulty eating for two weeks. Physical examination reveals a 1 cm white lesion on the buccal mucosa that cannot be rubbed off. Which of the following is the most likely diagnosis?

A. Oral cancer
B. Oral candidiasis
C. Aphthous ulcer
D. Necrotizing ulcerative gingivitis

A. The presence of leukoplakia in a smoker over the age of 40 should be biopsied to rule out the presence of oral cancer.

A 25 year-old female presents to the emergency department after an episode of substernal chest pain with radiation to the middle of her back that came on suddenly and lasted for about four minutes this morning while in bed. She denies previous episodes. Examination is unremarkable, but she appears jittery. Toxicology screen is positive for cocaine. Which of the following medications is contraindicated in this patient?

A. Lorazepam (Ativan)
B. Diltiazem (Cardizem)
C. Nitroglycerin (Nitrostat)
D. Propanolol (Inderal)

D. Pure Beta blockers, such as propranol, can cause a paradoxical hypertension because of unopposed alpha-adrenergic effects.

Which immunoglobulin is the first to respond during the primary immune response for a gram positive bacterial infection?

A. IgM
B. IgG
C. IgA
D. IgE

A. IgM is the first immunoglobin to respond during the acute exposure. This immunoglobulin promotes opsonization and phagocytosis. IgG is the immunoglobulin that responds during the secondary exposure. IgE is the immunoglobulin that responds during an allergic response as well as during a parasitic infection. IgA is an antibody found in colostrums and GI secretions.

A patient with obsessive compulsive disorder would most likely have which of the following findings?

A. Raw, red hands
B. Priapism
C. Memory impairment
D. Abdominal pain

A. Common manifestations of obsessive compulsive disorder include phobias of germ and contaminants which results in frequent handwashing leading to chafed and reddened hands. The other answers are inconsistent with obsessive compulsive disorder.

As a rule, solid foods such as cereal and fruits are best introduced into an infant’s diet at approximately

A. 1 to 3 weeks.
B. 4 to 8 weeks.
C. 4 to 6 months.
D. 10 to 14 months.

C. The inclusion of solid foods in the diet is best done at approximately 4-6 months. Before 4 to 6 months of age does not contribute significantly to the infant’s health and it increases their risk of allergies and atopy.

A 56 year-old female four days post myocardial infarction presents with a new murmur. On examination the murmur is a grade 3/6 pansystolic murmur radiating to the axilla. She is dyspenic at rest and has rales throughout all her lung fields. Blood pressure is 108/68 mmHg, pulse 70 bpm. Which of the following would be the definitive clinical intervention?

A. Intra-aortic balloon counterpulsation
B. Mitral valve replacement
C. Coronary artery bypass surgery
D. Immediate fluid bolus

B. MVR is the definitive intervention to correct MR caused by papillary muscle rupture.

During an influenza epidemic, a 6 year-old male is seen with fever and a severe sore throat. The parents report that his symptoms have not improved despite administration of aspirin. The next day, the parent calls to report that the child has persistent vomiting and increased lethargy. On examination, he is found to be delirious and disoriented with hyperactive reflexes. The liver edge is 3 cm below the right costal margin in the midclavicular line. Which of the following is the most likely diagnosis?

A. Reye’s syndrome
B. Measles encephalitis
C. Guillain-Barre syndrome
D. Acute bacterial meningitis

A. The suspected influenza associated with development of vomiting, progressive mental status changes, hyperreflexia, and hepatomegaly are consistent with a diagnosis of Reye’s syndrome.

What is the treatment of magnesium sulfate toxicity?

A. Nifedipine
B. Terbutaline
C. Potassium carbonate
D. Calcium gluconate

D. 10% calcium gluconate is used to treat magnesium sulfate toxicity.

The post The Audio PANCE and PANRE Board Review Podcast Episode 3 appeared first on The Audio PANCE and PANRE.

]]> This FREE series is limited to every 3rd episode,  you can download and enjoy the complete audio series by joining the PANCE and PANRE Exam Academy. Questions and Answers A 62 year-old homeless patient presents complaining of fever, weight loss,
This FREE series is limited to every 3rd episode,  you can download and enjoy the complete audio series by joining the PANCE and PANRE Exam Academy.
Questions and Answers
A 62 year-old homeless patient presents complaining of fever, weight loss, anorexia, night sweats and a chronic cough that recently became productive of purulent sputum that is blood streaked. On physical examination, the patient appears chronically ill and malnourished. Which of the following chest x-ray findings supports your suspected diagnosis?
A. Hyperinflation and flat diaphragms
B. Interstitial fibrosis and pleural thickening
C. Cavitary lesions involving the upper lobes
D. “Eggshell” calcification of hilar lymph nodes C. This patient most likely has tuberculosis.
A chest x-ray finding of cavitary lesions involving the upper lobes would support this suspected diagnosis.
Abduction of the shoulder against resistance helps localize pain in which of the following muscles of the shoulder girdle?
A. Supraspinatus
B. Infraspinatus
C. Teres minor
D. Subscapularis
A. Abduction against resistance tests the supraspinatus.
What is the recommended method for screening pregnant women for gestational diabetes?
A. Fasting blood sugar and 2 hour post prandial
B. 50 gram glucose load followed by a blood sugar in 1 hour
C. 75 gram glucose load followed by a blood sugar in 2 hours
D. 100 gram glucose load followed by a blood sugar at 1 hour, 2 hours, and 3 hours
B. One hour Glucola is the screening test for gestational diabetes. It is a 50 gram glucose load, with a serum glucose obtained 1 hour after the dose. Normal value is less than 140 mg/dL.
A 45 year-old smoker presents with a sore mouth and increasing difficulty eating for two weeks. Physical examination reveals a 1 cm white lesion on the buccal mucosa that cannot be rubbed off. Which of the following is the most likely diagnosis?
A. Oral cancer
B. Oral candidiasis
C. Aphthous ulcer
D. Necrotizing ulcerative gingivitis
A. The presence of leukoplakia in a smoker over the age of 40 should be biopsied to rule out the presence of oral cancer.
A 25 year-old female presents to the emergency department after an episode of substernal chest pain with radiation to the middle of her back that came on suddenly and lasted for about four minutes this morning while in bed. She denies previous episodes. Examination is unremarkable, but she appears jittery. Toxicology screen is positive for cocaine. Which of the following medications is contraindicated in this patient?
A. Lorazepam (Ativan)
B. Diltiazem (Cardizem)
C. Nitroglycerin (Nitrostat)
D. Propanolol (Inderal)
D. Pure Beta blockers, such as propranol, can cause a paradoxical hypertension because of unopposed alpha-adrenergic effects.
Which immunoglobulin is the first to respond during the primary immune response for a gram positive bacterial infection?
A. IgM
B. IgG
C. IgA
D. IgE
A. IgM is the first immunoglobin to respond during the acute exposure. This immunoglobulin promotes opsonization and phagocytosis. IgG is the immunoglobulin that responds during the secondary exposure. IgE is the immunoglobulin that responds during an allergic response as well as during a parasitic infection. IgA is an antibody found in colostrums and GI secretions.
A patient with obsessive compulsive disorder would most likely have which of the following findings?
A. Raw, red hands
B. Priapism
C. Memory impairment
D. Abdominal pain
A. Common manifestations of obsessive compulsive disorder include phobias of germ and contaminan...]]>
The Physician Assistant Life | Smarty PANCE full clean 7:51 21 The Audio PANCE and PANRE Board Review Podcast Episode 1 http://podcast.thepalife.com/episode-1/ Fri, 11 Jul 2014 22:21:20 +0000 http://podcast.thepalife.com/?p=1 http://podcast.thepalife.com/episode-1/#respond http://podcast.thepalife.com/episode-1/feed/ 0 <p>This FREE series is limited to every 3rd episode,  you can download and enjoy the complete audio series by joining the PANCE and PANRE Exam Academy. Which of the following factors in patients with chronic venous insufficiency predisposes them to development of skin ulcers? A. Increased intravascular oncotic pressure B. Leakage of fibrinogen and growth factors […]</p> <p>The post <a rel="nofollow" href="http://podcast.thepalife.com/episode-1/">The Audio PANCE and PANRE Board Review Podcast Episode 1</a> appeared first on <a rel="nofollow" href="http://podcast.thepalife.com">The Audio PANCE and PANRE</a>.</p> THE-AUDIO-PANCE-AND-PANRE-300x3001

This FREE series is limited to every 3rd episode,  you can download and enjoy the complete audio series by joining the PANCE and PANRE Exam Academy.

Which of the following factors in patients with chronic venous insufficiency predisposes them to development of skin ulcers?

A. Increased intravascular oncotic pressure
B. Leakage of fibrinogen and growth factors into the interstitial space
C. Decreased capillary leakage
D. Inherited deficiency of protein C B. Leakage of fibrinogen and growth factors into the interstitial space, leukocyte aggregation and activation, and obliteration of the cutaneous lymphatic network can predispose a patient to skin ulcers

A 26 year-old monogamous female presents with cyclic pelvic pain that has been increasing over the last 6 months. She complains of significant dysmenorrhea and dyspareunia. She uses condoms for birth control. On physical examination her uterus is retroverted and non-mobile, and she has a palpable adnexal mass on the left side. Her serum pregnancy test is negative. Which of the following is the most likely diagnosis?

A. Ovarian cancer
B. Endometriosis
C. Functional ovarian cyst
D. Pelvic inflammatory disease B. With endometriosis, the uterus is often fixed and retroflexed in the pelvis. The palpable mass is an endometrioma or “chocolate cyst”. The patient with endometriosis also often has dysmenorrhea, dyspareunia, and dyschezia.

At what age does the first tooth usually erupt in an infant?

A. 2-4 months
B. 6-8 months
C. 10-12 months
D. 14-16 months B. The first tooth in an infant to erupt is the central incisor at the average age of 6-8 months.

A 7 year-old boy wets the bed on most nights. Which of the following is the preferred pharmacological agent to decrease the incidence of bed wetting episodes?

A. Imipramine (Tofranil)
B. Phenytoin (Dilantin)
C. Pramipexole (Mirapex)
D. Hyoscyamine (Urised) A. Imipramine is an anti-cholinergic and when given before bedtime has been shown to decrease the incidence of bed wetting.

A newborn is being evaluated for perioral cyanosis while feeding associated with sweating. Vital signs are rectal temperature, 37.8 degrees C (100 degrees F), blood pressure 80/45 mmHg, pulse 180/min, and respirations 40/min. A grade 3/6 harsh systolic ejection murmur with a single loud S2 is heard at the left upper sternal border. Electrocardiogram (ECG) shows right ventricular hypertrophy with right axis deviation. Chest x-ray shows a boot-shaped heart and decreased pulmonary vascular markings. Which of the following is the most likely diagnosis?

A. Atrial septal defect
B. Total anomalous pulmonary venous return
C. Coarctation of the aorta
D. Tetralogy of Fallot D. This is a common presentation for tetralogy of fallot.

Which of the following is considered to be the modality of choice for the identification of a pituitary macroadenoma that is suspected on the basis of a visual field deficit?

A. Skull x-ray
B. PET scan
C. CT of the brain
D. MRI of the brain D. MRI of the brain provides the best visualization of pituitary tumors.

A 2 year-old male presents with a four day history of fever and general malaise. On examination the vitals reveal an oral temperature of 102 degrees F. The child appears to have rubor on the trunk which started one day prior to this visit. Physical examination reveals a maculopapular rash with defervescence. Which of the following is the most appropriate management at this time?

A. Ibuprofen (Motrin)
B. Aspirin
C. Amoxicillin
D. Valacyclovir (Valtrex) A. Motrin is indicated for management of the fever in Roseola infantum caused by the herpesvirus.

A 42 year-old female experiences pain on the plantar surface of her left foot in the area of the third metatarsal head. The pain is associated with wearing tight shoes and is relieved by removing shoes. Examination reveals a palpable mass and reproduction of pain with deep palpation of the third intermetatarsal space. The patient has tried wearing wider shoes with metatarsal cushions and taking NSAIDS but her symptoms persist. What is the best therapeutic option at this point?

A. Casting of the involved foot
B. Physical therapy
C. Steroid injection
D. Surgical excision C. Steroid injection is the treatment of choice for Morton’s neuroma when conservative measures fail.

Which of the following is the most likely to develop into a persistent cough in the adult patient?

A. Pertussis
B. Allergic rhinitis
C. Pharyngitis
D. Heart failure A. Pertussis is suspected in patients with persistent cough that lasts longer than 2-3 weeks. Allergic rhinitis, pharyngitis and heart failure are all potential causes of acute cough.

A 30 year-old patient presents with weight loss, diarrhea, and steatorrhea. Labs reveal that the antiendomysial antibody (AEA) is positive. What is the most likely diagnosis?

A. Celiac sprue
B. Ulcerative colitis
C. Whipple’s disease
D. Zollinger-Ellison syndrome A. Celiac sprue is not only characterized by these classic symptoms. The antiendomysial antibody has a 90-95% sensitivity and 90-95% specificity for celiac sprue.

A patient sustained a 6 cm laceration on his anterior tibia that was primarily closed in the emergency department. What is the most appropriate time frame for removal of these sutures?

A. 1-2 days
B. 3-5 days
C. 6-8 days
D. 7-14 days D. Suture removal is based upon the area of the body that was sutured. Facial sutures are placed for 3-4 days, scalp sutures for 5-7 days, trunk sutures are placed for 6-8 days, and sutures on the extremity are placed for 7-14 days. Sutures on the extremities can stay for longer periods of time if the area is under maximal tension.

A hospitalized patient is found with confirmed pulseless ventricular tachycardia. IV access is obtained following the second shock given. Which of the following medications is to be administered immediately?
A. Amiodarone
B. Magnesium
C. Atropine
D. Epinephrine D. Epinephrine should be given as soon as IV access is obtained before or after the second shock.

A patient with advanced AIDS complicated by toxoplasmosis presents with altered mental status, recent onset of seizures, and focal neurologic deficits. Which of the following diagnostic studies is most helpful?

A. Toxoplasma gondii antibody titers
B. CT scan of the brain
C. Lumbar puncture
D. MRI of brain D. An MRI showing multiple isodense or hypodense ring-enhancing mass lesions is the most useful test for such a patient.

The post The Audio PANCE and PANRE Board Review Podcast Episode 1 appeared first on The Audio PANCE and PANRE.

]]>
This FREE series is limited to every 3rd episode,  you can download and enjoy the complete audio series by joining the PANCE and PANRE Exam Academy. Which of the following factors in patients with chronic venous insufficiency predisposes them to develo...
This FREE series is limited to every 3rd episode,  you can download and enjoy the complete audio series by joining the PANCE and PANRE Exam Academy.
Which of the following factors in patients with chronic venous insufficiency predisposes them to development of skin ulcers?
A. Increased intravascular oncotic pressure
B. Leakage of fibrinogen and growth factors into the interstitial space
C. Decreased capillary leakage
D. Inherited deficiency of protein C B. Leakage of fibrinogen and growth factors into the interstitial space, leukocyte aggregation and activation, and obliteration of the cutaneous lymphatic network can predispose a patient to skin ulcers
A 26 year-old monogamous female presents with cyclic pelvic pain that has been increasing over the last 6 months. She complains of significant dysmenorrhea and dyspareunia. She uses condoms for birth control. On physical examination her uterus is retroverted and non-mobile, and she has a palpable adnexal mass on the left side. Her serum pregnancy test is negative. Which of the following is the most likely diagnosis?
A. Ovarian cancer
B. Endometriosis
C. Functional ovarian cyst
D. Pelvic inflammatory disease B. With endometriosis, the uterus is often fixed and retroflexed in the pelvis. The palpable mass is an endometrioma or “chocolate cyst”. The patient with endometriosis also often has dysmenorrhea, dyspareunia, and dyschezia.
At what age does the first tooth usually erupt in an infant?
A. 2-4 months
B. 6-8 months
C. 10-12 months
D. 14-16 months B. The first tooth in an infant to erupt is the central incisor at the average age of 6-8 months.
A 7 year-old boy wets the bed on most nights. Which of the following is the preferred pharmacological agent to decrease the incidence of bed wetting episodes?
A. Imipramine (Tofranil)
B. Phenytoin (Dilantin)
C. Pramipexole (Mirapex)
D. Hyoscyamine (Urised) A. Imipramine is an anti-cholinergic and when given before bedtime has been shown to decrease the incidence of bed wetting.
A newborn is being evaluated for perioral cyanosis while feeding associated with sweating. Vital signs are rectal temperature, 37.8 degrees C (100 degrees F), blood pressure 80/45 mmHg, pulse 180/min, and respirations 40/min. A grade 3/6 harsh systolic ejection murmur with a single loud S2 is heard at the left upper sternal border. Electrocardiogram (ECG) shows right ventricular hypertrophy with right axis deviation. Chest x-ray shows a boot-shaped heart and decreased pulmonary vascular markings. Which of the following is the most likely diagnosis?
A. Atrial septal defect
B. Total anomalous pulmonary venous return
C. Coarctation of the aorta
D. Tetralogy of Fallot D. This is a common presentation for tetralogy of fallot.
Which of the following is considered to be the modality of choice for the identification of a pituitary macroadenoma that is suspected on the basis of a visual field deficit?
A. Skull x-ray
B. PET scan
C. CT of the brain
D. MRI of the brain D. MRI of the brain provides the best visualization of pituitary tumors.
A 2 year-old male presents with a four day history of fever and general malaise. On examination the vitals reveal an oral temperature of 102 degrees F. The child appears to have rubor on the trunk which started one day prior to this visit. Physical examination reveals a maculopapular rash with defervescence. Which of the following is the most appropriate management at this time?
A. Ibuprofen (Motrin)
B. Aspirin
C. Amoxicillin
D. Valacyclovir (Valtrex) A. Motrin is indicated for management of the fever in Roseola inf...]]>
The Physician Assistant Life | Smarty PANCE full clean 8:16 1